ID,Context,Question,Choices,Answer,Solution,Subject 2463,A 35-year-old man is referred to you for the evaluation of persistently elevated diastolic BP despite lifestyle changes over the past 6 months. He feels well and has no other comorbid conditions other than the fact that he is overweight (body mass index (BMI) is 27.6 kg/m2). His father and grandfather had myocardial infarctions before age 50. He is worried about his cardiovascular risk. His BP is 130/102 mmHg and his exam is otherwise normal. A home BP log shows a home BP average of 126/97 mmHg. A basic evaluation for secondary causes of hypertension is negative. His diastolic BP has remained elevated despite a 6-month trial of lifestyle modification..,"In addition to ongoing lifestyle modification, which ONE of the following options would represent your BEST recommendation to him?","A. You should reassure him that his elevated diastolic BP does not have adverse prognostic implications because his systolic BP is normal B. You should inform him that although his cardiovascular risk is increased, isolated diastolic hypertension does not require treatment C. You should inform him that his cardiovascular risk is less than patients with systo-diastolic hypertension and that you would not treat him until his home diastolic BPs are > 100 mmHg D. You should inform him that his cardiovascular risk is increased and that you recommend that he start drug treatment",D,temp nan,Hypertension 2464,"You are asked to make a public health presentation about the current state of hypertension prevalence, diagnosis, and treatment in the United States at a local community fair..",Which ONE of the following statements regarding the epidemiology of hypertension is CORRECT?,"A. < 50% of hypertensive adults are aware that they have hypertension B. About 50% of people with hypertension are receiving treatment C. About 75% of people with hypertension are controlled and this rate is steadily improving D. About 50% of people with hypertension are controlled E. The prevalence of hypertension has steadily increased over the past decade",D,temp nan,Hypertension 2465,"A 67-year-old woman is seen during a routine follow-up visit for hypertension. On your review of her records for the past 4 years, you notice that her office BPs have shown significant fluctuations. Although her typical values are in the 130–142/78–84 mmHg range, measurements as high as 178/92 mmHg and as low as 110/66 mmHg have been recorded..",Which ONE of the following statements is CORRECT with regard to her long-term BP variability?,"A. Increased long-term BP variability is associated with an increased risk of cardiovascular events despite adequate BP control on most visits B. Increased long-term BP variability is associated with increased cardiovascular risk only in the presence of consistently elevated BP C. Beta-blockers decrease long-term BP variability D. Long-term BP variability has no relationship to cardiovascular events",A,temp nan,Hypertension 2466,,"Question 2466: Which ONE of the following statements is CORRECT regarding potassium intake, hypertension, and cardiovascular risk?","A. High potassium intake results in lower BP levels exclusively in individuals with low sodium intake B. Low potassium intake is associated with an increased risk of cardiovascular but not all-cause mortality C. High dietary potassium results in natriuresis and lower BP by decreasing aldosterone secretion D. Low dietary potassium promotes hypertension by increasing the activity of the sodium chloride cotransporter (on a high sodium diet) E. Dietary potassium has no significant role in BP regulation",D,temp nan,Hypertension 2467,"A 56-year-old man with type 2 diabetes mellitus is referred for the management of proteinuria and resistant hypertension. His antihypertensive medications include chlorthalidone, lisinopril, diltiazem, and doxazosin. Spironolactone had been tried with a favorable BP response, but he experienced significant gastrointestinal side effects and stopped taking it. Home BP readings average 148/90 mmHg. On physical examination, his BP is 160/92 mmHg, his pulse rate is 60/min, and there is 1+ leg edema. Laboratory studies show an eGFR (Chronic Kidney Disease Epidemiology Collaboration [CKD-EPI]) of 78 ml/min per 1.73 m2, a serum potassium level of 3.8 mEq/L, and a urine albumin/creatinine ratio of 1182 mg/g..",Which ONE of the following is the MOST appropriate treatment?,"A. Add atenolol B. Add minoxidil C. Add clonidine D. Add amiloride",D,temp nan,Hypertension 2468,"You are seeing a 32-year-old black woman for a health-maintenance visit in a primary care practice. She is asymptomatic. Her BP is 128/80 mmHg, her BMI is 30 kg/m2, and her physical examination is negative..",Which ONE of the following is the MOST appropriate time for her to have her blood pressure rechecked according to recommendations of the U.S. Preventive Services Task Force?,"A. In 6 months B. In 1 year C. In 3–5 years D. As needed, according to her symptoms",B,temp nan,Hypertension 2469,"A 56-year-old woman is referred for further evaluation and management of hypertension. She has had several elevated office BPs for the past year despite dietary sodium reduction, increased physical activity, and weight loss. She is otherwise well and is on no medications. Home BP readings average 134/85 mmHg. On physical examination, her BP is 152/94 mmHg..","In addition to continued monitoring of her BP, which ONE of the following is the MOST appropriate next step in management?","A. No change in management B. Obtain 24-hour ambulatory BP monitoring C. Obtain an echocardiogram D. Start drug treatment",B,temp nan,Hypertension 2470,"A 45-year-old man with hypertension is referred to you for the evaluation of treatment resistance. His antihypertensive medications include amlodipine 10 mg daily, valsartan 320 mg daily, chlorthalidone 25 mg daily, and metoprolol 50 mg twice daily. He has been adherent to therapy. A physical examination shows a BP of 160/98 mmHg. His body mass index is 30 kg/m2. His pulse rate is 60/min. The remainder of the examination is normal. Laboratory studies show a serum potassium level of 4.2 mEq/L and a serum creatinine (SCr) level of 0.8 mg/dl. The urinalysis is normal, and the urine albumin/creatinine ratio is 28 mg/g. You decide to order a 24-hour ambulatory BP monitoring study to confirm treatment resistance. Results of this study show the following average BPs..","24-hour: 134/83 mmHg Daytime (awake): 140/87 mmHg Nighttime (sleep): 122/78 mmHg Which ONE of the following is the MOST appropriate management?","A. No change in management B. Add doxazosin C. Increase the dose of metoprolol D. Add spironolactone",D,temp nan,Hypertension 2471,,"Question 2471: Which ONE of the following statements is CORRECT regarding strategies for population based management of hypertension?","A. Use of self-monitored BP at home results in worse BP control rates B. Home BP tele-monitoring supervised by a health care provider results in higher rates of BP control than usual care C. Self-treatment titration on the basis of home BP readings is suitable for more than 75% of hypertensive patients D. Self-treatment titration on the basis of home readings is safe for uncomplicated hypertensive patients, but not for patients with comorbid conditions E. Financial incentives to physicians do not result in any measurable improvement in BP control rates",B,temp nan,Hypertension 2472,"You are seeing a 31-year-old woman with autosomal dominant polycystic kidney disease. She is asymptomatic and feels well. She has liver cysts, but no other known extrarenal manifestations of autosomal dominant polycystic kidney disease. She is on no medications. On physical examination, her BP is 132/80 mmHg. Her home BP monitoring shows readings that average 130/82 mmHg. Laboratory studies show an eGFR (CKD-EPI) of 82 ml/min per 1.73 m2 and a potassium level of 4.1 mEq/L..",Which ONE of the following is the MOST appropriate target BP to attenuate increases in total kidney volume in this patient?,"A. 140/80 mmHg B. 130/80 mmHg C. 120/75 mmHg D. 110/75 mmHg",D,temp nan,Hypertension 2473,"A 70-year-old man with multiple system atrophy is referred to you for assistance in the management of orthostatic hypotension complicated by supine hypertension. He has poor orthostatic tolerance and has had multiple episodes of syncope. A trial of fludrocortisone had been terminated because of excessive edema and supine hypertension (BP approximately 200/110 mmHg). Midodrine caused pruritus and a severe headache, requiring discontinuation. He is taking no vasoactive medications. His supine BP is 152/86 mmHg. His standing BP is 80/52 mmHg (measured 3 minutes after standing), and is accompanied by significant dizziness. Laboratory studies are all normal..","In addition to usual nonpharmacologic measures, which ONE of the following is the MOST appropriate treatment recommendation to improve his orthostatic hypotension?","A. Droxidopa B. Pyridostigmine C. Clonidine D. Erythropoietin E. A caffeinated beverage with each meal",A,temp nan,Hypertension 2474,A 64-year-old woman with essential hypertension is evaluated in routine follow-up. She has seen information about the new BP guidelines published by the Joint National Committee (JNC) in 2014 and she is worried that her BP may be too low. She feels well and is asymptomatic. She does not have diabetes mellitus or coronary artery disease. Her medications include lisinopril 10 mg daily and hydrochlorothiazide (HCTZ) 12.5 mg daily. Her current BP is 132/74 mmHg. The remainder of the findings from the examination are unremarkable. Laboratory studies show a SCr level of 0.7 mg/dl..,"Which ONE of the following is the MOST appropriate management of her hypertension, according to the Eighth JNC 2014 evidence based guidelines for the management of high blood pressure in adults?","A. Discontinue lisinopril and HCTZ B. Discontinue lisinopril C. Discontinue HCTZ D. Make no changes to the current regimen",D,temp nan,Hypertension 2475,"A 56-year-old black woman is evaluated for inadequately controlled hypertension. She has stage 3 CKD, but does not have diabetes mellitus. Her home blood pressure readings range between 150–170/80–95 mmHg. She is currently only taking enalapril 40 mg daily. On physical examination, her body mass index (BMI) is 20 kg/m2 and her blood pressure is 165/95 mmHg. There is trace pretibial edema. The remainder of the examination is normal. Laboratory studies disclosed the following values: sodium 138 mEq/L, potassium 3.9 mEq/L, chloride 106 mEq/L, total CO2 22 mmol/L, SCr 1.9 mg/dl, and an eGFR of 44 ml/min per 1.73 m2. You decide to start a thiazide diuretic, and consider the risk of hyponatremia during your discussion of potential side effects of this intervention with the patient..",Which ONE of the following factors is associated with the greatest risk of thiazide-associated hyponatremia for this patient?,"A. Female sex B. Her eGFR C. Use of chlorthalidone over HCTZ at equipotent dosing D. Her BMI",D,temp nan,Hypertension 2476,,"Question 2476: Which ONE of the following MOST likely contributes the most to the pathogenesis of salt-sensitive hypertension in obese individuals?","A. Decreased expression of angiotensin 1 receptors in the proximal tubule B. Increased serum renin levels C. Inappropriately low atrial natriuretic peptide levels D. Decreased renal expression of Rac1",C,temp nan,Hypertension 2477,"A 66-year-old obese man is evaluated for poorly controlled hypertension. Ambulatory BP monitoring shows a mean daytime BP of 160/94 mmHg, and a mean nighttime BP of 152/92 mmHg without nocturnal dipping. He has sleep apnea, but finds it difficult to use the continuous positive airway pressure machine. His medications include lisinopril 40 mg daily, amlodipine 10 mg daily, and chlorthalidone 25 mg daily. A physical examination shows a BMI of 40 kg/m2, a BP of 162/94 mmHg, and a pulse rate of 68/min. His abdomen is obese. The remainder of the examination is unremarkable..","In addition to weight loss, which ONE of the following is the MOST effective nonpharmacologic intervention to lower BP further and improve the nocturnal BP pattern?","A. Nocturnal oxygen supplementation B. Psychotherapy C. Device-guided breathing D. Regular use of continuous positive airway pressure for at least 4 hours per night",D,temp nan,Hypertension 2478,"A 34-year-old white woman is referred for evaluation of recent-onset severe hypertension. She is asymptomatic. She does not have witnessed nocturnal choking or gasping, headaches, palpitations, or sweating. There is no family history of hypertension. Her regular medications are: losartan 100 mg daily, felodipine 10 mg daily, metoprolol 100 mg twice daily, and chlorthalidone 25 mg daily. A physical examination shows a BMI of 24 kg/m2, a BP of 172/102 mmHg without arm discrepancy or postural changes, and a pulse rate of 60/min. There is no radiofemoral delay. No abdominal bruits are detected. The remainder of the examination is normal. Laboratory studies disclose the following values: serum sodium 138 mEq/L, potassium 3.5 mEq/L, chloride 99 mEq/L, total CO2 29 mmol/L, BUN 12 mg/dl, SCr 1.2 mg/dl, TSH 2.0 U/L, supine plasma aldosterone 18 ng/dl per hour (reference range 2–5 ng/dl per hour), and standing plasma renin activity 12 ng/ml per hour (reference range 0.6–3.0 ng/ml per hour). An MRI of the abdomen shows no adrenal or kidney abnormalities..",Which ONE of the following is the MOST appropriate next step in the diagnostic evaluation of this patient?,"A. Measure plasma metanephrines B. Refer for adrenal vein sampling C. Refer for polysomnography D. Order a CT angiogram",D,temp nan,Hypertension 2479,"A 55-year-old man has severe resistant hypertension and is referred for further evaluation. He has no other medical problems. A physical examination shows a BP of 164/88 mmHg, a pulse rate of 72/min, and a body mass index (BMI) of 29 kg/m2. Laboratory studies show a serum creatinine (SCr) level of 1.0 mg/dl and potassium level of 3.9 mEq/L. You perform an appropriate workup for secondary hypertension. His plasma renin activity is 0.3 ng/ml per hour (reference range, 0.6–3.0 ng/ml per hour) and his plasma aldosterone level is 44 ng/dl (reference range, 2–5 ng/dl). He had a right adrenal adenoma shown on abdominal CT scan..",Which ONE of the following is the MOST appropriate next step in the management of this patient?,"A. Refer for surgery B. Add spironolactone 50 mg daily C. Add amiloride 5 mg twice daily D. Refer for adrenal vein sampling",D,temp nan,Hypertension 2480,,"Question 2480: Which ONE of the following indices best identifies individuals with primary aldosteronism who are MOST likely to benefit from surgery?","A. An aldosterone/renin ratio of 40 B. A selectivity index of 6 C. A lateralization index of 6 D. Contralateral suppression of aldosterone E. A low peripheral venous aldosterone/cortisol ratio",C,temp nan,Hypertension 2481,"A 26-year-old man develops an acute and severe elevation in BP to 220/140 mmHg during elective surgery for a hernia repair. The surgery is abandoned because of persistently high and uncontrolled BP. He has no prior history of hypertension, but does have a family history of hypertension..",Which ONE of the following is the MOST appropriate diagnostic test to order next?,"A. Aldosterone/renin ratio B. Polysomnography C. CT angiography D. Plasma metanephrines E. Plasma catecholamines",D,temp nan,Hypertension 2482,A 44-year-old man with paroxysmal hypertension has recently been diagnosed with an 8 cm paraaortic paraganglioma in the organ of Zuckerkandl observed on computed tomography imaging. His plasma catecholamines and metanephrines are ten times above the upper limit of the reference range. He is referred for surgical management..,"In addition to adequate preoperative combined alpha- and beta-blockade before surgery, which ONE of the following is the next MOST appropriate step in management?","A. 123-I-metaiodobenzylguanidine scintigraphy B. Fluorodeoxyglucose positron emission tomography C. Genetic testing D. Urine catecholamines",C,temp nan,Hypertension 2483,"A 72-year-old man with a history of poorly controlled hypertension is evaluated 3 months after a left-sided middle cerebral artery stroke. He wants advice about the optimal BP for which he should aim in order to prevent a future stroke. On physical examination, his BP is 156/80mmHg. He does not have carotid bruits or a cardiac murmur. He has decreased strength in the right arm graded at 5-/5. The remainder of the examination is normal. Review of a magnetic resonance imaging/angiography study of the brain performed during his hospitalization 3 months ago shows no evidence of large vessel disease and an infarct in the distribution of the left middle cerebral artery..","Which ONE of the following is associated with the highest 5-year risk of stroke, acute myocardial infarction, and death for this patient?","A. A discharge systolic BP of > 140 mmHg B. Maintenance of his systolic BP <140 mmHg C. Maintenance of his systolic BP <130 mmHg D. Maintenance of his systolic BP <120 mmHg",D,temp nan,Hypertension 2484,"A 65-year-old woman with hypertension and atrial fibrillation is evaluated in the emergency department for acute onset of headache, right sided weakness, and lethargy. A CT scan of her head reveals a left-sided frontal lobe hemorrhage. She has been adherent with warfarin anticoagulation, and her admission international normalized ratio (INR) is 2.4. A physical examination shows a BP of 184/88 mmHg. A neurologic examination shows lethargy, a dense right-leg paresis, and decreased strength in the right arm..","In addition to discontinuing warfarin, which ONE of the following is the MOST appropriate in the management of this patient?","A. Reversal of INR to 2.0 < 4 hours B. Reversal of INR to 1.3 < 4 hours C. Reversal of INR to < 1.3 plus lowering of systolic BP < 160 mmHg within 4 hours D. Reversal of INR to < 1.3 plus lowering of systolic BP < 140 mmHg within 4 hours",C,temp nan,Hypertension 2485,"An 84-year-old woman with hypertension is evaluated during a routine follow-up visit. Her daughter is concerned that her memory has gradually declined over the past year. She has been on a stable antihypertensive regimen over the past 10 years that includes lisinopril, amlodipine, and chlorthalidone. A physical examination shows a BP of 138/76 mmHg and a pulse rate of 66/min. She has no edema. The findings of the examination are otherwise unremarkable. Ambulatory BP monitoring shows a mean daytime BP of 140/80 mmHg, and a mean nighttime BP of 136/84 mmHg without nocturnal dipping. The results of a 24-hour urine test shows 130 mEq of sodium excretion..",Which ONE of the following findings is MOST predictive of cognitive decline in this woman?,"A. Her diastolic BP B. Her systolic BP C. Her dietary sodium intake D. The absence of nocturnal dipping",D,temp nan,Hypertension 2486,"A 72-year-old white man with a history of CKD is referred for evaluation of resistant hypertension. His average blood pressure is 170/95 mmHg on a regimen that includes optimally dosed valsartan, amlodipine, carvedilol, and chlorthalidone. A physical examination shows a BMI of 24 kg/m2, and a BP of 172/96 mmHg without arm discrepancy. He has grade 2 hypertensive retinopathy and an S4 gallop. He does not have edema. The remainder of the examination is unremarkable. Laboratory studies show a stable SCr level of 2.1 mg/dl (eGFR, 38 ml/min per 1.73 m2)..",Which ONE of the following is MOST likely to be associated with resistant hypertension in this man?,"A. His BMI B. Underlying CKD C. His age D. His race",B,temp nan,Hypertension 2488,"A 55-year-old woman with hypertension, hyperlipidemia, and active smoking is evaluated in follow-up for hypertension. She asymptomatic. She is currently taking lisinopril 40 mg daily and chlorthalidone 25 mg daily for her hypertension. Her office BP is 139/78 mmHg in the seated position and 135/76 mmHg after standing for 2 minutes. The findings from her physical examination and laboratory tests are unremarkable..","Based on the findings of the Systolic Blood Pressure Intervention Trial (SPRINT), which ONE of the following would be the MOST appropriate next step in her management?","A. Continue the same treatment B. Decrease the dose of lisinopril to allow her systolic BP to increase above 140 mmHg C. Add amlodipine, targeting a systolic BP of < 140 mmHg D. Add amlodipine, targeting a systolic BP of < 120 mmHg E. Stop lisinopril and substitute amlodipine",D,temp nan,Hypertension 2489,The SPRINT study compared a systolic BP target of 140 mmHg with a target of 120 mmHg in patients without diabetes who had hypertension and a high risk for cardiovascular disease..,Which ONE of the following statements is correct about SPRINT?,"A. The study results were difficult to interpret because the separation of systolic BP between the two groups was only 4 mmHg (130 mmHg versus 126 mmHg) B. The low BP target (120 mmHg) was associated with acute kidney injury in 20% of patients C. The low BP target (120 mmHg) resulted in a 38% reduction in the risk of heart failure D. The standard BP target (140 mmHg) resulted in a 14% reduction in the risk of myocardial infarction E. Among study subjects older than 75 years, randomization to the low BP target (120 mmHg) group resulted in a 22%-higher rate of cardiovascular events",C,temp nan,Hypertension 2490,"A 28-year-old woman is referred to you for the evaluation of hypertension that has been present for the past 6 months. She has no significant medical problems and is taking no medications. On physical examination, her BP is 150/98 mmHg. There are no other relevant findings. As part of her evaluation, a computed tomography angiogram shows a segment of multifocal left main renal artery stenosis, consistent with medial fibroplasia..",Which ONE of the following is the MOST appropriate next step in her management?,"A. Start losartan, aspirin, and atorvastatin B. Obtain a renal scintigraphy scan with captopril C. Refer for percutaneous renal angioplasty D. Refer for percutaneous renal angioplasty and stenting E. Refer for renal artery bypass",C,temp nan,Hypertension 2491,,"Question 2491: Which ONE of the following statements about 25-hydroxyvitamin D (25OHD) deficiency and hypertension is correct?","A. 25OHD deficiency is associated with higher rates of hypertension in individuals below age 50 but not among older patients B. 25OHD replacement in elderly patients with 25OHD deficiency and isolated systolic hypertension effectively lowers systolic BP compared with placebo C. 25OHD replacement is an effective therapy for orthostatic hypotension D. 25OHD treats the white-coat effect in patients with white-coat hypertension, but does not lower ambulatory BP E. 25OHD replacement has no effect on BP in hypertensive patients with 25OHD deficiency",E,temp nan,Hypertension 2492,"One of your patients with hypertension who is currently treated with amlodipine and benazepril, informs you that he would like to start drinking beetroot juice on the basis of what he has read on the Internet. He asks you for your opinion on its safety and efficacy with regard to hypertension..",Which ONE of the following options is correct about the effects of beetroot juice on BP?,"A. Beetroot juice lowers BP by decreasing aldosterone production B. Beetroot juice has an acute BP-lowering effect over the first 4–7 days, but tachyphylaxis develops after this initial period C. Beetroot juice lowers ambulatory BP by about 8/5 mmHg compared with placebo D. Beetroot juice increases pulse wave reflection E. Beetroot juice is a well known cause of hypertension and should be avoided",C,temp nan,Hypertension 2493,"A 32-year-old woman with type 1 diabetes mellitus, hypertension, and hyperlipidemia desires pregnancy. She asks for your advice about how to achieve the most optimal pregnancy outcome. Her BP has been well controlled on enalapril and nifedipine. Other medications include subcutaneous injections of insulin and simvastatin. She has been adherent with her medications and physician visits. On physical examination, the body mass index (BMI) is 28 kg/m2, and the BP is 122/72 mmHg. The remainder of the examination is unremarkable. Laboratory studies show a serum creatinine level of 1.2 mg/dl, a hemoglobin A1c of 7.8%, a total serum cholesterol level of 176 mg/dl, and an albumin-to-creatinine ratio of 1526 mg/g..",Which ONE of the following would you advise to optimize maternal and fetal outcomes?,"A. Make no changes to the current regimen B. Target the hemoglobin A1c to < 7.0% before conception C. Stop enalapril at least 2 months before conception D. Hold enalapril for the duration of breastfeeding",B,temp nan,Pregnancy and Kidney Disease 2494,"A 35-year-old woman with lupus nephritis and chronic hypertension is evaluated at 32 weeks of gestation for worsening kidney function. Her lupus nephritis was in full remission at the time of conception (off immunosuppressive therapy). Her current medications include nifedipine and labetalol. Her BP has increased from 132/68 to 150/86 mmHg. The physical examination shows no rash or synovitis. There is 1+ pitting pretibial edema. Laboratory studies show that her serum creatinine level has risen from 0.5 mg/dl in the first trimester to 1.0 mg/dl. The alanine aminotransferase (ALT) level is 236 U/L (reference range 0–35), and the aspartate aminotransferase (AST) level is 342 U/L (reference range 0–35). The serum C3 is 82 mg/dl (reference range 55– 120), and the serum C4 is 26 mg/dl (reference range 15–48). The leukocyte count is 5600/µl, the hemoglobin is 8.6 g/dl, and the platelet count is 76,000/µl. A 24-hour urine collection shows 3260 mg protein, increased from 260 mg/d in the first trimester..","Which ONE of the following laboratory results is MOST suggestive of the syndrome of hemolysis, elevated liver enzymes, and low platelets rather than worsening lupus nephritis?","A. Low hemoglobin B. Low platelet count C. Increased AST and ALT D. Increasing proteinuria",C,temp nan,Pregnancy and Kidney Disease 2495,"A 38-year-old woman with chronic hypertension who is 26-weeks pregnant asks you if she should liberalize or more strictly control her BP to prevent adverse pregnancy outcomes. Her current medications are labetalol at 300 mg twice daily and a prenatal vitamin. The BP is 128/85 mmHg, and she has 1+ bilateral ankle edema. Fetal growth is appropriate for gestational age..",Which ONE of the following is the MOST appropriate management?,"A. Implement a 2-g sodium-restricted diet B. Add methyldopa C. Stop labetalol D. Add hydrochlorothiazide E. Make no changes to the current regimen",E,temp nan,Pregnancy and Kidney Disease 2496,"A 38-year-old woman, gravida 4, para 3, is evaluated at 37 weeks of gestation for new-onset hypertension and fetal bradycardia. She complains of a headache and vague right upper quadrant abdominal discomfort. On physical examination, the BP is 165/110 mmHg. A repeat BP reading 4 hours later is 162/112 mmHg. There is 1+ leg edema. Laboratory studies show serum creatinine of 1.1 mg/dl, albumin of 3.4 g/dl, leukocyte count of 7600/µl, hemoglobin of 10 g/dl, and platelets of 96,000/µl. Urinalysis shows 3+ protein, 1+ blood, and two to five dysmorphic erythrocytes per high-power field. The urine protein-to-creatinine ratio is 2100 mg/g. Serologic studies show positive antinuclear antibody titers of > 1:640 and negative double–stranded DNA titers (dsDNA)..","In addition to admission for fetal monitoring, which ONE of the following is the MOST appropriate management recommendation?","A. Induction of labor and delivery B. Kidney biopsy now C. Pulse methylprednisolone at 500 mg intravenously once daily for 3 days D. Antenatal betamethasone for fetal lung maturation",A,temp nan,Pregnancy and Kidney Disease 2497,"A 38-year-old woman is evaluated in the immediate postpartum period after induction of labor and normal vaginal delivery for preeclampsia with severe features. Her reflexes remain brisk, and she complains of a slight headache. Her medications include intravenous magnesium, intravenous labetalol, and oral nifedipine. On physical examination, the BP is 150/90 mmHg. There is 1+ pretibial edema. The remainder of the examination is unremarkable. Laboratory studies show a serum creatinine of 1.2 mg/dl..",Which ONE of the following is the MOST appropriate management?,"A. Discontinue intravenous magnesium, intravenous labetalol, and oral nifedipine B. Discontinue intravenous magnesium but continue intravenous labetalol and oral nifedipine C. Discontinue intravenous magnesium and initiate furosemide D. Continue intravenous magnesium and transition to oral antihypertensive medications",D,temp nan,Pregnancy and Kidney Disease 2498,"A 32-year-old woman with autosomal dominant polycystic kidney disease and chronic hypertension who is 8-weeks pregnant is referred for management of hypertension. Her BP had been previously well controlled on losartan at 50 mg daily. She discontinued it when she decided to become pregnant several months ago. Her office BP is 162/95 mmHg, and her home BP readings have been 150–160/90–95 mmHg. The remainder of the examination is normal. Her serum electrolytes, creatinine, and urinalysis are normal..",Which ONE of the following is the MOST appropriate management?,"A. Labetalol B. Continued observation C. Lisinopril D. Spironolactone E. Atenolol",A,temp nan,Pregnancy and Kidney Disease 2499,"A 37-year-old woman, gravida 2, para 1, is seen for chronic hypertension at 8-weeks gestation. She has a history of preeclampsia complicating her first pregnancy 3 years ago. She says that she has been eating well and adheres to a well-balanced Western diet. Her medications are labetalol and a prenatal vitamin. A physical examination shows that the BMI is 32 kg/m2. The BP is 130/80 mmHg. There is no edema. She is concerned that she may develop preeclampsia again and asks for advice about how to prevent it from occurring..",Which ONE of the following is the MOST appropriate intervention to reduce her risk of preeclampsia?,"A. Supplemental vitamins C and E B. Weight loss targeting a BMI of < 30 kg/m2 C. Aspirin at 81 mg daily D. Calcium carbonate at 600 mg twice daily",C,temp nan,Pregnancy and Kidney Disease 2500,"A 32-year-old woman is seen in follow-up one month after an initial episode of preeclampsia. She delivered at 34-weeks gestation, and her baby weighed 1063 g. She currently is on no medications and has no other past medical history. Her BP is 122/78 mmHg, and the remainder of her physical examination is normal. The serum creatinine level is 0.6 mg/dl. She asks about the effect of preeclampsia on her future health..",Which ONE of the following would you advise regarding the effect of her prior episode of preeclampsia on her long-term health?,"A. It will have no effect B. It increases the risk of hyperthyroidism C. It increases the risk of cardiovascular disease D. She has a 5% risk of developing ESRD over the next 20 years E. It increases her risk for endometrial cancer",C,temp nan,Pregnancy and Kidney Disease 2501,A 19-year-old woman with a history of vesicoureteral reflux and frequent urinary tract infections during childhood is evaluated at 12 weeks of gestation. Her last urinary tract infection was at age 6 years old. Her physical examination is normal. The serum creatinine level is 0.4 mg/dl. The urinalysis is normal. An intravenous urogram at age 6 years old showed no scarring. A recent kidney ultrasound is normal. She asks about the effect of her history of vesicoureteral reflux on her pregnancy..,Which ONE of the following would you advise regarding the effect of vesicoureteric reflux on her pregnancy?,"A. She is at increased risk for preeclampsia B. She is at increased risk of urinary tract infection C. She is at increased risk for gestational hypertension D. There is an increased risk for low birth weight",B,temp nan,Pregnancy and Kidney Disease 2502,"A 20-year-old woman at 25-weeks gestation is evaluated for AKI. She has had nausea, vomiting, and abdominal pain for 2 days. On physical examination, she is afebrile and in moderate distress from the abdominal pain. The BP is 140/80 mmHg, and the pulse rate is 130/min. She has scleral icterus. The abdomen is gravid, with mild diffuse tenderness to palpation. Laboratory studies show sodium of 140 mEq/L, potassium of 3.2 mEq/L, chloride of 106 mEq/L, total CO2 of 12 mmol/L, BUN of 26 mg/ dl, creatinine of 1.6 mg/dl, and glucose of 34 mg/dl. The AST is 1256 U/L, the ALT is 1540 U/L, the total bilirubin is 2.5 mg/dl (direct bilirubin 1.6 mg/dl), the alkaline phosphatase is 249 U/L, the albumin is 3.5 g/dl, and the international normalized ratio is 2.68. The leukocyte count is 20,000/µl, the hemoglobin is 10.1 g/dl, and the platelet count is 102,000/µl. Serologic studies for hepatitis A, B, and C and autoimmune disorders are negative. The urinalysis shows 1+ protein. An abdominal ultrasound shows hepatomegaly with patent hepatic vasculature and normal portal flow direction on Doppler..",Which ONE of the following is the MOST likely diagnosis?,"A. Budd–Chiari syndrome B. Syndrome of hemolysis, elevated liver enzymes, and low platelets C. Thrombotic thrombocytopenic purpura D. Acute fatty liver of pregnancy (AFLP)",D,temp nan,Pregnancy and Kidney Disease 2503,"A 27-year-old woman, para 1, undergoes emergent cesarean section at 32 weeks for fetal distress and bradycardia. She was being followed weekly for poor fetal growth. The surgery is complicated by hemorrhage, and significant pooling of blood is noted in the pelvis. The BP rises from 119/69 to 150/97 mmHg. The pulmonary and cardiac examinations are normal. Admission lab tests: Creatinine 0.4 mg/dl, AST 13 U/L, ALT 12 U/L, Hemoglobin 12.2 g/dl, Platelet count 215,000 per ml, Prothrombin time 12.2/1.0 s/internationalnormalized ratio. Postoperative lab tests were: Creatinine 1.7 mg/dl, AST 1554 U/L, ALT 486 U/L, Hemoglobin 8.3 g/dl, Platelet count 32,000 per ml, Prothrombin time 13.3/1.3 s/international normalized ratio. The urine output is 50 ml/h. The peripheral blood smear reveals one to two schistocytes per high power field (< 1%) as well as decreased and giant platelets. The urinalysis shows trace protein, 1+ blood, and hyaline casts..",Which ONE of the following is the MOST appropriate management?,"A. Transfuse platelets and packed red blood cells B. Provide plasma Exchange C. Eculizumab D. Dexamethasone E. Recombinant factor VIIa",A,temp nan,Pregnancy and Kidney Disease 2504,"A 26-year-old woman is evaluated 2 months after recovering from AFLP. She is interested in pursuing another pregnancy, but is concerned about her risk of recurrent AFLP..",Detection of an abnormality in which ONE of the following is MOST likely to predict her risk for recurrent AFLP?,"A. A disintegrin and metalloproteinase with a thrombospondin type 1 motif, member 13 activity B. Long–chain 3-hydroxyl CoA dehydrogenase C. Factor H activity D. Multidrug resistance P-glycoprotein 3 E. Palmitoyltransferase 1 deficiency",B,temp nan,Pregnancy and Kidney Disease 2505,"A 27-year-old woman, gravida 1, para 0, develops new-onset hypertension at 35-weeks gestation. She is asymptomatic. On physical examination, she is not in acute distress. The BP is 146/90 mmHg. There is trace pretibial edema. The urinalysis shows 2+ protein. Laboratory studies show serum creatinine of 0.8 mg/dl, AST of 12 U/L (reference range 0–35), ALT of 25 U/L (reference range 0–35), leukocyte count of 7,400/µl, hemoglobin of 9.8 g/dl, and platelet count of 138,000/ml. Fetal testing is reassuring..",Which ONE of the following is the MOST appropriate management?,"A. Kidney biopsy B. Immediate delivery (within 24 hours) C. Observation with expectant medical management D. Intravenous magnesium",C,temp nan,Pregnancy and Kidney Disease 2506,,"Question 2506: Which ONE of the following physiologic changes occurs by the end of the third trimester of pregnancy?","A. GFR is decreased by 10% compared to pre-pregnancy levels B. GFR returns to pre-pregnancy levels C. Renal blood flow is increased by 50% compared to pre-pregnancy levels D. Renal blood flow returns to pre-pregnancy levels",D,temp nan,Pregnancy and Kidney Disease 2507,"A 23-year-old woman, para 1, is evaluated at 27-weeks gestation for new-onset hypertension confirmed on repeat testing. She reports a new headache and visual scotomata for the past 2 days. A physical examination shows a BP of 154/94 mmHg that persists on repeat testing. The remainder of the examination is normal. The complete blood count, renal function, liver chemistries, and urinalysis are normal..",Which ONE of the following is the MOST likely diagnosis?,"A. Gestational hypertension B. Migraine headache C. Central venous thrombosis D. Preeclampsia",D,temp nan,Pregnancy and Kidney Disease 2508,"A 26-year-old woman is evaluated for AKI 1 day after successful induction of labor and delivery for preeclampsia. The serum creatinine level has risen from 1.1 to 1.3 mg/dl over the past 24 hours. The urine output has averaged 50 ml/h. She complains of weakness and nausea. Medications include labetalol and intravenous magnesium. On physical examination, she is lethargic and unable to sit up or stand because of weakness. Her weight is 70 kg. The BP is 110/60 mmHg. The pulse rate is 56/min. The cardiac and pulmonary examinations are normal. There is 1+ peripheral edema. The neurologic examination is significant for loss of deep tendon reflexes and generalized weakness graded at 3/5. Laboratory studies show sodium of 138 mEq/L, potassium of 5.6 mEq/L, chloride of 105 mEq/L, total CO2 of 19 mmol/L, BUN of 56 mg/dl, creatinine of 1.3 mg/dl, glucose of 92 mg/dl, calcium of 8.6 mg/dl, magnesium of 10.1 mg/dl, and phosphorus of 5.6 mg/dl. The urinalysis shows 1+ protein and occasional fine granular casts. An electrocardiogram shows sinus bradycardia at 54 beats per minute with a P to R interval of 0.24 seconds, increased QRS duration of 0.13 seconds, and an increased Q to T interval of 0.52 seconds..","In addition to holding intravenous magnesium and labetalol and administering intravenous calcium, which ONE of the following is the MOST appropriate next step in management?","A. Intravenous normal saline plus furosemide B. Urgent hemodialysis C. No additional interventions D. Intravenous insulin (1-U/kg bolus and 1–10 U/kg per hour as a continuous infusion)",A,temp nan,Pregnancy and Kidney Disease 2509,"A 24-year-old woman with ESRD caused by diabetic kidney disease is found to be pregnant at 6-weeks gestation. She has been on hemodialysis for 5 years and is currently maintained on hemodialysis thrice weekly for 4 hours via a right radiocephalic arteriovenous fistula. Her equilibrated Kt/V is 1.3. Her monthly laboratory studies have been maintained in the desired range. Her BP is 130/80 mmHg, and the rest of the examination is unremarkable..",Which ONE of the following dialysis strategies is MOST appropriate for this woman?,"A. Continue current dialysis regimen B. Hemodialysis totaling 16 hours/week C. Hemodialysis totaling 20 hours/week D. Hemodialysis totaling greater than or equal to 36 hours/week",D,temp nan,Pregnancy and Kidney Disease 2510,"A 32-year-old woman, gravida 2, para 1, with a 31-week singleton gestation presents for a routine prenatal visit. She has no chronic medical problems, and her first pregnancy was uncomplicated, with a spontaneous vaginal delivery at 39-weeks gestation. Thus far, her BP readings have been normal during pregnancy. She reports a new headache starting 2 days ago. The BP is 135/82 mmHg, increased from 118/65 mmHg at her last visit. Urinalysis shows 1+ protein, which is new compared with prior urinalyses..",Which ONE of the following features is most useful for excluding the diagnosis of preeclampsia within the next week?,"A. The absence of edema B. The absence of preeclampsia in her first pregnancy C. A low soluble fms-like tyrosine kinase-1 to placental growth factor ratio D. A normal serum uric acid",C,temp nan,Pregnancy and Kidney Disease 2511,"A 35-year-old woman with chronic hypertension and type 2 diabetes mellitus is evaluated at 3 weeks postpartum. She did not require treatment for hypertension during pregnancy. However, her BP is now rising to pre-pregnancy levels. A physical examination shows a BP of 150/95 mmHg. There is trace pretibial edema. The remainder of the examination is normal. Laboratory studies show normal serum electrolytes, a serum creatinine level of 0.8 mg/dl, and a urine albumin-to-creatinine ratio of 800 mg/g. She is breastfeeding..",Which ONE of the following is the MOST appropriate treatment?,"A. Hydrochlorothiazide B. Atenolol C. Enalapril D. Metoprolol",C,temp nan,Pregnancy and Kidney Disease 2512,"A 23-year-old woman with ESRD secondary to reflux nephropathy who received a living donor kidney transplant 3 years ago is interested in becoming pregnant. Current medications are tacrolimus, mycophenolate mofetil, and amlodipine. Laboratory studies show a serum creatinine level of 0.8 mg/dl. The urine albumin-to-creatinine ratio is 15 mg/g..",Which ONE of the following would you recommend?,"A. She should have a cesarean section to avoid trauma to the graft during labor B. She should start aspirin at 81 mg daily immediately after pregnancy is diagnosed C. Switching mycophenolate mofetil to azathioprine should be delayed until immediately after pregnancy is diagnosed D. She should start prednisone after delivery to prevent transplant rejection",B,temp nan,Pregnancy and Kidney Disease 2513,A 38-year-old woman with autosomal dominant polycystic kidney disease with a known mutation of PKD1 wishes to discuss possible pregnancy outcomes associated with preimplantation genetic testing (PGT). Her only medication is ramipril. Her BP is 120/70 mmHg. The rest of the physical examination is normal. Laboratory studies show that serum creatinine is 1.3 mg/dl. The albumin to-creatinine ratio is 50 mg/g..,Which ONE of the following would you advise regarding the effect of PGT on her pregnancy outcomes?,"A. The probability of achieving pregnancy is reduced compared with spontaneous conception B. PGT is associated with an increased risk of fetal anomalies compared with routine assisted reproductive technology C. PGT is associated with an increased risk of fetal growth restriction D. PGT is associated with an increased risk of impaired cognitive development",A,temp nan,Pregnancy and Kidney Disease 2514,A 34-year-old woman who is a deceased donor kidney transplant recipient informs you that she is pregnant. An ultrasound examination confirms viability of a healthy fetus at 16-weeks gestation. Her immunosuppression includes mycophenolate mofetil and tacrolimus..,"In addition to repeat fetal imaging at 20–22 weeks, which ONE of the following is the MOST appropriate management?","A. No change in management B. Stop mycophenolate mofetil and start azathioprine C. Stop mycophenolate mofeil and start sirolimus D. Stop mycophenolate mofetil and continue tacrolimus monotherapy E. Immediate termination",B,temp nan,Pregnancy and Kidney Disease 2515,"A 23-year-old woman who received a deceased donor kidney transplant 3 years ago is evaluated after a urine pregnancy test returned positive. She is at approximately 6-weeks gestation. She has a history of ESRD caused by lupus nephritis. Her immunosuppression was changed to tacrolimus, azathioprine, and prednisone 3 months ago after preconception counseling and planning. Her BP is 130/80 mmHg. The cardiac and pulmonary examinations are normal. The allograft is nontender and does not have a bruit. There is no peripheral edema. Laboratory studies show a stable serum creatinine level of 1.2 mg/dl and a glucose level of 96 mg/dl. A urinalysis is normal..",Which ONE of the following additional investigations should be requested for this woman now (at 6-weeks gestation)?,"A. Fetal cardiac monitoring B. Echocardiography C. Donor–specific antibody titers D. Anti-Ro and anti-La antibodies",D,temp nan,Pregnancy and Kidney Disease 2516,"A 38-year-old woman with type 2 diabetes mellitus, proteinuric CKD, and hypertension is evaluated at 36-weeks gestation for a 1-week history of increasing leg edema, dyspnea, and weight gain. On physical examination, she is in modest respiratory distress at rest, with a respiratory rate of 28/min. The oxygen saturation on ambient air is 86%, and it improves to 92% on 3 L/min oxygen via nasal cannula. The BP is 146/80 mmHg. The pulse is 120/min. The jugular venous pressure is elevated at 10 cm H2O. The lungs show bibasilar crackles. The abdomen is gravid. There is 2+ pitting pretibial edema. Laboratory studies show sodium of 132 mEq/L, potassium of 4.2 mEq/L, chloride of 105 mEq/L, total CO2 of 26 mmol/L, BUN of 15 mg/dl, creatinine of 1.1 mg/dl, glucose of 96 mg/dl, and albumin of 2.6 g/dl. The urine protein-to creatinine ratio is 3800 mg/g. An echocardiogram shows a global reduction in left ventricular systolic function with an left ventricular ejection fraction of 35%..","In addition to admission for fetal monitoring and urgent delivery, which ONE of the following is the MOST appropriate management?","A. Begin intravenous insulin B. Sodium restriction and compression stockings C. Furosemide D. Labetalol plus intravenous nitrates",C,temp nan,Pregnancy and Kidney Disease 2517,"A 27-year-old kidney transplant recipient is evaluated at 33-weeks gestation for an asymptomatic rise in the serum creatinine. She developed ESRD because of reflux nephropathy, received a deceased donor kidney transplant 3 years ago, and has enjoyed excellent allograft function without acute rejection. The serum creatinine was noted to rise from 0.9 mg/dl 2 weeks ago to the current level of 1.3 mg/dl on surveillance studies. Medications include tacrolimus, prednisone, labetalol, aspirin, and a prenatal vitamin. The BP is 120/77 mmHg. She does not have edema. Laboratory studies show a normal complete blood count, a serum creatinine of 1.3 mg/dl, and a tacrolimus level of 10 ng/ml (confirmed on repeat testing) that increased from 5 ng/ml over the past 2 months. A urinalysis shows 1+ protein, negative blood, and two to three leukocytes per high power field. The urine protein-to-creatinine ratio is 305 mg/g. An ultrasound of the renal allograft shows mild hydronephrosis..",Which ONE of the following is the MOST appropriate next step in management?,"A. Nephrostomy tube placement B. Reduce the tacrolimus dose C. Pulse intravenous methylprednisolone D. Dexamethasone followed by induction of labor and delivery",B,temp nan,Pregnancy and Kidney Disease 2518,"A 29-year-old woman, gravida 1, para 0, with known Fabry disease is now 26-weeks pregnant. Before pregnancy, she was asymptomatic, except for occasional acroparesthesias. She has not received treatment with enzyme replacement therapy with α-galactosidase-β. Over the past 2 weeks, however, she has noted onset of worsening acroparesthesias. On physical examination, the BP is 108/75 mmHg. The neurologic examination shows diminished light–touch sensation in the legs. The remainder of the examination is normal. Laboratory studies show a serum creatinine of 0.9 mg/dl. The urine albumin-to-creatinine ratio is 360 mg/g, increased from 15 mg/g before pregnancy..",Which ONE of the following is the MOST appropriate management?,"A. No additional management B. Enzyme replacement therapy C. Gabapentin D. Diltiazem",B,temp nan,Pregnancy and Kidney Disease 2519,"A 36-year-old woman is evaluated for microscopic hematuria. She is now in the early first trimester. Her BP, physical examination, kidney function, and urine protein excretion are normal. A urinalysis shows trace blood, negative protein, and 3 to 5 dysmorphic erythrocytes per high power field. A urine culture is negative. A kidney ultrasound is normal. She is concerned about the potential impact on her pregnancy..","In the absence of a change in her clinical findings during pregnancy, which of the following would you advise regarding the impact of the hematuria on her pregnancy?","A. It increases her risk for gestational hypertension B. It increases the risk of preeclampsia three-fold C. It places her at increased risk for preterm labor D. It does not increase the risk for a small gestation baby",D,temp nan,Pregnancy and Kidney Disease 2520,"A 32-year-old woman with ESRD on intensive hemodialysis is admitted at 37-weeks gestation with increased BP (160/98 mmHg) and headache. Fetal growth has been poor, and umbilical artery Doppler studies show absent end diastolic flow. She is admitted for urgent delivery for severe preeclampsia. The obstetrician wishes to commence intravenous magnesium..",Which ONE of the following is the MOST appropriate dosing regimen for intravenous magnesium for this patient?,"A. A reduction in the loading dose only B. A reduction in the maintenance dose only C. A reduction in the loading dose and the maintenance dose D. No change in the loading or maintenance dose",C,temp nan,Pregnancy and Kidney Disease 2521,"A 29-year-old woman, gravida 3, para 2, is referred for evaluation of new-onset proteinuria and hematuria. She is currently at 32-weeks gestation. Her first two pregnancies were term spontaneous vaginal deliveries. She has no prior medical history. Thus far, her pregnancy has been uneventful. She is on no medications except prenatal vitamins. She complains of bilateral stiffness in her hands but otherwise, feels well. On physical examination, the BP is 125/64 mmHg, and the cardiovascular examination is normal. There is no rash or synovitis. There is 1+ bilateral lower extremity edema. The urinalysis shows 2+ protein and 3+ blood. The urine sediment reveals 10–20 dysmorphic erythrocytes per high-power field with no casts. A 24-hour urine collection shows 2.4 g protein excretion. Serum electrolytes are normal. The serum creatinine is 1.1 mg/dl (increased from 0.6 mg/dl at her first prenatal visit), the hemoglobin is 10.4 g/dl, and the platelet count is 146,000/µl. Serologic testing reveals a positive antinuclear antibody with a titer of 1:320, positive anti–dsDNA antibodies, and low serum C3 and C4. Serologic studies for HIV, hepatitis B virus, and hepatitis C virus are negative. A kidney ultrasound reveals bilateral hydronephrosis..",Which ONE of the following is the MOST appropriate initial management strategy?,"A. Referral to urology for bilateral ureteral stent placement B. Immediate admission, betamethasone for fetal maturation, and induction of labor C. Empiric therapy with prednisone and azathioprine, followed by adjusted treatment after delivery based on kidne biopsy performed postpartum D. Supportive management and induction of delivery at term followed by kidney biopsy and treatment after delivery",C,temp nan,Pregnancy and Kidney Disease 2522,"A 32-year-old woman with biopsy-proven FSGS is seen for preconception counseling. Her BP is well controlled (128/72 mmHg) on candesartan and amlodipine. She has 2.8 g urine protein on 24-hour urine collection along with significant renal insufficiency, with a serum creatinine level of 2.6 mg/dl. She asks about her risks of pregnancy related complications..",Which ONE of the following statements is CORRECT regarding her risk of CKD progression and preterm delivery?,"A. Her risk of progression to ESRD during or shortly postpartum is about 20% B. Her risk of progression to ESRD during or shortly postpartum is about 50% C. Her risk of preterm delivery (< 37-weeks gestation) is about 20% D. Her risk of extreme preterm delivery (< 34-weeks gestation) is about 20%",A,temp nan,Pregnancy and Kidney Disease 2523,"A 24-year-old Asian man who is a graduate student is discovered to have asymptomatic microscopic hematuria during a routine examination for a sports-related injury. The BP is 118/72 mmHg. His body mass index (BMI) is 33 kg/m2. The remainder of the history and physical examination is unremarkable. The serum creatinine level is 0.78 mg/dl. The urine sediment reveals 10–15 erythrocytes per high power field, of which 50% are dysmorphic. A spot urine albumin-to-creatinine ratio is 39 mg/g. The fasting blood glucose is 110 mg/dl..",Which ONE of the following lesions is MOST likely to be found on renal biopsy?,"A. FSGS B. Thin basement membrane nephropathy C. Minimal change disease D. IgA nephropathy E. Early diabetic glomerulosclerosis",D,temp nan,Primary & Secondary Glomerular Diseases 2524,An 82-year-old man is referred for evaluation of increased albuminuria detected on a recent spot urine albumin-to-creatinine ratio (33 mg/g) on routine follow-up for diet–controlled type 2 diabetes. He does not smoke. He takes no medication other than an occasional nonsteroidal anti–inflammatory drug for osteoarthritis. His BP is 130/65 mmHg. The BMI is 20 kg/m2. His physical examination is unremarkable. The serum creatinine is 0.95 mg/dl (eGFR 74 ml/min per 1.73 m2). A complete blood count is normal. The hemoglobin A1c is 5.7%. The urine dipstick shows no protein or blood. The addition of sulfosalicylic acid (eight drops of 20% sulfosalicylic acid to 10 ml urine) yields no turbidity..,Which ONE of the following is the next MOST appropriate step in this patient’s management?,"A. Start low-dose ramipril B. Check the urine protein-to-creatinine ratio C. Measure or estimate 24-hour creatinine excretion rate D. Reduce protein intake to 0.8 g/kg E. Start metformin therapy",C,temp nan,Primary & Secondary Glomerular Diseases 2525,"A 12-year-old boy has experienced multiple relapses of steroid–sensitive nephrotic syndrome since age 6 years. At age 10 years, he underwent a kidney biopsy during a relapse that revealed minimal-change disease. Despite attempts to reduce the frequency of relapses with a short course of oral cyclophosphamide (8 weeks) and subsequently, a 6-month course of levamisole, he continues to relapse two to three times per year. Each relapse is initially responsive to corticosteroids, but the proteinuria worsens when the steroid dose is tapered to ,0.4 mg/kg per day. His therapy has been complicated by mood swings, abdominal striae, growth retardation, and osteoporosis. He recently had a florid relapse manifest with .15 g/d of protein excretion. His serum albumin level is 1.2 g/dl, and the serum creatinine level is 1.23 mg/dl (eGFR 40 ml/min per 1.73 m2)..",Which ONE of the following is the next MOST appropriate step in this patient’s management?,"A. Repeat kidney biopsy B. Mycophenolate mofetil plus steroids C. Tacrolimus plus steroids D. Retreat with oral cyclophosphamide and steroids E. Rituximab",E,temp nan,Primary & Secondary Glomerular Diseases 2526,"A 77-year-old woman is referred to you for evaluation of steroid–resistant minimal-change disease and persisting nephrotic syndrome. She had gradual onset of worsening edema about 8 months ago and was found to have a 24-hour urine protein excretion of 18 g/d and serum albumin of 2.1 g/dl. The serum creatinine is 1.21 mg/dl (eGFR 43 ml/min per 1.73 m2). A kidney biopsy showed minimal changes by light microscopy (15 glomeruli, four of which were globally sclerotic). Immunofluorescence microscopy showed no staining for IgG and C3. Electron microscopy was not performed (no glomeruli were available for analysis). After 6 weeks of oral corticosteroids, she continues to show evidence of severe nephrotic syndrome..",Which ONE of the following would you do NEXT?,"A. Repeat kidney biopsy with electron microscopy B. Perform laser dissection/mass spectrometry on prior kidney biopsy C. Stain prior kidney biopsy for B7.1 deposits D. Stain prior kidney biopsy with Congo Red E. Stain prior kidney biopsy for phospholipase A2 receptor (PLA2R) antigen",D,temp nan,Primary & Secondary Glomerular Diseases 2527,"A 48-year-old black woman with nephrotic syndrome (serum albumin 3.1 g/dl and urine protein excretion 5.2 g/d) is evaluated in consultation. The serum creatinine is 1.91 mg/dl (eGFR 47 ml/min per 1.73 m2). There is no family history of kidney disease. A kidney biopsy shows a lesion of FSGS not otherwise specified involving eight of 22 glomeruli in the specimen. Immunofluorescence microscopy shows segmental IgM and C3 deposition, and electron microscopy shows > 80% foot process effacement. No microcytic changes or tubuloreticular structures are present. An HIV test is negative..","Before starting therapy, which ONE of the following is the MOST appropriate next step in this patient’s diagnostic evaluation at this time?","A. Serum soluble urokinase–type plasminogen activator receptor level B. Genetic testing for apolipoprotein A1 (APOL1) high–risk alleles C. Genetic testing for an NPHS2 gene mutation D. Genetic testing for polymorphisms of the angiotensin–converting enzyme gene E. No additional diagnostic studies are required",E,temp nan,Primary & Secondary Glomerular Diseases 2528,"A 15-year-old Hispanic boy is discovered to have proteinuria of 3.6 g/d and a serum creatinine of 1.45 mg/dl (eGFR 70 ml/min per 1.73 m2). The serum albumin is 4.1 g/dl. The urine sediment is unremarkable. His BMI is 31 kg/m2. His BP is 139/88 mmHg. A kidney biopsy reveals 19 glomeruli, three of which are globally sclerotic. No glomerular hypertrophy is observed. Electron microscopy reveals only minor podocyte foot process effacement. Immunofluorescence studies are negative..",Which ONE of the following is the MOST appropriate treatment for this patient?,"A. Oral angiotensin receptor blockers (ARBs) B. Oral high–dose steroids C. Mycophenolatemofetil plus low-dose steroids D. Corticotropin injection gel E. Cyclosporine plus low-dose steroids",A,temp nan,Primary & Secondary Glomerular Diseases 2529,,"Question 2529: In patients with IgA nephropathy with eGFRs.60 ml/min per 1.73 m2 and persistent proteinuria (0.75 g/d) after a 3- to 6-month trial of angiotensin blockade, which ONE of the following has been consistently shown to reduce protein excretion over the short term (e.g., 2 years) in randomized clinical trials?","A. Oral glucocorticoids B. Mycophenolate mofetil C. Omega-3 fatty acids D. Cyclophosphamide E. Azathioprine",A,temp nan,Primary & Secondary Glomerular Diseases 2530,"A 30-year-old woman with renal biopsy–proven lesion of presumed primary FSGS not otherwise specified is seen in follow-up. At the time of diagnosis, the serum albumin was 2.1 g/dl, and the urine protein excretion was 18 g/d. She was initially treated with a 6-month course of high–dose oral corticosteroids (prednisone at 1 mg/kg per day) and renin-angiotensin system inhibition. The urine protein excretion is now 12 g/d, and the serum albumin is 2.9 g/dl. The serum creatinine is 1.21 mg/dl (eGFR 61 ml/min per 1.73 m2). She has gained 12 kg in weight..",Which ONE of the following is the MOST appropriate management?,"A. Continue steroids at reduced dosage (0.5 mg/kg per day) for another 3 months B. Oral cyclosporine and low-dose steroids for greater than or equal to 4 months C. Rituximab and tapering doses of steroids D. Corticotropin injection gel and tapering of oral steroids E. Mycophenolate mofetil and tapering dosage of steroids",B,temp nan,Primary & Secondary Glomerular Diseases 2531,A 56-year-old man is evaluated after recently undergoing kidney biopsy to evaluate the nephrotic syndrome. Light microscopy shows a segmental glomerulosclerotic lesion in at least one glomerulus. He previously was in excellent health and has taken no medications or herbal preparations apart from rare use of acetaminophen. He does not have evidence of an underlying neoplasm or underlying viral infection..,"Which ONE of the following BEST describes the criteria for establishing a presumptive diagnosis of primary FSGS in a patient without evidence of an adverse drug reaction, cancer, or chronic viral infection?","A. A segmental glomerulosclerotic lesion in at least one glomerulus in a biopsy specimen examined by light microscopy B. As in A plus focal and segmental IgM de-position by immunofluorescence microscopy C. As in A plus widespread effacement of the podocyte foot processes by electron microscopy D. As in A plus elevated serum levels of soluble urokinase–type plasminogen activator receptor by immunoassay E. As in A plus an elevated plasma level of a permeability-inducing factor by bioassay",C,temp nan,Primary & Secondary Glomerular Diseases 2532,A 68-year-old man develops nephrotic syndrome and is found to have a lesion of membranous nephropathy on kidney biopsy..,Which ONE of the following findings would indicate that a primary form of the disorder is MOST likely present?,"A. Extensive deposition of C3 in the capillary loops by immunofluorescence microscopy B. Coalescent electron–dense deposits in the subepithelial space by electron microscopy C. Hyperexpression of PLA2R antigen by immunohistochemical studies of pronase digested, paraffin–preserved specimens D. IgG subclass 2 dominance of the IgG deposits by immunofluorescence microscopy with negative staining for PLA2R E. Increased infiltration of capillary loops by polymorphonuclear leukocytes by light microscopy",C,temp nan,Primary & Secondary Glomerular Diseases 2533,"A 46-year-old woman develops nephrotic syn-drome and is found to have a stage 1 membranous nephropathy lesion on kidney biopsy. The BP is 142/72 mmHg. Moderate pedal edema is present. The serum creatinine is 1.1 mg/dl (eGFR 79 ml/min per 1.73 m2), and the serum cholesterol is 320 mg/dl. The serum albumin is 3.2 g/dl, and the urine protein excretion is 4.8 g/d. The anti PLA2R antibody is positive at low titer by ELISA..","In addition to treatment with an angiotensin–converting enzyme inhibitor (ACEI) and ARB, a statin agent, and a low-sodium diet, which ONE of the following regimens would you prescribe NEXT?","A. Intravenous rituximab at 1000 mg for two doses 2 weeks apart B. Cyclical oral cyclophosphamide and steroids for 6 months C. Oral mycophenolate mofetil monotherapy for 6 months D. Oral tacrolimus monotherapy for 6 months E. Continued supportive care and observation",E,temp nan,Primary & Secondary Glomerular Diseases 2534,"A 30-year-old woman is referred for assessment and management of proteinuria (3 g/d) discovered during an evaluation for edema. She is otherwise asymptomatic. Review of prior laboratory studies shows a normal serum creatinine, normal serum complement levels, and negative serologies for antinuclear antibodies, HIV, hepatitis B virus, and hepatitis C virus. A kidney biopsy is performed and shows a lesion of mild membranous nephropathy by light microscopy and special stains. By electron microscopy, stage 1 subepithelial electron–dense deposits are seen. A few mesangial electron–dense deposits are also observed. Immunofluorescence microscopy shows very strong and intense granular C3 deposits along the capillary walls. Staining for Igs and kappa- and lambda -light chains is negative. Serum anti–PLA2R antibodies are negative..",Which ONE of the following should be done NEXT to identify the cause of the lesion?,"A. Pronase digestion of paraffin-embedded material and stain for PLA2R antigen B. Pronase digestion of paraffin-embedded material and staining for monoclonal IgG (light and heavy chains) C. Staining of biopsy specimen for C1q and C4d D. Staining of the biopsy with Congo Red or thioflavine T",B,temp nan,Primary & Secondary Glomerular Diseases 2535,"A 48-year-old man with nephrotic syndrome is found to have a lesion of membranous nephropathy on kidney biopsy. He is a nonsmoker. Because of a chronic cough and mild dyspnea, a computed tomography scan of the chest is per-formed that reveals bilateral hilar adenopathy and scattered interstitial lesions at the base of both lungs. No pleural effusion or thickening is observed. Laboratory studies show serum creatinine of 1.3 mg/dl, serum albumin of 2.8 g/dl, and a urine protein excretion of 10.5 g/d. The serum PLA2R antibody is positive (by ELISA), and a special stain of the kidney biopsy specimen reveals hyperexpression of PLA2R antigen in the capillary loops..",Which ONE of the following is the MOST likely diagnosis of his renal disease?,"A. Sarcoidosis causing secondary membranous nephropathy B. Lung cancer causing secondary membranous nephropathy C. Hodgkin disease causing secondary membranous nephropathy D. SLE causing secondary membranous nephropathy E. Primary membranous nephropathy",A,temp nan,Primary & Secondary Glomerular Diseases 2536,,"Question 2536: Which ONE of the following is preferred initial therapy for a 30-year-old man with IgA nephropathy, 1.2 g proteinuria per day, eGFR 80 ml/min per 1.73 m2 and BPof 128/75 mmHg?","A. Dietary salt and protein restriction only B. Oral ACEIs or ARBs C. Dihydropyridine calcium channel blockers D. Oral steroids plus ACEI or ARB E. Oral calcitriol",B,temp nan,Primary & Secondary Glomerular Diseases 2537,"A 59-year-old man is found to have nephrotic syndrome (serum albumin of 1.9 g/dl and urine protein excretion of 8.2 g/d). His serum creatinine is 1.48 mg/dl (eGFR 51 ml/min per 1.73 m2). A serum protein electrophoresis reveals a small M spike of around 0.2 g/dl, but a free light–chain assay is normal. A renal biopsy reveals amyloidosis by light (Congo Red stain positive) and electron microscopy. Immunofluorescence microscopy shows no staining for antisera against monoclonal light or heavy chains. Stains for serum amyloid A are negative..",Which ONE of the following is the MOST appropriate next step in this patient’s diagnostic evaluation?,"A. Stain the specimen for serum amyloid P component B. Perform serum immunofixation assay C. Perform a urinary electrophoresis D. Perform a laser dissection/mass spectrometry analysis of the biopsy specimen E. Perform Ig isotype and subclass stains after pronase digestion of the paraffin-embedded specimen",D,temp nan,Primary & Secondary Glomerular Diseases 2538,"A 63-year-old woman is found to have persistent hematuria, marked proteinuria (5.1 g/d), and impaired kidney function: serum creatinine of 2.2 mg/dl and eGFR 23 ml/min per 1.73 m2. The serum C3 level is 38 mg/dl (reference range 55–120 mg/dl). A serum protein electrophoresis is unremarkable, except for a decrease in albumin and an increase in beta globulins. A kidney biopsy reveals dense deposit disease..",Which ONE of the following is the next BEST serologic study to help define the etiology of her disease?,"A. Antistreptolysin O titers B. Soluble C5b-9 levels C. C3 nephritic factor D. Factor H levels E. Serum immunofixation and free light–chain assays",ANSWER FOR ID: 2538 NOT FOUND,temp nan,SUBJECT FOR ID: 2538 NOT FOUND 2539,,"Question 2539: In a patient presenting with membranoproliferative GN who has no serologic evidence of hepatitis B or C infection and whose biopsy shows predominantly IgM on immunofluorescence, which ONE of the following diagnoses should be seriously considered?","A. Occult hepatitis B B. An underlying autoimmune disorder C. Solid organ cancer D. Cocaine abuse",B,temp nan,Primary & Secondary Glomerular Diseases 2540,"A 40-year-old Asian man is found to have 4 g/d proteinuria and mild glomerular hematuria during an evaluation for new–onset leg edema. The serum creatinine concentration was 1.2 mg/dl. Complement levels were normal, and antinuclear antibodies were negative. The patient was hepatitis B surface antibody positive and hepatitis B surface antigen negative. A kidney biopsy was performed. The pathology was interpreted as membranous nephropathy with atypical features, with staining for both IgG and IgA on immuno-fluorescence microscopy. Electron microscopy showed subepithelial and mesangial deposits..",Which ONE of the following tests would be of the MOST help to confirm this patient’s diagnosis?,"A. Antibody levels to double-stranded DNA B. Staining the kidney biopsy for hepatitis B surface antigen and core antigen C. Measurement of serum anti–PLA2R antibodies D. Stool for occult blood",B,temp nan,Primary & Secondary Glomerular Diseases 2541,"A 46-year-old man with known hepatitis C virus infection was found to have proteinuria, hematuria, and lower extremity necrotic skin lesions. Circulating cryoglobulins were present. Kidney biopsy showed cryoglobulinemic membranoproliferative GN. The patient’s serum creatinine concentration was 1.5 mg/dl, and a 24-hour urine collection showed 4 g proteinuria. Viral genotyping suggested a genotype that may be less susceptible to IFN..",Which ONE of the following is the BEST course of treatment for this patient?,"A. Rituximab plus plasma exchange B. Rituximab plus plasma exchange plus cyclo-phosphamide C. Sofosbuvir plus simeprevir D. Sofosbuvir plus simeprevir plus rituximab",D,temp nan,Primary & Secondary Glomerular Diseases 2542,"A 29-year-old black patient with long-standing HIV secondary to past intravenous drug use injured his leg and was admitted to the hospital with an active cellulitis in the area of the injury. Staphylococcus was cultured from the leg. He had been on combined antiretroviral therapy for years, and his HIV viral load has been undetectable. He had never had kidney problems but developed renal insufficiency and nephrotic-range proteinuria with acanthocytes and red blood cell casts in his urine sediment..",Which ONE of the following is the MOST likely diagnosis?,"A. Collapsing FSGS B. Postinfectious GN C. IgA–dominant, infection–related GN D. Hepatitis C–associated membranous nephropathy",C,temp nan,Primary & Secondary Glomerular Diseases 2543,"A 24-year-old man with microscopic hematuria is found to have thin basement membrane disease on kidney biopsy. No other glomerular basement membrane abnormalities were noted on electron microscopy. His eGFR and BP are normal. A 24-hour urine collection showed 185 mg protein. His general health has been excellent, and he does not have evidence of hearing impairment. He recalls that, as a child, he was told that he could not play on the school football team, because he had some blood in his urine, but no additional evaluation was pursued at that time. He had no family history of kidney disease, but his father thought a doctor once told him that he had blood in his urine..",Which ONE of the following would be the BEST advice for this patient?,"A. Kidney prognosis is excellent, and there is no need for concern or additional follow-up B. Begin an ACEI C. Kidney prognosis is likely to be good long term, but uncertainty exists; therefore, yearly follow-up is advisable D. Undergo genetic testing to achieve a molecular diagnosis",C,temp nan,Primary & Secondary Glomerular Diseases 2544,"A 29-year-old man is referred for ankle swelling. During the initial evaluation, the patient is found to have proteinuria of 5 g/d. The serum creatinine concentration was 0.82 mg/dl, the serum albumin was slightly low at 3.4 g/dl, and total cholesterol was elevated at 336 mg/dl. A kidney biopsy showed FSGS with a well preserved tubulointerstitial compartment but extensive (> 90%) foot process effacement by electron microscopy. The patient said his father was on dialysis, and proteinuria was found in his sister during pregnancy, but it never resolved. A genetic cause for the patient’s FSGS was suspected. The patient had genotyping done and was found to have a pathogenic mutation in transient receptor potential channel 6. An ACEI was started, but proteinuria only decreased to 3.5 g/d after 3 months..",Which ONE of the following treatments would you choose now?,"A. A calcineurin inhibitor B. Prednisone at 1 mg/kg per day for 16 weeks C. Cyclophosphamide D. Mycophenolate mofetil",A,temp nan,Primary & Secondary Glomerular Diseases 2545,"An 84-year-old man is found to have proteinuria, hematuria, and renal insufficiency with a serum creatinine of 3.8 mg/dl. A kidney biopsy is performed, and it shows a crescentic necrotizing GN with no immune deposits. Serologic testing reveals the patient to be anti–myeloperoxidase-ANCA positive. He does not seem to have any extrarenal signs or symptoms of vasculitis..",Which ONE of the following choices for treatment is MOST correct for this patient?,"A. Treat only with corticosteroids B. Initiate therapy with intravenous pulse methylprednisolone at 1 g/d for 3 days followed by oral prednisone at 1 mg/kg per day and intravenous cyclophosphamide at 1 g/m2 six times or oral cyclophosphamide at 2 mg/kg per day C. Treat with cyclophosphamide and cortico-steroids as outlined in choice B but reduce the doses by greater than or equal to 20% D. Treat only with mycophenolate mofetil",C,temp nan,Primary & Secondary Glomerular Diseases 2546,"A 52-year-old woman is referred for management of pauciimmune necrotizing GN. She initially was found to have hematuria, proteinuria, a serum creatinine of 4.5 mg/dl, and an eGFR of 16 ml/min per 1.73 m2 during an evaluation for epistaxis, migratory arthralgias, mild hearing loss, and marked fatigue. She did not have hemoptysis. Serologic testing showed negative antinuclear antibodies, normal serum complement levels, negative serologies for hepatitis B and C viruses, and a positive anti–proteinase-3-ANCA. Kidney biopsy showed a pauciimmune, necrotizing GN. A chest radiograph was normal..","In addition to intravenous methylprednisolone (500–1000 mg/d three times) followed by oral prednisone beginning at 1 mg/kg per day, which ONE of the following approaches is the MOST appropriate induction therapy for this patient?","A. Methotrexate at 20–25 mg/wk B. Either intravenous cyclophosphamide or intravenous rituximab C. Mycophenolate mofetil at 1 g twice daily D. Plasmapheresis",B,temp nan,Primary & Secondary Glomerular Diseases 2547,"63-year-old man with proteinase-3-ANCA–associated vasculitis was treated with cyclophosphamide and corticosteroids and attained remis-sion after 3 months of therapy. The choice of maintenance therapy is between rituximab, azathioprine, mycophenolate, and 6 more months of cyclophosphamide (extended cyclophosphamide) before switching to azathioprine. These immunosuppressive agents have been ranked below in the order of preferred use..",Which ONE of the following choices lists of maintenance drugs is in the PREFERRED order of use?,"A. Rituximab > azathioprine > mycophenolate > extended cyclophosphamide B. Extended cyclophosphamide > mycophenolate > azathioprine C. Azathioprine = mycophenolate > extended cyclophosphamide D. Azathioprine > mycophenolate > extended cyclophosphamide",A,temp nan,Primary & Secondary Glomerular Diseases 2548,"A 29-year-old woman is seen in consultation in the intensive care unit after developing hemoptysis and increased respiratory distress. She has multiple sclerosis that is in remission after recent treatment with alemtuzumab. On physical examination, she is tachypneic with increased work of breathing. The temperature is 36.0C. The oxygen saturation on ambient air is 85%. She has crackles in both lower lung fields and trace leg edema. Laboratory studies show a serum creatinine concentration of 2 mg/dl. The serum lactate dehydrogenase and haptoglobin levels are normal. The leukocyte count is 11,000/ml, the hemoglobin is 9 g/dl, and the platelet count is 326,000/µl. The urinalysis shows 1+ protein, 2+ blood, and numerous acanthocytes, and erythrocyte casts. A chest radiograph shows a diffuse alveolar filling process. A bronchoscopy confirms the presence of diffuse pulmonary hemorrhage..",Which ONE of the following is the MOST appropriate treatment for this patient?,"A. Plasmapheresis, high-dose corticosteroids, and cyclophosphamide B. Cyclosporine A C. Rituximab D. Intravenous Ig",A,temp nan,Primary & Secondary Glomerular Diseases 2549,"An 80-year-old man is admitted for additional evaluation of a 2-week history of progressive fatigue and darkening of the urine. He does not have pulmonary symptoms or fever. On physical examination, he is afebrile. He has 1+ leg edema. The remainder of the examination is unremarkable. Laboratory studies show an initial serum creatinine concentration of 6.4 mg/dl that increased from 0.9 mg/dl 3 months ago. He is oliguric, with a 24- hour urine volume of only 200 ml. A kidney biopsy shows necrotizing crescentic GN involving 100% of 22 glomeruli in the specimen, with linear deposition of IgG along glomerular basement membranes. Serologic studies show dual positivity for antiglomerular basement mem-brane antibodies and myeloperoxidase-ANCAs. A chest radiograph was normal. He requires initiation of dialysis on the third hospital day for progressive pulmonary congestion and hyperkalemia..",Which ONE of the following is the MOST appropriate management for this patient?,"A. Provide supportive care and management for ESRD B. High-dose corticosteroids, plasmapheresis, and cyclophosphamide C. Intravenous rituximab at 1000 mg two times 2 weeks apart D. High-dose corticosteroids plus rituximab",A,temp nan,Primary & Secondary Glomerular Diseases 2550,"A 30-year-old white woman was treated for proliferative lupus nephritis (LN) with the Euro lupus protocol of corticosteroids, low-dose cyclophosphamide for 3 months, and maintenance with azathioprine. When treatment was started, her serum creatinine concentration was 1.2 mg/dl, and she had 2 g/d proteinuria. She had 15 red blood cells per high power field on urinalysis, and several of these were acanthocytes. Her complement levels were very low, and she had high–titer antidouble–stranded DNA antibodies. After a year of treatment, the proteinuria has fallen to 600 mg/d, the serum creatinine concentration is 0.7 mg/dl, and the urine sediment is unremarkable. She does not have any extrarenal SLE symptoms. Serum complement levels are now normal, and anti-DNA antibodies are not detected..",Which ONE of the following is the MOST likely prognosis for this patient?,"A. The long-term prognosis is favorable, be-cause the proteinuria declined to < 0.7 g/d after 1 year B. The long-term prognosis is guarded, be-cause her baseline eGFR was < 60 ml/min per m2 C. The long-term prognosis is guarded, be-cause baseline complement C3 and C4 levels were very low and antidouble–stranded DNA antibodies greatly elevated at the beginning of therapy D. The long-term prognosis is favorable, because the urine sediment became inactive with treatment",A,temp nan,Primary & Secondary Glomerular Diseases 2551,"A 40-year-old Hispanic woman who had been treated successfully for proliferative LN in the past with intravenous cyclophosphamide and corticosteroids was seen for a relapse of active LN. After 6 months of induction therapy, the patient received another year of maintenance cyclophosphamide given as quarterly intravenous pulses. The patient achieved complete renal remission 9 months after treatment began and remained in remission for 2 years after maintenance cyclophosphamide had been discontinued. In addition to treatment for LN, the patient had received a course of oral cyclophosphamide for central nervous system lupus several years before. As a result, the patient’s cumulative cyclophosphamide dose was 20 g. She now has low serum complement levels, an increase in serum creatinine from 1 to 1.3 mg/dl, and an increase in proteinuria from 350 mg/d to 2.5 g/d. You decide to treat the patient with oral prednisone and mycophenolate mofetil at 3 g/d. Mycophenolate mofetil is started at 500 mg twice a day, but she is unable to tolerate mycophenolate mofetil or mycophenolate sodium because of gastrointestinal symptoms..",Which ONE of the following would be the BEST way to treat this relapse?,"A. Add intravenous-pulse cyclophosphamide at a dose of 1 g/m2 monthly for 6 months to corticosteroids B. Add low–dose Euro lupus cyclophosphamide for 3 months to corticosteroids C. Add tacrolimus to corticosteroids D. Add abatacept to corticosteroids",B,temp nan,Primary & Secondary Glomerular Diseases 2552,"30-year-old black woman is evaluated 3 years after being diagnosed with severe LN (class 4 [G]). She received induction therapy with 6 months of intravenous cyclophosphamide (1 g/m2) and prednisone. She then was placed on maintenance mycophenolate mofetil at 2 g/d, low-dose prednisone (5 mg/d), and hydroxychloroquine. The patient achieved a complete renal remission after 1 year of overall therapy. You are considering tapering the intensity of her immunosuppression, because she has not had relapses and has normal kidney function and urinalyses. She has been enrolled in a clinical trial that assessed APOL1 risk allele status in addition to a number of novel biomarkers..",Which ONE of the following criteria would MOST favor tapering of this patient’s immunosuppression?,"A. The absence of high–risk APOL1 risk alleles B. A sustained remission for at least 24 months C. A high number of CD81 T cells in her urine D. The absence of C3b autoantibodies",B,temp nan,Primary & Secondary Glomerular Diseases 2553,"46-year-old man on hemodialysis three times per week is transferred to your unit. He feels relatively well. A review of his most recent laboratory testing shows calcium of 9.2 mg/dl, phosphorus of 6.1 mg/dl, alkaline phosphatase of 146 U/ml (reference range 36–92 U/ml), hemoglobin of 10.1 g/dL, and intact parathyroid hormone (PTH) of 324 pg/ml (reference range17–70 pg/ml). His current regimen includes sevelamer carbonate at 2400 mg three times daily with meals and paricalcitolat 2µg intravenously three times weekly with each dialysis..",Which ONE of the following is the MOST appropriate next step in management of this man's hyperphosphatemia?,"A. Increase sevelamer to 3200 mg three times daily B. Add calcium carbonate at 1.25 g at night C. Increase the dialysis treatment time by 30 minutes D. Increase paricalcitol to 3 µg intravenously three times weekly E. Conduct a detailed evaluation of his diet",E,temp nan,Disorders of Divalent Ions Renal Bone Disease & Nephrolithiasis 2554,,"Question 2554: Which ONE of the following is NOT associated with elevated serum alkaline phosphatase values?","A. Hip fracture B. Mortality C. Coronary calcification D. Osteitis fibrosa E. Adynamic bone",E,temp nan,Disorders of Divalent Ions Renal Bone Disease & Nephrolithiasis 2555,,"Question 2555: Which ONE of the following about parathyroidectomy for patients on dialysis is CORRECT?","A. Outpatient minimally invasive parathyroidectomy can achieve excellent control of secondary hyperparathyroidism B. All surgical approaches, such as partial, subtotal, or total parathyroidectomy with or without autotransplantation, are effective C. Hospital admission is uncommon after parathyroidectomy D. Patients with a history of parathyroidectomy have an increased risk for all-cause and cardiovascular mortality compared with patients with similar degrees of hyperparathyroidism",B,temp nan,Disorders of Divalent Ions Renal Bone Disease & Nephrolithiasis 2556,"A 65-year-old man with ESRD has been on hemodialysis for 2 months and transfers to your dialysis unit. Laboratory values are calcium of 10.2 mg/dl, phosphorus of 6.5 mg/dl, intact PTH of 645 pg/ml (reference range 17–70 pg/ml), and alkaline phosphatase of 212 U/ml (reference range 36–92 U/ml). He has been taking calcium acetate at 2001 mg three times daily with each meal..",Which ONE of the following is the MOST appropriate treatment?,"A. No additional intervention B. Begin cinacalcet therapy C. Increase calcium acetate to 2668 mg three times daily with meals D. Refer for parathyroidectomy E. Add calcitriol at 0.25 µg intravenously three times weekly",B,temp nan,Disorders of Divalent Ions Renal Bone Disease & Nephrolithiasis 2557,"A 74-year-old woman maintained on hemodialysis for the past 4 years is evaluated during dialysis rounds for complaints of frequent pain in the hips, legs, shoulders, and lower back. She has a history of severe peripheral vascular disease, coronary artery disease, renal artery stenosis, and chronic atrial fibrillation. Her regimen includes paricalcitol at 2 µg three times weekly with each dialysis and calcium acetate at 667 mg three times daily with meals. A review of her laboratory studies shows that the serum calcium has ranged from 9.2 to 9.5 mg/dl, that the serum phosphorus has ranged from 4.7 to 5.4 mg/dl, that the serum alkaline phosphatase has ranged from 310 to 349 IU/ml (reference range 36–92 U/ml), and that the intact PTH has ranged from 303 to 434 pg/ml (reference range 17–70 pg/ml) over that past 12 months. The γ-glutamyl transferase is normal, and the 25-hydroxy vitamin D level is 32 (reference range 30–50 ng/ml). She had a dual–energy x-ray absorptiometry (DEXA) scan showing a T score of 22.9 at the hip and 22.5 at the lumbar spine..",Which ONE of the following is the MOST appropriate management for this patient?,"A. Denosumab B. Increase calcium acetate C. Zoledronic acid D. Cinacalcet E. Increase paracalcitol",D,temp nan,Disorders of Divalent Ions Renal Bone Disease & Nephrolithiasis 2558,"A 68-year-old woman with ESRD caused by diabetes mellitus maintained on peritoneal dialysis for 4 years is evaluated in the peritoneal dialysis clinic. Two weeks ago, she sustained a stress fracture of the right femur. A right hip radiograph shows a linear cortical defect crossing the right femoral neck typical for a stress fracture and diffuse osteopenia. Her primary physician started her on alendronate and ordered a DEXA scan. This showed a T score of -22.1 for the spine and -22.5 for the hip. Her dialysate calcium is 2.0 mEq/L. Her laboratory data show PTH of 17 pg/ml (reference range 17–70 pg/ml), calcium of 10.2 mg/dl, phosphorus of 5.2 mg/dl, and alkaline phosphatase of 83 U/ml (reference range 36–92 U/ml)..","In addition to discontinuing alendronate, which ONE of the following is the MOST appropriate management of this patient's CKD-mineral bone disorder?","A. Add calcitriol B. Add denosumab C. Increase the dialysate calcium to 2.5 mEq/L D. Add teriparatide",D,temp nan,Disorders of Divalent Ions Renal Bone Disease & Nephrolithiasis 2559,,"Question 2559: Which ONE of the following statements is CORRECT?","A. The effect of phosphate binders does not increase linearly with dose B. Noncalcium–containing phosphate binders have not been associated with decreased mortality C. Randomized trials have shown that phos-phate binders lead to decreased mortality D. Nutritional databases provide accurate phos-phate content of foods E. Ferric citrate as a phosphate binder does not correct iron deficiency",A,temp nan,Disorders of Divalent Ions Renal Bone Disease & Nephrolithiasis 2560,"A 52-year-old woman with bipolar disorder is evaluated for hypercalcemia. She complains of fatigue, and her family has noted that she has become more withdrawn over the past 2 months. Review of her parents' and brother's laboratory studies does not disclose hypercalcemia. Her medications are lithium and quetiapine. Laboratory studies show serum sodium of 138 mEq/L, potassium of 4.3 mEq/L, chloride of 100 mEq/L, total carbon dioxide (CO2) of 25 mmol/L, BUN of 22 mg/dl, creatinine of 1.3 mg/dl, calcium of 11.3 mg/dl, albumin of 4.3 g/dl, magnesium of 2.6 mg/dl, and phosphorus of 2.4 mg/dl. The PTH level is 136 pg/ml (reference range 17–70 pg/ml). The 25-hydroxy vitamin D level is 32 ng/ml (reference range 30–50 ng/ml), and the 1,25-dihydroxy vitamin D is 35 pg/ml (reference range 25–65 pg/ml). A urinalysis shows specific gravity of 1.012, trace protein, and no cells or casts. The 24-hour urine shows a volume of 3.1 L, creatinine of 1056 mg, and calcium of 99 mg..",Which ONE of the following is the MOST likely diagnosis?,"A. Sarcoidosis B. Familial hypocalciuric hypercalcemia C. Lithium–related primary hyperparathyroidism D. Familial isolated hyperparathyroidism",C,temp nan,Disorders of Divalent Ions Renal Bone Disease & Nephrolithiasis 2561,"A 70-year-old man with recently diagnosed mono-clonal Ig deposition disease is evaluated for hyperphosphatemia. He began therapy with bortezomib, lenalidomide, and dexamethasone 2 days ago. His urine output has been normal, and he complains of fatigue. Laboratory studies show sodium of 136 mEq/L, potassium of 4.2 mEq/L, chloride of 100 mEq/L, total CO2 of 20 mmol/L, BUN of 25 mg/dl, creatinine of 1.3 mg/dl (eGFR 55 ml/min per 1.73 m2), calcium of 9.1 mg/dl, albumin of 3.8 g/dl, phosphorus of 6.6 mg/dl, lactate dehydrogenase of 80 U/L (reference range 60–100 U/L), uric acid of 4.2 mg/dl, leukocyte count of 4700/µl, hemoglobin 9.6 of g/dl, and platelets of 131,000/µl. The serum IgG is 14,400 mg/dl (reference range 600–1700 mg/dl), IgM is 25 mg/dl (reference range 35–290 mg/dl), and IgA is 29 mg/dl (reference range 40–400 mg/dl). Serum free kappa-light chains are 1288 mg/dl, and serum free lambda light chains are 2.16 mg/dl..",Which ONE of the following is the MOST likely cause of this man's hyperphosphatemia?,"A. Tumor lysis syndrome B. Decreased GFR C. Elevated IgG kappa monoclonal protein D. Bortezomib E. Lenalidomide",C,temp nan,Disorders of Divalent Ions Renal Bone Disease & Nephrolithiasis 2564,"A 65-year-old woman with ESRD on maintenance hemodialysis is admitted for additional evaluation management of an exquisitely painful abdominal nodule. She has been maintained on hemodialysis for 6 years. She has coronary artery disease and atrial fibrillation. Medications are sevelamer, warfarin, carvedilol, and doxercalciferol. The dialysate calcium is 2.5 mEq/L. On physical examination, she is in moderate distress because of pain localized to the abdominal nodule. She is afebrile. Her body mass index is 38 kg/m2.A cardiac examination shows an irregular rhythm. Examination of the abdomen shows a 4x5-cm tender nodule in the right lower quadrant. Labo-ratory studies show intact PTH of 184 pg/ml (reference range 17–70 pg/ml), calcium of 9.6 mg/dl, and phosphorus of 5.8 mg/dl. A punch biopsy of the nodule shows scattered calcifications in the subcutaneous tissue in a concentric pattern. There are calcifications in the walls of subcutaneous small blood vessels and the adipose tissue around the vessels consistent with calcific uremic arteriolopathy..","In addition to discontinuing warfarin and doxercalciferol, which ONE of the following is the MOST appropriate management strategy for this woman?","A. Add cinacalcet B. Parathyroidectomy C. Wide excision of the abdominal lesion D. Add sodium thiosulfate",D,temp nan,Disorders of Divalent Ions Renal Bone Disease & Nephrolithiasis 2566,"You are asked to see a previously healthy 46-year-old man with a 5-month history of T5–T6-level paraplegia after a motor vehicle accident for hypercalcemia and hyperphosphatemia. He was initially admitted to the Neurologic Intensive Care Unit with pneumonia. After extensive antibiotic therapy and mechanical ventilation, he was extubated. It was then noted that the patient had a persistently elevated serum phosphate of 6.5–6.7 mg/dl. His serum phosphorous level had been 3.1 mg/dl during a previous hospitalization at the time of his initial accident. The physical examination is remarkable for T5–T6-level paraplegia and left elbow calcification. Laboratory studies show serum calcium of 10.4 mg/dl, phosphorus of 6.7 mg/dl, creatinine of 1.2 mg/dl, PTH of 10 pg/ml (reference range 17–70 pg/ml), hemoglobin of 13 g/L, and albumin of 2.8 g/dl. The 24-hour calcium excretion is 310 mg..",Which ONE of the following is the next BEST step in management?,"A. Start bisphosphonate therapy B. Start acute PTH infusions C. Perform Sestamibi scan of the parathyroids D. Start denosumab",A,temp nan,Disorders of Divalent Ions Renal Bone Disease & Nephrolithiasis 2567,"A 64-year-old man with advanced alcohol–related cirrhosis is admitted for management of intractable ascites. You are asked to see him for persistent hyperphosphatemia. The patient had no history of phosphate and calcium abnormalities in the past. His only medications are furosemide and lactulose. On physical examination, he has diffuse muscle wasting. He has marked ascites and 3+ leg edema. He has no asterixis. Laboratory stud-ies show serum calcium of 8.1 mg/dl, phosphorus of 6.4 mg/dl, creatinine of 0.8 mg/dl, PTH of 30 pg/ml (reference range 17–70 pg/ml), hemoglobin of 7.1 g/dL, total bilirubin of 21 mg/dl, and albumin of 1.3 g/dl..",Which ONE of the following is the MOST appropriate next step in the evaluation of this man’s hyperphosphatemia?,"A. Measure 24-hour urine phosphate B. Measure ionized calcium C. Identify the method of serum phosphate measurement D. Measure tubular reabsorption of phosphate",C,temp nan,Disorders of Divalent Ions Renal Bone Disease & Nephrolithiasis 2568,"A 45-year-old man is seen in consultation for longstanding hypercalcemia. At age 18 years old, he had kidney stones followed by a recurrent right renal calculus at age 26 years old. Further evaluation at that time showed a serum calcium level that was elevated between 10.7 and 11.5 mg/dl, a phosphorus level of 4.0 mg/d, a serum creatinine of 1.1 mg/dl, and 24-hour urine calcium of 369 mg. Despite the fact that all PTH levels were at or below the lower limit of the assay, the concern for hyperparathyroidism led to a neck exploration; 3 years later, a mediastinal exploration was unsuccessful in recovering abnormal parathyroid tissue. Histologic findings of these two operations included only two normal parathyroid glands. The patient's hypercalcemia and hypercalciuria persisted. Further workup revealed a normal angiotensin–converting enzyme level. The serum 1,25-dihydroxy vitamin D was 75 pg/ml (reference range 25–65 pg/ml). Treatment with oral prednisone at 30 mg/d for 1 week did not reduce the serum calcium. The patient takes no over the counter medications or vitamins. A family history revealed a cousin with a history of some sort of “calcium problem,” but no other familial diseases were described. A DEXA scan showed normal bone density. A chest radiograph was normal..",Which ONE of the following laboratory tests is MOST likely to explain this patient's condition?,"A. Intact PTH level B. PTH–related protein level C. 25-dihydroxy vitamin D level D. 24,25-dihydroxy vitamin D level E. Fibroblast growth factor 23 level",D,temp nan,Disorders of Divalent Ions Renal Bone Disease & Nephrolithiasis 2569,"A 54-year-old man with stage G3b CKD is evaluated for persistent hypercalcemia after undergoing autologous hematopoietic stem cell transplantation for multiple myeloma. Despite treatment with a bisphosphonate 2 weeks ago, the serum calcium level remains elevated. A physical examination shows a cachectic appearing man in no acute distress. He is confused but alert. The BP is 146/90 mmHg, and the pulse rate is 90/min. The rest of the examination is unremarkable. Laboratory studies show serum calcium of 12.8 mg/dl, phosphorus of 4.6 mg/dl, creatinine of 2.2 mg/dl, hemoglobin of 7.1 g/dL, albumin of 3.3 g/dl, and total protein of 7.7 g/dl..","In addition to saline infusions, which ONE of the following is the MOST appropriate treatment of this man's hypercalcemia?","A. Standard dose denosumab B. Weight-based dose of denosumab C. Weight-based dose of zoledronic acid D. Standard dose calcitonin",B,temp nan,Disorders of Divalent Ions Renal Bone Disease & Nephrolithiasis 2570,"A 23-year-old man is evaluated for anemia, renal insufficiency, and severe hypercalcemia (serum calcium of 13.2 mg/dl). His main hobby is weight-lifting, and he had injected paraffin oil into his chest, arms, and back for the past 3 years to enhance his physical appearance. After the last injection 6 months ago, he developed local infection requiring two courses of oral antibiotics. On physical examination, he is afebrile and in no acute distress. Many warm, minimally tender, dense nodules in the skin and subcutaneous tissue overlying the pectoral, trapezius, and biceps muscles are identified. The rest of his examination is normal. Laboratory studies reveal serum calcium of 13.2 mg/dl, phosphorus of 4.6 mg/dl, creatinine of 4.1 mg/dl, hemoglobin of 7.4 g/dL, PTH of 6 pg/ml (reference range 17–70 pg/ml), albumin of 4.3 g/dl, and total protein of 6.7 g/dl..",A biopsy of the affected area is MOST likely to show which ONE of the following pathologic lesions?,"A. Lymphomatous infiltration B. Necrotizing fasciitis C. Giant cell granuloma formation D. Tumoral calcinosis E. Scleroderma",C,temp nan,Disorders of Divalent Ions Renal Bone Disease & Nephrolithiasis 2571,"A 65-year-old woman with known multiple myeloma is referred for management of persistent hypercalcemia. She has been treated with a variety of agents for hypercalcemia in the past, but despite a recent infusion of zoledronic acid, her hypercalcemia has recurred. You are consulted regarding using denosumab as therapy. A physical examination reveals a cachectic-appearing woman in no acute distress. She is mildly con-fused. Laboratory studies show serum calcium of 13.7 mg/dl, phosphorus of 3.6 mg/dl, creatinine of 1.9 mg/dl, hemoglobin of 7.8 g/dL, albumin of 4.2 g/dl, and total protein of 7.9 g/dl..",Which ONE of the following is the MOST likely potential complication of denosumab therapy in this woman?,"A. Prolonged hypocalcemia B. Prolonged hypomagnesemia C. AKI D. Induction of FSGS E. Prolonged hypophosphatemia",A,temp nan,Disorders of Divalent Ions Renal Bone Disease & Nephrolithiasis 2572,"A 45-year-old woman with metastatic breast cancer is admitted for evaluation of marked confusion and somnolence that developed gradually over the past 2 weeks. She developed a severe headache associated with nausea and vomiting 2 days before admission and sustained a generalized seizure on the day of admission. On physical examination, she is lethargic and confused. Her BP is consistently in the normal range throughout her hospital course. A neurologic examination shows diffuse muscle weakness graded at 5-/5. A 2-cm mass is palpable in her left breast. Laboratory tests show hemoglobin of 10.4 g/dl, serum creatinine of 2.5 mg/dl (increased from 1.2 mg/dl 1 year ago), calcium of 14.5 mg/dl (after correction for an albumin level of 2.3 g/dl), and phosphorus of 7.5 mg/dl. Cerebrospinal fluid analysis is nor-mal. Magnetic resonance imaging (MRI) of the brain shows findings consistent with posterior reversible encephalopathy syndrome. A photoemission tomography-computed tomography (CT) scan revealed multiple metastatic lesions in liver and lung but no metastatic brain lesions..",Which ONE of the following is the MOST likely cause of the MRI findings in this woman?,"A. Hypercalcemia B. Masked hypertension C. Hyperphosphatemia D. AKI",A,temp nan,Disorders of Divalent Ions Renal Bone Disease & Nephrolithiasis 2573,,"Question 2573: Which ONE of the following statements regarding the effects of hypomagnesemia in pa-tients receiving chronic hemodialysis therapy is CORRECT?","A. Hypermagnesemia but not hypomagnesemia is associated with an increased risk of premature mortality B. Hypomagnesemia but not hypermagnesemia is associated with an increased risk of premature mortality C. Both hypomagnesemia and hypermagnesemia increase the risk of premature mortality in dialysis patients D. Supplementation with magnesium reduces the risk of increased premature mortality",C,temp nan,Disorders of Divalent Ions Renal Bone Disease & Nephrolithiasis 2574,"A 69-year-old woman with primary hyperparathyroidism is referred for management of hypercalcemia. She has severe coronary artery disease complicated by three myocardial infarctions, and she has required coronary artery bypass grafting and multiple percutaneous interventions over the past 5 years. Her most recent coronary intervention was 4 months ago. At this point, she experiences mild chest pain on exertion. The patient refuses to undergo parathyroidectomy because of concern regarding her cardiovascular risk for surgery. A physical examination reveals a BP of 110/80 mmHg. The rest of the physical examination is unremarkable. Laboratory studies show serum calcium of 11.7 mg/dl, phosphorus of 2.6 mg/dl, creatinine of 1.4 mg/dl, PTH of 157 pg/ml (reference range 17–70 pg/ml), hemoglobin of 11.8 g/L, and albumin of 4.3 g/dl..",Which ONE of the following is the MOST ap-propriate management strategy for this woman?,"A. Cinacalcet therapy B. Calcitonin C. Estrogen D. Zoledronic acid E. Observation",A,temp nan,Disorders of Divalent Ions Renal Bone Disease & Nephrolithiasis 2575,"A 54-year-old woman is referred for evaluation of possible hyperparathyroidism. A screening examination revealed an elevated serum calcium level of 10.6 mg/dl. She notes that her mood has been somewhat depressed over the past 6 months, a difficulty that she attributes to some minor financial difficulties. She has no other symptoms. A physical examination is within normal limits. Laboratory studies show serum calcium of 10.3 mg/dl, phosphorus of 3.5 mg/dl, creatinine of 0.9 mg/dl, PTH of 69 pg/ml (reference range 17–70 pg/ml), and albumin of 3.8 g/dl..",Which ONE of the following tests should be checked NEXT in this woman's diagnostic evaluation?,"A. A repeat PTH level B. A 25-hydroxy vitamin D level C. A serum estrogen level D. No further testing is required",B,temp nan,Disorders of Divalent Ions Renal Bone Disease & Nephrolithiasis 2576,"An 80-year-old woman with a history of open-angle glaucoma is admitted for evaluation of profound muscle weakness. She had been living independently and was last seen by her daughter 1 day before admission. At that time, she complained of a 2-week history of progressive muscle weakness, but she had no other specific complaints. She had a hip fracture 2 years ago that required surgery and a myocardial infarction 6 years ago. Medications include acetazolamide, which was started 2 weeks ago for glaucoma. A physical examination shows profound and diffuse muscle weakness. There are no focal neurologic deficits. The remainder of the examination is normal. Laboratory studies show serum sodium of 134 mEq/L, potassium of 3.5 mEq/L, chloride of 100 mEq/L, total CO2 of 22 mmol/L, BUN of 18 mg/dl, creatinine of 1.2 mg/dl, calcium of 9.1 mg/dl, phosphorus of 0.6 mg/dl, and albumin of 4.3 g/dl. The fractional excretion of phosphorus is 25%..",Which ONE of the following is the MOST likely cause of this patient's hypophosphatemia?,"A. Oncogenic osteomalacia B. Primary hyperparathyroidism C. Secondary hyperparathyroidism caused by vitamin D deficiency D. Acetazolamide E. Acute respiratory alkalosis",D,temp nan,Disorders of Divalent Ions Renal Bone Disease & Nephrolithiasis 2577,"You are asked to see a 47-year-old man who underwent a partial hepatectomy 3 days ago for treatment of an isolated metastatic lesion from colon cancer for hypophosphatemia. Before surgery, the patient had been in good health, and the only manifestation of his colon carcinoma was occasional rectal bleeding. The surgery went well, but postoperatively, the patient was found to have a serum phosphate level of 0.9 mg/dl. He has been maintained on nasogastric suction. His only complaint is generalized weakness. Pre-operative laboratory studies showed a serum calcium of 9.6 mg/dl and a serum phosphorus of 4.1 mg/dl. The patient had been on no medications before surgery. The surgical procedure required the transfusion of 4 U packed red blood cells. There were no episodes of hypotension during surgery. On physical examination, he is afebrile, and his vital signs are normal. Laboratory studies show serum sodium of 133 mEq/L, potassium of 3.8 mEq/L, chloride of 100 mEq/L, total CO2 of 24 mmol/L, BUN of 14 mg/dl, creatinine of 1.1 mg/dl, calcium of 9.0 mg/dl, phosphorus of 0.8 mg/dl, and albumin of 4.2 g/dl. The fractional excretion of phosphorus is 18%..",Which ONE of the following is the MOST likely cause of this man's hypophosphatemia?,"A. The release of a phosphaturic factor from the liver B. A paraneoplastic phosphate wasting disorder C. Primary hyperparathyroidism D. Phosphate malabsorption E. Vitamin D deficiency",A,temp nan,Disorders of Divalent Ions Renal Bone Disease & Nephrolithiasis 2578,"A 65-year-old man is assessed in the emergency department for the acute onset of severe right flank pain. He had two episodes of symptomatic nephrolithiasis over the past 15 years, although no stone has ever been analyzed. He does not have visible hematuria, fever, or a change in bowel habits. His only medication is atorvastatin at 40 mg/d. The serum calcium and phosphorus levels have always been normal. On physical examination, he is writhing in pain. He is afebrile. There is moderate right flank and right lower quadrant abdominal tenderness. The rest of the examination is unremarkable. Laboratory studies reveal serum sodium of 137 mEq/L, potassium of 4.1 mEq/L, chloride of 101 mEq/L, total CO2 of 25 mmol/L, BUN of 18 mg/dl, creatinine of 1.2 mg/dl, calcium of 9.2 mg/dl, and phosphorus of 3.9 mg/dl. A urinalysis shows pH 5.2, 1+ protein, and 2+ blood. A microscopic examination shows 8–10 monomorphic erythrocytes per high power field and rare leukocytes. No crystals are seen..","After treatment with analgesics, which ONE of the following diagnostic studies should be performed NEXT?","A. Spiral CT scan without contrast B. Spiral CT scan with contrast C. Point of care ultrasonography D. MRI without contrast E. Abdominal radiographs",C,temp nan,Disorders of Divalent Ions Renal Bone Disease & Nephrolithiasis 2579,"A 48-year-old man is evaluated for a long-standing history of recurrent nephrolithiasis. He emigrated from North Africa to the United States 12 years ago. Since arriving in the United States, he has passed six calcium stones. Because of his eco-nomic status, he has not received consistent medical care but has been told that his serum calcium was elevated in the past. His father had a history of recurrent nephrolithiasis. He now has medical insurance and seeks further evaluation for his condition. A physical examination is un-remarkable. Laboratory studies show serum sodium of 136 mEq/L, potassium of 3.8 mEq/L, chloride of 99 mEq/L, total CO2 of 23 mmol/L, BUN of 36 mg/dl, creatinine of 2.1 mg/dl, calcium of 11.1 mg/dl, and phosphorus of 4.8 mg/dl; 1,25-dihydroxy vitamin D level is 47.6 pg/ml (reference range 20–42 pg/ml), and 24,25-dihydroxy vita-min D level is not detected in the serum. The serum PTH level is 8 pg/ml (reference range 17–70 pg/ml). The 24-hour urine calcium excretion is 561 mg. A recent chest radiograph is normal..",Which ONE of the following is the MOST appropriate treatment?,"A. Calcitonin B. Zoledronic acid C. Ketoconazole D. Prednisone E. Parathyroidectomy",C,temp nan,Disorders of Divalent Ions Renal Bone Disease & Nephrolithiasis 2580,,"Question 2580: Which ONE of the following statements regarding the relationship between kidney stones and cardiovascular events is CORRECT?","A. Although there is an excess risk of cardiovascular events associated with passing a kidney stone, the risk is attributable to an increased incidence of hypertension in the population B. When corrected for other factors that may predispose to cardiovascular events, kidney stone formation is not associated with cardiovascular events C. Although there is an excess risk of cardio-vascular events associated with passing a kidney stone, the risk is predominantly found in older individuals D. Even correcting for diabetes, hypertension, and albuminuria, kidney stone formation is associated with an increased risk of cardiovascular events E. Those individuals who form multiple kidney stones are at increased risk for cardio-vascular events compared with those who form a single stone",D,temp nan,Disorders of Divalent Ions Renal Bone Disease & Nephrolithiasis 2581,"A 57-year-old man is evaluated in the emergency department for moderate right flank pain. He has been in good health, except for three episodes of nephrolithiasis. Although no stone analysis was performed after passage of the first two kidney stones, his last stone was analyzed and found to contain calcium oxalate. He has been treated with a thiazide diuretic for hypercalciuria, and he has been adherent with therapy. After institution of hydrochlorothiazide, his urinary calcium fell from 280 to 190 mg/d. Laboratory studies reveal normal serum calcium, phosphate, and serum creatinine levels. Urinalysis shows many erythrocytes with normal morphology. A kidney ultrasound shows mild hydroureter to ureterovesicular junction. The pain remits, and he passes a 5-mm stone..",Which ONE of the following is the next MOST appropriate step in management?,"A. Follow-up ultrasonography imaging in 10 days B. Repeat evaluation of urinary parameters in 2 weeks C. The stone should be sent for chemical analysis D. No further diagnostic studies are necessary E. The patient should undergo a spiral CT study at this time",C,temp nan,Disorders of Divalent Ions Renal Bone Disease & Nephrolithiasis 2582,"A 41-year-old man is evaluated in the emergency department for severe left flank pain. He had one previous episode of nephrolithiasis 2 years ago. The stone was analyzed and found to contain 100% calcium oxalate. His management has been limited to an increase in his daily fluid intake. Laboratory studies reveal normal serum calcium, phosphate, and serum creatinine levels. Urinalysis shows many monomorphic erythrocytes. An ultrasound examination shows minimal hydroureter to the midureter with a 6-mm stone, with urine passage around the stone. The patient is treated with analgesics with good relief..",Which ONE of the following would lead to the MOST rapid stone passage and relief of symptoms for this patient?,"A. Infusion of 0.9% saline B. Tamsulosin C. Silodosin D. Tadalafil",C,temp nan,Disorders of Divalent Ions Renal Bone Disease & Nephrolithiasis 2583,"A 62-year-old man with ESRD is evaluated during a follow-up visit 6 months after an episode of refractory culture–negative peritonitis that required catheter removal. He has since been treated with hemodialysis. He now hopes to switch back to peritoneal dialysis (PD) because that modality better suits his lifestyle. Since starting hemodialysis, his residual renal function has declined, and he is now anuric. His dialysis adequacy while on PD was consistently above recommended guidelines, with weekly Kt/V values greater than 2.2. On physical examination, his weight is 68 kg. His abdomen is soft with normal bowel sounds and no masses evident on palpation. The remainder of the examination is normal..",Which ONE of the following would be the BEST advice regarding his outcomes of returning to PD?,"A. PD is unlikely to be successful because of adhesions and inadequate peritoneal mem-brane transport B. His peritonitis-free survival after switching back to PD is comparable with that of those who did not require transfer to hemodialysis after experiencing peritonitis C. PD is not recommended because he is anuric D. PD is not recommended because of an in-creased risk for recurrent peritonitis after an episode of culture-negative peritonitis",B,temp nan,ESRD & Dialysis 2584,"A 62-year-old woman with ESRD maintained on PD for the last 8 years is evaluated in the PD clinic. Over the last 3 months, she has developed increased dyspnea on exertion and leg edema. She has lost 3 kg in weight in association with intermittent diffuse abdominal pain, constipation, anorexia, and nausea. She has had no problems with inflow. She had peritonitis due to coagulase-negative staphylococcus 6 months ago that cleared after a 2-week course of intraperitoneal cefazolin. On physical examination, the abdo-men is firm to palpation. There are basilar crackles in both lung fields. She has 2+ leg edema. A rapid 2-L exchange shows no prob-lem with inflow and return of 1900 ml of clear effluent. Results of her peritoneal equilibrium tests over the past several years performed using 4.25% dextrose-containing fluid are as follows: 4 years ago D/Pcreatinine (dialystate to plasma creatinine ratio) was 0.65 and D/Pglucose (dialystate to plasma glucose ratio) was 0.41. 3 years ago ago D/Pcreatinine was 0.82 and D/Pglucose was 0.29. 2 years ago D/Pcreatinine was 0.68 and D/Pglucose was 0.40. 1 year ago D/Pcreatinine was 0.52 and D/Pglucose was 0.48. Currently the D/Pcreatinine is 0.38 and D/Pglucose is 0.59..",Which ONE of the following is the MOST likely cause of this patient’s ultrafiltration failure?,"A. A peritoneal dialysate leak B. Increased lymphatic absorption C. Encapsulating peritoneal sclerosis D. A high transport membrane",C,temp nan,ESRD & Dialysis 2585,"A 45-year-old woman with slowly progressive advanced CKD is evaluated for symptoms of worsening fatigue and intermittent nausea over the past several months. An arteriovenous fistula was placed 4 weeks ago and is maturing as expected. A conversation about dialysis options ensues, and she asks about home–suitable dialysis modalities..",Which ONE of the following would be the BEST advice regarding the differences between PD and home hemodialysis (HHD)?,"A. She can expect an increased risk of all-cause mortality with HHD B. HHD would confer a lower risk of hospitalization compared with PD C. Technique failure is more common in HHD patients compared with PD patients D. If she chooses HHD, dialysis should begin in center and then transition to home to facilitate access maturation",B,temp nan,ESRD & Dialysis 2586,"A 27-year old woman with ESRD due to lupus nephritis maintained on continuous cycling peritoneal dialysis (CCPD) is evaluated in the emergency department for 2 days of abdominal pain, nausea, and vomiting. This morning, she noted that her dialysate effluent was somewhat darker than usual. She does not have fever, rash, or arthralgias. On physical examination, she is afebrile. The abdomen shows mild diffuse tenderness. The PD catheter exit site shows no erythema, purulent drainage, or tenderness. An emergency department nurse obtains 50 ml peritoneal fluid from the 250-ml day dwell that the patient infused just 15 minutes ago..",Peritoneal fluid analysis shows a cell count of 45 total nucleated cells/mm3 with 36% neutrophils Which ONE of the following is the next MOST appropriate step in management?,"A. Repeat the cell count after infusing 1 L of dialysate that dwells for 1–2 hours B. Check serum complement levels C. Obtain a computed tomography scan of the abdomen and pelvis D. Discharge home and repeat the cell count in 24 hours",A,temp nan,ESRD & Dialysis 2587,"Your patient, who is maintained on CCPD, is admitted to a hospitalist service for severe PD–related peritonitis. The hospitalist, who recently rotated on the nephrology service during training, asks if PD should be performed with dextrose-containing solution or icodextrin..",Which ONE of the following would you advise regarding the use of icodextrin during PD-associated peritonitis?,"A. It leads to a faster resolution of peritonitis B. It results in improved fluid balance C. It decreases sodium sieving D. Its use is associated with increased mortality",B,temp nan,ESRD & Dialysis 2588,A 50-year-old woman with ESRD secondary to polycystic kidney disease recently initiated in-center hemodialysis. She has a well matured right upper extremity arteriovenous fistula and uses a rope ladder or rotating site cannulation technique (needle placement that is rotated along the length of the fistula). She has not had any access-related events. She would like to transition from standard in–center hemodialysis to frequent hemodialysis performed at home 5 d/wk. She has heard about buttonhole needling and inquiries about advantages and disadvantages of that technique. You begin a discussion about the relative merits and risks of using the buttonhole needling technique..,Which ONE of the following outcomes can she expect if she decides to implement the buttonhole needling technique?,"A. An increased risk of infectious complications B. An increased rate of access survival C. A decrease in cannulation-associated pain D. An increased rate of hematoma formation",A,temp nan,ESRD & Dialysis 2589,"A 56-year-old man who recently started hemodialysis is evaluated during weekly hemodialysis rounds. He indicates that his treatments have been difficult, because he feels “drained” for several hours after undergoing dialysis. He receives 3.5 hours of hemodialysis thrice weekly, with an average blood flow of 400 ml/min and a dialysate flow of 800 ml/min. The dialysate composition is sodium 135 mEq/L, potassium 2 mEq/L, total buffer 37 mEq/L, and calcium 2.5 mEq/L. His average interdialytic weight gain is 1.0 kg, and the average predialysis and postdialysis BPs are 142/76 and 130/68 mmHg, respectively. On physical examination, he does not have postural hypotension following dialysis. He does not have edema. The remainder of the examination is unremarkable. The monthly laboratory studies show single-pool Kt/V of 1.4, hemoglobin of 11.2 g/dl, sodium of 138 mEq/L, potassium of 4.2 mEq/L, chloride of 103 mEq/L, total CO2 of 23 mmol/L, calcium of 9.0 mg/dl, phosphorus of 4.8 mg/dl, and albumin of 3.8 g/dl. His residual renal function is estimated to be approximately 5 ml/min per 1.73 m2..",Which ONE of the following interventions is MOST likely to reduce the duration of the recovery time after dialysis for this man?,"A. An increase in dialysate potassium B. A reduction in the blood flow rate C. An increase in dialysate sodium to 140 mEq/L D. A decrease in dialysis frequency to twice weekly",C,temp nan,ESRD & Dialysis 2590,A 57-year-old man with ESRD due to hyper-tensive nephrosclerosis is evaluated during routine hemodialysis rounds. He started hemodialysis 3 weeks ago and has had to curtail his exercise regimen because of the time committed to dialysis and his ongoing part–time work. He recently saw a report on the internet about an exercise regimen that used modest resistance exercise with weights and elastic bands during the first 2 hours of dialysis..,Which ONE of the following outcomes can he expect from implementing such an exercise regimen during dialysis?,"A. A decline in dialysis adequacy B. An increase in musculoskeletal injuries C. An increase in cardiovascular events D. An improvement in peak oxygen consumption",D,temp nan,ESRD & Dialysis 2591,"A 54-year-old woman with ESRD is evaluated during dialysis rounds. She complains of several weeks of increased pruritus but is otherwise asymptomatic. You review her most recent lab-oratory results with her. During this discussion, you explain that her serum parathyroid hormone (PTH) has shown progressive elevation over the past 6 months and that it is now outside of the target range. This has occurred despite optimization of the serum calcium, phosphorus, and vitamin D status. Her medications include sevelamer 2400 mg thrice daily with meals and paricalcitol 8 µg with hemodialysis. Her most recent laboratory studies show a corrected calcium of 9.8 mg/dl, a serum phosphorus of 4.9 mg/dl, and a PTH level of 730 pg/ml. In addition to continuing vitamin D sterol and phosphate binder therapy, you recommend initiation of cinacalcet therapy. She is concerned about the cost of this agent, but indicates that she would be willing to take it if cinacalcet offered significant benefit..",Which ONE of the following would you indicate is the MOST likely benefit of cinacalcet therapy in this patient?,"A. It will reduce her mortality risk B. It will reduce her risk of fracture C. It will likely delay or obviate the need for future parathyroidectomy D. It will decrease her risk for cardiovascular events",C,temp nan,ESRD & Dialysis 2592,"A 45-year-old man with ESRD secondary to hypertensive nephrosclerosis is evaluated for severe hyperparathyroidism refractory to medical therapy. He now has evidence of myopathy and intractable pruritus. His current regimen includes optimally dosed cinacalcet, sevelamer, and doxercalciferol. His most re-cent laboratory studies show hemoglobin of 11.5 g/dl, serum albumin of 3.4 g/dl, PTH of 1832 pg/ml, corrected serum calcium of 10.3 mg/dl, and serum phosphorus of 6.4 mg/dl. You recommend parathyroidectomy, and he asks about what he might expect following this procedure..",Which ONE of the following would you advise him regarding the effect of parathyroidectomy?,"A. He can expect a reduced risk of hospitalization for the first year after surgery B. He has about a 10% risk of persistent hyperparathyroidism C. He is unlikely to develop hypocalcemia D. He can expect an increased risk of mortality for 1 year following surgery",B,temp nan,ESRD & Dialysis 2593,,"Question 2593: Which ONE of the following long-term outcomes (over a median follow-up of 3.6 years) was found in the intervention group in the Frequent Hemodialysis Network (FHN) Daily Trial?","A. An increase in left ventricular mass B. Higher Physical Health Composite scores C. Improved relative mortality D. A reduction in hospitalization rates",C,temp nan,ESRD & Dialysis 2594,,"Question 2594: Which ONE of the following outcomes has been found to be associated with use of dialyzable β-blockers compared with nondialyzable β-blockers in older patients undergoing intermittent hemodialysis?","A. Increased 6-month mortality B. Increased risk of ventricular tachycardia C. Increased risk of myocardial infarction D. Increased risk of sudden cardiac death",A,temp nan,ESRD & Dialysis 2595,"A 57-year-old man with ESRD maintained on thrice weekly in–center hemodialysis is evaluated for recurrent intradialytic hypotension. His prescribed dry weight is 90 kg, and he typically requires 6 L of volume removal during dialysis. He frequently cannot achieve his dry weight because his systolic BP decreases by an average of 40 mmHg, with intra-dialytic systolic BPs as low as 85 mmHg. He receives dialysis three times weekly for 4 hours. The dialysate sodium concentration is 138 mEq/L..",Which ONE of the following ultrafiltration goal targets is MOST appropriate to decrease his risk of intradialytic hypotension?,"A. 6%–7% Weight gain B. 5%–6% Weight gain C. 4%–5% Weight gain D. < 4% Weight gain",D,temp nan,ESRD & Dialysis 2596,"A 62-year-old man with ESRD on maintenance hemodialysis is evaluated for hyperlipidemia during a routine office visit. He has diet–controlled type 2 diabetes mellitus and hypertension. He does not have chest pain or dyspnea, and he walks 2 miles daily without difficulty. His medications are aspirin, irbesartan, amlodipine, and carvedilol. He has never received treatment with statins. Recent laboratory studies show total cholesterol of 223 mg/dl, LDL cholesterol of 124 mg/dl, HDL cholesterol of 46 mg/dl, and triglycerides of 267 mg/dl..","In addition to lifestyle and dietary modification, which ONE of the following is the MOST appropriate management?","A. Pravastatin plus ezetimibe B. Pravastatin C. Fenofibrate D. No additional management",D,temp nan,ESRD & Dialysis 2597,One of your patients with ESRD treated with hemodialysis recently read about the cardiovascular benefits of cocoa flavanols. She asks for your opinion about the efficacy of these compounds in mitigating cardiovascular risk..,Which ONE of the following options is CORRECT about the cardiovascular effects of cocoa flavanols in ESRD?,"A. They lower systolic BP B. They reduce C–reactive protein levels C. They enhance flow–mediated brachial artery dilation D. They have no effect on platelet aggregation",C,temp nan,ESRD & Dialysis 2598,"A 69-year-old man with ESRD, hypertension, obstructive sleep apnea, and type 2 diabetes mellitus is evaluated for hypertension during weekly dialysis rounds. Over the past 2 weeks, he has felt excessively tired and is sleeping through most of the day. His wife indicates that he has been using continuous positive airway pressure as prescribed at night, but she has witnessed several episodes of apnea during daytime naps. Review of his dialysis records over the last week shows an average predialysis BP of 168/94 mmHg and an average postdialysis BP of 154/90 mmHg. He usually achieves his target weight, although he occasionally sustains an asymptomatic fall in BP to approximately 100/ 60 mmHg during treatment. He has been adherent with his dialysis treatments, and his inter-dialytic weight gains are 1.5 kg. Medications include carvedilol 25 mg twice daily, fosinopril 40 mg daily, amlodipine 10 mg daily, long- and short-acting insulin, and epoetin with each dialysis treatment. The dialysis prescription is for 225 minutes three times per week, with an average blood flow rate of 450 ml/min, and a dialysate flow rate of 800 ml/min. The dialysate composition is sodium 138 mEq/L, potassium 2.0 mEq/L, total buffer 37 mEq/L, and calcium 2.5 mEq/L. The physical examination shows a pulse of 72 per mi-nute and trace pedal edema. The remainder of the examination is unremarkable. The monthly laboratory studies show a single-pool Kt/V of 1.5 and a hemoglobin of 10.4 g/dl..",Which ONE of the following is the MOST appropriate next step in managing this man’s hypertension?,"A. Decrease the dialysate sodium to 136 mEq/L B. Decrease epoetin C. Gradual lowering of the target weight D. Increase carvedilol",C,temp nan,ESRD & Dialysis 2599,"As medical director, you are reviewing the policies of your outpatient dialysis facility with your nursing staff..",Which ONE of the following measures should be included in the policies of your outpatient dialysis center?,"A. Isolation of patients who test positive for hepatitis C antibody B. Use of product water for dialysate only when microbial counts are < 50 CFU Immunization of all patients against Staphylococcus aureus C. Hand hygiene after removal of gloves and other personal protective equipment D. Monthly interdisciplinary team meetings with every patient",D,temp nan,ESRD & Dialysis 2600,You have been appointed to the quality oversight committee of your health care system. You have been charged with developing recommendations for improving the value of care for patients with ESRD. The committee chair asks you to make a recommendation for the most important intervention that would reduce the cost of care for patients with ESRD..,Which ONE of the following would you recommend as the BEST strategy to reduce the overall cost of care for patients with ESRD?,"A. Increase organ procurement by instituting opt–out donation policies B. Increase the utilization of home–suitable dialysis modalities C. Referral to nephrologists when patients are anticipated to require renal replacement therapy (RRT) at least 1 year in advance D. Provision of financial incentives to care providers for achievement of laboratory goals before the initiation of dialysis",C,temp nan,ESRD & Dialysis 2601,"An 87-year-old man with ESRD is admitted to the hospital for management of line sepsis related to his tunneled dialysis catheter. A left forearm arteriovenous fistula was placed 2 weeks ago. He has a prolonged hospital course resulting in severe deconditioning. He is no longer able to ambulate independently, and requires placement at a skilled nursing facility for physical therapy. Following discharge, the patient returns to his outpatient hemodialysis center for continued renal replacement therapy (RRT). As the medical director of his dialysis unit and his provider, you are invested in reducing his risk for a repeat hospitalization..",Which ONE of the following is MOST likely to be associated with a decreased risk of a readmission within 30 days of discharge for this patient?,"A. His inability to ambulate B. His age C. His stable residence in a nursing home D. An additional provider visit within one month following discharge",D,temp nan,ESRD & Dialysis 2602,"A 47-year-old woman with ESRD, type 2 diabetes mellitus, hypertension, and hyperlipidemia is admit-ted to your dialysis facility. She has a mature and functional arteriovenous fistula and is well informed about RRTs. She tells you that she has several family members who also have ESRD, and she expresses concern that two of her first cousins died during routine maintenance dialysis treatments. You review her history and laboratory studies before her hemodialysis treatment. These include a serum potassium level of 5.2 mEq/L, a serum calcium level of 9.2 mg/dl, a serum magnesium level of 2.4 mg/dl, a serum phosphorus level of 4.5 mg/dl, and a serum PTH level of 482 pg/ml. Her current medications include propanolol, long-acting insulin, atorvastatin, and ergocalciferol..",Which ONE of the following MOST strongly increases her risk for sudden cardiac arrest?,"A. Propanolol therapy B. The serum potassium concentration of 5.2 mEq/L C. The serum magnesium of 2.4 mg/dl D. Her family history E. The serum PTH concentration of 482 pg/ml",D,temp nan,ESRD & Dialysis 2603,"A 68-year-old man with autosomal dominant polycystic kidney disease has been on conventional hemodialysis for 7 years. His arteriovenous fistula provides stable vascular access, he achieves a single-pool Kt/V urea of 1.7, and his BP and electrolytes are stable. He dialyzes with a high-flux membrane for 4 hours three times per week. His predialysis hemoglobin is 11g/dl without erythropoiesis–stimulating agent support, and his serum albumin his 3.6 g/dl. He complains that his exercise tolerance has deteriorated over the past five years. He has read newspaper reports of a procedure called hemodiafiltration, and he asks you if he should consider changing to this treatment modality..",Which ONE of the following is MOST likely to result from a change to hemodiafiltration in this patient?,"A. A reduction in his left ventricular mass B. An increase in the serum albumin concentration C. Stabilization of systemic markers of inflammation D. A decrease in quality of life",C,temp nan,ESRD & Dialysis 2604,"You are reviewing a case from your hospital’s acute dialysis unit of a patient who suffered a cardiac arrest immediately after his dialysis session. The patient was a 72-year-old man with ESRD due to diabetic nephropathy who was admitted for foot cellulitis. He had been receiving three times per week hemodialysis sessions via an arteriovenous graft for the past 7 years. You review his medications, recent laboratory results, and the technical aspects of his dialysis treatment. In addition to his usual medications that included calcium acetate, carvedilol, enalapril, and amlodipine, he was also receiving ceftazidime and vancomycin. The patient had what appeared to be an uncomplicated dialysis session on the day of the event, receiving 2.2 L ultrafiltration over a 225-minute session (corresponding to an ultrafiltration rate of 9.8 ml/min and an ultrafiltration volume of 3.2% postdialysis weight). He did not have antecedent hives, rash, pruritus, angioedema, or bronchospasm. The dialysate had a sodium concentration of 140 mEq/L, potassium of 2.0 mEq/L, final buffer concentration of 36 mEq/L, and calcium of 2.0 mEq/L. Predialysis laboratory studies on the day of the arrest showed hemoglobin 9.8 g/dl, sodium 140 mEq/L, potassium 5.1 mEq/L, chloride 105 mEq/L, total CO2 20 mmol/L, calcium 8.9 mg/dl, phosphorus 5.9 mg/dl, and albumin 3.2 g/dl. You discuss the episode during a quality assurance and safety meeting..",Which ONE of the following factors in this case has been shown to be associated with sudden cardiac death in patients with ESRD receiving hemodialysis?,"A. A dialysate calcium level of 2.0 mEq/L B. A dialysate sodium concentration of 140 mEq/L C. A dialysate potassium concentration of 2 mEq/L D. His ultrafiltration rate E. Vancomycin use",A,temp nan,ESRD & Dialysis 2605,"A 57-year-old man with CKD due to FSGS is evaluated in the clinic. He has hypertension, hyperlipidemia, and gout. His medications are losartan, furosemide, allopurinol, and pravastatin of 40 mg daily. He is on the active kidney transplant waiting list, and a living donor trans-plant is planned in approximately 2 months. He has mild uremic symptoms, and you are now transitioning him to a hemodialysis regimen. His last fasting lipid profile performed 2 weeks ago while on the current dose of simvastatin showed total cholesterol 150 mg/dl, LDL cholesterol 67 mg/dl, HDL cholesterol 52 mg/dl, and triglycerides 155 mg/dl..",Which ONE of the following is the MOST appropriate management of this patient’s statin therapy?,"A. Discontinue pravastatin now B. Discontinue pravastatin at the time of transplantation C. Increase pravastatin to 80 mg daily D. Continue pravastatin at the current dose",D,temp nan,ESRD & Dialysis 2606,"A 52-year-old woman with ESRD due to IgA nephropathy and hypertension is evaluated during an interval clinic visit. She has been on three-times-per-week hemodialysis for 3 years, and she has been medically stable. She walks several miles each day without difficulty and continues to work part time. She denies depressed mood, chest pain, or dyspnea. When you ask her how her life is at home, she indicates that she and her partner have had some difficulties due to a lack of sexual activity. On physical examination, her BP is 130/78 mmHg. She has a well matured left radiocephalic fistula. The remainder of the examination is normal..",Which ONE of the following is MOST likely contributing to her sexual inactivity?,"A. Sexual dysfunction B. Lack of interest C. Physical debility D. An arteriovenous fistula rather than a tunneled central venous access",B,temp nan,ESRD & Dialysis 2607,"A 54-year-old man with ESRD is admitted for management of presumed catheter–related bacteremia. He had no pre–ESRD nephrology care and recently started maintenance hemodialysis on an urgent basis for symptomatic uremia. Two days ago, he developed acute onset of fever to 40 C, chills, and rigors during dialysis. After obtaining blood cultures, he received intravenous vancomycin and ceftazidime at the outpatient dialysis center before admission. His tunneled dialysis catheter is removed because of erythema and purulent exudate at the exit site. His fever is now resolved. He does not have back pain, cough, dental complaints, or rash. On physical examination, he is now afebrile. The BP is 136/88 mmHg. There is a 1-cm area of erythema at the tunneled dialysis catheter removal site. He has no stigmata of emboli. He has no obvious dental issues on gross inspection of the oropharynx. The cardiac examination shows a grade 2 early peaking systolic crescendo-decrescendo murmur along the right sternal border without radiation. The remainder of the examination is unremarkable. The blood cultures drawn before implementation of antibiotic therapy grew S. aureus within 12 hours. The results of sensitivity testing and repeat blood cultures are pending. Laboratory studies show sodium 138 mEq/L, potassium 3.8 mEq/L, chloride 102, total CO2 25 mmol/L, BUN 45 mg/dl, and creatinine 6.4 mg/dl..","In addition to continued antibiotic therapy, which ONE of the following is the next BEST step in management?","A. Arrange for placement of a tunneled dialysis catheter today B. Transesophageal echocardiography C. Magnetic resonance imaging of the spine D. Bone scintigraphy (bone scan) E. Examination for an occult dental source of bacteremia",B,temp nan,ESRD & Dialysis 2608,"A 52-year-old-man with ESRD on maintenance hemodialysis is diagnosed with proximal calcific uremic arteriolopathy with exquisitely painful lesions on his thighs and lower abdomen. He has type 2 diabetes, hypertension, and obesity. He does not smoke. He is dialyzed for 4 hours on a three times weekly schedule. The dialysate composition is sodium 140 mEq/L, potassium 2 mEq/L, total buffer 36 mEq/L, and calcium 2.5 mEq/L. His medications are insulin detemir once daily, insulin as part before each meal, irbesartan, carvedilol, felodipine, lanthanum carbonate with meals, and intravenous epoetin with each dialysis treatment. On physical examination, the BP is 160/94 mmHg. A representative skin lesion is shown in Figure 2. His laboratory studies show single-pool Kt/V 1.6, hemoglobin 11.4 g/dl, sodium 139 mEq/L, potassium 5.3 mEq/L, chloride 103 mEq/L, total CO2 20 mmol/L, calcium 9.4 mg/dl, phosphorus 5.8 mg/dl, albumin 3.5 g/dl, and PTH 396 pg/ml..",Which ONE of the following has MOST likely increased this man’s risk for calcific uremic arteriolopathy?,"A. Hypertension B. Irbesartan C. Dialysate calcium of 2.5 mEq/L D. Intravenous epoetin E. Multiple daily insulin injections",E,temp nan,ESRD & Dialysis 2609,"A 44-year-old woman with ESRD on maintenance hemodialysis has been reading information on the internet about the accumulation of indoxyl sulfate and cardiovascular risk in ESRD. She asks about whether there are any interventions that could potentially lower her serum indoxyl sulfate levels. She is maintained on three-times-per-week hemodialysis, with an average blood flow rate of 400 ml/min and a dialysate flow rate of 800 ml/min. The monthly laboratory studies show an average single–pool Kt/V of 1.3..",Which ONE of the following interventions is MOST likely to lower this woman’s serum levels of indoxyl sulfate?,"A. Decreased dietary fiber B. An oral charcoal adsorbent C. Increased blood flow rate to 450 ml/min D. Increased dialysis time to achieve a Kt/V of 1.7",B,temp nan,ESRD & Dialysis 2610,"A 72-year-old woman has ESRD and chronic atrial fibrillation. She is noted to have increasing frequency of symptomatic atrial fibrillation during her hemodialysis sessions. Today, she was found to have a pulse rate of 120 per minute 30 minutes into her dialysis treatment. Her medications include amlodipine of 5 mg daily, aspirin, and epoetin. On physical examination, her systolic BP is 110 mmHg, and her pulse is irregular, with a rate of 134/min. Her lungs are clear, and she has no edema. Her heart rate decreases to > 100 per minute with a decrease in the ultrafiltration rate, but she is unable to achieve her dry weight, despite an increase in her dialysis time by 15 minutes..",Which ONE of the following is the BEST initial pharmacologic intervention for maintaining rate control in this woman with atrial fibrillation during dialysis?,"A. Metoprolol B. Enalapril C. Bisoprolol D. Digoxin",C,temp nan,ESRD & Dialysis 2611,"A 56-year-old woman with ESRD is evaluated during her initial interdisciplinary team meeting after recently starting hemodialysis on an urgent basis. She did not receive pre-ESRD care and is currently dialyzing via a tunneled central venous catheter. Her dialysis facility is a nonprofit center that serves a diverse population of patients, 40% of which have incomes below the poverty level. She currently has employer group health insurance coverage. She asks about moving forward with a transplant evaluation..",Which ONE of the following factors is MOST likely to be associated with an increased rate of referral for transplantation within 1 year of starting dialysis for this woman?,"A. The absence of pre–ESRD nephrology care B. Dialysis at a nonprofit dialysis facility C. The proportion of patients treated at her center who are below the poverty level D. Her insurance status",D,temp nan,ESRD & Dialysis 2612,"A 76-year-old woman with ESRD due to membranous nephropathy on maintenance PD is found to have asymptomatic atrial fibrillation during a routine office visit. She has an excellent functional status and does not have a history of falls or bleeding complications. She has hypertension that is controlled on lisinopril. On physical examination, her body mass index is 25 kg/m2. Her cardiac examination shows an irregularly irregular rhythm and no murmur or rub. The remainder of the examination is un-remarkable. After a detailed discussion regarding the benefits and risks of anticoagulation, she is in favor of anticoagulation to reduce her risk of stroke. Her laboratory studies show hemoglobin 11.3 g/dl, platelet count 256,000/µl, albumin 4.2 g/dl, calcium 9.0 mg/dl, phosphorus 4.2 mg/dl, and PTH 176 pg/ml..",Which ONE of the following is the MOST appropriate treatment strategy to reduce the subsequent risk of stroke for this woman?,"A. Enoxaparin 70 mg once daily B. Rivaroxaban 20 mg daily C. Warfarin with time in the therapeutic range > 70% D. Dabigatran 75 mg twice daily E. Apixaban 5 mg twice daily",C,temp nan,ESRD & Dialysis 2613,"A 67-year-old woman with chronic systolic heart failure and stage 3a:A1 CKD is evaluated for persistent congestive symptoms despite recent initiation of furosemide 2 weeks ago. During the first week after initiation of furosemide, the weight decreased by 4 kg. Over the past week, however, her weight has remained stable, and she continues to have orthopnea and dyspnea on exertion. Medications are furosemide at 40 mg daily, lisinopril at 30 mg daily, and spironolactone at 25 mg daily. On physical examination, the BP is 115/80 mmHg, and the pulse rate is 68/min. The jugular venous pressure is 8 cm H2O. There are bibasilar crackles and 2+ leg edema. Laboratory studies show sodium of 134 mEq/L, potassium of 5.0 mEq/L, chloride of 96 mEq/L, total CO2 of 28 mmol/L, BUN of 34 mg/dl, and creatinine of 2.1 mg/dl (increased from 1.8 mg/dl 2 weeks ago). A urinalysis shows no protein, cells, or casts..","In addition to ongoing assessment of this woman’s serum electrolytes and kidney function, which ONE of the following is the MOST appropriate next step in management?","A. Discontinue furosemide B. Discontinue lisinopril C. Increase furosemide to 80 mg/d D. Increase furosemide to 40 mg twice daily",D,temp nan,Electrolyte Acid-Base Disorders 2614,"A 72-year-old man with recently diagnosed non-small cell lung cancer is referred for evaluation of asymptomatic hyponatremia. He was recently found to have a serum sodium level of 125 mEq/L, despite adherence to 1.5 L/d of fluid restriction. Review of the medical record indicates that his serum sodium was 139 mEq/L on laboratory studies obtained 1 year ago. He is not on any medications. On physical examination, he is alert and oriented. The BP is 138/82 mmHg, and the pulse is 70/min. He has no edema. The remainder of the examination is unremarkable. Laboratory studies show sodium of 125 mEq/L, potassium of 4.9 mEq/L, chloride of 91 mEq/L, total CO2 of 24 mmol/L, BUN of 7 mg/dl, and creatinine of 0.8 mg/dl. His urinary studies show osmolality of 560 mOsm/kg, sodium of 80 mEq/L, and potassium of 60 mEq/L..","In addition to continued fluid restriction, which ONE of the following is the BEST initial treatment for this patient?","A. Decrease water intake further to 1.0 L/d B. Decrease water intake further to 750 ml/d C. 100 ml 3% saline intravenously over 6 hours D. Furosemide at 40 mg/d plus sodium chloride tablets at 1 g three times daily",D,temp nan,Electrolyte Acid-Base Disorders 2615,"A 75-year-old man with recently diagnosed pancreatic cancer and hypertension is seen in consultation for hyponatremia. According to family members, he was in his usual state of health until the day before admission, when he developed progressive confusion and agitation. His medications are lisinopril and chlorthalidone. Review of his medical record shows that his serum sodium level was 134 mEq/L 6 weeks ago. On physical examination, he is intermittently agitated and disoriented. The BP is 140/85 mmHg, and the pulse rate is 85/min. There are no postural changes in BP or pulse. The neurologic examination shows no focal deficits. The remainder of examination is unremarkable. Laboratory results show sodium of 115 mEq/L, potassium of 3.9 mEq/L, chloride of 80 mEq/L, total CO2 of 25 mmol/L, BUN of 6 mg/dl, creatinine of 0.8 mg/dl, glucose of 106 mg/dl, calcium of 9.0 mg/dl, and serum osmolality of 236 mOsm/kg. The urine osmolality is 380 mOsm/kg, the urine sodium is 40 mEq/L, and the urine potassium is 30 mEq/L..","In addition to fluid restriction and discontinuing chlorthalidone, which ONE of the following is the MOST appropriate next step in management?","A. No additional interventions B. 3% saline to increase serum sodium concentration 4–6 mEq/L C. Tolvaptan D. Furosemide plus intravenous isotonic saline with potassium chloride at 40 mEq/L",B,temp nan,Electrolyte Acid-Base Disorders 2616,,"Question 2616: Which ONE of the following is MOST likely to be associated with the development of hyponatremia in individuals involved in heavy exercise?","A. Urine sodium plus urine potassium equal to 36 mEq/L B. Urine osmolality of 200 mOsm/kg after exercise C. Weight gain .3 kg after exercise D. An increase in urinary osmolality during exercise",C,temp nan,Electrolyte Acid-Base Disorders 2617,"A 70-year-old man with refractory nocturnal polyuria is evaluated 1 week after starting desmopressin for enuresis. He has heart failure with a preserved ejection fraction that has been well compensated. Optimization of treatment for lower urinary tract symptoms stemming from benign prostatic hypertrophy has failed to control his nocturia. Before starting desmopressin, the first morning void urine osmolality was 286 mOsm/kg. After starting desmopressin, the frequency of nocturia decreased from five to two times per night. On physical examination, he is alert and oriented. The BP is 130/78 mmHg, and the pulse rate is 72/min. An S4 gallop is present. There is trace pretibial edema. The remainder of the examination is normal. Laboratory studies show that the serum sodium level has decreased to 133 mEq/L from a baseline level of 138 mEq/L. The urine osmolality is 526 mOsm/kg..","In addition to close follow-up of the serum sodium level, which ONE of the following is the MOST appropriate management for this man’s enuresis, hyponatremia, and heart failure?","A. Discontinue desmopressin B. Discontinue desmopressin plus a 100-ml bolus of 3% saline C. Continue desmopressin, decreasing fluid in-take in response to thirst only D. Continue desmopressin and increase dietary sodium",C,temp nan,Electrolyte Acid-Base Disorders 2618,"A 46-year-old man with central diabetes insipidus (DI) is evaluated in the emergency department for a 1-day history of generalized fatigue. Over the past 2 weeks, he increased his fluid intake because of an upper respiratory tract infection. His only medication is intranasal desmopressin at 10 mg twice daily. On physical examination, he is alert and oriented. The remainder of the examination is normal. Laboratory studies show serum sodium of 116 mEq/L, potassium of 3.8 mEq/L, chloride of 82 mEq/L, total CO2 of 24 mmol/L, BUN of 5 mg/dl, creatinine of 0.7 mg/dl, and glucose of 120 mg/dl. The urine osmolality is 826 mOsm/kg. He is admitted, and desmopressin therapy is withheld. The urine volume subsequently rises to 500 ml/h. A repeat serum sodium level obtained 4 hours after the initial laboratory studies is 124 mEq/L, and the urine osmolality is now 36 mOsm/kg..","In addition to ongoing monitoring of the serum sodium level and neurologic status, which ONE of the following is the MOST appropriate treatment for this patient?","A. Restart intranasal desmopressin at 10 mg twice daily B. Desmopressin at 4 mg subcutaneously plus intravenous 5% dextrose in water (D5W) to achieve a serum sodium of 118–122 mEq/L C. Restart intranasal desmopressin at 10 mg twice daily plus a 100-ml bolus of 3% saline intravenously D. Start intravenous D5W at 250 ml/h",B,temp nan,Electrolyte Acid-Base Disorders 2619,"You are asked to see a 65-year-old woman with stage 3b:A3 CKD from diabetic kidney disease and chronic systolic heart failure in consultation 1 week in advance of elective coronary angiography (CA). Medications are metformin, simvastatin, lisinopril, chlorthalidone, and aspirin. A physical examination shows a BP of 132/80 mmHg. There is trace ankle edema. The remainder of the examination is normal. Laboratory studies show sodium of 138 mEq/L, potassium of 5.2 mEq/L, chloride of 104 mEq/L, total CO2 of 23 mmol/L, BUN of 32 mg/dl, creatinine of 1.9 mg/dl, and eGFR of 32 ml/min per 1.73 m2..",Which ONE of the following is the BEST management recommendation?,"A. Discontinue metformin 2–3 days before CA B. Discontinue lisinopril 1 day before CA C. Make no changes to the current medication regimen D. Discontinue chlorthalidone",A,temp nan,Electrolyte Acid-Base Disorders 2620,"A 78-year-old-man with advanced chronic obstructive lung disease, congestive heart failure, and stage G3b CKD is admitted for treatment of pneumonia. Over the past 3 days, his oral intake has been minimal. His home medications include inhaled fluticasone-salmeterol and furosemide. On physical examination, his breathing is labored, with use of accessory respiratory muscles. The BP is 96/60 mmHg, the pulse rate is 120/min, and the temperature is 38.1C. The oxygen saturation is 98% on 2 L/min supplemental oxygen via nasal cannula. The jugular venous pressure is 5 cm H2O. The lung examination shows scattered wheezes and poor air movement. There is no peripheral edema. Laboratory studies show se-rum sodium of 145 mEq/L, potassium of 3.0 mEq/L, chloride of 100 mEq/L, total CO2 of 40 mmol/L, BUN of 45 mg/dl, and creatinine of 2.5 mg/dl (increased from a recent baseline level of 1.8 mg/dl). An arterial blood gas on 2 L/min of supplemental oxygen shows pH 7.47, PaCO2 of 55 mmHg, bicarbonate of 38 mEq/L, and PaO2 of 65 mmHg. The patient is treated with antibiotics, bronchodilators, and systemic corticosteroids. You are asked to help manage his acid-base disorder..","In addition to potassium supplementation and continuous monitoring of the volume status, which ONE of the following is the MOST appropriate management strategy for this man’s acid-base disorder?","A. Discontinue furosemide and begin intravenous 0.9% saline at 125 ml/h B. Replace furosemide with acetazolamide at 500 mg daily C. Add spironolactone at 50 mg twice daily D. Ammonium chloride at 20 mEq in 1 L 0.9% saline intravenously over 24 hours",A,temp nan,Electrolyte Acid-Base Disorders 2621,"A 50-year-old man with bipolar disorder is referred for persistent polyuria. He had previously been maintained on lithium for 15 years, but it was discontinued 6 months ago because of a progressive rise in the serum creatinine level from 0.8 to 1.4 mg/dl. A water deprivation test 2 months ago was consistent with nephrogenic DI. He is now maintained on hydrochlorothiazide at 25 mg daily. Laboratory studies show serum osmolality of 298 mOsm/kg, sodium of 141 mEq/L, potassium of 3.9 mEq/L, chloride of 102 mEq/L, total CO2 of 24 mmol/L, BUN of 28 mg/dl, creatinine of 1.4 mg/dl, and glucose of 90 mg/dl. A 24-hour urine collection shows a volume of 4.0 L, a sodium of 70 mEq/L, a potassium of 26 mEq/L, and a urine osmolality of 240 mOsm/kg. A urinalysis shows specific gravity of 1.006, trace protein, and negative glucose..",Which ONE of the following is the MOST appropriate next step in management?,"A. Increase hydrochlorothiazide to 25 mg twice daily B. Add amiloride at 10 mg daily C. Add intranasal desmopressin at 10 mg daily D. Decrease dietary sodium intake to 2–3 g/d",D,temp nan,Electrolyte Acid-Base Disorders 2622,"A 53-year-old man is admitted to the intensive care unit with urosepsis. His serum sodium level has risen slowly from 140 to 155 mEq/L over the past 5 days. He is lethargic, but responds appropriately to simple questions. His BP is 130/70 mmHg off vasopressor agents. The pulse rate is 80/min. His weight is 60 kg. The remainder of the examination is unremarkable. His current urine output is 2 L/d. Laboratory studies show serum osmolality of 329 mOsm/kg, sodium of 155 mEq/L, potassium of 3.6 mEq/L, chloride of 122 mEq/L, total CO2 of 20 mmol/L, BUN of 36 mg/dl, creatinine of 1.5 mg/dl, and glucose of 88 mg/dl. Urinary studies show sodium of 60 mEq/L, potassium of 40 mEq/L, and osmolality of 315 mOsm/kg..","Assuming insensible water losses of 0.8 L/d, which ONE of the following is the total amount of free water that should be administered to decrease the serum sodium to 145 mEq/L over the next 24 hours?","A. 4.0 L B. 3.2 L C. 2.3 L D. 1.6 L",A,temp nan,Electrolyte Acid-Base Disorders 2623,"A 25-year-old woman marathon runner becomes confused after completing the run in 5 hours and 15 minutes. Her serum sodium at end of the run is 125 mEq/L. On physical examination, she is obtunded. The BP is 140/80 mmHg, and the pulse rate is 86/min. Skin turgor is normal. The remainder of the examination is normal..",Which ONE of the following is MOST important in the pathogenesis of her hyponatremia?,"A. Excessive water intake and retention B. Sequestration of water in the gastrointestinal tract C. Sodium chloride loss in sweat D. Decreased extracellular fluid volume",A,temp nan,Electrolyte Acid-Base Disorders 2624,"A 70-year-old woman with type 2 diabetes mellitus, stage G3b CKD, and hypertension is admitted for treatment of hypovolemia after 1 week of recalcitrant nausea, vomiting, and diarrhea. Her medications include metformin at 500 mg twice daily, irbesartan at 150 mg/d, and chlorthalidone at 25 mg daily. On physical examination, she is alert, oriented, and does not appear toxic. The skin turgor is reduced. She is afebrile, with a BP of 100/60 mmHg and a pulse rate of 112/min. The remainder of the physical examination is unremarkable. Laboratory studies show serum sodium of 143 mEq/L, potassium of 4.9 mEq/L, chloride of 112 mEq/L, total CO2 of 16 mmol/L, BUN of 46 mg/dl, creatinine of 2.2 mg/dl (increased from a recent baseline level of 1.4 mg/dl), and glucose of 102 mg/dl. A complete blood count shows a leukocyte count of 5600/microliter, hemoglobin of 13.3 g/dl, and platelet count of 356,000/microliter. An arterial blood gas on ambient air shows pH 7.32, PaCO2 of 32 mmHg, bicarbonate of 18 mEq/L, and PaO2 of 94 mmHg. The arterial lactate level is 4 mmol/L. Urinalysis shows specific gravity of 1.018, trace protein, and no glucose. The urine sediment is bland. The urine sodium level is < 10 mEq/L..","In addition to discontinuing her current medi-cations and volume expansion with isotonic crystalloid, which ONE of the following is the next BEST step in management?","A. Obtain a serum metformin level to guide further therapy B. Continuous RRT at 25 ml/kg per hour C. Acute hemodialysis with a blood flow of 450 ml/h for 4 hours D. Observation of clinical status after correction of hypovolemia",D,temp nan,Electrolyte Acid-Base Disorders 2625,,"Question 2625: Compared with furosemide, which ONE of the following statements is CORRECT regarding torsemide?","A. Torsemide has increased bioavailability and a longer half-life B. Torsemide inhibits proximal tubular sodium reabsorption C. Torsemide is more kaliuretic D. Torsemide is more potent because it directly inhibits the activity of the epithelial sodium channel (ENaC) in the collecting duct",A,temp nan,Electrolyte Acid-Base Disorders 2626,"You are asked to make management recommendations for a 70-year-old woman with recently diagnosed SIADH. She is asymptomatic and has a normal physical examination. Laboratory studies show serum sodium of 128 mEq/L, potassium of 4.0 mEq/L, chloride of 94 mEq/L, total CO2 of 24 mmol/L, BUN of 6 mg/dl, creatinine of 0.6 mg/dl, glucose of 80 mg/dl, and osmolality of 263 mOsm/kg. Urine studies show sodium of 40 mEq/L, potassium of 22 mEq/L, and osmolality of 250 mOsm/kg. The daily urine volume is 2 L. You recommend fluid restriction and liberalization of dietary sodium intake..","Assuming 0.8 L insensible losses of water per day, which ONE of the following is the maximum amount of solute-free water that she can drink without a further decrease in the serum sodium?","A. 0 L B. 1.8 L C. 2.3 L D. 2.8 L",B,temp nan,Electrolyte Acid-Base Disorders 2627,"You are asked to see a 65-year-old woman with chronic obstructive lung disease and congestive heart failure in consultation for alkalemia. She was admitted with 2 days of increased dyspnea, fever, nausea, vomiting, and cough. Medications are ipratropium, furosemide, and enalapril. On physical examination, she is tachypneic, with a respiratory rate of 24/min. The BP is 90/60 mmHg, the pulse rate is 108/min, and the temperature is 38.3C. The lung examination shows right basilar crackles. There is no edema. A dense right lower infiltrate is seen on a chest radiograph. Laboratory studies show sodium of 140 mEq/L, potassium of 3.4 mEq/L, chloride of 85 mEq/L, total CO2 of 35 mmol/L, BUN of 35 mg/dl, creatinine of 1.2 mg/dl, and glucose of 82 mg/dl. An arterial blood gas on ambient air shows pH 7.45, PaCO2 of 54 mmHg, bicarbonate of 36.5 mEq/L, and PaO2 of 52 mmHg..",Which ONE of the following BEST describes this woman’s acid-base status?,"A. Simple metabolic alkalosis B. Metabolic alkalosis and metabolic acidosis C. Respiratory acidosis and metabolic alkalosis D. Respiratory acidosis, metabolic alkalosis, and metabolic acidosis",D,temp nan,Electrolyte Acid-Base Disorders 2628,"A 20-year-old woman is referred for hypokalemic metabolic alkalosis. Over the past several years, she has noted progressive worsening of polyuria, thirst, salt craving, weakness, and fatigue. She does not have vomiting and is on no medications. On physical examination, the BP is 110/70 mmHg, and the pulse rate is 74/min. The remainder of the examination is normal. Laboratory studies show sodium of 140 mEq/L, potassium of 2.2 mEq/L, chloride of 88 mEq/L, total CO2 of 42 mmol/L, BUN of 12 mg/dl, creatinine of 0.9 mg/dl, glucose of 90 mg/dl, calcium of 9.2 mg/dl, and magnesium of 1.7 mg/dl. The plasma renin activity is 102 ng/ml per hour (reference range 50.6–3.0 ng/ml per hour), and the plasma aldosterone concentration is 56.5 ng/dl (reference range 52–5 ng/dl). Urinary electro-lytes are sodium of 45 mEq/L, potassium of 50 mEq/L, and chloride of 60 mEq/L. The urine calcium-to-creatinine ratio is 280 mg/g..",Which ONE of the following is the MOST likely diagnosis?,"A. Primary hyperaldosteronism B. Gitelman syndrome C. Bartter syndrome D. Surreptitious vomiting E. Liddle syndrome",C,temp nan,Electrolyte Acid-Base Disorders 2629,"A 65-year-old man is evaluated 3 weeks after receiving a deceased donor kidney transplant. He is doing well and asymptomatic. His medications include tacrolimus, mycophenolate mofetil, prednisone, carvedilol, dapsone, and acyclovir. On physical examination, the BP is 160/94 mmHg. The allograft is nontender and without bruit. He has trace pretibial edema. The remainder of the examination is normal. Laboratory studies show sodium of 138 mEq/L, potassium of 5.8 mEq/L, chloride of 109 mEq/L, total CO2 of 18 mmol/L, BUN of 18 mg/dl, creatinine of 1.4 mg/dl, and glucose of 108 mg/dl. A 12-hour tacrolimus trough level is 8 ng/ml. An ultrasound of the kidney transplant shows normal vasculature and no hydronephrosis..","In addition to a low-potassium diet, which ONE of the following is the BEST management of this patient’s hyperkalemia?","A. Reduce tacrolimus to achieve trough levels of 4–5 ng/ml B. Start chlorthalidone C. Start fludrocortisone D. Start sodium bicarbonate",B,temp nan,Electrolyte Acid-Base Disorders 2630,"A 55-year-old woman with stage G4 CKD due to diabetic kidney disease is referred for management of persistent hyperkalemia. She has been adherent with dietary potassium restriction. Her medications include insulin glargine, omeprazole, metoprolol, lisinopril, amlodipine, and furosemide. The physical examination shows a BP of 130/82 mmHg, background diabetic retinopathy, and decreased sensation in the feet. The remainder of the examination is normal. Laboratory studies show sodium of 140 mEq/L, potassium of 5.9 mEq/L, chloride of 109 mEq/L, total CO2 of 22 mmol/L, BUN of 26 mg/dl, creatinine of 4.2 mg/dl, eGFR of 17 ml/min per 1.73 m2, glucose of 138 mg/dl, calcium of 8.9 mg/dl, magnesium of 1.8 mg/dl, and phosphorus of 5.4 mg/dl. You prescribe patiromer calcium..",This patient is at risk for which ONE of the following complications related to treatment with patiromer calcium?,"A. Hypomagnesemia B. Hypercalcemia C. Metabolic alkalosis D. Hypophosphatemia",A,temp nan,Electrolyte Acid-Base Disorders 2631,"A 50-year-old woman is referred for evaluation of new-onset hypertension. Two weeks ago, she noted onset of headache and muscle cramps. She is an avid tea drinker and recently starting drinking an imported tea from Egypt. She has not smoked cigarettes. On physical examination, she is a healthy-appearing woman in no acute distress. The body mass index is 19.5 kg/m2. The BP is 220/100 mmHg, and the pulse rate is 88/min. There are no abdominal or flank bruits, and there is no radiofemoral delay. Laboratory studies show sodium of 146 mEq/L, potassium of 2.9 mEq/L, chloride of 104 mEq/L, total CO2 of 32 mmol/L, BUN of 9 mg/dl, creatinine of 0.6 mg/dl, calcium of 9.2 mg/dl, magnesium of 1.8 mg/dl, and phosphorus of 2.4 mg/dl. Urine electrolytes show potassium of 50 mEq/L, so\dium of 45 mEq/L, and chloride of 40 mEq/L. The plasma renin activity is 0.1 ng/ml per hour (reference range 50.6–3.0 ng/ml per hour), and the plasma aldosterone concentration is < 1.0 ng/dl (reference range 52–5 ng/dl). The plasma deoxycorticosterone level is normal. The 24- hour urine free cortisol is 170.25 microgram/g creatinine (reference range < 24 microgram/g), the 24-hour urine cortisone is 42 microgram/g creatinine (reference range 58–65 microgram/g), and the 24-hour urine cortisol-to-cortisone ratio is 4.0..",Which ONE of the following is the MOST likely cause of this patient’s hypokalemia?,"A. Decreased activity of 11β-hydroxysteroid dehydrogenase B. Liddle syndrome C. Cushing syndrome D. Ectopic adrenocorticotropic hormone syndrome",A,temp nan,Electrolyte Acid-Base Disorders 2632,"An 82-year-old man with stage G5 CKD and hypothyroidism is evaluated during a follow-up visit for hyperkalemia. His medications are amlodipine, furosemide, and levothyroxine. He has decided to forego dialysis in favor of medical management. His serum potassium level is 6.2 mEq/L, despite adherence to a potassium-restricted diet. You prescribe sodium polystyrene sulfonate..",Which ONE of the following would you advise this patient with regard to sodium polystyrene sulfonate therapy?,"A. The frequency of colonic necrosis is approximately 0.1% B. It can be safely coadministered with his other medication C. There is no evidence supporting its use for the treatment of hyperkalemia in CKD D. Use of a sorbitol-free preparation eliminates the risk of colonic necrosis",A,temp nan,Electrolyte Acid-Base Disorders 2633,,"Question 2633: Which ONE of the following MOST likely contributes to the pathogenesis of salt-sensitive hypertension in individuals consuming a low-potassium diet?","A. Increased aldosterone production B. Increased activity of the NCC in the distal convoluted tubule C. Increased activity of the ENaC in the aldosterone-sensitive portion of the distal tubule and collecting duct D. Inhibition of WNK1 in the distal convoluted tubule",B,temp nan,Electrolyte Acid-Base Disorders 2634,"A 24-year-old man is evaluated for weakness and recently documented hypokalemic metabolic alkalosis. He indicates that he has always craved salty foods and had intermittent leg cramps since age 15 years old. Over the past 2 months, he noted onset of diffuse myalgias, polyuria, weakness, and wors-ening leg cramps. He is on no medications and does not have vomiting or diarrhea. On physical examination, his stature is normal. The BP is 118/60 mmHg, and the pulse rate is 88/min. The neuro- logic examination shows mild diffuse weakness. The remainder of the examination is normal. Laboratory studies show sodium of 142 mEq/L, potassium of 3.2 mEq/L, chloride of 102 mEq/L, total CO2 of 30 mmol/L, BUN of 9 mg/dl, creatinine of 0.8 mg/dl, glucose of 87 mg/dl, calcium of 9.6 mg/dl, magnesium of 1.4 mg/dl, and phosphorus of 2.8 mg/dl. Urine electrolytes show potassium of 50 mEq/L, sodium of 63 mEq/L, and chloride of 87 mEq/L. The plasma renin activity is 11 ng/ml per hour (reference range 50.6–3.0 ng/ml per hour), and the plasma aldosterone concentration is 17 ng/dl (reference range 52–5 ng/dl). A urinalysis shows specific gravity of 1.015, pH 7, and no protein or blood. A comprehensive urine diuretic screen is negative..",Which ONE of the following diagnostic tests is MOST appropriate to perform next?,"A. 24-hour urine collection for potassium B. Genetic testing for mutations in the CLCNKB gene C. Urine calcium-to-creatinine ratio D. Fractional chloride clearance in response to hydrochlorothiazide at 50 mg",C,temp nan,Electrolyte Acid-Base Disorders 2635,"A 65-year-old woman is referred for evaluation of hypokalemia and hypomagnesemia. She recently completed concurrent radiochemotherapy for stage 3 nonsmall cell lung cancer. The chemotherapy regimen included cisplatin plus vinblastine. On physical examination, the BP is 110/60 mmHg, and the pulse rate is 96/min. There are decreased breath sounds at the left lung base. There is no edema. The remainder of the examination is normal. Laboratory studies show sodium of 133 mEq/L, potassium of 3.2 mEq/L, chloride of 101 mEq/L, total CO2 of 29 mmol/L, BUN of 32 mg/dl, creatinine of 1.3 mg/dl, calcium of 8.9 mg/dl, magnesium of 1.1 mg/dl, and phosphorus of 3.5 mg/dl. The urine sodium is 72 mEq/L, the urine chloride is 56 mEq/L, and the urine potassium is 65 mEq/L. The urine calcium-to-creatinine ratio is 26 mg/g..",Dysfunction of which ONE of the following nephron segments is contributing the MOST to this woman’s hypomagnesemia?,"A. The thick ascending limb of the loop of Henle B. The medullary collecting duct C. The descending limb of the loop of Henle D. The early distal convoluted tubule",D,temp nan,Electrolyte Acid-Base Disorders 2636,"You are asked to see a 60-year-old woman with transfusion-dependent myelodysplastic syndrome hospitalized for weakness and newly documented hypokalemic metabolic acidosis. Her medications are ruxolitinib and deferasirox. The physical ex-amination shows a normal BP and pulse, pallor, no edema, and mild diffuse weakness. Laboratory studies show sodium of 138 mEq/L, potassium of 3.4 mEq/L, chloride of 109 mEq/L, total CO2 of 19 mmol/L, BUN of 18 mg/dl, creatinine of 1.3 mg/dl, glucose of 86 mg/dl, calcium of 8.0 mg/dl, magnesium of 1.6 mg/dl, phosphorus of 1.1 mg/dl, albumin of 3.8 g/dl, total protein of 6.6 g/dl, uric acid of 2.8 mg/dl, leukocyte count of 2500/ml, hemoglobin of 6.3 g/dl, and platelet count of 32,000/ml. A urinalysis shows pH 5.0 and trace protein. A venous blood gas shows pH 7.32 and PvCO2 of 36 mmHg. The daily urine ammonium excretion is 40 mmol. The urinary phosphate level is 25 mg/dl, and the urine creatinine level is 100 mg/dl. The urine protein-to-creatinine ratio is 400 mg/g, and the albumin-to-creatinine ratio is 30 mg/g. A urine electrophoresis shows low-molecular weight proteinuria. No monoclonal spike is present on urine and serum electrophoresis..",Which ONE of the following is the MOST likely diagnosis?,"A. Hypokalemic distal renal tubule acidosis (type 1) B. Tumor-induced osteomalacia C. Spurious hypophosphatemia D. Bicarbonate loss through the gastrointestinal tract E. Proximal renal tubular acidosis",E,temp nan,Electrolyte Acid-Base Disorders 2637,"A 38-year-old man is seen in consultation for recurrent kidney stones. He has had symptomatic nephrolithiasis, averaging one stone per year, since age 25 years. Analysis of a recently extracted stone shows that is composed of calcium phosphate. Laboratory studies show sodium of 138 mEq/L, potassium of 4.0 mEq/L, chloride of 108 mEq/L, total CO2 of 20 mmol/L, BUN of 13 mg/dl, creatinine of 1.2 mg/dl, calcium of 9.1 mg/dl, and phosphorus of 3.2 mg/dl. A urinalysis shows no protein, glucose, or blood. A 24-hour urine collection shows volume of 2.12 L, pH 6.1, calcium of 124 mg (target level < 250 mg), oxalate of 33 mg (target level 20–40 mg), citrate of 102 mg (target level > 450 mg), uric acid of 392 mg (target level < 800 mg), sodium of 122 mEq (reference range 50–150 mEq), ammonium of 28 mmol (reference range 15–60 mmol), and phosphate of 358 mg (reference range 600–1200 mg)..",Which ONE of the following is the MOST likely diagnosis?,"A. Type 4 renal tubular acidosis B. Proximal renal tubular acidosis (type 2) C. Dent disease D. Incomplete distal renal tubular acidosis (type 1)",D,temp nan,Electrolyte Acid-Base Disorders 2638,,"Question 2638: Which ONE of the following patients is at an increased risk for profound metabolic alkalosis and hypovolemia in response to hydrochlorothiazide?","A. A 20-year-old man with glucocorticoid-remediable aldosteronism (GRA) B. A 23-year-old man with Liddle syndrome C. An 18-year-old woman with sensorineural hearing loss and goiter D. A 74-year-old woman with hypertension on no additional medications",C,temp nan,Electrolyte Acid-Base Disorders 2639,"A 24-year-old woman with cystic fibrosis is hospitalized for treatment of Pseudomonas pneumonia. Her hospital course is complicated by ileus requiring nasogastric suction. Twelve days into her hospital course, she develops paresthesias and diffuse muscle weakness. Her medications are pancrelipase, fat-soluble vitamins, calcium carbonate, insulin as part, high-dose ceftazidime, and gentamicin. On physical examination, the BP is 110/80 mmHg, and the pulse rate is 102/min. The lung examination shows inspiratory crackles at the right base. There is no edema. Laboratory studies show sodium of 138 mEq/L, potassium of 2.3 mEq/L, chloride of 93 mEq/L, total CO2 of 34 mmol/L, BUN of 39 mg/dl, creatinine of 1.3 mg/dl, calcium of 8.4 mg/dl, magnesium of 1.2 mg/dl, and phosphorus of 3.2 mg/dl. A venous blood gas shows pH 7.43 and PvCO2 of 53 mmHg. The urine sodium is 78 mEq/L, the urine chloride is 68 mEq/L, and the urine potassium is 52 mEq/L..",Which ONE of the following is the MOST likely cause of this woman’s acid-base abnormality?,"A. Nasogastric suction B. Gentamicin C. Posthypercapnic alkalosis D. Sodium chloride loss in sweat E. Ceftazidime",B,temp nan,Electrolyte Acid-Base Disorders 2640,"A 33-year-old woman with Sjogren syndrome is referred for evaluation of recurrent calcium phosphate kidney stones. She has had three episodes of symptomatic nephrolithiasis over the past 2 years, the most recent of which was 5 days ago. She passed a 3-mm stone after several hours of symptoms, and chemical analysis confirm that it was com-posed of calcium phosphate. Serum studies show sodium of 139 mEq/L, potassium of 3.8 mEq/L, chloride of 106 mEq/L, total CO2 of 21 mmol/L, BUN of 16 mg/dl, creatinine of 1.1 mg/dl, calcium of 9.3 mg/dl, and phosphorus of 2.8 mg/dl. The urine pH is 6.1. A 24-hour urine collection is significant for hypocitraturia (citrate of 158 mg; target level > 450 mg). The daily urine ammonium excretion is 32 mmol (reference range 515–60 mmol/d). A kidney ultrasound shows mild medullary nephrocalcinosis..",Which ONE of the following diagnostic tests is MOST likely to establish the cause of recurrent kidney stone formation in this woman?,"A. Ammonium chloride challenge test B. Fractional excretion of bicarbonate C. Repeat evaluation of urinary parameters in 2 weeks D. Urine osmolal gap",A,temp nan,Electrolyte Acid-Base Disorders 2641,"A 25-year-old man with a history of type 1 diabetes mellitus is admitted for management of persistent diarrhea and weakness 3 months after undergoing living related kidney transplantation. Admission laboratory studies disclose significant hyperkalemia, with a serum potassium level of 6.5 mEq/L. His medications include insulin as part, tacrolimus, mycophenolate mofetil, trimethoprim-sulfamethoxazole at 400/80 mg three times weekly, valganciclovir, and metoprolol. On physical examination, he appears fatigued, but he is alert and oriented. The BP is 130/64 mmHg supine and 96/50 mmHg standing. The pulse rate increases from 80/min supine to 110/min standing. The abdominal examination shows normal bowel sounds and no peritoneal signs. The allograft is nontender and does not have a bruit. There is no edema. Laboratory studies show sodium of 142 mEq/L, potassium of 6.5 mEq/L, chloride of 115 mEq/L, total CO2 of 18 mmol/L, BUN of 40 mg/dl, creatinine of 1.4 mg/dl (increased from 1.2 mg/dl 1 week ago), glucose of 270 mg/d, creatinine kinase of 31 U/L, leukocyte count of 8000/µl, hemoglobin of 9.2 g/dl, and platelet count of 119,000/µl. An arterial blood gas shows pH 7.32, PaCO2 of 32 mmHg, and bicarbonate of 18 mEq/L. The tacrolimus trough is increased at 12 ng/ml. The urinalysis shows specific gravity of 1.015, 1+ glucose, and no protein, blood, or ketones. An electrocardiogram shows peaked T waves and a prolonged PR interval. A renal trans-plant ultrasound is normal..","In addition to therapy with intravenous insulin and calcium, which ONE of the following is the next BEST step in the management of this man’s hyperkalemia?","A. Fludrocortisone B. Patiromer C. Isotonic sodium bicarbonate infusion D. Hemodialysis",C,temp nan,Electrolyte Acid-Base Disorders 2642,"You are asked to evaluate a 66-year-oldwoman1 day after admission for management of acute decompensated heart failure for a rising serum creatinine level and persistent hypervolemia. She initially is treated with bumetanide at 3 mg intravenously every 12 hours. Over the initial 24 hours of hospitalization, the urine volume is 3 L. On physical examination, she remains in mild respiratory distress, with a respiratory rate of 24/min. The oxygen saturation is 98% on 2 L/min supplemental oxygen. The BP is 110/60 mmHg, and the pulse rate is 106/min. The jugular venous pressure is 10 cm H2O. The lung examination shows bibasilar crackles. The estimated sodium output from each dose of bumetanide is estimated to be 140 mEq using a spot urine estimation equation. Laboratory studies show sodium of 132 mEq/L, potassium of 3.8 mEq/L, chloride of 89 mEq/L, total CO2 of 30 mmol/L, BUN of 52 mg/dl (increased from 40 mg/dl on admission), creatinine of 2.2 mg/dl (increased from 1.9 mg/dl on admission), and glucose of 67 mg/dl. A urinalysis shows specific gravity of 1.012, pH 5.4, no protein, and no blood. The urinary sediment shows many hyaline casts..","In addition to ongoing close follow-up of this woman’s kidney function, which ONE of the following is the MOST appropriate management?","A. Begin isolated ultrafiltration B. Redose intravenous bumetanide only after the serum creatinine is less than or equal to 2.0 mg/dl C. Decrease intravenous bumetanide to once daily D. Continue current regimen",D,temp nan,Electrolyte Acid-Base Disorders 5444," A 72-year-old man with hepatitis C–related cirrhosis complicated by ascites and grade 2 esophageal varices is admitted for management of increasing ascites and edema. He had been adherent with escalating doses of furosemide and spironolactone. A paracentesis shows no evidence of spontaneous bacterial peritonitis. He is initially treated with intravenous diuretics, but subsequently develops AKI with a rise in the serum creatinine (SCr) level from 0.8 mg/dl on admission to 1.8 mg/dl 3 days later. The kidney function continues to worsen, and oliguria ensues over the next 2 days despite discontinuation of diuretics and treatment with albumin 1 g/kg per day. On physical examination, the BP is 100/65 mmHg. He has modest ascites and 2+ leg edema. Laboratory studies now show serum creatinine of 3.4 mg/dl, albumin 2.6 g/dl, and bilirubin 4.5 mg/dl. The fractional excretion of sodium is 0.05%. The urinalysis is negative for protein, and the urinary sediment shows no casts or cellular elements. An abdominal ultrasound shows moderate ascites. The kidney size and echogenicity are normal. There is no hydronephrosis..",Which ONE of the following is the MOST likely diagnosis?,"1. Abdominal compartment syndrome 2. Acute tubular necrosis 3. Prerenal azotemia 4. Hepatorenal síndrome",D,temp nan,Acute Kidney Injury & Critical Care Nephrology 5445," Hepatology consults you to consider initiation of albumin dialysis for a 35-year-old woman with acute hepatic failure due to acetaminophen toxicity. Her course has been complicated by oliguric AKI due to hepatorenal syndrome and hepatic encephalopathy refractory to medical therapy. On physical examination, she is disoriented and lethargic. The BP is 96/60 mmHg. The sclerae are icteric. There are moderate ascites and 1+ leg edema. Serum studies show that the serum creatinine level is 2.8 mg/dl, the albumin is 1.8 g/dl, the bilirubin is 23 mg/dl, and the ammonia is 244 mmol/L (reference range 9–33)..",Which ONE of the following is MOST likely to result from implementation of albumin dialysis compared with standard medical therapy for this patient?,"1. Lower serum ammonia level 2. Improved mental status 3. Decreased inpatient mortality risk 4. Increased risk of gastrointestinal bleeding",B,temp nan,Acute Kidney Injury & Critical Care Nephrology 5446," A 42-year-old woman with alcoholic cirrhosis is admitted to the general medical ward for management of refractory ascites. Her course is complicated by oliguric AKI due to hepatorenal syndrome. She has intermittent encephalopathy that has improved with lactulose therapy. On physical examination, she is currently well oriented. The BP is 96/ 58 mmHg (mean arterial pressure [MAP] 71 mmHg). She has moderate ascites and 21 leg edema. Laboratory studies show sodium 132 mEq/L, potassium 3.6 mEq/L, chloride 100 mEq/L, total carbon dioxide (CO2) 20 mmol/L, BUN 45 mg/dl, and creatinine 2.1 mg/dl (increased from a recent baseline level of 1.1 mg/dl)..","In addition to intravenous albumin, which ONE of the following is the next MOST appropriate step in management?","1. Norepinephrine 2. Midodrine plus octreotide or terlipressin alone 3. Dopamine 4. Transjugular intrahepatic portosystemic shunt",B,temp nan,Acute Kidney Injury & Critical Care Nephrology 5447," The chief quality officer at your institution asks for your advice about interventions to reduce the risk of progression of aminoglycoside-induced nephrotoxicity to severe AKI..",Which ONE of the following interventions would you recommend as MOST likely to decrease the risk of progression to severe AKI in patients receiving aminoglycosides?,"1. Electronic alerts that identify patients who develop stage 1 AKI 2. Daily measurement of aminoglycoside levels 3. Preferential use of a three times daily dosing regimen 4. Daily serum creatinine measurements",D,temp nan,Acute Kidney Injury & Critical Care Nephrology 5448," You are asked to see a 63-year-old man in consultation for AKI 2 days after a combined two-vessel coronary artery bypass and mitral valve replacement. His urine volume over the past 12 hours has decreased to 250 ml, and the serum creatinine level has increased from a baseline of 1.5 mg/dl (2 weeks preoperatively) to 2.2 mg/dl. He was easily extubated postoperatively but remains on 2–3 L/min supplemental oxygen via nasal cannula. His medications before surgery included furosemide 80 g twice daily. On physical examination, he is alert and is not in distress. The BP is 110/78 mmHg. There are decreased breath sounds at the lung bases and trace pretibial edema. A urinalysis shows 21 protein and several muddy brown casts..",Which ONE of the following is MOST likely to predict the development of stage 3 AKI and the impending need for renal replacement therapy in this man?,"1. A fractional excretion of urea > 50% 2. A spot urine albumin-to-creatinine ratio > 30 mg/g 3. Urine volume < 200 ml over 2 hours after 1.5 mg/kg of intravenous furosemide 4. A urine neutrophil gelatinase–associated lipocalin level > 150 ng/ml",C,temp nan,Acute Kidney Injury & Critical Care Nephrology 5450," A 54-year-old man hospitalized in the intensive care unit (ICU) for the last 4 days in the setting of documented fungemia and septic shock following recent autologous stem cell transplant is seen in consultation for AKI. The MAPs are in the 60- to 65-mmHg range with support from norepinephrine and vasopressin infusions. He is oliguric, with a urine volume of only 150 ml/d for the last 2 days. Review of his medical record shows that his fluid balance is 7 L positive over his ICU stay and 9.5 L positive for his hospital stay. On physical examination, he is intubated and sedated. The lung bases show crackles, and there is 3+ leg and presacral edema. Laboratory studies show sodium 132 mEq/L, potassium 6.1 mEq/L, chloride 96 mEq/L, total CO2 15 mmol/L, BUN 86 mg/dl, and creatinine 6.5 mg/dl (increased from 1.2 mg/dl 10 days ago). The leukocyte count is 2100/µl (with an absolute neutrophil count of 1200/µl), the hemoglobin is 7.3 g/dl, and the platelet count is 102,000/µl. You decide to initiate him on continuous renal replacement therapy for his kidney dysfunction, volume overload, and electrolyte abnormalities..",Which ONE of the following anticoagulation strategies is MOST likely to yield the longest filter life and the lowest risk of bleeding complications for this patient?,"1. Regional citrate anticoagulation 2. Unfractionated heparin-infused prefilter 3. No anticoagulation 4. Regional unfractionated heparin-protamine",D,temp nan,Acute Kidney Injury & Critical Care Nephrology 5451," A 32-year-old kidney transplant recipient is evaluated in the emergency department for cough, fever, and hypotension. On physical examination, she is in moderate distress. She is febrile to 39.0 C. The BP is 95/60 mmHg, and the heart rate is 110–120/min. She has decreased lung sounds localized to the left base with a corresponding area of egophony. There is no peripheral edema. A urinary catheter is placed, and the initial urine output is 0.5 ml/kg per hour. Mycophenolate mofetil is held, and she is started on “stress dose” corticosteroids. Laboratory studies show sodium 138 mEq/L, potassium 3.9 mEq/L, chloride 98 mEq/L, total CO2 18 mmol/L, BUN 18 mg/dl, creatinine 1.1 mg/dl (increased from 0.9 mg/dl 2 weeks ago), leukocyte count 17,600/µl, hemoglobin 8.2 g/dl, and platelet count 293,000/µl. A chest radiograph shows a dense left lower lobe infiltrate..","In addition to an appropriately targeted anti-biotic regimen, which ONE of the following is the MOST appropriate initial management strategy for this patient?","1. Intravenous isotonic crystalloid and/or albumin to achieve a goal MAP of >70 mmHg 2. Intravenous vasopressors to achieve a goal MAP > 70 mmHg 3. Transfusion with 2 U packed red blood cells targeting a central venous oxygen saturation of > 70% 4. Intravenous isotonic crystalloid solution and/or albumin tailored to her hemodynamic status with addition of vasopressors if unresponsive to fluids/albumin",D,temp nan,Acute Kidney Injury & Critical Care Nephrology 5452," A 75-year-old man with type 2 diabetes mellitus is admitted to the ICU for management of community- acquired pneumonia. His course is complicated by septic shock and acute respiratory distress syndrome. He requires mechanical ventilator support and vasopressor infusions to maintain the MAP > 65 mmHg. Serial laboratory studies reveal that the hemoglobin level has slowly drifted down from 9.8 to 7.9 g/dl. He does not have evidence of active bleeding..",Which ONE of the following is the MOST likely outcome when using a transfusion threshold of hemoglobin less than or equal to 9 g/dl versus a hemoglobin less than or equal to 7 g/dl in this patient?,"1. Improved survival 2. Decreased risk of AKI 3. Decreased duration of mechanical intubation 4. Increased risk for acute lung injury 5. Decreased requirement for vasoactive drugs",D,temp nan,Acute Kidney Injury & Critical Care Nephrology 5453," A 72-year-old woman is hospitalized for treatment of urosepsis. She requires resuscitation with 6 L fluids over the initial 48 hours of hospitalization..","On the basis of published retrospective cohort studies, resuscitation with which ONE of the following types of intravenous fluids is LEAST likely to be associated with the development of severe AKI in this patient?","1. 0.9% Saline 2. Isotonic balanced electrolyte solution 3. 5% dextrose in water (D5W) with sodium bicarbonate at 100 mEq/L 4. Hydroxyethyl starch",B,temp nan,Acute Kidney Injury & Critical Care Nephrology 5454," A 56-year-old man with a history of head and neck cancer currently undergoing chemotherapy is admitted to the ICU with presumed sepsis. He presented with a fever to 38.9C and purulent sputum emanating from his tracheostomy tube. Despite 3 L intravenous fluid resuscitation with lactated ringers, he remains hypotensive with blood pressures of 85/60 and a heart rate of 110–120/min. He is supported with mechanical ventilation using the assist control mode with a fraction of inspired oxygen of 0.4 with 5 mmHg of positive end expiratory pressure. Given his persistent hypotension, the decision is made to start him on a vasoactive medication. His care team debates the relative merits of norepinephrine versus vasopressin..",Which ONE of the following outcomes is MOST likely to be associated with use of nor-epinephrine versus vasopressin in this patient?,"1. Increased risk of AKI 2. Decreased number of AKI-free days 3. Increased risk of RRT 4. Decreased mortality risk",C,temp nan,Acute Kidney Injury & Critical Care Nephrology 5455,,"Question 5455 In which ONE of the following clinical scenarios is remote ischemic preconditioning MOST likely to be useful in preventing the development of AKI?","1. Coronary artery bypass surgery 2. Open abdominal aortic aneurysm repair 3. Radiocontrast exposure 4. Aortic valve replacement surgery",C,temp nan,Acute Kidney Injury & Critical Care Nephrology 5456," A 67-year-old woman is hospitalized for the treatment of pneumonia complicated by sepsis and stage 1 AKI. She is enrolled in a clinical trial of intravenous alkaline phosphatase therapy for patients with sepsis..",Which ONE of the following outcomes would MOST likely be associated with treatment with intravenous alkaline phosphatase in this patient on the basis of currently available published literature?,"1. Decreased rate of progression to stage 2 AKI 2. Higher levels of urinary kidney injury marker-1 3. Higher levels of C-reactive protein 4. Decreased mortality",A,temp nan,Acute Kidney Injury & Critical Care Nephrology 5457," You are asked to see a 68-year-old man with pneumonia admitted to the ICU with respiratory failure for persistent oliguria. Over the past 24 hours, the urine output has decreased to 480 ml (0.3 ml/kg per hour), despite prompt initiation of appropriate antibiotics. On physical examination, he is intubated and sedated. The temperature is 38.6C, the BP is 105/85 mm Hg, and the pulse is 110/min. The central venous pressure is 7 cm H2O. Crackles are present at the right lung base. There is no edema. Mechanical ventilation is currently provided with 7cmH2O of positive end expiratory pressure, a tidal volume of 6 ml/kg, and an a fraction of inspired oxygen of 0.5. Laboratory studies show sodium 139 mEq/L, potassium 4.3 mEq/L, chloride 103 mEq/L, total CO2 20 mmol/L, BUN 26 mg/dl, and creatinine 1.4 mg/dl (increased from 1.1 mg/dl 2 days ago)..",Which ONE of the following is the MOST appropriate next step in management on the basis of the Fluid and Catheter Treatment Trial Lite Fluid Management Protocol?,"1. Give a fluid bolus and reassess in 1 hour 2. Give dopamine 1 µg/kg per minute 3. Give intravenous furosemide 20 mg and reassess in 4 hours 4. No intervention and reassess in 4 hours",A,temp nan,Acute Kidney Injury & Critical Care Nephrology 5458," A 55-year-old woman with advanced AIDS is admitted for management of sepsis due to biliary tract obstruction and acute cholangitis. On physical examination, she is jaundiced and febrile to 38.4C. The MAP is persistently < 65 mmHg despite appropriate administration of fluid boluses. There are crackles present in the left lower lung zone. There is no edema. Laboratory studies show sodium 130 mEq/L, potassium 3.2 mEq/L, chloride 96 mEq/L, total CO2 14 mmol/L, BUN 10 mg/dl, creatinine 1.3 mg/dl (unchanged from 10 days ago), total bilirubin 8.6 mg/dl (direct 8.3 mg/dl), alkaline phosphatase 834 mg/dl (reference range 538–126 U/L), alanine aminotransferase 186 U/L (reference range 50–35), and aspartate aminotransferase 152 U/L (reference range 50–35). The leukocyte count is 1500/µl, the hemoglobin is 8.6 g/dl, and the platelet count is 127,000/µl. A urinalysis shows trace protein, 3+ bilirubinuria, and many hyaline casts. The urine albumin-to-creatinine ratio is 43mg/g (urine albumin 56mg/dl). The intensivist asks you about whether a urine tissue inhibitor of metalloproteinase-2 X IGF binding protein-7 would be helpful in predicting this woman’s risk for the development of AKI..",Which ONE of the following findings decreases the utility of measuring urine tissue inhibitor of metalloproteinase-2 X IGF binding protein-7 product in the early detection of AKI in this woman?,"1. Her age 2. Her bilirubinuria 3. Her critical illness status 4. Her albuminuria",B,temp nan,Acute Kidney Injury & Critical Care Nephrology 5459," A 65-year-old woman is evaluated for the acute onset of oliguria following cardiac bypass surgery. She has a baseline serum creatinine (SCr) of 1.0 mg/dl and a urine albumin-to-creatinine ratio of 28 mg/g. On postoperative day 1, she develops hypotension requiring vasopressor support. The urine output is recorded to be < 0.1 ml/kg per hour for a consecutive period of 3 hours before improving to 0.4 ml/kg per hour over the next 3 hours. The a serum creatinine on postoperative day 2 is 1.2 mg/dl..",Which ONE of her clinical parameters is MOST highly associated with an increased risk for mortality?,"1. Severe oliguria lasting over 3 hours 2. Her baseline SCr level 3. Her baseline level of albuminuria 4. Her SCr level on postoperative day 2",A,temp nan,Acute Kidney Injury & Critical Care Nephrology 5460," You are asked to comment on the risk of peri-operative AKI in a 70-year-old black man who is scheduled for nontraumatic hip arthroplasty. He has stage G3b:A1 CKD, diabetes mellitus, and well compensated congestive heart failure. He does not have chest pain on exertion, short-ness of breath, or orthopnea. On physical examination, the BP is 132/82 mmHg, and the heart rate is 62/min. The jugular venous pressure is < 5cm H2O. The lungs are clear. The cardiac examination shows an S4 gallop. There is no edema. The remainder of the examination is normal. Laboratory studies show creatinine of 1.8 mg/dl (stable) and an eGFR of 43ml/min per 1.73m2..",Which ONE of the following is the STRONGEST predictor of AKI in nontraumatic hip arthroplasty in this man?,"1. Black race 2. Diabetes mellitus 3. History of congestive heart failure 4. Baseline CKD",D,temp nan,Acute Kidney Injury & Critical Care Nephrology 5461," A 65-year-old man with stage G3a:A1 CKD, type 2 diabetes mellitus, and chronic obstructive pulmonary disease is seen for acute bronchitis and hypertension. Review of his home BPs over the past 2 months shows an average BP of 156/94 mmHg. His medications are irbesartan 300 mg daily, chlorthalidone 25 g daily, pravastatin 40 mg daily, and metformin 500 mg twice daily. On physical examination, he is not in acute respiratory distress. The BP is 160/90 mmHg. The lung examination shows diffuse rhonchi. There is no edema. The remainder of the examination is normal. Laboratory studies show a stable serum creatinine of 1.4 mg/dl (eGFR 51 ml/min per 1.73 m2). You decide to add a dihydropyridine calcium blocker to address the hypertension and a macrolide antibiotic to treat the pulmonary infection..",Prescription of which ONE of the following medication combinations is associated with the GREATEST risk for hospitalization with AKI over the next month?,"1. Amlodipine and azithromycin 2. Amlodipine and clarithromycin 3. Nifedipine and azithromycin 4. Nifedipine and clarithromycin",D,temp nan,Acute Kidney Injury & Critical Care Nephrology 5462," A 70-year-old woman who is kidney transplant recipient is admitted for management of urosepsis. She is initially treated with ceftriaxone but continues to have recurrent high fevers 1 day after admission. On physical examination, she appears toxic. The temperature is 39.2 C, the BP is 110/78 mmHg, and the heart rate is 100/min. There is minimal allograft tenderness. The remainder of the examination is normal. Laboratory studies show that the serum creatinine is 1.2 mg/dl and unchanged from her baseline level. A urinalysis shows large leukocyte esterase and 100 leukocytes per high-power field. No bacteria are seen. Blood and urine cultures show no growth so far. You recommend addition of vancomycin..",Her internist raises concerns about potential nephrotoxicity.,"1. Vancomycin-associated AKI only occurs when co-administered with piperacillin-tazobactam 2. The risk of AKI is not increased unless vancomycin levels exceed the therapeutic range for > 3 days 3. She is not at risk for vancomycin-associated AKI, because her baseline renal function is normal 4. Her risk of AKI is twofold higher than if she were not treated with vancomycin",D,temp nan,Acute Kidney Injury & Critical Care Nephrology 5463," A 34-year-old combat victim is flown to a military hospital after sustaining multiple gunshot wounds to the head, chest, and abdomen..",Which ONE of the following places this trauma victim at increased risk for rhabdomyolysis?,"1. Chest injury 2. Head injury 3. Abdominal injury 4. Gunshot versus explosive mechanism injury",C,temp nan,Acute Kidney Injury & Critical Care Nephrology 5464," A 77-year-old man with stage G3a CKD is evaluated in the office after being diagnosed with Helicobacter pylori several days ago. He has hypertension, type 2 diabetes mellitus, and coronary heart disease. Medications are metformin, carvedilol, lisinopril, chlorthalidone, and pravastatin. His gastroenterologist has recommended treatment with clarithromycin, amoxicillin, and esomeprazole..",Which ONE of his medications should not be coprescribed with clarithromycin to avoid an increased risk of AKI?,"1. Lisinopril 2. Pravastatin 3. Carvedilol 4. Metformin",B,temp nan,Acute Kidney Injury & Critical Care Nephrology 5465," A 76-year-old man is admitted for management of acute decompensated heart failure. Furosemide is administered by continuous infusion at a dose of 10 mg/h. In addition, he is continued on his home regimen of lisinopril and metoprolol. On day 3 of treatment, he has achieved a net negative output of 12 L, but his BUN and creatinine have risen from20 mg/dl and1.4 mg/dlto65mg/dl and 3.0 mg/dl, respectively..",Which ONE of the following statements BEST describes his renal status?,"1. Stage 1 AKI 2. Stage 2 AKI 3. Stage 3 AKI 4. Type 5 cardiorenal syndrome",B,temp nan,Acute Kidney Injury & Critical Care Nephrology 5466," A 60-year-old man is admitted to the neurosurgical ICU after falling and sustaining a traumatic subdural hematoma. He is evaluated with two contrasted computed tomography scans over the first 24 hours of admission. On hospital day 3, the urine output decreases to 0.25 ml/kg per hour. He is undergoing continuous monitoring of his intracranial pressure, which remains under tenuous control. Laboratory studies show sodium 135 mEq/L, potassium 6.5 mEq/L, chloride 101 mEq/L, total CO2 19 mmol/L, BUN 53 mg/dl, and creatinine 2.8 mg/dl (increased from 1.4 mg/dl on admission). A urinalysis shows three to five muddy brown casts per high-power field. The fractional excretion of sodium is 0.9%. You recommend initiation of renal replacement therapy..",Which ONE of the following is the BEST choice of renal replacement therapy for this man on the basis of the most recent Kidney Disease Improving Global Outcomes guidelines?,"1. Intermittent hemodialysis (4 hours 3 d/week) 2. Continuous RRT (24 h/day) 3. Intermittent hemodiafiltration (4 hours 3 d/week) 4. Prolonged intermittent RRT (8 h/day)",B,temp nan,Acute Kidney Injury & Critical Care Nephrology 5467," A 47-year-old woman with advanced heart failure awaiting heart transplantation is seen in consultation for persistent kidney dysfunction. Her cardiologist has recommended placement of a ventricular assist device (VAD) before transplant because of refractory heart failure despite optimal pharmacologic and device treatment. Her eGFR has been stable at 30 ml/min per 1.73 m2 over the past 2 weeks. She asks about the effect of VAD placement on her renal prognosis..",Which ONE of the following would you advise regarding the expected renal outcomes of VAD placement?,"1. She is likely to experience at least a transient improvement in kidney function 2. If her eGFR increases to > 60 ml/min per 1.73 m2, her 1-year survival following transplant is likely to be worse than that in those with normal pre-VAD eGFR 3. Her risk of post-VAD AKI is equivalent to that in those with normal pre-VAD kidney function 4. She can expect a sustained increase in eGFR of > 50 ml/min per 1.73 m2 above the pre-VAD eGFR at 1 year while awaiting transplant",A,temp nan,Acute Kidney Injury & Critical Care Nephrology 5468,,"Question 5468 Which ONE of the following statements BEST characterizes the relationship between AKI and long-term risk of CKD?","1. The risk of CKD after AKI is similar across individuals with varying levels of baseline kidney function 2. Advanced CKD prior to AKI attenuates the effect of AKI on the long-term risk of progressive CKD 3. The level of CKD prior to AKI has no prognostic effect on post-AKI risk of CKD 4. The severity of AKI and not the level of baseline CKD is the major determinant of long-term risk of CKD",B,temp nan,Acute Kidney Injury & Critical Care Nephrology 5469," A 52-year-old man with recently diagnosed cirrhosis secondary to hepatitis C is evaluated in 1 week following a hospitalization for increased ascites, spontaneous bacterial peritonitis, and transient AKI. The SCr rose to 2.3 mg/dl but is now 1.1 mg/dl. His medications are spironolactone, furosemide, ranitidine, and lactulose. On physical examination, the BP is 112/76 mmHg, and the heart rate is 76/min. The abdomen is soft with minimal ascites. There is no leg edema..",Addition of which ONE of the following would you recommend to decrease this man’s long-term risk of AKI and hepatorenal syndrome?,"1. Norfloxacin or rifaximin 2. Carvedilol 3. Pentoxifylline 4. Midodrine",A,temp nan,Acute Kidney Injury & Critical Care Nephrology 5470," A 52-year-old man with metastatic melanoma, gout, and gastroesophageal reflux disease is seen in consultation for AKI complicating nivolumab therapy. He has had mild diarrhea but has been eating and drinking without difficulty. His medications include allopurinol and omeprazole. On physical examination, his BP is 136/92 mmHg, and the heart rate is 85/min without postural changes. There is no rash. The remainder of the examination is normal. Laboratory studies show sodium 138 mEq/L, potassium 5.2 mEq/L, chloride 107 mEq/L, total CO2 20 mmol/L, BUN 32 mg/dl, and creatinine 1.9 mg/dl (increased from 0.9 mg/dl 1 week ago). The leukocyte count is 4900/µl, the hemoglobin is 12.2 g/dl, and the platelet count is 247,000/µl. A kidney ultrasound shows normal-sized kidneys and no hydronephrosis. A urinalysis shows trace protein, 15–20 leukocytes per high-power field, and occasional leukocyte casts..","In addition to holding nivolumab, which ONE of the following is the MOST appropriate initial treatment?","1. No further treatment 2. Prednisone 0.5–1 mg/kg per day 3. Plasma exchange 4. Rituximab",B,temp nan,Acute Kidney Injury & Critical Care Nephrology 5471," An 80-year-old man is hospitalized for AKI. His family noted onset of increased confusion 2 days ago. He had been eating and drinking without difficulty until 1 day before admission. His medications are allopurinol, pantoprazole, amlodipine, and atorvastatin. On physical examination, he is somnolent, and oriented only to self. The BP is 160/94 mmHg, and the heart rate is 65/min. The jugular venous pressure is increased to 8 cm H2O. Examination of the chest shows bibasilar crackles. There is moderate tenderness in the suprapubic region, but there are no peritoneal signs. There is 2+ pretibial edema. The remainder of the examination is unremarkable. Laboratory studies show sodium 137 mEq/L, potassium 5.8 mEq/L, chloride 109 mEq/L, total CO2 16 mmol/L, BUN 28 mg/dl, and creatinine 1.6 mg/dl (increased from a recent baseline value of 0.7 mg/dl). The leukocyte count is 7800/µl, the hemoglobin is 11.9 g/dl, and the platelet count is 289,000/µl. A urinalysis shows trace protein, negative blood, one to three monomorphic erythrocytes per high-power field, and three to five leukocytes per high-power field..",Which ONE of the following diagnostic tests is MOST likely to influence management in this man?,"1. Serum creatine kinase 2. Urine eosinophils 3. Kidney ultrasound 4. Serum antineutrophil cytoplasmic antibody",C,temp nan,Acute Kidney Injury & Critical Care Nephrology 5472," A 20-year-old man is seen in consultation for AKI. Over the past 4 days, he has been smoking illicit “cigarettes” that he purchased at a party. He subsequently developed intermittent nausea, vomit-ing, and bilateral flank pain. On physical examination, the BP is 132/80 mmHg, and the heart rate is 82/min. The remainder of the examination is normal. Laboratory studies show sodium 142 mEq/L, potassium 5.4 mEq/L, chloride 110 mEq/L, total CO2 19 mmol/L, BUN 38 mg/dl, creatinine 3.7 mg/dl, calcium 9.2 mg/dl, magnesium 1.8 mg/dl, phosphorus 4.8 mg/dl, creatine kinase 156 U/L (reference range 570–185 U/L), leukocyte count 7900/µl, hemoglobin 13.3 g/dl, and platelets 289,000/µl. The fractional excretion of sodium is 6%. A urinalysis shows 1+ protein, negative blood, four to six renal tubular epithelial cells per high-power field, and occasional muddy brown casts. No crystals are seen. A kidney ultrasound shows normal-sized kidneys with increased echogenicity..",Which ONE of the following is the MOST likely cause of his AKI?,"1. Synthetic cannabinoids 2. Heroin 3. Levamisole-adulterated cocaine 4. Crystal methamphetamine",A,temp nan,Acute Kidney Injury & Critical Care Nephrology 5473," A 56-year-old woman with IgA lambda multiple myeloma undergoing treatment with cyclophosphamide, carfilzomib, and dexamethasone is seen in consultation for AKI. One year ago, she received an autologous stem cell transplant after treatment with melphalan. She was subsequently maintained on bortezomib and lenalidomide but transitioned to cyclophosphamide, carfilzomib, and dexamethasone 2 weeks ago after she was found to have a relapse of myeloma. Over the past 10 days, she noted onset of fatigue, nausea, anorexia, and dyspnea on exertion. On physical examination, there is pallor. The BP is 146/85 mmHg, and the heart rate is 98/min. Examination of the skin shows scattered ecchymoses and petechiae. The remainder of the examination is normal. Laboratory studies show sodium 136 mEq/L, potassium 5.7 mEq/L, chloride 108 mEq/L, total CO2 18 mmol/L, BUN 48 mg/dl, creatinine 2.6 mg/dl, calcium 8.6 mg/dl, albumin 3.8 g/dl, phosphorus 5.2 mg/dl, and lactate dehydrogenase 356 U/L (reference range 560–100 U/L). A peripheral blood smear shows 4% schistocytes. A direct Coombs antiglobulin test is neg-ative. A disintegrin and metalloproteinase with a thrombospondin type 1 motif, member 13 activity is 82%. Urinalysis shows trace protein and blood. Microscopic examination of the urinary sediment shows three to eight erythrocytes per high-power field with occasional acanthocytes..",Which ONE of the following therapeutic agents is MOST likely responsible for AKI in this woman?,"1. Carfilzomib 2. Lenalidomide 3. Melphalan 4. Cyclophosphamide",A,temp nan,Acute Kidney Injury & Critical Care Nephrology 2981," A 55-year-old man with hypertension and stage G3a CKD is evaluated during a routine office visit. His BP is 132/72 mmHg, and his body mass index (BMI) is 26 kg/m2. The remainder of the physical examination is unremarkable. He follows a low-salt diet, but asks for additional information regarding diet and the risk of CKD progression. Laboratory studies show sodium 138 mEq/L, potassium 4.2 mEq/L, chloride 103 mEq/L, total CO2 24 mmol/L, BUN 36 mg/dl, creatinine 1.8 mg/dl (eGFR of 48 ml/min per 1.73 m2), and albumin 3.6 g/dl. The urine albumin-to-creatinine ratio is 17 mg/g..",Which ONE of the following diets should you tell him is associated with the lowest risk of CKD progression?,"A. A very low–carbohydrate diet B. liberal protein diet (> 1.2 g/kg per day) 1. A Dietary Approaches to Stop Hypertension-style diet that restricts red and processed meat 2. A calorie-restricted diet (< 800 kcal/d)",ANSWER FOR ID: 2981 NOT FOUND,temp nan,SUBJECT FOR ID: 2981 NOT FOUND 2982," A 67-year-old man with stage 4 CKD due to diabetes mellitus is evaluated during a routine office visit. His BP is well controlled with an angiotensin receptor blocker. His hemoglobin A1c is 6.4%. The albumin-to-creatinine ratio is 75 mg/g. He is concerned about his risk for ESRD and wishes to delay dialysis as long as possible. He recently read an article on the Internet about pentoxifylline and asks whether he should start this medication..",Which ONE of the following should you tell him BEST describes the current state of evidence regarding the use of pentoxifylline in treating diabetic kidney disease?,"1. Excess adverse events have led to early termination of clinical trials 2. Five-year follow-up data on renal outcomes is available 3. ESRD has been examined as a primary outcome 4. Short-term data indicate a slower decline in eGFR compared with placebo",ANSWER FOR ID: 2982 NOT FOUND,temp nan,SUBJECT FOR ID: 2982 NOT FOUND 5416," A 55-year-old woman with type 2 diabetes mellitus and coronary heart disease is seen in consultation for albuminuria. Three separate measurements of the urine albumin-to-creatinine ratio over the past 6 months have yielded results in the 30- to 300-mg/g range. Her BP has been well controlled on irbesartan. Her hemoglobin A1c has remained > 9% despite aggressive lifestyle interventions and therapy with metformin and glipizide. Other medications are aspirin and atorvastatin. On physical examination, the BP is 128/80 mmHg. The BMI is 38 kg/m2. There is mild background diabetic retinopathy. The remainder of the examination is normal. Laboratory studies show a serum creatinine level of 0.9 mg/dl, normal liver chemistries, and a hemoglobin A1c of 9.5%. She is reluctant to start insulin, and you decide to prescribe empagliflozin. You counsel her regarding the benefits and risks of empagliflozin therapy..","In addition to improved glycemic control, weight loss, and a reduction in the risk of progression to macroalbuminuria, which ONE of the following should you advise is an additional possible benefit of empagliflozin therapy?","A. A reduced risk of death from renal disease B. A reduced risk of cardiovascular death C. A reduction in LDL cholesterol D. A decreased risk of ketoacidosis",ANSWER FOR ID: 5416 NOT FOUND,temp nan,SUBJECT FOR ID: 5416 NOT FOUND 5417," A 58-year-old black man returns for follow-up of stage 4 CKD attributed to longstanding hypertension. His eGFR has declined by 4 ml/min per 1.73 m2 over the past year, and his urine albumin-to-creatinine ratio is 425 mg/g. He is concerned about needing dialysis during his lifetime. He asks about why there is a higher incidence of ESRD among blacks in the United States..",Which ONE of the following factors does NOT explain the higher incidence rate of ESRD among blacks compared with whites in the United States?,"1. A faster rate of GFR decline 2. A lower mean GFR at baseline 3. A lower competing risk of mortality 4. A higher prevalence of albuminuria",ANSWER FOR ID: 5417 NOT FOUND,temp nan,SUBJECT FOR ID: 5417 NOT FOUND 5418," A 17-year-old young woman with CKD secondary to primary FSGS is seen during a routine office visit. She is asymptomatic. The physical examination reveals that her height is 162 cm and that her weight is 52 kg. She has no edema. Laboratory studies show a serum creatinine level of 1.0 mg/dl. You apply both the Chronic Kidney Disease Epidemiology Collaboration (CKD-EPI) formula and the Schwartz formula to estimate her GFR..",Which ONE of the following would you predict with regard to the difference in GFR as predicted by the Schwartz formula compared with the CKD-EPI formula in this patient?,"1. No difference in the calculated GFR 2. The calculated GFR is 30% lower using the Schwartz formula 3. The calculated GFR is 30% higher using the Schwartz formula 4. The calculated GFR is 50% higher using the Schwartz formula",ANSWER FOR ID: 5418 NOT FOUND,temp nan,SUBJECT FOR ID: 5418 NOT FOUND 5419," A 28-year-old black woman with stage 3b CKD due to primary FSGS informs you that her daughter and her sister have CKD. She has a cousin who is on dialysis. She is a postdoctoral fellow in a research laboratory and curious about the scientific basis for the increased incidence of ESRD in the black population. You begin to discuss the current state of evidence regarding APOL1 high-risk variants..",Which ONE of the following BEST describes how APOL1 high-risk variants may cause kidney disease?,"1. Prevention of podocyte replication 2. Enhanced hypertension-related barotrauma 3. Enhanced endocytic trafficking 4. Increased intracellular potassium efflux that initiates programmed podocyte death",ANSWER FOR ID: 5419 NOT FOUND,temp nan,SUBJECT FOR ID: 5419 NOT FOUND 5420," A 78-year-old man with type 2 diabetes, stage 4 CKD, and coronary artery disease is evaluated during a routine office visit. He has not experienced episodes of hypoglycemia. His medications are lisinopril, rosuvastatin, empagliflozin, furosemide, and bedtime neutral protamine Hagedorn insulin. His physical examination reveals a BP of 132/86 mmHg and evidence of diabetic retinopathy and neuropathy. His hemoglobin A1c is 7.6%. The eGFR by the Modification of Diet in Renal Disease (MDRD) formula is 27 ml/min per 1.73 m2, and this value is confirmed on repeat testing. Serial results of his eGFR, as calculated by the MDRD formula are: 1 year ago the eGFR was 36 ml/min per 1.73m2; 6 months ago the eGFR was 32 ml/min per 1.73m2; the current value is 27 ml/min per 1.73 m2..",Which ONE of the following is the MOST appropriate management?,"1. Add glipizide, targeting a hemoglobin A1c of < 7% 2. Discontinue empagliflozin 3. No change in management 4. Discontinue neutral protamine Hagedorn insulin",ANSWER FOR ID: 5420 NOT FOUND,temp nan,SUBJECT FOR ID: 5420 NOT FOUND 5421," A 79-year-old man with stage 3b CKD, hyper-tension, and history of congestive heart failure is referred for management of atherosclerotic renal artery disease. He recently had magnetic resonance angiography that showed a 75% ostial stenosis in the right renal artery. His BP is well controlled at 124/62 mmHg on a regimen of amlodipine 10 mg daily, lisinopril 20 mg daily, and furosemide 40 mg twice daily. He asks for your advice regarding stenting of the right renal artery stenosis..",Which ONE of the following should you tell him about stenting of his right renal artery lesion?,"1. It will decrease his risk of ESRD 2. It will delay progression of his CKD 3. It will decrease his risk of death from renal or cardiovascular causes 4. It will not impact the rate of CKD progression",ANSWER FOR ID: 5421 NOT FOUND,temp nan,SUBJECT FOR ID: 5421 NOT FOUND 5422," A 56-year-old man with type 2 diabetes mellitus and stage G3b CKD due to diabetic kidney disease is seen during a routine office visit. His primary care physician recently found that he had asymptomatic hyperuricemia on screening laboratory studies. He does not have gout. He subsequently searched the Internet and found some articles that suggested possible benefit of treating hyperuricemia to delay progression to ESRD. Laboratory studies show an eGFR of 36 ml/min per 1.73 m2 and a serum uric acid of 9.2 mg/dl. He asks you whether he should start urate-lowering therapy..",Which ONE of the following would you advise about treating his hyperuricemia?,"1. There is no indication to begin urate-lowering therapy, because it only slows GFR decline when started in individuals with an eGFR > 45 ml/min per 1.73 m2 2. He should start febuxostat, because it has been shown to delay progression to ESRD 3. He should not start urate-lowering therapy, because existing evidence does not support efficacy in delaying the need for renal replacement therapy 4. There is no need to begin urate-lowering therapy, because it is only effective in individuals who have both CKD and gout",ANSWER FOR ID: 5422 NOT FOUND,temp nan,SUBJECT FOR ID: 5422 NOT FOUND 5423,,"Question 5423 Which ONE of the following is CORRECT regarding the association of dietary potassium, as assessed by urinary excretion, with CKD incidence in the setting of diabetes with an eGFR greater than or equal to 60 ml/min per 1.73 m2?","1. Low potassium intake is associated with faster annual eGFR decline 2. High potassium intake is associated with faster annual eGFR decline 3. The effect of potassium intake on incident CKD depends on the sodium intake 4. Potassium intake is not associated with incident CKD",ANSWER FOR ID: 5423 NOT FOUND,temp nan,SUBJECT FOR ID: 5423 NOT FOUND 5424," A 64-year-old woman with stage 4 CKD attributed to type 2 diabetes mellitus is referred for evaluation of her kidney disease. Her eGFR has declined from 29 ml/min per 1.73 m2 6 months ago to a current value of 25 ml/min per 1.73 m2. Her medications are lisinopril 40 mg daily, glipizide 5 mg daily, and rosuvastatin 20 mg daily. On physical examination, her BP is 152/88 mmHg, and she has trace edema. Her serum potassium level is 5.5 mEq/L, and her hemoglobin A1c is 7.2%. The albumin-to-creatinine ratio is 1100 mg/g..",Which ONE of the following is the MOST appropriate treatment?,"1. Atrasentan 2. Finerenone 3. Linagliptin 4. Furosemide",ANSWER FOR ID: 5424 NOT FOUND,temp nan,SUBJECT FOR ID: 5424 NOT FOUND 5425,,"Question 5425 Which ONE of the following BEST describes existing evidence regarding the use of oral adsorbents for reducing uremic toxin levels and slowing CKD progression?","1. Oral adsorbents delay need for RRT 2. Existing studies show no benefit 3. Poor adherence limits the use of oral adsorbents 4. Oral adsorbents reduce serum indoxyl sulfate levels more than placebo",ANSWER FOR ID: 5425 NOT FOUND,temp nan,SUBJECT FOR ID: 5425 NOT FOUND 5426,,"Question 5426 Which ONE of the following is CORRECT regarding the urinary biomarker α1-microglobulin?","1. It is freely filtered and then metabolized by the proximal tubule 2. Increased urinary excretion signifies glomerular injury 3. Urinary levels do not correlate with diabetes duration 4. Increased urinary levels do not predict tenofovir nephrotoxicity",ANSWER FOR ID: 5426 NOT FOUND,temp nan,SUBJECT FOR ID: 5426 NOT FOUND 5427," A 49-year-old woman with type 2 diabetes mellitus and gastroesophageal reflux disease is referred for further evaluation of progressive CKD. She had moderate dyspepsia 5 years ago but has had no symptoms since starting esomeprazole. Prior attempts to discontinue esomeprazole led to recurrent symptoms of gastroesophageal reflux. Her medications include metformin 500 mg twice daily, empagliflozin 10 mg daily, irbesartan 75 mg daily, and pravastatin 20 mg daily. Her eGFR has declined steadily from > 60 ml/min per 1.73 m2 1 year ago (serum creatinine 1.1 mg/dl) to 49 ml/min per 1.73 m2 (serum creatinine 1.4 mg/dl). On physical examination, the BP is 132/80 mmHg. She has background diabetic retinopathy. The neurologic examination reveals significant loss of light touch and vibratory sensation in the feet. There is no edema. Laboratory studies show serum sodium of 138 mEq/L, potassium level of 4.2 mEq/L, chloride level of 103 mEq/L, total CO2 of 24 mmol/L, BUN of 36 mg/dl, creatinine level of 1.4 mg/dl, and magnesium level of 2.3 mg/dl. A complete blood count with differential is normal. The hemoglobin A1c is 6.8%. The urinalysis shows trace protein and no cells or casts. The urine albumin-to-creatinine ratio is 108 mg/g..",Which ONE of the following is the MOST appropriate management?,"1. No change in management 2. Discontinue metformin 3. Discontinue empagliflozin 4. Discontinue esomeprazole and start ranitidine",ANSWER FOR ID: 5427 NOT FOUND,temp nan,SUBJECT FOR ID: 5427 NOT FOUND 5428," A 53-year-old black man with hypertension and HIV infection is seen in the office during a routine follow-up visit. He recently was enrolled in a trial investigating the influence of clinical modifiers and APOL1 risk status on CKD risk. He was found to carry two high-risk APOL1 variants. On physical examination, his BMI is 41 kg/m2, and the BP is 156/90 mmHg. Home BP readings confirm sustained systolic hypertension, with an average systolic BP of 154 mmHg. The remainder of the examination is unremarkable. A dietary consultation recently revealed that his diet was high in sodium and phosphate but low in potassium..",Which ONE of the following modifies his in-creased risk of CKD derived from his APOL1 risk status?,"1. His systolic hypertension 2. His obesity 3. His low potassium intake 4. His HIV status",ANSWER FOR ID: 5428 NOT FOUND,temp nan,SUBJECT FOR ID: 5428 NOT FOUND 5429," A 58-year-old Hispanic man from the Dominican Republic is seen in follow-up for stage G3b:A3 CKD due to primary FSGS. His maternal aunt required dialysis, and he has a cousin with CKD. You discuss his possible enrollment in a clinical trial investigating the impact of APOL1 risk variants on progression of FSGS. You begin a discussion about APOL1 risk variants and CKD progression..",Which ONE of the following should you tell him about APOL1 risk variants?,"1. The variants are limited to individuals with black race 2. APOL1 risk variants may be present in persons with African ancestry 3. APOL1 variants are not present in Hispanic/Latino persons 4. Individuals carrying two APOL1 high-risk variants will always develop CKD",ANSWER FOR ID: 5429 NOT FOUND,temp nan,SUBJECT FOR ID: 5429 NOT FOUND 5430," A 49-year-old woman with stage 3b CKD is evaluated in the office. She has hypertension, atrial fibrillation, and hyperlipidemia. She completed the Kidney Disease Quality of Life-36 survey and scored 1.2 SDs below the mean. Her father died at age 51 years old after a myocardial infarction. She has been adherent with a 2.5-g sodium-restricted diet. On physical examination, the BP is 138/79 mmHg. Funduscopic examination reveals very mild nonproliferative retinopathy. The remainder of the physical examination is normal. Laboratory studies show serum creatinine level of 1.9 mg/dl and urine albumin-to-creatinine ratio of 380 mg/g..",Which ONE of the following is associated with an increased risk of progressive CKD in this woman?,"1. Her low health-related quality of life 2. Her parental history of cardiovascular disease 3. Her dietary sodium intake 4. Her retinopathy",ANSWER FOR ID: 5430 NOT FOUND,temp nan,SUBJECT FOR ID: 5430 NOT FOUND 5431," A 60-year-old man is seen in follow-up for stage 4 CKD due to diabetic kidney disease. He was recently hospitalized for chest pain. His evaluation included an echocardiogram that showed a left ventricular ejection fraction of 55%, mild eccentric left ventricular hypertrophy (LVH), and an elevated pulmonary artery systolic pressure (PASP) of 48 mmHg..",Which ONE of the following regarding this man’s echocardiographic abnormalities is CORRECT?,"1. The elevated PASP is uncommon in stages 3 and 4 CKD, with a frequency of < 5% 2. The elevated PASP is associated with an increased risk of progression to ESRD 3. His LVH is not associated with an increased risk of cardiovascular events because it is eccentric 4. His LVH is associated with an increased risk of renal death",ANSWER FOR ID: 5431 NOT FOUND,temp nan,SUBJECT FOR ID: 5431 NOT FOUND 5432," A 70-year-old man with stage 4 CKD and chronic systolic heart failure is evaluated in follow-up. He has type 2 diabetes mellitus and coronary artery disease. His medications are losartan 50 mg daily, atorvastatin 20 mg daily, and carvedilol 12.5 mg daily. On physical examination, the BP is 126/76 mmHg. The jugular venous pressure is 8 cm H2O. The lungs are clear to auscultation. A grade 2 holosystolic murmur is heard best at the apex and radiates to the axilla. A soft S3 gallop is audible. There is trace pretibial edema. Laboratory studies show hemoglobin of 10.5 g/dl, sodium of 132 mEq/L, potassium of 5.6 mEq/L, total CO2 of 23 mmol/L, BUN of 56 mg/dl, and creatinine of 2.3 mg/dl (eGFR524 ml/min per 1.73 m2). The electrocardiogram shows a widened QRS complex (130 milliseconds). An echocardiogram shows a left ventricular ejection fraction of 25%. He would like to discuss additional therapeutic options to optimize his outcome..",Which ONE the following treatment options MOST likely has the greatest potential to im-prove his survival?,"1. Cardiac resynchronization therapy 2. Implantable cardioverter defibrillator 3. Spironolactone 4. Darbepoetin",ANSWER FOR ID: 5432 NOT FOUND,temp nan,SUBJECT FOR ID: 5432 NOT FOUND 5433," A 72-year-old woman with stage 5 CKD who will soon start hemodialysis is evaluated during a routine follow-up visit. Her primary care physician recently diagnosed her with atrial fibrillation and started anticoagulation with warfarin because of aCHA2DS2-VASC score of four. She detests the need for frequent international normalized ratio monitoring and asks whether she could switch to one of the newer direct oral anticoagulants that she sees advertised on television. You explain that there is a paucity of evidence supporting the use of direct oral anticoagulants for stroke prevention in advanced CKD and that current guidelines recommend warfarin. After a detailed discussion of the benefits and risks, she insists on changing to a direct oral anticoagulant..",Which ONE of the following agents is the MOST appropriate direct oral anticoagulant to prevent stroke in this woman?,"1. Edoxaban 2. Dabigatran 3. Rivaroxaban 4. Apixaban",ANSWER FOR ID: 5433 NOT FOUND,temp nan,SUBJECT FOR ID: 5433 NOT FOUND 5434," A 62-year-old man with stage 4 CKD due to diabetic nephropathy is evaluated in your clinic during a follow-up visit after a recent hospitalization for a left middle cerebral artery lacunar stroke. He was found to have an 80% stenosis of the left internal carotid artery. His medications are sevelamer 800 mg three times daily with meals and atorvastatin 20 mg once daily. On physical examination, the BP is 156/94 mmHg, consistent with several recent home BP readings. The neurologic examination reveals a right hemiparesis. Laboratory studies show creatinine of 2.8 mg/dl (eGFR of 28 ml/min per 1.73 m2), total cholesterol level of 184 mg/dl, calcium of 9.1 mg/dl, phosphorus of 5.2 mg/dl, uric acid of 7.5 mg/dl, hemoglobin of 10.3 g/dl, and ferritin of 210 ng/ml..",Which ONE of the following BEST predicts progression of his atheromatous disease?,"1. His serum ferritin level 2. His serum uric acid level 3. His total cholesterol level 4. His systolic BP 5. His treatment with a phosphate binder",ANSWER FOR ID: 5434 NOT FOUND,temp nan,SUBJECT FOR ID: 5434 NOT FOUND 5435," A 64-year-old white man with type 2 diabetes and hypertension is seen in consultation for progressive CKD. He drinks two alcoholic beverages per day and has less than a high school education. His eGFR has fallen from 55 ml/min per 1.73 m2 to the current value of 46 ml/min per 1.73 m2 over the past 2 years. On physical examination, the BP is 130/70 mmHg. The remainder of the examination is normal. His most recent laboratory studies show total cholesterol 160 mg/dl, low-density cholesterol 90 mg/dl, high-density cholesterol 39 mg/dl, calcium 9.2 mg/dl, phosphorus 3.6 mg/dl, parathyroid hormone 50 pg/ml, and high-sensitivity C-reactive protein 4.2 mg/L (reference range < 3 mg/L)..",Which ONE of the following factors or findings noted in this patient has been found to be MOST strongly associated with incident peripheral artery disease?,"1. His elevated high-sensitivity C-reactive protein 2. His serum calcium 3. His use of alcohol 4. His education status 5. His race/ethnicity",ANSWER FOR ID: 5435 NOT FOUND,temp nan,SUBJECT FOR ID: 5435 NOT FOUND 5436," A 47-year-old man with type 2 diabetes mellitus is seen for evaluation and management of stage 3b CKD. He has hypertension and hyperlipidemia, and he has smoked one pack of cigarettes per day for the past 30 years. His medications are metformin 500 twice daily, furosemide 40 mg daily, linagliptin 5 mg daily, enalapril 40 mg daily, amlodipine 10 mg daily, metoprolol tartrate 50 mg twice daily, and atorvastatin 40 mg once daily. On physical examination, the BP is 124/78 mmHg, and the heart rate is 64/min. The posterior tibial and dorsalis pedis pulses are not palpable. There is no peripheral edema. Laboratory studies show eGFR of 39 ml/min per 1.73 m2, total CO2 of 23 mmol/L, hemoglobin A1c of 7%, total cholesterol of 156 mg/dl, LDL cholesterol of 90 mg/dl, HDL cholesterol of 32 mg/dl, and serum triglycerides of 170 mg/dl..",Which ONE of the following interventions is MOST likely to improve this man’s cardiovascular outcomes?,"1. Addition of niacin 2. Smoking cessation 3. Addition of sodium bicarbonate 4. Increase atorvastatin",ANSWER FOR ID: 5436 NOT FOUND,temp nan,SUBJECT FOR ID: 5436 NOT FOUND 5437," A 48-year-old woman with stage 3a CKD, morbid obesity, type 2 diabetes mellitus, and hypertension is seen in consultation before planned bariatric surgery. Her eGFR is 56 ml/min per 1.73 m2..",Which ONE of the following should you tell her about the renal outcomes of bariatric surgery?,"1. An improvement in the serum creatinine level after surgery always reflects an improvement in kidney function 2. Bariatric surgery can reduce the risk of de-cline in eGFR and the risk of ESRD 3. Bariatric surgery is associated with a reduction in fat but not muscle mass 4. Malabsorptive procedures are not associated with an increased risk of kidney stones",ANSWER FOR ID: 5437 NOT FOUND,temp nan,SUBJECT FOR ID: 5437 NOT FOUND 5438," A 24-year-old woman with type 1 diabetes mellitus and hypertension is referred for preconception counseling. Her urinary albumin excretion increased to 50 mg/g 1 year ago but has been < 30 mg/g since starting lisinopril. Her medications include insulin as part via continuous subcutaneous infusion and simvastatin. Her eGFR, as estimated by the CKD-EPI formula, is 92 ml/min per 1.73 m2..",Which ONE of the following would you advise during counseling about her peripartum management and the renal risks of pregnancy?,"1. Pregnancy will not increase her risk for a faster decline in GFR 2. She should stop lisinopril at least 6 weeks before conception 3. She is at increased risk for preterm labor 4. She should continue simvastatin during pregnancy",ANSWER FOR ID: 5438 NOT FOUND,temp nan,SUBJECT FOR ID: 5438 NOT FOUND 5439," A 65-year-old man with stage 3b CKD and type 2 diabetes mellitus is admitted for further management of unstable angina. His functional status has been excellent, and he has been walking about 2 miles several times per week before admission. Coronary angiography shows significant three-vessel disease (90% stenoses in the left circumflex artery, left anterior descending artery, and right coronary artery). His ejection fraction is 48%..",Which ONE of the following interventions has the GREATEST potential to reduce his risk of repeat percutaneous intervention or myocardial infarction over the next 3 years?,"1. Coronary angioplasty with placement of a bare-metal stent 2. Coronary artery bypass surgery 3. Coronary angioplasty with placement of an everolimus-eluting stent 4. Optimal medical therapy without intervention",ANSWER FOR ID: 5439 NOT FOUND,temp nan,SUBJECT FOR ID: 5439 NOT FOUND 5440," A 57-year-old man with stage G3b CKD is evaluated in follow-up. His BP is 148/88 mmHg on losartan 100 mg daily, chlorthalidone 25 mg daily, amlodipine 10 mg daily, and metoprolol succinate 100 mg daily. The urine albumin-to-creatinine ratio is 650 mg/g. A 24-hour urine obtained to further evaluate his dietary intake shows a daily urine sodium excretion of 350 mmol (approximately 8 g) and urine potassium excretion of 86 mmol (about 3.4 g)..",Which ONE of the following MOST correctly characterizes the implications of his 24-hour urine sodium and potassium excretion?,"1. His urinary potassium excretion is not associated with CKD progression 2. His urinary sodium excretion is not associated with CKD progression 3. His urinary sodium excretion is not associated with mortality because he does not have diabetes mellitus 4. His urinary sodium excretion is associated with an increased risk for cardiovascular events",ANSWER FOR ID: 5440 NOT FOUND,temp nan,SUBJECT FOR ID: 5440 NOT FOUND 5441," A 54-year-old man with hepatitis C virus infection (genotype 1a) and stage 4 CKD is seen for management of CKD. His most recent laboratory data show an eGFR of 24 ml/min per 1.73 m2 and a hepatitis C virus RNA level of 1.1 million IU/ml. His hepatitis fibrosis stage is F0. His hepatologist would like to initiate treatment for hepatitis C virus infection..",Which ONE of the following is the MOST appropriate combination for treating his hepatitis C infection?,"1. Simeprevir and sofosbuvir 2. Ledipasvir and sofosbuvir 3. Paritaprevir and ritonavir 4. Grazoprevir and elbasvir",ANSWER FOR ID: 5441 NOT FOUND,temp nan,SUBJECT FOR ID: 5441 NOT FOUND 5442," A 72-year-old man with stage 3b CKD is seen in follow-up after a recent hospitalization for a transient ischemic attack. Magnetic resonance imaging of the brain showed several focal deposits of hemosiderin consistent with cerebral microbleeds. He asks about the significance of this finding..",Which ONE of the following statements about cerebral microbleeds in CKD is CORRECT?,"1. They are present in < 10% of the CKD population 2. They are associated with a higher risk of CKD progression 3. They do not portend a higher risk of adverse cardiovascular outcomes 4. They only predict adverse cardiovascular outcomes in diabetes",ANSWER FOR ID: 5442 NOT FOUND,temp nan,SUBJECT FOR ID: 5442 NOT FOUND 5443," A 62-year-old woman with type 2 diabetes mellitus, hypertension, and progressive CKD is evaluated in follow-up. Her medications are lisinopril 40 mg daily, atorvastatin 40 mg daily, chlorthalidone 25 mg daily, and metformin 500 mg twice daily. She has been on this regimen for the past 4 years. The physical examination reveals a BP of 132/75 mmHg and evidence of diabetic retinopathy and neuropathy. There is no peripheral edema. Laboratory studies show sodium of 138 mEq/L, potassium of 5.2 mEq/L, chloride of 102 mEq/L, total CO2 of 25 mmol/L, BUN of 56 mg/dl, creatinine of 2.6 mg/dl, eGFR of 30 ml/min per 1.73 m2, LDL cholesterol of 86 mg/dl, and hemoglobin A1c of 6.4%. Serial eGFR levels are: 3 years ago the eGFR was 47 ml/min per 1.73m2; 2 years ago the eGFR was 41 ml/min per 1.73m2; 1 year ago the eGFR was 35 ml/min per 1.73m2; the current value is 30 ml/min per 1.73 m2..",Which ONE of the following is the MOST appropriate management?,"1. Add empagliflozin 2. Discontinue metformin 3. Change hydrochlorothiazide to furosemide 4. Discontinue enalapril",ANSWER FOR ID: 5443 NOT FOUND,temp nan,SUBJECT FOR ID: 5443 NOT FOUND 2643,"A 68-year-old black man with ESRD is waitlisted for deceased donor kidney transplantation. His blood type is A, and he has a calculated panel reactive antibody (cPRA) level of 80%..","Since implementation of the new Kidney Allocation System (KAS) by the United Network for Organ Sharing on December 4, 2014, which ONE of the following factors now associates with him having an INCREASED likelihood of receiving a deceased donor kidney compared with the prior allocation system?","A. His race B. His cPRA C. His age D. His blood type",ANSWER FOR ID: 2643 NOT FOUND,temp nan,SUBJECT FOR ID: 2643 NOT FOUND 2644,"A 66-year-old man is referred for kidney trans-plantation. He has stage 5 CKD due to membranous nephropathy (MN), and he has been maintained on hemodialysis for 3 months. He uses a walker and needs help to get dressed each day. His examination shows a slow walking speed and difficulty standing from a sitting position. He asks about his prognosis regarding kidney transplantation..",Which ONE of the following should you tell him about the effect of his physical function on his transplantation outcomes?,"A. It will not affect his graft survival after kidney transplantation B. It is associated with increased waitlist mortality C. It is associated with a decreased risk for readmissions after transplantation D. Physical therapy can mitigate the effects of frailty on his transplant outcomes",ANSWER FOR ID: 2644 NOT FOUND,temp nan,SUBJECT FOR ID: 2644 NOT FOUND 2645,"A 56-year-old man with ESRD due to IgA nephropathy returns for his routine yearly re-evaluation visit with your local transplant program. He has been listed for deceased donor transplantation for 4 years. His blood type is B. No living donors have been identified. He previously was listed for an expanded criteria donor kidney, and he asks whether there are any other options to increase his chances of receiving a deceased donor kidney transplant. You begin a discussion about the benefits and risks of a Public Health Service Increased Risk Donor (PHS-IRD) kidney. He is ambivalent about receiving a kidney from an intravenous drug user and concerned about the risk of acquiring an infection from the procedure..",Which ONE of the following should you tell him about PHS-IRD kidneys?,"A. Transplantation of PHS-IRD kidneys is as-sociated with an increased waiting time B. Kidneys from individuals with increased risk behaviors currently make up < 10% of all deceased donors C. Kidneys from intravenous drug users are never used for transplantation D. His risk of contracting HIV, hepatitis C virus (HCV), or hepatitis B virus from a PHS-IRD donor is < 1%",ANSWER FOR ID: 2645 NOT FOUND,temp nan,SUBJECT FOR ID: 2645 NOT FOUND 2646,"A 59-year-old woman with stage 5 CKD is referred for transplantation. She donated a kidney to her sister 18 years ago and now has advanced CKD complicating diabetes mellitus and hypertension. She has had multiple pregnancies and received several blood transfusions after a recent gastrointestinal bleed. Her blood type is A, and her cPRA is 9%. She is concerned about the effect of the new KAS on her chances of receiving a kidney. A discussion ensues about kidney transplantation options..",Which ONE of the following should you tell her regarding her options for kidney transplantation?,"A. The quality of decreased donor kidneys has deteriorated since implementation of Kidney Allocation System (KAS), with a median kidney donor risk index of > 80% B. Prior living donors have improved access to transplantation since implementation of KAS C. Her projected waiting time for a deceased donor kidney is likely to be < 4 months D. She can be listed for transplantation only after initiation of dialysis",ANSWER FOR ID: 2646 NOT FOUND,temp nan,SUBJECT FOR ID: 2646 NOT FOUND 2647,"A 63-year-old man with ESRD and HCV infection (genotype 1) as a consequence of a remote history of intravenous drug use is listed for kidney trans-plantation. He is HCV treatment naive. He asks whether it would be better to opt for an HCV-positive kidney compared with a standard criteria HCV-negative kidney. His HCV viral load is persistently high at > 450,000 IU/ml..",Which ONE of the following statements should you tell him regarding the expected outcomes of receiving an HCV-positive kidney and management of HCV infection?,"A. Consent to receive an HCV-positive kidney may dramatically reduce his waiting time B. Receipt of an HCV-positive donor kidney is associated with worse allograft survival com-pared with waiting for an HCV-negative donor kidney C. Consent to receive an HCV-positive donor kidney is not recommended in order to avoid infection with a virus of different genotype D. HCV infection should be treated before transplantation because direct-acting antiviral agents are contraindicated in transplant recipients receiving calcineurin inhibitors",ANSWER FOR ID: 2647 NOT FOUND,temp nan,SUBJECT FOR ID: 2647 NOT FOUND 2648,Your transplant surgeon informs you that a de-ceased donor kidney with premortem AKI has been offered to a dialysis patient from an affiliated practice. The donor was 28 years old. The terminal serum creatinine level is 2.5 mg/dl. The preimplantation allograft biopsy shows fibrin thrombi. Glomerulosclerosis is not present. Your nephrology colleague asks you about the expected post-transplant prognosis..,Which ONE of the following should you tell your colleague about the expected prognosis of transplanting this kidney?,"A. The risk of delayed graft function (DGF) is equivalent to receiving a deceased donor kidney without AKI B. This donor’s preimplantation kidney biopsy predicts that DGF is certain C. The finding of fibrin thrombi on biopsy is associated with > 10% lower allograft survival at 1 year D. AKI in the donor kidney does not affect allograft function at 1 year",ANSWER FOR ID: 2648 NOT FOUND,temp nan,SUBJECT FOR ID: 2648 NOT FOUND 2649,"A 59-year-old woman with ESRD due to polycystic kidney disease is admitted for transplantation. She is asymptomatic. Her physical examination shows a body mass index (BMI) of 37 kg/m2. She has a low-level donor-specific antibody targeting HLA-A2 (mean fluorescence intensity of 1582). The flow cytometry crossmatch is negative. She receives the deceased donor kidney after 28 hours of cold ischemia time. The preimplantation biopsy shows no fibrin thrombi. Postoperatively, her urine output is 3 L over the initial 24 hours. However, the serum creatinine level rises from 5 mg/dl on postoperative day 1 to a peak of 6.2 mg/dl on postoperative day 7. She does not require dialysis, and the serum creatinine subsequently begins to decline to 1.4 mg/dl over the next 2 weeks..",Which ONE of the following statements about this patient’s delayed graft function (DGF) is CORRECT?,"A. The presence of the donor-specific antibody increases her risk of DGF B. Her BMI does not influence her risk of DGF C. Her DGF only affects allograft prognosis when fibrin thrombi are seen on preimplantation biopsy D. Her DGF only affects long-term allograft function if she develops rejection",ANSWER FOR ID: 2649 NOT FOUND,temp nan,SUBJECT FOR ID: 2649 NOT FOUND 2650,A 29-year-old woman is seen in consultation to assess her candidacy to donate a kidney to her father. She is planning to have a family within the next 3 years. All of her donor testing is normal. She is ABO compatible and has a negative crossmatch with her father. They are a one-haplotype match..,Which ONE of the following should you tell her about her pregnancy risk after live donor nephrectomy?,"A. Pregnancy after kidney donation is more likely to be complicated by low-birth weight infants B. Pregnancy after kidney donation is more likely to be complicated by preterm births C. Pregnancy after kidney donation is associated with an increased risk of gestational hypertension and preeclampsia D. The risk of neonatal mortality is higher among kidney donors compared with nondonors",ANSWER FOR ID: 2650 NOT FOUND,temp nan,SUBJECT FOR ID: 2650 NOT FOUND 2651,"A 23-year-old white man has been accepted as an altruistic living donor. He is completely healthy and does not smoke. On physical examination, his BP is 110/80 mmHg. His creatinine clearance is 106 ml/min. He asks about his future risk of developing kidney failure..",Which ONE of the following should you tell him about his risk of ESRD after kidney donation?,"A. It is higher compared with black donors B. It is no higher than that of nondonors in the general population C. It is three to five times higher than non-donors in the general population D. It may be lower than that of older donors",ANSWER FOR ID: 2651 NOT FOUND,temp nan,SUBJECT FOR ID: 2651 NOT FOUND 2652,"A 32-year-old woman with ESRD due to reflux nephropathy is seen in follow-up 6 weeks after receiving a 6/6-antigen mismatch kidney transplant from a living unrelated donor. She received in-duction therapy with antithymocyte globulin and is now maintained on tacrolimus, mycophenolic acid (MPA), and prednisone 15 mg daily, with a plan to taper prednisone to 5 mg daily by week 10. Her serum creatinine concentration is 1.2 mg/dl. She wishes to taper entirely off prednisone because of weight gain and onset of glucose intolerance..",Which ONE of the following should you advise her about the risks of steroid elimination after transplantation?,"A. It is associated with an increased risk of mortality B. It will lower her risk of infection C. It will lower her risk of developing diabetes mellitus D. It will lower her risk of malignancy E. It is associated with an increased risk of acute rejection",ANSWER FOR ID: 2652 NOT FOUND,temp nan,SUBJECT FOR ID: 2652 NOT FOUND 2653,,"Question 2653: Which ONE of the following statements about antibody induction therapy for kidney transplantation is CORRECT?","A. Basiliximab therapy associates with marked reduction in rejection risk in tacrolimus/mycophenolate/steroid-treated nonsensi-tized patients B. Rituximab induction reduces rejection risk in nonsensitized allograft recipients C. An alemtuzumab/prednisone-free regimen is becoming the most widely used strategy in the United States D. Induction therapy associates with approximately 50% rejection risk reduction com-pared with no induction therapy",ANSWER FOR ID: 2653 NOT FOUND,temp nan,SUBJECT FOR ID: 2653 NOT FOUND 2654,,"Question 2654: Which ONE of the following statements about use of mycophenolate after kidney transplantation is CORRECT?","A. Dose reductions stemming from adverse effects are associated with an increased risk of rejection and graft failure B. The risk of cytomegalovirus (CMV) viremia is greater with azathioprine compared with mycophenolate C. Mycophenolate is associated with a reduced risk of mortality compared with azathioprine D. Physical frailty does not affect the frequency of mycophenolate adverse events E. Dose-limiting adverse effects are more commonly seen in living compared with de-ceased donor allograft recipients",ANSWER FOR ID: 2654 NOT FOUND,temp nan,SUBJECT FOR ID: 2654 NOT FOUND 2655,"A 38-year-old kidney transplant recipient is admitted for management of an upper gastrointestinal bleed. She is found to have a bleeding gastric ulcer that is cauterized endoscopically. She is fatigued but otherwise asymptomatic. Her allograft function has been excellent since transplantation 4 months ago, and her serum creatinine has been stable at 1.3 mg/dl. Her medications include cyclosporine, mycophenolate mofetil, prednisone, and valganciclovir. Her hemoglobin stabilizes at 6.7–6.9 g/dl. Her internist recommends transfusion with 1 U packed red blood cells. You discuss the benefits and risks of transfusion as well as transfusion methods with her internist..",Which ONE of the following statements should you advise about the risks and methods of transfusion in this patient?,"A. There is no risk of allosensitization, because she is immunosuppressed B. Blood transfusion may induce donor-specific antibody and an increased risk of rejection C. Leukocyte filtration will eliminate the risk of allosensitization D. If she receives blood, it should be CMV negative E. If she receives blood, it should be irradiated",ANSWER FOR ID: 2655 NOT FOUND,temp nan,SUBJECT FOR ID: 2655 NOT FOUND 2656,"A 48-year-old woman with ESRD due to lupus nephritis is evaluated during her annual visit while waitlisted for transplantation. Her cPRA is 99%as a result of prior pregnancies and remote transfusions. She has been on the waitlist for 3 years. Her sister is her only potential living donor. However, their complement-dependent cytotoxicity crossmatch is positive. Her nephrologist consults you about her candidacy for desensitization. She has already been listed for kidney paired donation nationally..",Which ONE of the following should you advise about HLA antibody desensitization?,"A. HLA antibody desensitization using plasma-pheresis and intravenous Ig is associated with the same excellent outcomes as desensitization to permit ABO-incompatible transplantation B. Treatment of subclinical rejection detected by protocol biopsy abrogates the risk of allograft failure after HLA antibody desensitization C. The addition of rituximab to her regimen will increase the risk of infection and malignancies at 2 years D. HLA antibody desensitization is associated with inferior patient and graft survival com-pared with HLA-compatible transplantation E. Thrombotic microangiopathy after desensitization is mitigated by plasmapheresis and does not affect allograft outcome",ANSWER FOR ID: 2656 NOT FOUND,temp nan,SUBJECT FOR ID: 2656 NOT FOUND 2657,A 68-year-old woman with polycystic kidney disease receives a kidney donor profile index of 86 with a deceased donor kidney. Her achieved serum creatinine level is 1.8 mg/dl. She is tolerating maintenance immuno-suppression with tacrolimus and mycophenolate well..,"In addition to serum creatinine levels, which ONE of the following biomarkers should be used to best monitor her allograft stability?","A. Kidney injury marker-1 B. Neutrophil gelatinase–associated lipocalin C. Perforin D. Granzyme B E. Urine protein-to-creatinine ratio and/or urine albumin-to-creatinine ratio",ANSWER FOR ID: 2657 NOT FOUND,temp nan,SUBJECT FOR ID: 2657 NOT FOUND 2658,"A 54-year-old woman with a history of reflux nephropathy and recurrent urinary tract infection is seen six months after living donor kidney trans-plantation. She had prompt function of the allograft, and her serum creatinine level has been stable at 1.2 mg/dl for the last six weeks. She is concerned about her bone health because her 10-year probability of a hip fracture is estimated to be 3.2% based on the Fracture Risk Assessment Tool (FRAX). Her immunosuppression includes tacrolimus, mycophenolate mofetil, and prednisone. The serum creatinine level is 1.2 mg/dl (eGFR > 60 ml/min per 1.73 m2), the parathyroid hormone (PTH) level is 116 pg/ml (reference range 12 - 88 pg/ml), the serum calcium is 9.1 mg/dl, the phosphorus is 2.6 mg/dl, the serum alkaline phosphatase is normal, the 25-hydroxy-vitamin D level is 30 ng/ml (reference range 5 30–50 ng/ml), and the 1,25-dihydroxy-vitamin D level is 32 pg/ml (reference range 25–65 pg/ml)..",Which ONE of the following is the MOST ap-propriate management strategy for this woman?,"A. No further treatment B. Denosumab C. A bisphosphonate D. Teriparatide",ANSWER FOR ID: 2658 NOT FOUND,temp nan,SUBJECT FOR ID: 2658 NOT FOUND 2659,"A 56-year-old man has hypercalcemia and hyperparathyroidism 18 months after a second deceased donor kidney transplant. Laboratory studies show a serum creatinine of 1.0 mg/dl, a PTH of 1208 pg/ml (reference range 512–88 pg/ml), a serum calcium of 10.6–11.2 mg/dl, a serum albumin of 4.2 g/dl, a serum phosphorus of 1.4 mg/dl, and a serum 25-hydroxy-vitamin D level of 35 ng/ml. A dual energy x-ray absorptiometry scan shows a Z score of - 22.1 and a T score of - 22.7, which was a significant decrease of 7.1% compared with a previous measurement 1 year ago. Parathyroid scintigraphy shows four enlarged glands..",Which ONE of the following is the MOST appropriate next step in treatment?,"A. Subtotal parathyroidectomy B. Cinacalcet 30 mg daily and titrate until the serum PTH level is normalized C. Paricalcitol 1 microgram/d titrated to 2 microgram/d as tolerated D. Denosumab 60 mg administered every 6 months",ANSWER FOR ID: 2659 NOT FOUND,temp nan,SUBJECT FOR ID: 2659 NOT FOUND 2660,"A 31-year-old woman is seen for preconception counseling 5 years after successful kidney trans-plantation. She is maintained on stable doses of tacrolimus, mycophenolate mofetil (MMF), and prednisone. She had one episode of mild acute cellular rejection 3 months after transplantation that responded to pulse methylprednisolone. Her serum creatinine level has been stable in the 1.4- to 1.6-mg/dl range over the past year, and the urine protein-to-creatinine ratio has averaged 400 mg/d. A biopsy 2 years ago showed mild to moderate transplant glomerulopathy with no active inflammation. At that time, a donor-specific antibody to HLA-DQ7 was detected with a mean fluorescence intensity of 7000..",Which ONE of the following is the MOST appropriate management of her immunosuppression in preparation for and during pregnancy?,"A. Transition MMF to azathioprine > 6 weeks prior to attempts to conceive and plan to increase tacrolimus about 20%–25% during the second trimester to maintain therapeutic levels B. Wait until the first trimester to discontinue mycophenolate mofetil C. Transition mycophenolate mofetil to sirolimus now D. Discontinue tacrolimus after pregnancy is confirmed",ANSWER FOR ID: 2660 NOT FOUND,temp nan,SUBJECT FOR ID: 2660 NOT FOUND 2661,"A 54-year-old man with type 2 diabetes mellitus is evaluated for persistent hypertension 2 years after a living related kidney transplant. He has no prior cardiovascular history. His medications are tacrolimus, sirolimus (implemented because of gastrointestinal intolerance to mycophenolate), prednisone, amlodipine 10 mg daily, and hydrochlorothiazide 25 mg daily. The systolic BP has consistently been in the 150-to 159-mmHg range using home BP monitoring. On physical examination, the BP is 158/90 mmHg. There is no lower extremity edema. Laboratory studies show a serum creatinine level of 0.9 mg/dl, potassium of 4.3 mEq/L, and a urine protein-to-creatinine ratio of 200 mg/g..",Which ONE of the following is the MOST appropriate agent to add for BP control?,"A. Furosemide B. Hydralazine C. Labetalol D. Lisinopril",ANSWER FOR ID: 2661 NOT FOUND,temp nan,SUBJECT FOR ID: 2661 NOT FOUND 2662,A 62-year-old man with ESRD due to type 2 diabetes mellitus currently maintained on hemodialysis for 1 year is seen in consultation to evaluate his candidacy for kidney transplantation. His past medical history is significant for stable coronary artery disease that required placement of a drug-eluting stent in the left circumflex artery 3 years ago. His BMI is 36.5 kg/m2..,Which ONE of the following is the MOST appropriate statement regarding his expected transplant outcome?,"A. His risk of graft loss (death censored) is similar to that of nonobese transplant recipients B. His mortality risk is higher than nonobese transplant recipients C. His risk of DGF is higher than nonobese transplant recipients D. His mortality risk with transplant is higher than his nonobese waitlisted counterparts",ANSWER FOR ID: 2662 NOT FOUND,temp nan,SUBJECT FOR ID: 2662 NOT FOUND 2663,"A 64-year-old woman with ESRD due to chronic GN and a remote history of a transient ischemia attack and peptic ulcer disease has been on hemodialysis for 6 years. She is active on the kidney waitlist. She suffers from hypotension during dialysis and requires regular use of midodrine during dialysis to maintain systolic BP .90 mmHg. Her systolic BP on nondialysis days is 90–100 mmHg. Her medications include omeprazole, aspirin, sevelamer carbonate, magnesium oxide, and methoxy polyethylene glycol epoetin-beta. She had a normal exercise stress test within the past year. An echocardiogram shows severe left ventricular hypertrophy. Her laboratory studies are significant for a low serum magnesium of 1.3 mg/dl..","In considering her clinical risk factors, which ONE of the following clinical features is MOST highly associated with graft loss after transplant?","A. Midodrine use B. Hypomagnesemia C. Proton pump inhibitor use D. Aspirin use",ANSWER FOR ID: 2663 NOT FOUND,temp nan,SUBJECT FOR ID: 2663 NOT FOUND 2664,"A 44-year-old man with HIV-associated nephropathy has been on dialysis for 4 years. His HIV viral load is undetectable on highly active antiretroviral therapy, with a recent CD4 count of 480/ml and no opportunistic infections. He wishes to pursue kidney transplantation..","When considering his transplant referral and management, which ONE of the following is the MOST appropriate management?","A. Use of cyclosporine rather than tacrolimus for maintenance immunosuppression B. Use of thymoglobulin induction therapy C. Referral to a transplant center with extensive prior experience in HIV-positive transplantation D. Avoidance of HIV-positive deceased donors",ANSWER FOR ID: 2664 NOT FOUND,temp nan,SUBJECT FOR ID: 2664 NOT FOUND 2665,"A 62-year-old man was found to have BK virus (BKV) viremia (16,240 copies/ml) 6 months after deceased donor transplant while on tacrolimus, MPA, and prednisone. A biopsy of the allograft 6 months after transplantation showed no evidence of rejection or BKV nephropathy. His tacrolimus and MPA doses were initially reduced by 25%. MPA was further reduced by an additional 50%over the subsequent 3 months due to persistent viremia. Over the next 3 months, BKV viremia remained present at very low levels (1000–2000 copies/ml). Renal allograft function remained stable throughout the 6-month period, with serum creatinine of 1.2 mg/dl..","Compared with individuals who never developed BKV viremia, which ONE of the following is the MOST likely clinical outcome?","A. Development of de novo donor-specific antibodies B. Graft loss due to BKV nephropathy C. Development of a second viral infection D. Increased mortality",ANSWER FOR ID: 2665 NOT FOUND,temp nan,SUBJECT FOR ID: 2665 NOT FOUND 2666,"A 55-year-old man with ESRD secondary to type 2 diabetes mellitus is evaluated for future kidney transplantation. He has been on dialysis for 6 months, and he has a potential living donor, his wife, who is otherwise healthy. He is a nonsmoker. He has a history of squamous cell skin cancer on his scalp that was diagnosed and successfully resected with clear margins 1 year ago. The lesion was 2.1 cm in size. He had a 1-cm tubular adenomatous polyp resected 2 years ago during screening colonoscopy. He has no symptoms of urinary frequency, urgency, or hesitancy. He does not have hematuria and has not worked in aluminum smelting, rubber manufacture, or leather industries. There is no family history of colon cancer or prostate cancer..","Regarding his risk for malignancy after transplant, which ONE of the following is the MOST appropriate management recommendation?","A. An additional 1-year waiting time (total waiting time was 52 years) on the basis of prior skin cancer history B. Repeat colonoscopy now before proceeding with a living donor transplant C. Check a screening serum prostate-specific antigen level D. Refer him for screening cystoscopy",ANSWER FOR ID: 2666 NOT FOUND,temp nan,SUBJECT FOR ID: 2666 NOT FOUND 2667,A 55-year-old man with ESRD secondary to membra-nous nephropathy (MN) is seen to assess his candidacy for kidney transplantation. He still produces urine and has persistent proteinuria (6 g/d). An antiphospholipase A2 receptor autoantibody level drawn at the time of evaluation is negative. He is scheduled to receive a living unrelated kidney transplant from a friend..,Which ONE of the following is the MOST accurate regarding his risk for recurrent MN after transplant?,"A. He has a high (> 80%) likelihood of developing recurrent MN B. The absence of anti-phospholipase A2 receptor (anti-PLA2R) antibody indicates a low risk (< 20%) of recurrent MN C. Graft survival is significantly lower in those who develop recurrent MN than the general transplant population D. Recurrent MN with progressive proteinuria typically responds to rituximab",ANSWER FOR ID: 2667 NOT FOUND,temp nan,SUBJECT FOR ID: 2667 NOT FOUND 2668,"A 49-year-old woman with type 2 diabetes mellitus and stage 5 CKD undergoes an evaluation for kidney transplantation. She has no identified living donors. She has had insulin-requiring diabetes mellitus for 18 years. Her BMI is 28 kg/m2. Laboratory studies show an eGFR of 14 ml/min per 1.73 m2, a C-peptide level of 3.2 ng/ml, and a hemoglobin A1c of 6.8%. She wishes to discuss pancreas transplantation..",Which ONE of the following is the MOST accurate when discussing simultaneous pancreas-kidney (SPK) transplantation with her?,"A. She is an eligible candidate for SPK transplantation B. SPK transplant in type 2 diabetes mellitus recipients is associated with a higher risk of rejection compared with in type 1 diabetes mellitus recipients C. SPK transplant in type 2 diabetes mellitus recipients is associated with an increased risk of primary nonfunction compared with in type 1 diabetes mellitus recipients D. SPK transplant in type 2 diabetes mellitus recipients is associated with worse pancreas graft survival compared with in type 1 diabetes mellitus recipients",ANSWER FOR ID: 2668 NOT FOUND,temp nan,SUBJECT FOR ID: 2668 NOT FOUND 2669,A 55-year-old woman with type 1 diabetes and ESRD for 6 months is evaluated for kidney transplantation. She has consistently achieved a hemoglobin A1c level of about 7.2%–7.8% using an insulin pump. Her BMI is 31 kg/m2. She has not had emergency care or third party assistance for hypoglycemia. She does not have symptomatic cardiovascular disease. She has no potential living kidney donors and wishes to consider pancreas transplantation. Her C peptide is < 2 ng/ml..,Which ONE of the following should you tell her about simultaneous pancreas kidney (SPK) transplantation?,"A. She is not a candidate for SPK transplantation B. SPK transplant is associated with substantial improvements in health-related quality of life, even in the setting of early pancreas transplant failure C. SPK transplant is associated with substantial improvements in long-term survival compared with kidney transplant alone D. SPK transplant waiting time is substantially shorter than for deceased donor kidney trans-plant alone",ANSWER FOR ID: 2669 NOT FOUND,temp nan,SUBJECT FOR ID: 2669 NOT FOUND 2670,"In 2016, eligibility criteria for simultaneous liver-kidney transplant were developed and approved by the Organ Procurement and Transplantation Network..",Which ONE of the following liver transplant candidates is NOT eligible for simultaneous liver-kidney transplantation?,"A. A 64-year-old man with stage 3 CKD (eGFR of 54.5–50 ml/min per 1.73 m2) documented for 4 months and a recent fall in eGFR during a period of hypovolemia to 25 ml/min per 1.73 m2 B. A 19-year-old woman with primary hyper-oxaluria, nephrocalcinosis, and stage 5 CKD C. A 54-year-old man with HCV infection, type 2 diabetes mellitus, an eGFR of 35–40 ml/min per 1.73 m2, and proteinuria of 1 g/d D. A patient with type 2 hepatorenal syndrome with an eGFR of 15–25 ml/min per 1.73 m2 over the past 6 weeks",C,temp nan,Interventional Nephrology & Dialysis Therapy 2671,"A 62-year-old man with chronic GN is to undergo living unrelated kidney transplant. He has a history of squamous cell carcinoma successfully treated 5 years ago. He is CMV negative (serum IgG for CMV is negative), whereas his prospective donor is CMV IgG positive. Other past medical history includes chronic idiopathic diarrhea two to three times daily. An extensive evaluation of the diarrhea was unrevealing. The transplant center is considering use of tacrolimus combined with the mammalian target of rapamycin inhibitor everolimus and prednisone as initial maintenance immunosuppression..","Compared with tacrolimus/mycophenolate-based immunosuppression, which ONE of the following is the MOST likely clinical outcome of using tacrolimus/everolimus/prednisone in the absence of CMV prophylaxis?","A. Lower incidence of CMV viremia B. Lower incidence of acute rejection C. Lower incidence of mortality D. Lower incidence of nonskin malignancy",ANSWER FOR ID: 2671 NOT FOUND,temp nan,SUBJECT FOR ID: 2671 NOT FOUND 2672,"A 62-year-old man with chronic GN is to undergo living unrelated kidney transplant. His calculated panel reactive antibody score is 0%, and the HLA mismatch with the donor is four out of six. In addition to tacrolimus/mycophenolate-based immunosuppression with prednisone, the transplant center is planning use of rabbit antithymocyte globulin for induction therapy..","Compared with induction therapy with an IL-2 receptor antagonist, which ONE of the following is the MOST likely clinical outcome of using rabbit antithymocyte globulin for induction therapy?","A. A lower incidence of acute rejection B. A lower incidence of graft loss C. A lower incidence of mortality D. A lower incidence of malignancy",ANSWER FOR ID: 2672 NOT FOUND,temp nan,SUBJECT FOR ID: 2672 NOT FOUND 2673,Practice patterns regarding vascular access modalities at the time of hemodialysis (HD) initiation have changed in the United States between 2005 and 2014..,Which ONE of the following BEST depicts the changes in incident vascular access during this interval (2005–2014)?,"A. An increase in incident fistula rate, graft rate, and catheter rate B. No change in incident fistula rate, graft rate, and catheter rate C. An increase in incident fistula rate and a decline in graft and catheter rate D. An increase in incident fistula rate, a decline in graft rate, and no change in catheter rate E. A decrease in incident fistula rate, a decline in graft rate, and an increase in catheter rate",D,temp nan,Interventional Nephrology & Dialysis Therapy 2674,"A 61-year old man with ESRD due to diabetic kidney disease began maintenance HD 5 months ago. He is currently receiving dialysis through a tunneled right internal jugular catheter. He has a maturing arteriovenous fistula that was placed 6 weeks ago. The leukocyte count is 10,000/µL, the hemoglobin level is 9.2 g/dL, the Kt/V is 1.2, and the serum albumin level is 2.9 g/dL..","After successful transition to dialysis access through the arteriovenous fistula, which ONE of the following outcomes is MOST likely to occur?","A. A decrease in the hemoglobin level B. An increase in the leukocyte count C. An improvement in serum albumin D. An increase in erythropoietin requirement E. A decreased likelihood of survival over the next year",C,temp nan,Interventional Nephrology & Dialysis Therapy 2675,A 55-year-old woman is seen in the clinic for follow-up of stage 4 CKD. Her estimated GFR is 18 ml/min per 1.73 m2. Creation of an arterio-venous fistula is scheduled for next week..,Which ONE of the following factors is MOST likely to predict failure of arteriovenous maturation in this patient?,"A. Preexisting neointimal hyperplasia in the fistula vein biopsy specimen taken at the time of surgery B. Failure to perform preoperative vessel mapping C. Predialysis care < 12 months D. Fistula creation by a surgeon who has created > 25 fistulas during surgical training E. The fact that she is female",E,temp nan,Interventional Nephrology & Dialysis Therapy 2676,A 71-year-old man with ESRD due to diabetic nephropathy who has just started HD with a catheter is seen during weekly dialysis rounds. A discussion begins about the relative benefits and risks of an arteriovenous fistula versus an arteriovenous graft..,Which ONE of the following should you tell him about the different types of arteriovenous access?,"A. His survival with a catheter is likely to be similar to that achieved with a fistula or a graft B. The catheter dependence period is likely to be higher during the first 6 months with a graft than with a fistula C. Infection-related hospitalization is more likely with fistulas D. Septicemia-related or bacteremia-related hospitalization is more likely with fistulas E. The risk of death is lower with fistulas than with grafts",E,temp nan,Interventional Nephrology & Dialysis Therapy 2677,A 63-year-old man who has received maintenance HD through central venous catheters for the past 6 months is evaluated during HD rounds. His course has been complicated by recurrent episodes of bacteremia and catheter dysfunction that began 2 weeks after the first catheter was placed. Previous attempts at arteriovenous fistula creation failed because of nonmaturation. Vascular ultrasonography shows patent central veins and an absence of peripheral veins suitable for fistula creation..,Which ONE of the following strategies is MOST likely to provide the quickest transition away from catheter-based HD?,"A. Placement of a hybrid graft–catheter device B. Placement of a graft with a venous stent C. Placement of an early cannulation arterio-venous graft D. Placement of an arteriovenous fistula with sirolimus wrap around an arterial anastomosis",C,temp nan,Interventional Nephrology & Dialysis Therapy 2678,"A 35-year-old woman with ESRD due to focal and segmental glomerulosclerosis, who is receiving maintenance in-center HD through a right forearm arteriovenous fistula, is evaluated during routine HD rounds. Over the past 3 months she has required hospitalization on three occasions for acute decompensated heart fail-ure. She has persistent dyspnea after walking less than one block, and her symptoms only partially improve after each dialysis session. Attempts to lower her dry weight have been unsuccessful because of intradialytic hypotension even after her dialysis time is prolonged to 5 hours. Her interdialytic weight gains average 2 kilograms. On physical examination, her blood pressure is 108/70 mmHg, and her heart rate is 102/min. Her right arm arteriovenous fistula is serpiginous, tortuous, and widely dilatated with areas of aneurysmal dilatation. Her jugular venous pressure is 10 cm H2O. Hepatojugular reflux is present. Her cardiac apex is hyper-dynamic. She has modest ascites and trace pretibial edema. Her hemoglobin is 10.7 g/dl, thyroid stimulating hormone is 2 IU/L (reference range 0.6–3.3 IU/L), and free T3 and T4 are normal. A recent echocardiogram shows no significant valvular abnormalities, a small pericardial effusion, normal left ventricular size, mild concentric hypertrophy, normal systolic function, and grade 2 left ventricular diastolic dysfunction..",Which ONE of the following is the MOST appropriate next step in evaluation and management?,"A. Adjustment of erythropoiesis-stimulating therapy, targeting a hemoglobin of 12 g/dl B. Placement of a tunneled catheter for dialysis and ligation of arteriovenous fistula C. Measurement of access flow and cardiac output D. Aggressive paracentesis",C,temp nan,Interventional Nephrology & Dialysis Therapy 2679,"A 39-year-old man has ESRD due to recurrent AKI consequent to cocaine abuse and has required dialysis with tunneled catheters in the past. He has received dialysis through a left forearm arteriovenous graft for the past 6 months. He is found to have a clotted dialysis access when he presents for his scheduled HD treatment. He is in no respiratory distress, and his weight is 2 kilograms above his estimated dry weight. Examination of his left arm reveals no aneurysms or evidence of infection. He does not have facial or arm edema. During endovascular thrombectomy, a 90 percent stenosis of the venous anastomosis is noted. He undergoes successful declotting of the access with angioplasty of the stenosis followed by a stent placement. Central venography shows a 50 per-cent stenosis of the brachiocephalic vein..",Which ONE of the following represents the MOST appropriate next step in management?,"A. Angioplasty of the brachiocephalic vein stenosis without stent placement B. Angioplasty of the brachiocephalic vein stenosis plus a bare metal stent placement C. Angioplasty of the brachiocephalic vein stenosis plus placement of a stent graft D. Placement of a hybrid graft catheter device for central vein stenosis E. No additional intervention",E,temp nan,Interventional Nephrology & Dialysis Therapy 2680,"A 54-year-old kidney transplant recipient has progressive allograft dysfunction and is expected to require dialysis in 4 to 6 weeks. She is now ready to begin preparing for dialysis and is considering all options. She does not have uremic symptoms. She has no contraindications to peritoneal dialysis (PD). The physical examination is remarkable only for modest pedal edema. Laboratory studies show hemoglobin of 10.5 g/dl, potassium of 4.3 meq/L, total CO2 of 23 mmol/L, and serum creatinine level of 4.2 mg/dl. A left forearm brachiocephalic arteriovenous graft has not functioned for over 19 years. Vein mapping shows a patent left upper arm outflow cephalic vein from a previous arteriovenous graft. The veins in her right upper extremity are also suitable for placement of an arteriovenous fistula..",Which ONE of the following is the LEAST appropriate strategy to initiate dialysis in this patient?,"A. Placement of a secondary fistula using the out-flow vein of the previous arteriovenous graft B. Placement of a tunneled central vein catheter closer to the initiation of dialysis C. Placement of a right forearm arteriovenous fistula D. Placement of a PD catheter to start PD E. Placement of an arteriovenous graft closer to the initiation of dialysis",B,temp nan,Interventional Nephrology & Dialysis Therapy 2681,"A 38-year-old man with type 1 diabetes has progressive CKD and is anticipated to require dialysis in about 3 months. After considering his modality options, he indicates a preference for HD. He has recurrent osteomyelitis of the right foot and has been hospitalized four times over the past 6 months for debridement and intravenous antibiotics. He has required placement of multiple peripheral and central intravenous catheters. His most recent laboratory studies show an estimated GFR of 16 ml/min per 1.73 m2..",Which ONE of the following is the BEST preoperative imaging study to plan this patient’s access surgery?,"A. Vascular ultrasonographic mapping of peripheral and central veins B. Small-dose venography to assess peripheral and central veins C. Selective vasculature ultrasonographic mapping of the peripheral veins only if no suitable veins are identified on physical examination",B,temp nan,Interventional Nephrology & Dialysis Therapy 2682,A 45-year-old woman with ESRD is evaluated 2 weeks before scheduled surgery for HD access. Arteriovenous graft placement is planned based on the findings of preoperative vascular ultrasonography. Her surgeon asks you whether she would benefit from extended-release dipyridamole plus aspirin after graft placement..,Which ONE of the following should you tell the surgeon about the expected benefit of using extended-release dipyridamole plus aspirin after arteriovenous graft placement?,"A. Its use will be associated with a clinically meaningful prolongation of the duration of primary unassisted graft patency B. Its use will be associated with no clinically meaningful increase in arteriovenous graft patency C. Its use will improve cumulative patency and decrease graft failure at 1 year D. Its use will have no effect on graft outcomes",B,temp nan,Interventional Nephrology & Dialysis Therapy 2683,"A 45-year-old woman with ESRD who is receiving maintenance HD through a left fore-arm arteriovenous graft is noted to have had increasing venous pressures and declining urea clearance over the past month. A shuntogram shows a 70 percent stenosis at the venous anastomosis. Angioplasty of the lesion is per-formed, and a stent graft is placed at the venous anastomosis..",Which ONE of the following outcomes is predicted from stent graft placement at the venous anastomosis after angioplasty versus angioplasty alone?,"A. No benefit of stent graft placement com-pared with angioplasty alone B. Superior patency at 6, 12, and 24 months C. Increased requirement for subsequent interventions for stenosis at the venous anastomosis at 6 months after stent graft placement D. Higher rate of adverse events after stent graft placement",B,temp nan,Interventional Nephrology & Dialysis Therapy 2684,"A 58-year-old man with ESRD due to IgA nephropathy who is currently receiving maintenance home HD is evaluated during a routine monthly home therapies clinic visit. He received a kidney transplant 10 years ago that failed 5 years ago. Thrombosis occurred in a right forearm arteriovenous fistula at the time of the transplantation. A right upper arm arteriovenous fistula was created several months ago. He has been successfully using the access for home HD 5 days per week over the past 2 months. Blood flows above 450 ml/min have yielded high venous pressures, but no problems have been encountered at lower blood flow rates. On physical examination, he is obese. His body mass index is 41 kg/m2. Extensive collateralization is noted over the right side of his chest. His right arm has minimal edema. The fistula has a strong thrill, and it collapses when raised above the level of the heart. A fistulogram and central venography show bilateral brachiocephalic vein occlusion with extensive collateralization. His laboratory studies show that he is meeting urea clearance targets and that his acid-base balance, electrolytes, and CKD mineral and bone disorder chemistry results are acceptable..",Which ONE of the following is the next BEST step in management?,"A. Treat the left brachiocephalic vein occlusion using radiofrequency wire recanalization B. Establish a new catheter access using the “inside-out” technique and place a hybrid–graft catheter C. Stop using the fistula, do not intervene on the central venous occlusions, and transition the dialysis access to a translumbar catheter D. Continue to use the fistula with blood flows < 450 ml/min",D,temp nan,Interventional Nephrology & Dialysis Therapy 2685,"A 68-year-old man with stage 5 CKD is seen in the outpatient clinic 2 weeks after surgery for a left radiocephalic end-to-side fistula. On physical examination, the fistula is noted to have hyperpulsatility at the arterial anastomosis, a thrill 3 cm from the anastomosis, and a small caliber outflow vein. Ultrasonography of the fistula shows a juxta-anastomotic stenosis. He asks about the significance of this finding..",Which ONE of the following should you tell him about the significance of the juxta-anastomotic stenosis seen on ultrasonography?,"A. It is associated with an increased risk of unassisted fistula maturation failure in comparison with no stenosis B. The stenosis will persist on repeated ultrasonographic imaging at 6 weeks C. It carries a higher risk for unassisted maturation failure in comparison with distal stenoses D. It is more likely to be associated with intimal hyperplasia when compared to distal stenoses",A,temp nan,Interventional Nephrology & Dialysis Therapy 2686,You have recently been appointed medical director of your practice’s dialysis program. The program uses measurement of access flow as its surveillance strategy. The staff requests that you provide education about the various methods of access surveillance..,Which ONE of the following should you tell them regarding the diagnostic accuracy of available surveillance methods?,"A. Surveillance methods are equally sensitive in identifying both inflow and outflow stenosis in both fistula and graft B. Identification of access flows < 650 ml/min is the best method to detect outflow stenosis C. The optimal tests for identifying an inflow stenosis are access flows < 650 ml/min or the combination of a positive physical examination result plus access flows < 650 ml/min D. Combining physical examination results with a venous access pressure ratio > 0.5 improves diagnostic performance of the tests in detecting outflow stenosis",C,temp nan,Interventional Nephrology & Dialysis Therapy 2687,"A 42-year-old woman with ESRD due to lupus nephritis who is receiving maintenance in-center HD is evaluated during dialysis rounds. The dynamic and derived static venous pressures of the left brachiocephalic fistula have recently risen to > 0.55. On physical examination, the fistula has a smooth thrill with good augmentation, and there is absence of partial vein collapse upon arm elevation. A fistulogram identifies a 60 percent outflow stenosis that is successfully treated with angioplasty..",Which ONE of the following is the MOST likely outcome of this intervention?,"A. A significantly reduced risk of permanent access loss B. A reduced risk of subsequent fistula thrombosis C. A reduced rate of subsequent surgical interventions D. An increased rate of complication-related hospitalizations",B,temp nan,Interventional Nephrology & Dialysis Therapy 2688,"A 72-year-old man with ESRD due to diabetic nephropathy who is receiving maintenance HD is found to have a thrombosed arteriovenous fistula upon presentation to his usual scheduled dialysis treatment on a Friday before a holiday weekend. The dialysis staff refer the patient to the local emergency department. He is asymp-tomatic. On physical examination, his weight exceeds his dry weight by 2 kg. His blood pressure is 136/80 mm Hg. The lung fields are clear, and there is trace pedal edema. Laboratory studies show a potassium of 5.2 meq/L and a total CO2 of 20 mmol/L. He is admitted for observation, and a fistula thrombectomy is scheduled for the following Tuesday (4 days later) because interventional services are not available on Monday. You are concerned about delaying dialysis and contact the interventional radiologist in an attempt to expedite the pro-cedure. The radiologist offers catheter placement tomorrow. You express concerns about delaying endovascular thrombectomy..",Which ONE of the following should you tell the radiologist regarding outcomes in delaying fistula thrombectomy?,"A. Immediate procedural success is more likely when thrombectomy is performed within 24 hours of the time of thrombosis diagnosis B. Patients undergoing fistula thrombectomy within 24 hours of diagnosis have higher patency rates at 3 months than do patients treated later C. Thrombectomy performed > 48 hours after thrombosis diagnosis is associated with higher rates of complications D. Thrombectomy performed > 48 hours adversely influences his fistula patency at 3 and 6 months because of his diabetes diagnosis",B,temp nan,Interventional Nephrology & Dialysis Therapy 2689,"A 73-year-old woman with stage 5 CKD decides to pursue PD. Her past surgical history includes two Caesarean sections and an un-complicated laparoscopic appendectomy. She is curious about the PD catheter placement procedure, and she wonders whether her prior surgeries will complicate or preclude catheter placement..",Which ONE of the following should you tell her about PD catheter placement?,"A. Percutaneous placement is contraindicated because of her prior appendectomy B. Advanced laparoscopic placement has the lowest mechanical complication rate C. Advanced laparoscopic placement is associated with an increased risk for subsequent need to change her modality to HD D. Fluoroscopic catheter placement has a superior rate of 1-year overall catheter survival compared with advanced laparoscopic catheter placement",B,temp nan,Interventional Nephrology & Dialysis Therapy 2690,"A 56-year-old man with stage 5 CKD due to diabetic nephropathy is seen in follow up 1 week after PD catheter insertion. He has increased fatigue, loss of appetite, and dysguesia. He asks about the optimal timing for initiation of PD..",Which ONE of the following should you tell him based on the Timely PD study?,"A. Patients starting PD one week after catheter placement had significantly higher leaks in comparison with patients who started after 2 weeks B. Patients starting PD 1 week after catheter placement had significantly higher leaks in comparison with patients who started after 4weeks C. Patients with diabetes waiting 4 weeks after catheter placement to initiate PD had higher success with the technique D. Embedded catheters were a significant part of the study",B,temp nan,Interventional Nephrology & Dialysis Therapy 2691,,"Question 2691: Which ONE of the following statements about predialysis education is CORRECT?","A. Patient-targeted educational interventions increase the odds of receiving PD as the initial treatment modality B. Unplanned or planned dialysis initiation does not affect patient outcomes C. Patients with unplanned dialysis initiation do not benefit from education about therapy options D. Small group dialysis option education does not affect the rate at which patients choose PD",A,temp nan,Interventional Nephrology & Dialysis Therapy 2692,"A 42-year-old man receiving maintenance HD is dialyzed through a left upper extremity brachiocephalic fistula. During weekly rounds, you notice that he is wearing a glove on his left hand. He indicates that he has had pain and numbness in the left hand over the past 3 weeks and that his symptoms worsen during dialysis. His blood pressure is 130/80 mmHg. A physical examination reveals that his hand is slightly cool to touch. The left radial pulse is normal. There are no necrotic or ischemic ulcers. His left hand strength and sensation are normal. You suspect arteriovenous access ischemic steal syndrome..",Which ONE of the following is the MOST appropriate next step in management?,"A. Ultrasonographic flow measurements and left arm arteriogram B. Recommend hand-grip exercises only C. Start amlodipine and aspirin D. Referral for immediate surgical ligation E. Change his dialysis access to a tunneled central venous catheter and monitor the left hand",A,temp nan,Interventional Nephrology & Dialysis Therapy 2693,"A 67-year-old man with stage 4 CKD is seen in consultation in the surgical intensive care unit for oliguric AKI after emergency cholecystectomy for necrotizing cholecystitis. He requires initiation of intermittent HD for progressive fluid overload. A right internal jugular vein catheter is placed at the bedside under ultrasonographic guidance. He has a history of heparin-associated thrombocytopenia. The catheter is locked with a concentrated solution of trisodium citrate (46.7 percent). Several hours later he experiences perioral numbness, confusion, tachypnea, and hand paresthesias. The oxygen saturation is stable at 92 percent on ambient air. On physical examination, he is somewhat agitated and anxious. His blood pressure is 110/80 mmHg, his respiratory rate is 24/min, his heart rate is 96/min, and his temperature is 36.7C. Auscultation of the chest reveals symmetric breath sounds. There is 1+ peripheral edema. The surgical incision is without erythema or induration. A Trousseau sign is present. The remaining results of the examination are unremarkable. A basic metabolic panel shows sodium 143 meq/L, potassium 5.6 meq/L, chloride 96 meq/L, total CO2 29 mmol/L, BUN 96 mg/dl, and creatinine 5.6 mg/dl. The arterial blood gas shows pH 7.48, PCO2 32 mmHg, and PO2 82 mmHg. An electrocardiogram shows normal sinus rhythm with a prolonged QTc interval of 0.5 milliseconds..",Which ONE of the following is the MOST likely cause of his symptoms?,"A. Pulmonary embolus B. A complication from use of the citrate lock solution C. A panic attack D. Venous air embolism",B,temp nan,Interventional Nephrology & Dialysis Therapy 2694,"You are contacted by the dialysis staff for recommendations regarding the management of HD catheter dysfunction. The patient is a 56-year-old woman with ESRD due to focal and segmental glomerulosclerosis. She has exhausted all options for HD access and is currently dialyzed through a right femoral tunneled central venous catheter. Over the past two dialysis treatments, the catheter has been functioning poorly, delivering blood flows of < 300 ml/min. The blood flows do not improve after instillation of intracatheter thrombolytic therapy that dwelled for 48 hours..",Which ONE of the following is the MOST appropriate next step in management?,"A. Remove the catheter and place a translumbar or transhepatic catheter B. Flush the catheter with isotonic saline solution C. Exchange the tunneled catheter over a guide wire followed by warfarin anticoagulation D. Exchange the tunneled catheter over a guidewire with balloon angioplasty to disrupt the fibrin sheath",D,temp nan,Interventional Nephrology & Dialysis Therapy 2695,"A dialysis technician asks you to examine a patient’s access before cannulation. The patient is a 53-year-old man with ESRD due to diabetic kidney disease on in-center HD for 7 years. He has been dialyzed via a left upper extremity brachiocephalic fistula since dialysis initiation. Over the years, the diameter of the fistula increased in size. On physical examination, the fistula has hypopigmented, necrotic skin surrounded by thin, shiny skin overlying an area of aneurysm dilation. A small amount of dried blood with minimal fresh oozing of blood is visible at this site..",Which ONE of the following is the BEST next step in management?,"A. Advise the technician to avoid cannulation in the areas of breakdown, mark alternative sites for cannulation, and monitor the access closely B. Refer the patient immediately to the vascular surgeon for evaluation for impending rupture of the arteriovenous access C. Draw blood cultures and start the patient on empiric antibiotics D. Apply a pressure dressing, rest the fistula for several weeks, and refer the patient for a tunneled dialysis catheter placement",B,temp nan,Interventional Nephrology & Dialysis Therapy 2696,You are discussing dialysis options with a 36-year-old woman with stage 5 CKD due to autosomal dominant polycystic kidney disease. She has a left radiocephalic arteriovenous fistula that has an excellent thrill and that augments and collapses well on examination. She is listed for transplantation but is now symptomatic and ready to begin renal replacement therapy. She is interested in short daily home HD and wants your opinion on the effect of intensive HD on her arteriovenous access..,Which ONE of the following should you tell her regarding management of vascular access during frequent HD?,"A. A central venous catheter is the preferred access for frequent HD because it minimizes the risk of needle dislodgement during dialysis B. Her radiocephalic arteriovenous fistula is an excellent choice for vascular access for intensive HD C. Recent randomized controlled trials suggest that intensive HD does not increase the risk of vascular access events D. An immediate-use arteriovenous graft is preferable to a standard polytetrafluoroethylene graft for intensive HD",B,temp nan,Interventional Nephrology & Dialysis Therapy 2697,"A 32-year-old woman in stage 5 CKD is evaluated at a routine monthly office visit. She has fatigue, mild intermittent nausea, and dysguesia. She has maintained her weight, and the results of her current laboratory studies are acceptable. Six weeks ago, she underwent creation of a radiocephalic arteriovenous fistula. On physical examination, the fistula is poorly developed and does not augment well on examination. An ultrasonographic evaluation shows a superficial fistula, with a diameter of 4 mm and blood flow of 350 ml/min..",Which ONE of the following is the MOST appropriate next step in management?,"A. Recommend continuing hand grip exercises to assist in fistula maturation because it has not yet had adequate time to mature B. Refer the patient for a low-dose contrast angiogram, including direct arteriogram C. Repeated ultrasonographic measurements in 6 weeks to avoid radiocontrast exposure D. Abandon the fistula, monitor her closely, and place an immediate-use arteriovenous graft or catheter when she needs to start dialysis",B,temp nan,Interventional Nephrology & Dialysis Therapy 2698,"A 49-year old man is evaluated during HD rounds. He recently began HD through a right internal jugular central venous catheter. He has a nonmaturing radiocephalic fistula that was created 8 weeks ago. On physical examination, the fistula has a weak thrill and does not augment well on examination. A large side-branch accessory vein that is > 60 percent of the diameter of the main fistula takes off within 5 cm of the anastomosis. A thrill persists after occlusion of the fistula downstream from the side-branch vein. You estimate that about 30 to 40 percent of the outflow is diverted through the large side-branch vein. He asks for your recommendations regarding the significance of the accessory vein and its management..",Which ONE of the following should you tell him about current recommendations regarding accessory veins in nonmaturing fistulae?,"A. Accessory veins are significant contributors to nonmaturity in fistulas and must always be ligated to allow maturation B. An accessory vein is usually pathologic and is the result of an alternative channel that develops to bypass a stenotic area C. In nonmaturing fistulas, accessory veins should be ligated if they divert> .25 percent of the outflow D. Accessory veins are a common cause of late fistula failure, and surgical ligation is preferred over endovascular coil placement",C,temp nan,Interventional Nephrology & Dialysis Therapy 2699,"A 46-year-old woman with type 1 diabetes mellitus and hypertension who has required HD for the past 12 years is evaluated for permanent HD access. She has had a left forearm radiocephalic fistula that never matured, a left upper extremity brachiobasilic fistula, and a left upper extremity arteriovenous graft, all of which are thrombosed. No upper extremity veins are visible on physical examination. She has scars of previous central venous catheters on both sides. Venous mapping by ultrasonography on the right arm shows a 2.6-mm radial artery and a 3.2-mm brachial artery. The cephalic vein is not visible in the forearm but measures 3.2 mm at the elbow, 3.6 mm at the midarm, and 4.2 mm at the upper arm. The basilic vein measures 3.4 mm at the elbow and 4.8 mm in the axilla. Both veins are compressible throughout their course..",Which ONE of the following is the BEST next step in management?,"A. Plan right brachiocephalic fistula B. Plan a right forearm arteriovenous graft to allow veins in the upper extremity to develop for a secondary fistula C. Plan contrast venography of her central veins to evaluate for unsuspected central venous stenosis D. Plan a right brachiobasilic arteriovenous fistula because the cephalic vein is not visualized in the forearm",C,temp nan,Interventional Nephrology & Dialysis Therapy 2700,,"Question 2700: Which ONE of the following criteria is MOST consistent with a diagnosis of a stenotic lesion of an arteriovenous fistula by ultrasonography?","A. A reduction in luminal diameter by 25 percent B. Decreased peak systolic flow velocity C. A twofold increase in peak systolic flow velocity ratio D. Low-resistance Doppler waveform E. An increase in access flow volume",ANSWER FOR ID: 2700 NOT FOUND,temp nan,SUBJECT FOR ID: 2700 NOT FOUND 2701,"A 78-year-old man is evaluated because of a 2-month history of progressive generalized weakness, decreased appetite, nausea, and difficulty urinating. On physical examination, he is bradycardic and hypertensive. He has mild hypervolemia but is not in acute respiratory distress. Further evaluation shows a serum creatinine level of 11 mg/dl, anemia, hyperkalemia, and metabolic acidosis. The findings on bedside kidney ultrasonography show dilated pelvis in the right kidney..",Which ONE of the following diagnoses BEST describes the ultrasonography findings?,"A. Renal masses B. Hydronephrosis C. Dilatated renal vessels D. Acquired cystic disease of the kidney E. Autosomal dominant polycystic kidney disease",B,temp nan,Interventional Nephrology & Dialysis Therapy 2702,You hire two advanced practice providers without experience in dialysis care to assist in the treatment of patients in your dialysis program. They are interested in learning about physical examination findings that indicate arteriovenous fistula access stenosis..,Which ONE of the following findings should you tell them is associated with inflow stenosis?,"A. Edema in the access arm B. A high-pitched systolic bruit in the proximal venous segment of the access C. Absence of venous collapse upon arm elevation D. A discontinuous thrill with only a systolic component at the juxta-anastomotic site E. Augmentation of the pulse on fistula occlusion",D,temp nan,Interventional Nephrology & Dialysis Therapy 5535," You are asked to evaluate a 73-year-old retired dentist who has CKD secondary to focal and segmental glomerulosclerosis. He was first seen 3 years ago when his serum creatinine level was 2.3 mg/dl (eGFR 28 ml/min per 1.73 m2). He had received steroid therapy, and his hypertension has been managed with angiotensin blockade. He has not returned to your office for the past 2 years. One year ago, he had a myocardial infarction, requiring an angioplasty and has been followed up regularly by his cardiologist for control of hypertension and hypercholesterolemia. You obtain laboratory values, which reveal that his serum creatinine is now 2.8 mg/dl (eGFR 22 ml/min per 1.73 m2), serum calcium is 9.0 mg/ml, phosphorus is 3.6 mg/dl, alkaline phosphatase is 100 U/ml (reference range 36–92 U/ml), and intact PTH is 122 pg/ml (reference range 12-72 pg/ml). The 25-hydroxyvitamin D level is 30 ng/ml (reference range 20–80 ng/ml). The serum albumin is 3.6 g/dl, and the urine protein-to-creatinine ratio is 900 mg/g..",Which ONE of the following is the MOST appropriate management?,"1. Cholecalciferol 2000 units daily 2. Calcifediol 30 mg daily 3. No further evaluation or therapy is required 4. Start a phosphate binder 5. Start calcitriol 0.25 µg per day",C,temp nan,"Disorders of Divalent Ions, Renal Bone Disease and Nephrolithiasis" 5536," A 64-year-old man with ESRD has been receiving hemodialysis for 2 months and transfers to your unit. The initial laboratory values show that serum calcium is 10.2 mg/dl, the phosphorus is 5.7 mg/dl, the serum PTH is 594 pg/ml, and the alkaline phosphatase is 214 U/ml (reference range 36–92 U/ml). He had been prescribed sevelamer carbonate 2400 mg three times daily with meals..",Which ONE of the following is the next BEST step in management?,"1. Begin therapy with calcitriol 2 µg IV with each dialysis 2. Begin cinacalcet therapy 30 mg daily 3. Add lanthanum carbonate 1 gram 3 times daily with meals 4. Begin paricalcitol 4 µg with each dialysis",B,temp nan,"Disorders of Divalent Ions, Renal Bone Disease and Nephrolithiasis" 5537," A 74-year-old woman has been receiving hemodialysis for 4 years. She has a history of severe peripheral vascular disease, coronary artery disease, renal artery stenosis, and chronic atrial fibrillation. She describes having aches and pains in the hips, legs, shoulders, and lower back. She has taking paricalcitol 2 µg with each dialysis session and takes calcium acetate 1334 mg with each meal. Her PTH values have ranged from 312 to 479 pg/ml (reference range 12–72 pg/ml) over the past 12 months. Her serum calcium values have ranged from 9.8 to 10.1 mg/dl, serum phosphorus values have ranged from 4.7 to 5.4mg/dl, and alkaline phosphatase values have ranged between 389 and 492 U/ml (reference range 36–92 U/ml). The serum gamma glutamyl transpeptidase and bilirubin are normal. The 25-hydroxyvitamin D level is 32 ng/ml (reference range 20–80 ng/ml). Her primary doctor had obtained a dual-energy x-ray absorptiometry (DEXA) scan, which showed a T score of -2.9 at the hip and -2.5 at the lumbar spine..",Which ONE of the following is the MOST appropriate management?,"1. Increase paricalcitol 2. Increase calcium acetate 3. Calcifediol 30 µg daily 4. Add cinacalcet",D,temp nan,"Disorders of Divalent Ions, Renal Bone Disease and Nephrolithiasis" 5538," A 65-year-old man with ESRD who has been receiving maintenance hemodialysis for 2 months is evaluated during weekly dialysis rounds. Laboratory values are calcium 10.4 mg/dl, phosphorus 6.5 mg/dl, PTH 745 pg/ml (reference range 12–72 pg/ml), and alkaline phosphatase 212 U/ml (reference range 36–92 U/ml). He has been taking calcium acetate 2001 mg with each meal..",Which ONE of the following statements regarding cinacalcet is CORRECT?,"1. Addition of cinacalcet is appropriate based on this patient’s laboratory findings 2. Cinacalcet therapy is not associated with a reduction of serum phosphorus 3. Cinacalcet is associated with decreased cardiovascular and all-cause mortality 4. The addition of cinacalcet will likely lead to increased vascular calcification 5. Cinacalcet therapy will decrease PTH levels by 15% to 20%",A,temp nan,"Disorders of Divalent Ions, Renal Bone Disease and Nephrolithiasis" 5541," Both calcimimetic therapy and parathyroidectomy can effectively control secondary hyperparathyroidism in patients with ESRD..",Which ONE of the following statements is CORRECT?,"1. Cinacalcet and parathyroidectomy are as-sociated with a similar decrease in serum calcium 2. The effect of increasing bone mineral density with calcimimetic therapy and parathyroidectomy are comparable 3. Both cinacalcet and parathyroidectomy de-crease levels of fibroblast growth factor-23 (FGF23) 4. Etelcalcetide reduces serum PTH but does not affect levels of FGF23",C,temp nan,"Disorders of Divalent Ions, Renal Bone Disease and Nephrolithiasis" 5542," It has been demonstrated that levels of FGF23 increase early in the course of CKD and have been associated with an increase in mortality..",Which ONE of the following is CORRECT?,"1. FGF23 increases the production of inflammatory cytokines 2. FGF23 increases erythropoietin production 3. Erythropoietin therapy decreases the levels of FGF23 in circulation 4. Increased FGF23 levels are associated with decreased cardiac output 5. Iron deficiency decreases FGF23 production",A,temp nan,"Disorders of Divalent Ions, Renal Bone Disease and Nephrolithiasis" 5543," Your internal medicine colleague asks you about the evaluation and management of bone disease in an 80-year-old woman with advanced stage 4 CKD. Over the past 3 years, she has experienced worsening kyphoscoliosis and vertebral body collapse, with associated pain. Laboratory studies show a serum PTH level of 436 pg/ml (reference range 12–72 pg/ml), calcium level of 9.3 mg/dl, phosphorus of 4.9 mg/dl, alkaline phosphatase level of 156 U/ml (reference range 36–92 U/ml), and serum creatinine level of 2.9 mg/dl (eGFR 16 ml/min per 1.73 m2). A DEXA scan shows a T-score of -2.8 at the hip and -2.9 at the lumbar spine..",Which ONE the following should you tell your colleague about the evaluation and management of bone disease in CKD stage G3a–G5D?,"1. Measurements of DEXA are useful in predicting the underlying bone histologic features in patients with CKD stage G3a–G5D 2. DEXA is predictive of fracture risk in adults with CKD stage G3a–G5D 3. Parathyroidectomy has not been associated with increases in bone density 4. Denosumab may lead to hypercalcemia in patients with CKD",B,temp nan,"Disorders of Divalent Ions, Renal Bone Disease and Nephrolithiasis" 5544," A 43-year-old man received a deceased donor kidney transplant after 7 years of dialysis. Two months later, the serum creatinine is 1.4 mg/dl (stable), the serum calcium is 11.2 mg/dl, the phosphorus is 1.7 mg/dl, and the serum PTH is 246 pg/ml (reference range 12–72 pg/ml). A recent DEXA scan shows normal bone mineral density..",Which ONE of the following statements is CORRECT?,"1. He should be prescribed paricalcitol to decrease PTH 2. He has tertiary hyperparathyroidism and needs urgent parathyroidectomy 3. The majority of such patients are expected to achieve remission of hyperparathyroidism after 1 year of cinacalcet therapy 4. Cinacalcet therapy would be expected to lower serum phosphorus further",C,temp nan,"Disorders of Divalent Ions, Renal Bone Disease and Nephrolithiasis" 5545," A 27-year-old man is evaluated for hypocalcemia. He has been experiencing paresthesias for the past 6 months and recently had an episode in which he experienced a prolonged and painful cramp of his right hand. He has otherwise been in good health. A physical examination reveals that Chvostek and Trousseau signs are present. The serum electrolytes and creatinine are normal. The serum calcium is 6.7 mg/dl, the ionized calcium is 0.5 mmol/L (reference range 1.10 to 1.35 mmol/L), the serum magnesium is 2.0 mg/dl, and the serum phosphorus is 5.8 mg/dl..",Which ONE of the following genetic abnormalities BEST explains this clinical condition?,"1. A gain of function mutation in the calcium sensing receptor gene 2. A loss of function mutation in the calcium sensing receptor gene 3. A mutation in the adaptor-related protein complex 2 sigma subunit 1 (AP2S1) gene 4. Loss of function mutation in G-protein α- 11",A,temp nan,"Disorders of Divalent Ions, Renal Bone Disease and Nephrolithiasis" 5546," A 52-year-old African American woman with ESRD, hypertension, and diabetes mellitus has been treated with multiple daily insulin injections for the past 10 years and maintenance in-center hemodialysis for the past 4 years. The patient’s serum calcium and phosphate levels have been maintained with in the range suggested by the KDIGO 2017 Clinical Practice Guideline. PTH levels have ranged between 486 and 592 pg/ml (reference range 12–72 pg/ml). She had been treated with activated vitamin D in an attempt to control her hyperparathyroidism..","In assessing this patient’s risk for calcific uremic arteriolopathy, which ONE of the following factors is MOST likely associated with an increased risk of the complication?","1. Hypertension 2. Her ethnicity 3. Her use of activated vitamin D 4. Her treatment with multiple insulin injections",D,temp nan,"Disorders of Divalent Ions, Renal Bone Disease and Nephrolithiasis" 5547," A 64-year-old woman with a 15-year history of essential hypertension successfully managed with hydrochlorothiazide therapy is evaluated during a routine follow-up visit and is found to have hypercalcemia. Her physical examination is unremarkable. She describes having increased difficulty sleeping, and she notes persistent constipation. A colonoscopy performed last year was unremarkable. Laboratory values show serum sodium of 145 mEq/L, potassium of 4.6 mEq/L, chloride of 105 mEq/L, total CO2 of 25 mmol/L, BUN of 22 mg/dl, creatinine of 1.4 mg/dl, calcium of 11.4, and phosphorus of 2.1 mg/dl. The 24-hour urine calcium excretion is 262 mg. The results of radiologic studies including chest radiograph and abdominal computed tomography (CT) are normal..",Which ONE of the following is the MOST likely primary cause of her hypercalcemia?,"1. Primary hyperparathyroidism 2. Familial hypocalciuric hypercalcemia 3. Thiazide-induced hypocalciuria 4. Occult malignancy 5. Sarcoidosis",A,temp nan,"Disorders of Divalent Ions, Renal Bone Disease and Nephrolithiasis" 5548," A 67-year-old man with recent onset of ESRD resulting from diabetic kidney disease is evaluated 1 week after starting hemodialysis. He is asymptomatic and is up to date on all age-appropriate cancer screenings. Routine laboratory studies show a hemoglobin of 11.1 gm/dl, albumin of 3.5 g/dl, creatinine of 6.7 mg/dl, calcium of 9.8 mg/dl, phosphorus of 3.7 mg/dl, and PTH of 363 pg/ml (reference range 12–72 pg/ml)..","At this point, which ONE of the following would you recommend next?","1. Begin calcitriol 2. Begin cinacalcet therapy 3. Begin phosphate binder therapy 4. Measure ionized calcium 5. Measure PTH-related protein",D,temp nan,"Disorders of Divalent Ions, Renal Bone Disease and Nephrolithiasis" 5549," A 61-year-old man is referred for evaluation of hypercalcemia. He has had 2 months of progressive weakness, bone pain, nausea, and anorexia. Over the past 3 weeks, he has experienced intense thirst and polyuria associated with weight loss of 8 kg. Laboratory studies showed a serum calcium of 12.2 mg/dl and a serum PTH of 12 pg/ml (reference range 12-72 pg/ml). The serum sodium was 144 mEq/L, potassium was 3.6 mEq/L, chloride was 96 mEq/L, total CO2 was 30 mmol/L, BUN was 35 mg/dl, creatinine was 1.6 mg/dl (increased from 0.9 mg/dl 1 month ago), and glucose was 78 mg/dl. The patient denied use of oral vitamin D supplements, but the 25-hydroxyvitamin D concentration exceeded the analytic range of the automated analyzer. Further studies showed a monoclonal protein immunoglobulin G with elevated serum free lambda light chains (73.4 mg/dl; reference range 0.83–2.7 mg/dl). The 1,25-dihydroxyvitamin D level was 28 pg/ml (reference range 25–65 pg/ml)..",Which ONE of the following is the MOST likely cause of this patient’s laboratory profile?,"1. Surreptitious intake of vitamin D 2. Sarcoidosis 3. Monoclonal protein causing an artefactual increase in 25-hydroxyvitamin D 4. Plasma cell-induced stimulation of 25-hydroxyvitamin D synthesis",C,temp nan,"Disorders of Divalent Ions, Renal Bone Disease and Nephrolithiasis" 5550," A 27-year-old man with ESRD secondary to focal and segmental glomerulosclerosis is evaluated 3 weeks after deceased donor kidney transplantation. He had delayed graft function requiring 2 weeks of postoperative hemodialysis. A recent biopsy showed no evidence of rejection and acute tubular injury. Laboratory studies show a sodium of 138 mEq/L, potassium of 5.7 mEq/L, chloride of 107 mEq/L, total CO2 of 19 mmol/L, BUN of 46 mg/dl, creatinine of 3.8 mg/dl, glucose of 126 mg/dl, calcium of 10.3 mg/dl, magnesium of 1.0 mg/dl, and phosphorus of 5.2 mg/dl..","In addition to the patient’s immunosuppressive medications, which ONE of the following fac-tors is associated with an increased risk for the development of diabetes mellitus in the year after transplantation?","1. Hypomagnesemia 2. Hyperkalemia 3. Hypercalcemia 4. Hyperphosphatemia",A,temp nan,"Disorders of Divalent Ions, Renal Bone Disease and Nephrolithiasis" 5551," A 58-year-old woman is referred for evaluation and management of sarcoidosis-related hypercalcemia. She presented with fever, arthralgias, and fatigue of 2 weeks duration. Laboratory studies showed a serum calcium of 11.4 mg/dl, a serum phosphorus of 4.0 mg/dl, a BUN of 42 mg/dl, a creatinine of 1.6 mg/dl, a PTH level of 10 pg/ml (reference range 12–72 pg/ml), a 25-hydroxyvitamin D level of 35 ng/ml (reference range 20–80 ng/ml), and a 1,25-dihydroxy vitamin D level of 106 pg/ml (reference range 25–65 pg/ml). CT of the chest revealed pretracheal and paratracheal, subcarinal, and bilateral hilar lymph node enlargement. Transbronchial needle aspiration of a subcarinal lymph node revealed noncaseating granulomatous lymphadenitis. The patient was treated with prednisone 40 mg daily. The acute kidney injury and hypercalcemia improved, but she gained 9 kg in weight, she had difficult-to-control hyperglycemia, and a Cushingoid appearance developed..","At this point, which ONE of the following would you recommend for treatment of this woman’s hypercalcemia?","1. Zoledronic acid 2. Calcitonin 3. Denosumab 4. Phenytoin 5. Cinacalcet",A,temp nan,"Disorders of Divalent Ions, Renal Bone Disease and Nephrolithiasis" 5552," You are asked to see a 44-year-old man in consultation for severe hypercalcemia. He has had progressive fatigue and generalized bone pain over the past 3 months. He does not have confusion, polyuria, polydipsia, renal colic, nausea, constipation, or abdominal pain. He was previously in excellent health and was taking no medications except for acetaminophen on an as needed basis for bone pain. Laboratory data showed a serum sodium of 140 mEq/L, potassium of 4.5 mEq/L, chloride of 115 mEq/L, total CO2 of 24 mmol/L, anion gap of 1 mmol/L, BUN of 20 mg/dl, creatinine of 0.9 mg/dl, glucose of 78 mg/dl, serum calcium of 15.9 mg/dL, albumin of 3.9 g/dl, phosphorus of 2.3 mg/dl, PTH of 19 pg/ml (reference range 12–72 pg/ml), 25-hydroxyvitamin D level of 32 ng/ml (reference range 20–80 ng/ml), hemoglobin of 10.5 g/dl, and erythrocyte sedimentation rate of 45 mm/h (reference range 0–22 mm/h)..",Which ONE of the following should you recommend at this time?,"1. Zoledronic acid 2. Hemodialysis 3. Ionized calcium 4. Calcitonin 5. PTH-related protein level",C,temp nan,"Disorders of Divalent Ions, Renal Bone Disease and Nephrolithiasis" 5553,,"Question 5553 Which ONE of the following statements regarding the association between uric acid and kidney stone formation is CORRECT?","1. Increased serum uric acid is a risk factor for stone formation in men without a previous history of stone formation 2. Lowering serum uric acid reduces the risk of calcium stone formation 3. Increased serum uric acid is a risk factor for stone formation in women without a previous history of stone formation 4. Serum uric acid predicts the risk of uric acid stone, but not calcium stone, formation 5. Patients with hyperuricemia usually have hyperuricosuria",A,temp nan,"Disorders of Divalent Ions, Renal Bone Disease and Nephrolithiasis" 5554," A 22-year-old man with a history of a motor vehicle accident 5 months ago is evaluated in the emergency department for worsening and possibly infected sacral decubitus ulcers and intermittent substernal chest pain. He sustained multiple sacral, pelvic, and femur fractures, and he has remained bedridden at home. He has had intractable pain that has precluded full participation in home physical therapy. A physical examination reveals multiple sacral and heel decubitus ulcers with serosanguinous drainage. He is unable to walk, sit, or roll over without assistance secondary to pelvic girdle pain. The result of his cardiopulmonary examination is normal except for resting tachycardia. Laboratory studies reveal normal levels of serum sodium, potassium, chloride, bicarbonate, and serum troponin. The EKG shows shortened QoTc (b180 ms) and QaTc (b300 ms) intervals as well as ST segment/T wave abnormalities..",Which ONE of the following is the next BEST step in the evaluation of this patient?,"1. Refer for cardiac catheterization 2. Chest CT 3. Serum calcium 4. Serum magnesium 5. Cardiac ultrasonography",C,temp nan,"Disorders of Divalent Ions, Renal Bone Disease and Nephrolithiasis" 5555," A 50-year-old woman is referred for evaluation of recurrent nephrolithiasis. She has had a long history of recurrent urinary tract infections and has received multiple antibiotics in an attempt to suppress her infections. The results of her laboratory studies are serum sodium 138 mEq/L, potassium 3.9 mEq/L, chloride 105 mEq/L, total CO2 24 mmol/L, BUN 18 mg/dl, creatinine 1.2 mg/dl, calcium 8.9 mg/dl, albumin 4.0 g/dl, magnesium 2.3 mg/dl, and uric acid 4 mg/dl. A urinalysis reveals a pH of 7 and many leukocytes and bacteria on microscopic examination. Stone analysis in the past was consistent with struvite stones. A flat plate radiograph of the abdomen shows a right staghorn calculus..","In addition to a urine culture, which ONE of the following is the MOST appropriate next diagnostic test for this patient?","1. 24-hour urine metabolic evaluation 2. Urine osmolal gap 3. Serum PTH level 4. Serum 1,25-dihydroxyvitamin D level",A,temp nan,"Disorders of Divalent Ions, Renal Bone Disease and Nephrolithiasis" 5556," A 29-year-old man was evaluated in the emergency department because of 5 days of persistent emesis. He had previously been in good health, with no interactions with the health care system. He admitted to habitual use of cannabis. He described muscle weakness as well. A physical examination revealed a well-nourished man, who vomited once during the initial evaluation. The blood pressure was 110/80 mmHg, and the heart rate was 110 mmHg. He had minimal weakness of his upper and lower extremities. Otherwise, the physical examination was normal. Laboratory studies showed a serum sodium of 141 mEq/L, potassium of 3.7 mEq/L, chloride of 105 mEq/L, total CO2 of 21 mmol/L, BUN of 18 mg/dl, and creatinine of 0.84 mg/dl..","At this point, which ONE of the following would you expect on further laboratory assessment?","1. Hypophosphatemia 2. Hyperkalemia 3. Hyperammonemia 4. Hypoalbuminemia",A,temp nan,"Disorders of Divalent Ions, Renal Bone Disease and Nephrolithiasis" 5557," A 32-year-old woman is referred for evaluation of hypophosphatemia. She had previously been in excellent health, but over the past year she has experienced fatigue, diffuse muscle pain, and generalized weakness. She is not taking any medications. She has no family history of bone or mineral metabolism disorders. She had normal growth and development during childhood. She does not use alcohol or tobacco. Her general physician treated her for iron deficiency anemia because of heavy menstrual bleeding 7 weeks earlier. She received 2 doses of parenteral ferric carboxymaltose 500 mg before presentation to your office. She does not have diarrhea or weight loss. The result of a physical examination is unremarkable. The results of laboratory studies, including serum electrolytes, calcium, magnesium, and albumin, are all normal except for a serum phosphorus level of 1.6 mg/dl..",Which ONE of the following is the MOST likely cause of her hypophosphatemia?,"1. Oncogenic osteomalacia 2. Occult hyperparathyroidism 3. Poor phosphate intake 4. Ferric carboxymaltose infusions 5. Autosomal dominant hypophosphatemia",D,temp nan,"Disorders of Divalent Ions, Renal Bone Disease and Nephrolithiasis" 5558," A 75-year-old woman is evaluated for diffuse joint and bone pain, most prominent in the low back and hips, for the past 6 months. She has had no other significant medical problems. She takes ibuprofen for her pain. The physical examination shows diffuse muscle weakness. The rest of the examination results are unremarkable. Laboratory studies show normal serum electrolytes, a BUN of 16 mg/dl, creatinine of 1.0 mg/dl, calcium of 9.8 mg/dl, phosphorus of 1.6 mg/dl, fractional excretion of filtered phosphate of 18%, PTH of 20 pg/ml (reference range 12–72 pg/ml), and 1,25-dihydroxyvitamin D level of 30 pg/ml (reference range 25–65 pg/ml). Radiographs of the pelvis and hips reveal osteopenia with thinning of the cortex and Looser pseudofractures. Spine radiographs show concave vertebral bodies (codfish vertebrae) with large and biconvex vertebral disks..",Which ONE of the following is the MOST likely cause of this patient’s clinical problem?,"1. Ankylosing spondylitis 2. Adynamic bone disease 3. Tumor-induced osteomalacia 4. Osteitis fibrosa cystica",C,temp nan,"Disorders of Divalent Ions, Renal Bone Disease and Nephrolithiasis" 5559,,"Question 5559 Higher levels of which ONE of the following factors, even if within the normal range, has been shown to be SIGNIFICANTLY associated with an increased risk of new-onset nephrolithiasis in an otherwise healthy male population with otherwise normal serum chemistry values?","1. PTH level 2. Serum calcium 3. Serum phosphorus 4. Serum calcitonin 5. Serum 1,25-dihydroxyvitamin D",E,temp nan,"Disorders of Divalent Ions, Renal Bone Disease and Nephrolithiasis" 5560," A 40-year-old man is evaluated for new-onset flank pain and hematuria. Ultrasonography revealed three stones in the left kidney but no hydronephrosis. The results of renal function and routine laboratory studies, including calcium, phosphate, total CO2, and uric acid were all within normal limits. The urine pH was 6.8. His symptoms remitted after 2 days of analgesic medication. He was seen in follow-up, and an abdominal flat plate radiograph was normal. However, repeated ultrasonography showed virtually no change from the earlier ultrasound examination..",Which ONE of the following is the MOST likely cause of this patient’s kidney stones?,"1. Hypercalciuria 2. Renal tubular acidosis 3. Hypocitraturia 4. Increased 2,8-dihydroxyadenine excretion 5. Increased uric acid excretion",D,temp nan,"Disorders of Divalent Ions, Renal Bone Disease and Nephrolithiasis" 5561," A 56-year-old woman is seen in consultation for symptomatic hypomagnesemia. She had been in good health except for recurring symptoms of gastrointestinal reflux, for which she was prescribed esomeprazole. Over the past 6 months, she has experienced generalized weakness and intermittent muscle cramps and spasm. She has a history of hypertension for which she was prescribed a thiazide diuretic. She has had no other medical problems. A physical examination reveals a positive Chvostek sign. Laboratory studies show a serum sodium of 138 mEq/L, potassium of 3.9 mEq/L, chloride of 105 mEq/L, total CO2 of 24 mmol/L, BUN of 18 mg/dl, creatinine of 1.2 mg/dl, calcium of 9.6 mg/dl, phosphorus of 4.2 mg/dl, and magnesium of 0.8 mg/dl. The fractional excretion of magnesium is 0.7%. A 24-hour urine determination shows a daily magnesium excretion of 4 mg..",Which ONE of the following is the MOST likely explanation of her hypomagnesemia?,"1. Renal losses induced by a specific mutation in the TRMP6 gene 2. Renal losses induced by thiazide diuretics 3. Renal losses induced by a defect in the Claudin-16 gene 4. Decreased gastrointestinal absorption",D,temp nan,"Disorders of Divalent Ions, Renal Bone Disease and Nephrolithiasis" 5562," You are monitoring a 28-year-old woman who has had three episodes of nephrolithiasis in the past 7 years. Studies in the past have been unrevealing, with normal values for calcium, phosphate, urinary pH, urine calcium, and serum PTH. She has been treated with increased fluid intake and liberal dietary calcium. She takes vitamin C 1000 mg daily and asks you about the impact of this on her risk for subsequent kidney stones..",Which ONE of the following should you tell her about the use of vitamin C regarding kidney stone risk?,"1. Her vitamin C use places her at increased risk for recurrent stones 2. Increased stone risk with vitamin C supplementation is seen only in men 3. Cohort studies do not include sufficient numbers of vitamin C users to assess risk 4. There is no increased risk of kidney stones with vitamin C because it is chemically converted to oxalate only after excretion in the urine 5. Her use of vitamin C supplements does not place her at increased risk because she is not hypercalciuric",B,temp nan,"Disorders of Divalent Ions, Renal Bone Disease and Nephrolithiasis" 5563," A 45-year-old woman who has been in generally good health is found to have an asymptomatic increase in her serum calcium of 10.3 mg/dl. Subsequent evaluation showed persistent mild hypercalcemia and a PTH level of 78 pg/ml (reference range 12–72 pg/ml). A 24-hour urine determination reveals a daily calcium excretion of 246 mg. The results of all other laboratory studies and routine radiology studies are normal..",Recent studies would suggest which ONE of the following as the BEST clinical approach to her care?,"1. Parathyroidectomy 2. Observation on a twice-a-year schedule 3. Cinacalcet 4. Estrogen therapy 5. Denosumab",B,temp nan,"Disorders of Divalent Ions, Renal Bone Disease and Nephrolithiasis" 2733,"A 20-year-old white man is referred for asymptomatic hematuria discovered during a routine health examination for a new job. You see him with a nephrology fellow. His medical history is unremarkable, and he has been healthy all his life. He has never seen any blood in his urine, even with a cold or sore throat. His father has normal kidney function but was told he had blood in his urine. His mother is healthy with no history of any kidney problems. The patient’s physical examination is normal. His urine dipstick shows a specific gravity of 1.020, no protein, and large blood. Microscopic examination of the urine sediment demonstrates > 10 erythrocytes per high power field; about 30%of the erythrocytes are acanthocytes. The serum creatinine level is 0.8 mg/dl. Under your super-vision the fellow talks with the patient about his kidney disease but makes one mistake..",Which ONE of the following statements made by the fellow is WRONG?,"A. The fellow tells the patient that his sister may also have hematuria B. The fellow advises the patient that his kidney disease will most likely not result in ESRD, but progressive kidney disease can occur in a small number of patients C. The fellow does not advocate a kidney biopsy at this time but notes that a biopsy would most likely show alterations in the thickness of the glomerular basement membrane (GBM) D. The fellow tells the patient that his kidney disease is caused by a mutation in the gene that encodes the alpha-5 chain of type IV collagen",D,temp nan,Secondary Glomerular Disease 2734,"A 30-year-old white woman is seen in consultation for a second opinion concerning steroid-resistant focal and segmental glomerulosclerosis (FSGS). Her initial evaluation, done elsewhere, showed that she had 4 g per day of proteinuria, a serum albumin of 3.2 g/dl, lipiduria, and a serum creatinine concentration of 1.3 mg/dl. Analysis of a kidney biopsy specimen showed an FSGS pattern of glomerular injury with nearly complete podocyte foot process effacement. She was treated with high-dose prednisone for 2 months without response. Her course was complicated by the development of Cushingoid features and weight gain. Thereafter, her management consisted of a low-sodium diet, a maximally tolerated dose of an angiotensin-converting enzyme inhibitor, and a diuretic. The patient’s father is on dialysis and had heavy proteinuria preceding the onset of ESRD. Currently the patient has about 2.5 g per day of proteinuria and poorly controlled hypertension. Her estimated GFR has fallen from 45ml/min per 1.73 m2 1year ago to 23ml/min per 1.73m2. She seeks your advice because of progressive CKD. A physical examination reveals a BP of 160/92 mmHg, a body mass index of 32 kg/m2, and trace pedal edema..",Which ONE of the following would be the MOST helpful in her management?,"A. Genetic analysis for hereditary forms of FSGS B. Repeat kidney biopsy C. Bariatric surgery because the FSGS is likely secondary to her obesity D. Forgo kidney transplantation because the disease is likely to recur in the allograft",A,temp nan,Secondary Glomerular Disease 2735,"A 48-year-old man is referred by your nephrology colleague for consultation regarding the evaluation and management of crescentic glomerulonephritis (GN). He presented with the nephritic syndrome and was found to have a serum creatinine level of 2 mg/dl. A physical examination reveals bilateral ankle edema, clear lungs, a normal cardiac examination, and no skin lesions. His urine dipstick is positive for protein and blood, and his urine sediment shows erythrocyte casts and several acanthocytes. A kidney biopsy performed 1 week ago shows a membranoproliferative pattern on light microscopy with < 20% cellular and fibrocellular crescents, one area of glomerular capillary fibrinoid necrosis, and strong linear GBM IgG staining by immunofluorescence; however, no circulating anti-GBM antibodies were found. The IgG was lambda-light chain restricted..",Which ONE of the following should you tell your colleague about this man’s glomerular disease?,"A. The IgG in the kidney biopsy may be monoclonal B. The patient will most likely have a positive assay for an anti-neutrophil cytoplasmic antibody (ANCA) C. The patient will most likely require dialysis within 1 year D. Treatment with cyclophosphamide, corticosteroids, and plasmapheresis should be started immediately",A,temp nan,Secondary Glomerular Disease 2736,"You are asked to consult on a 56-year-old man who has been receiving sunitinib, a growth factor inhibitor, after unilateral nephrectomy for renal cell carcinoma. He has a history of diabetes mellitus and has had long-standing proteinuria of about 1.5 g/d and a serum creatinine concentration that has been about 2 mg/dl for the past 8 years. The patient’s oncologist found that the serum creatinine concentration recently increased to 3.2 mg/dl. When the test was repeated a month later, the serum creatinine level was 3.3 mg/dl. In general, the patient has been doing well and has been tolerating sunitinib. The BP has averaged about 140/ 85 mmHg on a modest dose of an angiotensin receptor blocker. His diabetes has been well controlled, with a hemoglobin A1c of 7.2%. The patient has had no recent infections, treatment with anti-biotics, new medications, or radiologic studies with intravenous contrast, and he was not taking any over-the-counter medications including nonsteroidal anti-inflammatory drugs. A urinalysis showed 3+ protein and 1+ blood, with about 6 erythrocytes per high power field, 0–2 leukocytes per high power field, and no bacteria. A 24-hour urine showed that proteinuria was increased to 3 g/d. The results of liver function studies were normal except for a mildly increased serum lactate dehydrogenase of 326 U/L (reference range, 60–100 U/L). The patient was anemic, with a hemoglobin of 11 g/dl, a leukocyte count of 5600/µL, and a platelet count of 160,000/µL. The international normalized ratio was normal..",Which ONE of the following recommendations do you provide to the oncologist?,"A. The worsening kidney function is most likely due to progression of diabetic nephropathy, and the patient needs better control of his BP B. The worsening kidney function may be due thrombotic microangiopathy from the sunitinib, and you would like to do a kidney biopsy for diagnosis before recommending any changes to the patient’s cancer therapy C. The worsening kidney function may be due to thrombotic microangiopathy from the sunitinib, which can be easily diagnosed by examining a peripheral blood smear for schistocytes and assessing the patient’s haptoglobin and complement levels D. The worsening kidney function may be due to thrombotic microangiopathy from the sunitinib, and you recommend stopping the drug immediately",B,temp nan,Secondary Glomerular Disease 2737,"You are asked to see a 62-year-old woman with peripheral vascular disease hospitalized for treat-ment of foot cellulitis for AKI. Her serum creatinine concentration was 1.4 mg/dl on admission but was known to be 1.0 mg/dl about 6 months ago. Her systolic BP on admission was 100 mmHg, and she initially received intrave-nous saline with stabilization of the systolic BP at 115 mmHg. However, over the next few days her serum creatinine slowly increased to 2.3 mg/dl, and you are called to see her for AKI. Her urine output over the preceding 24 hours was about 800 ml. Blood cultures obtained on admission were positive for methicillin-resistant Staphylococcus aureus, and she has been receiving intravenous vancomycin. On physical examination she does not appear toxic. The BP is 120/74 mmHg. The right foot cellulitis has a receding margin of erythema. Ultrasonography of the kidneys is un-remarkable. The urinalysis shows a specific gravity of 1.020, 3+ protein, 3+ blood, no bacteria, 4 leukocytes per high power field, and 5–10 erythrocytes per high power field, several of which are acanthocytes. There are occasional granular casts. Blood cultures have been negative for the past 24 hours. A vancomycin trough is 24 µg/ml (reference range, 10–20 µg/ml) on the day of the consultation. The serum C3 level is low at 62 mg/dl (reference range 88–171 mg/dl) and myeloperoxidase (MPO)-ANCA is positive at 49 U/ml (reference range < 21 U/ml)..",Which ONE of the following should you recommend?,"A. Supportive care because the patient most likely has postinfectious GN, and this will resolve with treatment of the infection B. Discontinue vancomycin because this patient most likely has AKI due to vancomycin toxicity C. Supportive care because the patient most likely has acute tubular necrosis due to hemodynamic compromise before being admitted to the hospital and given fluids D. A kidney biopsy to verify your suspicion that this patient has staphylococcal-associated IgA GN E. High-dose intravenous corticosteroids plus cyclophosphamide for ANCA-associated nephritis",D,temp nan,Secondary Glomerular Disease 2738,,"Question 2738: Which ONE of the following statements about hepatitis C virus cryoglobulinemia and GN is CORRECT?","A. Interferon-based antiviral therapies do not decrease the chances of developing cryoglobulinemia and GN in patients infected with hepatitis C, genotype 1 B. GN cannot develop after successful treatment of hepatitis C with direct-acting anti-viral agents in patients who have achieved a sustained remission C. Late treatment with interferon-based anti-viral therapies is as effective as early treatment in preventing the kidney manifestations of hepatitis C D. Cryoglobulinemia can occur with hepatitis C and hepatitis B",D,temp nan,Secondary Glomerular Disease 2739,"A patient with light-chain amyloidosis and known kidney involvement is being treated by a hematologist with a proteasome inhibitor–based regimen. The patient comes to your clinic and wants to know your opinion about his chances of requiring permanent renal replacement therapy. All the patient’s records are being sent to you but have not yet arrived. You call the hematologist, who is aboard a plane that is about to leave, and you have time to ask one question before the cell phone is switched off..",Which ONE of the following questions will be MOST helpful to you in providing the patient with an immediate answer?,"A. How much interstitial fibrosis and tubular atrophy were present on the kidney biopsy specimen taken before treatment was started B. How much proteinuria did the patient have when treatment was started C. What was the patient’s eGFR when treatment was started D. What was the magnitude and time course of the patient’s hematologic response to therapy",D,temp nan,Secondary Glomerular Disease 2740,A patient was referred to your clinic with impaired kidney function and proteinuria. Ex-amination of a kidney biopsy specimen revealed a membranoproliferative pattern of glomerular injury on light microscopy. Immunofluorescence showed IgG deposits with k-light chains. IgG subclass staining showed only IgG3. Your nephropathologist thinks that this is proliferative GN with monoclonal immuno-globulin deposits..,Which ONE of the following is the BEST next test(s) to help you decide on therapy?,"A. Urine protein electrophoresis B. Serum immunofixation plus serum free light chains C. Serum complement levels D. Laser dissection/mass spectrometry analysis of the kidney biopsy",B,temp nan,Secondary Glomerular Disease 2741,A patient with non–small cell lung cancer is to be treated with the checkpoint inhibitor pembrolizumab. The patient’s oncologist refers the patient to you to determine if the patient is at risk for kidney injury while taking this medication and to offer advice on how to prevent kidney problems. The patient has no history of kidney disease and has a normal serum creatinine level and urinalysis. The patient is not taking any other prescribed medications but has been using a proton pump inhibitor for 3 years because of dyspepsia from nonsteroidal anti-inflammatory drugs used for chronic back pain..,Which ONE of the following should you recommend?,"A. Serologic testing for lupus B. Hemoglobin A1c and fasting glucose level C. Avoid nonsteroidal anti-inflammatory drugs and proton pump inhibitors D. Begin a low-dose angiotensin converting enzyme inhibitor",C,temp nan,Secondary Glomerular Disease 2742,"You are asked to consult on a 57-year-old woman who presented to the hospital with hemoptysis and kidney failure. The patient had been coughing up blood for about 2 weeks and was initially treated with antibiotics for bronchitis. She is found to have a serum creatinine concentration of 4 mg/dl. Her baseline kidney function is unknown, but she has no prior history of any medical problems. She does not take any medications routinely and has no family history of kidney disease. A urinalysis shows 10 erythrocytes per high power field, of which about 25% are acanthocytes. Additionally, you find two erythrocyte casts. Serologic studies for lupus, ANCA vasculitis, and anti-GBM disease are ordered and are pending. You perform an urgent kidney biopsy on the initial day of consultation because of concern for a rapidly progressive pulmonary renal syndrome. Examination of the biopsy specimen shows necrotizing crescentic GN with linear IgG immunofluorescence along the GBMs. You start plasmapheresis, high-dose corticosteroids, and cyclophosphamide. Eventually the serologic data report comes back. The patient does not have antinuclear antibodies but does have anti-GBM antibodies and proteinase 3 (PR3)-ANCA..",Which ONE of the following is the most appropriate management?,"A. You stop cyclophosphamide and begin treatment with rituximab B. You continue treatment for anti-GBM disease with plasmapheresis, steroids, and cyclophosphamide. and after the patient goes into remission you taper off all immunosuppression C. You continue treatment for anti-GBM disease with plasmapheresis, steroids, and cyclophosphamide. and after the patient goes into remission you provide the patient with maintenance immunosuppression D. You stop immunosuppression because short-term and long-term kidney survival in patients with double-positive anti-GBM disease and ANCA vasculitis is very poor, and the patient is likely to require long-term dialysis",C,temp nan,Secondary Glomerular Disease 2743,"A 24-year-old Chinese woman is referred to you because of hematuria and proteinuria. She also describes joint pain and a facial rash, and lately she has noticed some unusual mouth ulcers. The urine sediment shows several acanthocytes and two erythrocyte casts. A 24-hour urine collection has 3 grams of protein. Her serum creati-nine is 1.4 mg/dl. A kidney biopsy specimen shows class IV lupus nephritis. You recommend starting high-dose corticosteroids in combination with cyclophosphamide. The patient refuses cyclophosphamide because of what she has learned on the internet about premature ovarian failure..",Which ONE of the following is the BEST choice for induction therapy for this patient?,"A. A combination of low-dose tacrolimus and low-dose mycophenolate mofetil (MMF) with prednisone B. Azathioprine plus prednisone C. Rituximab alone D. Belimumab",A,temp nan,Secondary Glomerular Disease 2744,"A 30-year-old black woman you treated for class IV lupus nephritis continues to have 1.5 g per day of proteinuria. She received a diagnosis of SLE and lupus nephritis about 3 years ago. She completed induction therapy with “Euro-lupus” cyclophosphamide and subsequently received maintenance therapy with mycophenolate mofetil (MMF) 1 g twice daily until now. She was also treated with prednisone, which was tapered off about 9 months ago. The patient initially had a good response to treatment, with a reduction in proteinuria from 4 g per day to 1.5 g per day during the first year of treatment. The urine sediment became bland, and the hematuria re-solved. Her serum creatinine fell from 1.8 mg/dl at the time of diagnosis to 1.1 mg/dl after 6 months and has remained stable since that time. Serum C3 and C4 levels have normalized. She continues to be positive for antinuclear antibodies, but antibodies to double-stranded DNA are now negative. The patient wants to discontinue MMF in preparation for pregnancy..",Which ONE of the following is the BEST course of action for this patient?,"A. Stop MMF, begin azathioprine, and tell patient it is OK to conceive immediately B. Stop MMF, begin tacrolimus, and tell the patient it is OK to conceive immediately C. Talk to the patient about repeating a kidney biopsy to determine if her lupus nephritis is still active or if the proteinuria represents chronic disease and immunosuppression can be safely tapered D. Re-treat the patient with high-dose cyclo-phosphamide to cause the proteinuria to remit",C,temp nan,Secondary Glomerular Disease 2745,A patient with class IV lupus nephritis is being treated with prednisone and MMF after initial induction therapy with “Euro-lupus” cyclophosphamide. She wants to avoid dialysis in the future and seeks your opinion as to what else she should be doing to prevent her disease from progressing to ESRD..,Which ONE of the following do you suggest?,"A. Add a calcineurin inhibitor to the regimen B. Add an antimalarial agent to the regimen C. Add rituximab to the regimen D. Use azathioprine for maintenance instead of MMF",B,temp nan,Secondary Glomerular Disease 2746,"A 23-year-old man with ESRD secondary to lupus nephritis is evaluated for a possible lupus flare. He has been doing well with maintenance hemodialysis for the past 6 years. He has been maintained on immunosuppressive agents since starting dialysis and has had no extrarenal flares of lupus. He has a history of antiphospholipid antibodies but has not had antiphospholipid syndrome. Over the past 2 weeks he has experienced a cough, pleuritic chest pain, joint aches, and a mouth ulcer. It is cold and flu season, and several patients in the dialysis unit have upper respiratory infections. On physical examination, there is a shallow ulcer in the left buccal mucosa. There is no synovitis. Laboratory studies show a normal complete blood count and an increased sedimentation rate of 84 mm/h (reference range < 15 mm/h). Both C3 and C4 have been persistently low over the past several months..",Which ONE of the following factors makes it LESS likely that he is experiencing a lupus flare?,"A. The fact that he is male B. The history of IgM antibodies to cardiolipin and β2-glycoprotein C. Dialysis vintage > 5 years D. Low complement factor C4 E. His age",C,temp nan,Secondary Glomerular Disease 2747,"A 32-year-old black woman with pure class V lupus nephritis was successfully treated with MMF and corticosteroids and achieved complete remission 4 years ago. Since the time of re-mission, her kidney function has been normal, and she has not had proteinuria. She has not used any immunosuppressive agents for over a year. She returns to your office because of a 6-week history of ankle edema. On physical examination, her BP is 140/90 mmHg. She has a mild facial rash and moderate ankle edema. The urinalysis shows 1+ blood and 1+ protein by dipstick. The urinary sediment shows many acanthocytes. Additional laboratory studies show a C3 level of 74 mg/dl, which is just below the lower limit of normal for your laboratory (reference range, 88–171 mg/dl). Anti-dsDNA antibodies are negative. The serum albumin is 3.4 g/dl, and the serum creatinine level is 1.1 mg/dl. The patient’s baseline serum creatinine is generally 0.7 to 0.8 mg/dl. A 24-hour urine collection contains 5 grams of protein. You recommend a repeat kidney biopsy, but the patient declines. A urine pregnancy test is negative..","In addition to dietary sodium restriction, treatment with renin-angiotensin-aldosterone system (RAAS) blockade, plus diuretic therapy, which ONE of the following is the MOST appropriate next step in management?","A. Treat the patient with rituximab monotherapy for recurrent class V lupus nephritis B. Treat the patient with a calcineurin inhibitor for recurrent class V lupus nephritis C. Recommend close follow-up without immunosuppression D. Restart MMF and corticosteroids to treat a presumptive diagnosis of class V plus class III or IV lupus nephritis",D,temp nan,Secondary Glomerular Disease 2748,"You are asked to see a 50-year-old man in consultation for PR3-ANCA–associated small vessel vasculitis. He has experienced upper re-spiratory symptoms, epistaxis, and malaise for several weeks. His initial evaluation revealed a nasal ulcer and a serum creatinine level of 1.4 mg/dl. A urinalysis showed 2+ blood, 2+ protein, and many erythrocyte casts. A kidney biopsy revealed pauci-immune necrotizing GN with focal crescents. His chest radiograph was clear, and sinus radiographs showed opacities in the maxillary sinuses. Serologic studies showed a negative antinuclear antibody panel, negative anti-GBM antibody, and a positive ANCA sub-typed as PR3-ANCA. He was given two 500-mg intravenous boluses of methylprednisolone over 2 days and prednisone at a dose of 1 mg/kg per day..",Which ONE of the following is the MOST appropriate treatment recommendation?,"A. No additional immunosuppressive therapy B. Intravenous cyclophosphamide plus plasma-pheresis C. Rituximab D. Mycophenolate mofetil",C,temp nan,Secondary Glomerular Disease 2749,"A 45-year-old man is seen in consultation for MPO-ANCA–associated vasculitis with kidney involvement. His induction regimen was intravenous cyclophosphamide, corticosteroids, and five sessions of plasmapheresis. The patient did well and went into remission with a Birmingham Vasculitis Activity Score of zero, stabilization of serum creatinine at 1.5 mg/dl, and a reduction in proteinuria from 2.8 g to 1.0 g per day. He had been prescribed azathioprine for maintenance therapy. He is seeking a second opinion on the likelihood of disease relapse. You review all his clinical and laboratory data and conclude that he has a low risk of relapse..","In addition to his MPO-ANCA status (as compared to PR3-ANCA), which ONE of the following BEST supports your conclusion?","A. ANCA titers decreased but were still positive when maintenance therapy was started B. The patient was treated with intravenous cyclophosphamide C. ANCA became undetectable after induction therapy D. The patient was treated with plasma exchange",C,temp nan,Secondary Glomerular Disease 2751,"A 26-year old patient you have been treating for pauci-immune GN is gradually progressing to ESRD. Extrarenal signs and symptoms of vasculitis have resolved, but the patient has remained MPO-ANCA positive. The patient has no additional medical problems. You initiate a discussion regarding renal replacement therapy..",Which ONE of the following should you tell her regarding management options for ESRD?,"A. Patients with ESRD due to ANCA-associated vasculitis have inferior survival while using hemodialysis compared with patients with ESRD from other causes B. Patients with ESRD due to ANCA-associated vasculitis who receive a kidney transplant consistently have inferior allograft survival compared with patients receiving transplants for other reasons C. Because the patient is still ANCA positive, transplantation should be delayed until she is ANCA negative for at least 6 months because of a higher risk of disease recurrence D. Because the patient did not have lung or upper respiratory vasculitis, her chances of disease relapse while she is using dialysis are lower",D,temp nan,Secondary Glomerular Disease 2752,You have successfully induced remission in a patient with ANCA-associated nephritis and have begun giving him azathioprine maintenance therapy. The patient has had 18 months of total therapy and 12 months of complete remission and wants to stop immunosuppression. You agree that immunosuppression can be tapered..,In which ONE of the following situations is withdrawal of therapy BEST supported?,"A. The patient had mild kidney disease and minimal extrarenal signs or symptoms of vasculitis, so he is unlikely to experience relapse B. The patient has been PR3-ANCA positive throughout his treatment course C. The patient was PR3-ANCA positive, be-came negative with rituximab treatment, but became PR3-ANCA positive again at month 16 D. The patient had moderately severe MPO-ANCA–associated nephritis, received induction therapy with oral cyclophosphamide, and became ANCA negative when maintenance therapy was initiated",D,temp nan,Secondary Glomerular Disease 2753,You are asked to see a 75-year-oldman in consultation for rapidly progressive GN. He does not have extrarenal manifestation of vasculitis. Ex-amination of a kidney biopsy specimen reveals findings consistent with anti-GBM GN with 40%crescents. His serum creatinine level at presentation was 6.4 mg/dl. He asks about his prognosis..,Which ONE of the following should you tell him about his prognosis?,"A. His serum creatinine level at presentation is an independent predictor of dialysis de-pendency at 1 year B. His age is associated with a reduced hazard of dialysis dependency at 1 year C. His percentage of crescents shown by ex-amination of a kidney biopsy specimen is predictive of dialysis dependency at 1 year D. Recent studies indicate that renal survival in anti-GBM nephritis is now > 50% in survivors",A,temp nan,Secondary Glomerular Disease 2754,"A 28-year-old woman with a recent diagnosis of Fabry disease based on clinical features and genetic testing is referred for an opinion regarding enzyme replacement therapy with agalsidase-b. Her initial evaluation revealed 900 mg of daily urinary protein excretion and a serum creatinine level of 0.7 mg/dl. She has acroparesthesias and intermittent mild abdominal pain and diarrhea. On physical examination, her BP is 132/80 mmHg. A corneal examination reveals verticillata. Several angiokeratomas are present in the upper thighs. The urinalysis shows 1+ protein, negative blood, and rare fine granular casts. A serologic evaluation investigating other potential causes of glomerular disease was unremarkable. An echocardiogram shows mild left ventricular hypertrophy..","In addition to RAAS blockade, which ONE of the following is the BEST treatment recommendation for this woman?","A. Begin enzyme replacement therapy only if the daily protein excretion remains above 500 mg per day after 6 months of RAAS inhibition B. Begin enzyme replacement therapy now C. Delay enzyme replacement therapy until the serum creatinine level exceeds 1.5 mg/dl D. Do not recommend enzyme replacement therapy because she has a favorable renal prognosis without treatment",B,temp nan,Secondary Glomerular Disease 2755,"A 42-year-old Greek man is evaluated for progressive CKD. His sister has known microscopic hematuria. His BP is 126/80 mmHg. The result of physical examination is otherwise normal. Laboratory studies show a serum creatinine level of 1.8 mg/dl and a urine protein-to-creatinine ratio of 1456 mg/g. A urinalysis shows 1+ blood, 1+ protein, and15to20dysmorphic erythrocytes per high power field. A kidney biopsy specimen reveals FSGS on light microscopy. Electron microscopy shows thinning of the GBM with focal regions of podocyte foot process effacement..",Genetic testing will MOST likely reveal a mutation in which ONE of the following genes?,"A. NPHS1 (nephrin) B. ACTN4 (alpha-actinin 4) C. COL4 (type IV chain of collagen) D. NPHS2 (podocin)",C,temp nan,Secondary Glomerular Disease 2756,"A 62-year-old man with hepatitis C virus infection is seen in consultation for rapidly progressive GN in association with mixed cryoglobulinemia. He sought medical attention after 2 weeks of worsening digital pain and ulcers, rash, weakness, and arthralgias. His serum creatinine level was found to have risen from 1.2 mg/dl 1 month ago to 7 mg/dl. He requires initiation of hemodialysis for progressive oliguria. A physical examination reveals palpable purpura with ulceration over the trunk and extremities, digital necrosis of several fingers, and synovitis involving the hands, wrists, knees, and ankles. Laboratory studies show serum C3 low at 73 mg/dl (reference range 88–171 mg/dl), C4 low at 4mg/dl (reference range 15–48 mg/dl), and serum cryoglobulins with a polyclonal increase in IgG and a monoclonal anti-IgG IgM rheumatoid factor consistent with type II cryoglobulinemia. Testing for hepatitis C is positive with a viral load of 2,180,385 IU/ml and genotype 1b. The results of serologic studies for HIV and hepatitis B are negative. A kidney biopsy shows a pattern of membranoproliferative GN with cryoprecipitates in capillary loops on light microscopy. Immunofluorescence microscopy reveals diffuse IgM deposition in the capillary loops..","In addition to intravenous “pulse” cortico-steroids, which ONE of the following is the MOST appropriate management?","A. Antiviral therapy without additional immunosuppression B. Rituximab or cyclophosphamide plus plasmapheresis without antiviral therapy C. Antiviral therapy plus rituximab D. Antiviral therapy, rituximab or cyclophosphamide, plus plasmapheresis",D,temp nan,Secondary Glomerular Disease 2757,"A 62-year-old man with hepatitis B–related cirrhosis complicated by portal hypertension is seen in follow-up after a recent diagnosis of membranous glomerulopathy. On physical examination, the BP is 136/80 mmHg. He has minimal ascites and 1+ leg edema. Laboratory studies show a daily urinary protein excretion of 4 g per day, serum creatinine level of 1.6 mg/dl (stable compared with his most recent creatinine value 1 month ago), positive hepatitis B surface and e antigens, negative serologic results for hepatitis C and D, serum alanine transaminase of 89 U/L, and hepatitis B virus DNA of 2500 U/ml. The kidney biopsy specimen shows membranous nephropathy with positive staining for hepatitis B antigen..",Which ONE of the following is the MOST appropriate management?,"A. Pegylated interferón B. Entecavir C. Rituximab D. No antiviral or immunosuppressive therapy",B,temp nan,Secondary Glomerular Disease 2758,"A 32-year-old woman with mitral valve endocarditis due to methicillin-sensitive Streptococcus mutans is seen in consultation for AKI. A small (3 mm) mitral vegetation and mild mitral re-gurgitation were found on an initial transesophageal echocardiogram. She has been receiving intravenous penicillin G for 6 weeks, and she has not had fever for the past month, repeated blood cultures have been negative, and a repeated trans-esophageal echocardiogram did not show persistent vegetations or other abnormalities. The serum creatinine level increased from 0.8 mg/dl on admission to 4.5 mg/dl. Urinalysis shows erythrocyte casts and 1+ proteinuria. Her complete blood count shows white blood cell count of 6800/mL, hemoglobin of 9.2 g/dl, and platelets of 146,000/mL. Serum C3 is slightly low, C4 is normal, and test results for ANA, ANCA, anti–double-stranded DNA, hepatitis B and C, and HIV are negative. An electrocardiogram is normal. You do a kidney biopsy and find 35%cellular crescents without fibrinoid necrosis, and diffuse granular mesangial and segmental granular capillary wall staining for IgG (3+), IgM (2+), C3 (3+), C1q (3+), kappa-light chains (2–3+), and lambda-light chains (1–2+), but no staining for IgA or phospholipase A2 receptor antigen. Electron microscopy shows scattered subepithelial and mesangial deposits..",Which ONE of the following is the MOST appropriate management?,"A. Supportive care without further antibiotics or immunosuppression B. Refer for mitral valve replacement C. Eculizumab D. Cyclophosphamide plus corticosteroids",D,temp nan,Secondary Glomerular Disease 2759,"A 55-year-old man is referred for further evaluation and management of GN. He presented with increased edema and was found to have microscopic hematuria and proteinuria. He did not have rash, abdominal pain, diarrhea, or arthralgias. A kidney biopsy specimen showed membranoproliferative GN with mesangial and subendothelial deposits. Immunofluorescence microscopy revealed intense mesangial and subendothelial deposits of monotypic IgA, C3, and kappa-light chains, with negative staining for lambda-light chains. Electron microscopy showed no organized deposits. His BP has been well controlled by losartan over the past 2 months. His laboratory studies show a serum creatinine level of 1.4 mg/dl, albumin of 3.4 g/dl, hemoglobin of 9.6 g/dl, and a urine protein-to-creatinine ratio of 2600 mg/g..",Which ONE of following is the next BEST step in this patient’s management?,"A. Begin prednisone 1 mg/kg per day for 6 to 8 weeks B. Rituximab 1 g every other week for 2 doses C. Conduct further hematologic investigation to search for a B cell clone D. Check the antitissue transglutaminase antibody level",C,temp nan,Secondary Glomerular Disease 2760,"A 51-year-old woman with longstanding type 1 diabetes mellitus is seen in consultation for nephrotic syndrome that developed over the past 3 months. Laboratory studies show a serum creatinine level of 1.14 mg/dl, an albumin of 2.7 g/dl, and serum protein electrophoresis and immunofixation demonstrating trace monoclonal free l-light chains that are too low to accurately quantify. The serum free kappa-light chain level is 3.16 mg/dl, the lambda-light chain level is 2.83 mg/dl, and the free light chain ratio is 1.12. A urinalysis demonstrates 3+ protein and oval fat bodies. The urine protein-to-creatinine ratio is 7267 mg/g, and the albumin-to-creatinine ratio is 4987 mg/g. The white blood cell count is 9000/µL, hemoglobin is 11.8 g/dl, and the platelet count is 339,000/µL. A kidney biopsy shows findings consistent with IgG3 lambda-light heavy chain deposition disease. A bone marrow biopsy is consistent with small lymphocytic lymphoma, and flow cytometry reveals a clone of a CD5/CD23 dual-positive mature B cell population..",Which ONE of the following is the MOST appropriate initial therapy?,"A. Cyclophosphamide, bortezomib plus dexamethasone B. Referral for autologous hematopoietic cell transplantation now C. Anti-B cell therapy with rituximab plus bendamustine D. Plasmapheresis",C,temp nan,Secondary Glomerular Disease 2761,"A 56-year-old man is referred for evaluation and management of AKI. Three months ago, he noted the onset of fatigue, blurred vision, and diplopia. A physical examination revealed bilateral upper eyelid edema. Hard masses were palpable under the upper lids. Images of 2-[18F]-fluoro-2-deoxy- D- glucose positron emission tomography/CT displayed enlargement and increased uptake in the bilateral lacrimal and submandibular glands, aorta, and kidneys. Laboratory studies showed a serum creatinine of 3.2 mg/dl, C3 of 46 mg/dl (reference range 88–171 mg/dl), C4 of 38 mg/dl (reference range 15–48 mg/dl), positive antinuclear antibodies at 1:40, negative anti–double-stranded DNA, and positive PR3-ANCA at 132 U/ml (reference range < 20 U/ml). A kidney biopsy specimen showed pauci-immune crescentic necrotizing GN. A biopsy specimen from an enlarged lacrimal gland showed whirling fibrotic tissue encasing lymphoplasmacytic infiltrates..","In this case, ANCA-associated vasculitis is MOST likely overlapping with which ONE of the following diagnoses?","A. IgG4-related disease B. Sarcoidosis C. Kikuchi disease D. Cogan’s syndrome",A,temp nan,Secondary Glomerular Disease 2762,A 54-year-old woman is referred for further evaluation and management of recently diagnosed fibrillary GN. She presented with a serum creatinine of 3.4 mg/dl and proteinuria of 5.4 g per day. She has many questions about her prognosis and management..,Which ONE of following should you tell her about fibrillary GN?,"A. The majority of persons with this lesion have occult solid tumors B. > 50% of patients treated with rituximab can expect improvement or stabilization of kidney function C. Further evaluation should include testing for hepatitis C virus infection D. Future kidney transplantation is contraindicated because recurrent fibrillary GN always occurs",C,temp nan,Secondary Glomerular Disease 2763,"You are caring for a 64-year-old man who has been maintained on three-times-per-week hemodialysis (HD) for the past 4 years. He indicates that he is tolerating his HD treatments well. The monthly predialysis laboratory studies show single-pool Kt/V of 1.25, hemoglobin of 10.8 g/dl, sodium of 138 mEq/L, potassium of 4.2 mEq/L, chloride of 103 mEq/L, total CO2 of 23 mmol/L, calcium of 9.0 mg/dl, phosphorus of 4.8 mg/dl, and al-bumin of 3.8 g/dl. He recently learned that sudden cardiac death (SCD) is the leading cause of death in ESRD, and he asks about your assessment of his risk..",Which ONE of the following should you tell him has been associated with an INCREASED risk of sudden cardiac arrest in patients with ESRD?,"A. An abnormal exercise stress test result B. Elevated serum lipid levels C. A QRS-T angle of greater than or equal to 75 degrees on signal-averaged electrocardiogram D. A serum potassium level > 5 mEq/L on nondialysis days E. Ventricular premature beats on 48-hour ambulatory electrocardiogram monitoring",C,temp nan,End Stage Renal Disease & Dialysis 2764,You are treating a 57-year-old man who has ESRD due to diabetic nephropathy. He has been on HD for 2 years. Over the past year his body mass index (BMI) has increased from 44 kg/m2 to 49 kg/m2. The dialysis center’s dietitian is concerned about the impact of the increasing weight on his outcomes. His recent laboratory studies show an equilibrated Kt/V of 1.3 and a C-reactive protein of 75 mg/L (reference range < 10 mg/L)..,Which ONE of the following should you tell the dietician about elevated BMI in patients who are treated with HD?,"A. Elevated BMI will predispose him to cardiac complications and increase his mortality risk B. Elevated BMI will worsen metabolic control of his diabetes and increase his mortality risk C. As long as he can maintain his vascular access (and avoid catheter-based vascular access) his mortality risk will be similar to that of other ESRD patients on dialysis D. Elevated BMI is associated with improved survival in patients who have evidence of systemic inflammation E. Elevated BMI is a surrogate indicator of nutritional status and thus is associated with improved survival on dialysis",D,temp nan,End Stage Renal Disease & Dialysis 2765,"A 57-year-old woman with ESRD is evaluated during routine dialysis rounds. She complains of several weeks of increased pruritus aggravated by the recent cold weather. During a discussion of her recent laboratory studies, you explain that her serum parathyroid hormone (PTH) concentration is currently elevated above the target range. This has occurred despite optimization of the serum calcium and phosphorus laboratory values. Her medications include calcium acetate 1334 mg three times daily with meals and paricalcitol 4 µg with HD. She was unable to tolerate oral cinacalcet because of nausea. Her most recent laboratory studies show corrected calcium of 9.7 mg/dl, serum phosphorus of 4.5 mg/dl, and PTH of 1130 pg/ml. In addition to continuing her diet, vitamin D sterol, and phosphate binder therapy, you recommend initiation of intravenous (IV) etelcalcetide therapy with her dialysis treatments..",Which ONE of the following should you tell her about the benefits of etelcalcetide therapy?,"A. It will be less likely to cause hypocalcemia than cinacalcet B. It will have fewer gastrointestinal side effects than cinacalcet C. It will decrease the need to adjust calcium concentration in her dialysate compared with cinacalcet D. It is not inferior to cinacalcet in lowering PTH concentrations E. It will reduce her risk for cardiovascular events, including SCD (sudden cardiac death)",D,temp nan,End Stage Renal Disease & Dialysis 2766,"A 72-year-old man with stage 5 CKD due to autosomal dominant polycystic kidney disease is evaluated during a scheduled office visit. He now has dysgeusia, pruritus, and intermittent nausea. He has received predialysis modality education. He has read about complications of different treatments, and he expresses concern about the high reported rate of SCD (sudden cardiac death) in patients with ESRD maintained on HD. He asks whether peritoneal dialysis (PD) offers any advantage over HD with regard to the risk of SCD..",Which ONE of the following should you tell him about the risk of SCD?,"A. The risk of SCD is the same in HD and PD patients B. The risk of SCD is higher in HD than in PD patients C. The risk of SCD is higher in HD than in PD patients, but only during the first 6 months of treatment D. The risk of SCD is higher in HD than in PD patients, but only in patients with known coronary artery disease E. PD patients have lower risks of SCD compared with HD patients",A,temp nan,End Stage Renal Disease & Dialysis 2767,"You are evaluating a 57-year-old man with ESRD maintained on in-center HD for recurrent intra-dialytic hypotension (IH). The patient’s prescribed dry weight is 90 kg, and he typically requires 2.5 to 3 L of ultrafiltration during his dialysis sessions. He frequently cannot achieve his dry weight because his systolic BP de-creases from typical predialysis measurements of approximately 140 to 150 mmHg to intra-dialytic systolic BPs as low as 85 mmHg. He is not symptomatic with these events. He receives 4 hours of HD three times weekly. A newly hired advanced practice provider accompanies you on rounds and asks about the significance of the IH..",Which ONE of the following should you tell them about the significance of IH?,"A. Asymptomatic IH does not increase mortality risk B. IH does not have an impact on dialysis recovery time C. A drop in systolic pressure to < 90 mmHg is associated with increased mortality risk D. The frequency of IH is reduced by blood volume monitoring–guided ultrafiltration biofeedback E. IH is not associated with increased risk of hospital readmission",C,temp nan,End Stage Renal Disease & Dialysis 2768,"A retired 62-year-old man with ESRD is on maintenance HD and is under your care. He is planning a trip to Europe, and he is working with the clinic social worker to arrange for dialysis treatments while he is away. When the social worker presents clinic options for him, the patient learns that some of the clinics offer hemodiafiltration (HDF) as well as conventional HD. He asks you for information regarding the differences between the two modalities..","Compared with conventional HD, which ONE of the following should you tell him about HDF?","A. HDF improves patient-reported posttreatment recovery times B. HDF provides superior β-2 microglobulin clearance C. HDF is consistently associated with im-proved survival D. HDF reduces the frequency of myocardial stunning E. HDF is associated with an increased risk of bacteremia",B,temp nan,End Stage Renal Disease & Dialysis 2769,A recent quality improvement analysis at your dialysis center revealed a high rate of hospitalizations for fluid overload in patients maintained on HD. Your senior partners ask you to review the current status of lung ultrasonography and on-line optical technology in managing volume in ESRD patients before they will consider implementing these interventions..,Which ONE of the following should you report about the use of lung ultrasonography and on-line optical technology?,"A. Lung ultrasonography is equivalent to the clinical assessment of interstitial lung edema assessed by the absence of crackles and peripheral edema B. Moderate interstitial lung edema by lung ultrasonography is associated with in-creased risk of death C. The assessment of volume status with on-line optical technology is likely to be equivalent to the clinical assessment of volume status D. Lung ultrasonography is a reliable tool to detect underhydration",B,temp nan,End Stage Renal Disease & Dialysis 2770,"A 70-year-old man with ESRD and obesity is admitted to the medicine service for management of acute myocardial infarction. He requires placement of a right coronary artery stent and is discharged 3 days later. His medications are tailored in such a manner that he is discharged on four medications, decreased from ten at the time of admission..",Which ONE of the following factors is associated with an INCREASED risk of hospital readmission within 30 days of discharge?,"A. His age B. His sex C. The decrease in the number of his pre-scribed medications D. His diagnosis of acute myocardial infarction E. His obesity",D,temp nan,End Stage Renal Disease & Dialysis 2771,"You are treating a 75-year-old woman with ESRD on maintenance HD. You recently noted on rounds that her heart rate has been increased with her treatments in the preceding week. On physical examination, the heart rate is 118 beats/min and it is irregularly irregular. She denies any chest pain, dyspnea, palpitations, or change in her exercise tolerance. An electrocardiogram confirms the diagnosis of atrial fibrillation (AF). You begin a discussion about the implications of this diagnosis and the benefits and risks of anticoagulation..",Which ONE of the following should you tell her about the risks associated with AF and treatment with warfarin?,"A. AF is associated with an increased risk of ischemic stroke B. AF is associated with increased mortality only in the first 90 days after diagnosis C. Asymptomatic supraventricular rhythms are not associated with increased mortality risk D. Warfarin use in HD patients does not increase the risk of bleeding",A,temp nan,End Stage Renal Disease & Dialysis 2772,An 82-year-old woman with ESRD on HD is diagnosed with atrial fibrillation. She is discharged from the emergency department on apixaban 5 mg twice daily. She weighs 55 kg. She declines warfarin therapy because of the need for repeated blood monitoring..,Which ONE of the following is the MOST appropriate management?,"A. Reduce apixiban to 2.5 mg twice daily B. Change apixiban to rivaroxaban C. No change in management D. Recommend adding a supplemental 2.5-mg dose after each HD session",A,temp nan,End Stage Renal Disease & Dialysis 2773,"You are treating a 51-year-old man with ESRD on HD. Five years ago he required aortic valve re-placement with a St. Jude prosthetic valve, and he has been maintained on warfarin since that time. His medications include calcium acetate, cinacalcet, ergo-calciferol, and intravenous (IV) epoetin. On physical examination, he is thin, with a BMI of 20 kg/m2. He complains of pain and tenderness over a 5-cm ulcerative lesion on the lateral aspect of his left calf concerning for a lesion resulting from calcific uremic arteriolopathy. His laboratory studies show a hemoglobin of 10.5 g/dl, calcium of 8.1 mg/dl, phosphorus of 7.8 mg/dl, and PTH of 650 pg/ml..","In addition to warfarin, which ONE of the following factors in this patient has been as-sociated with an INCREASED risk of calcific uremic arteriopathy (calciphylaxis)?","A. His low BMI B. His low serum calcium level C. Use of IV epoetin D. Ergocalciferol",D,temp nan,End Stage Renal Disease & Dialysis 2774,"A 72-year-old woman with ESRD is found to have an elevated coronary calcium score on a study ordered by her primary care physician. She is concerned that her dialysate calcium level is set at 3.5 mEq/L. She asks whether her dialysate calcium should be reduced to 2.5 mEq/L. Her recent bone and mineral indices show calcium 8.7 mg/dl, phosphorus 5.6 mg/dl, and PTH 286 mg/dl..",Which ONE of the following should you tell her has been shown to result from use of a dialysate calcium level of 2.5 mEq/L compared with 3.5 mEq/L over a period of 2 years?,"A. A significant reduction in mortality B. No change in histologically evident low-turnover bone disease C. A decrease in mean cancellous bone volume D. A reduction in the progression of coronary artery calcification",D,temp nan,End Stage Renal Disease & Dialysis 2775,,"Question 2775: In the International Dialysis Outcomes and Practice Patterns Study (DOPPS), which ONE of the following factors has the STRONGEST association with increased interdialytic weight gains in ESRD patients treated with HD?","A. Higher dialysate sodium concentration B. Lower normalized protein catabolic weight C. Higher serum sodium concentration D. Lower pre-HD systolic BP",A,temp nan,End Stage Renal Disease & Dialysis 2776,"A 47-year-old woman has ESRD attributed to autosomal dominant polycystic kidney disease. She has received HD on a three-times-weekly schedule for the past 6 years. Her monthly laboratory tests show a calcium of 9.8 mg/dl, phosphorus of 4.9 mg/dl, and PTH of 524 pg/ml. She is concerned about the elevated PTH level. She tells you that she has been reading about bone disease in ESRD and is curious about the role of fibroblast growth factor-23 (FGF23) in mediating abnormalities in her bone and mineral metabolism..",Which ONE of the following should you tell her about FGF23?,"A. Elevated FGF23 levels in ESRD consistently suppress serum PTH levels B. FGF23 serum levels fall as serum PTH levels rise in ESRD C. Elevated FGF23 serum levels in ESRD are associated with an increased risk of cardio-vascular events, infections, and mortality D. Increased FGF23 levels in ESRD increase 1-α hydroxylation of 25-hydroxyvitamin D",C,temp nan,End Stage Renal Disease & Dialysis 2777,Your internal medicine colleague calls you because of her concerns about the deteriorating cognitive function in one of your patients. The patient is a 62-year-old woman with ESRD due to diabetic kidney disease. She initiated HD 1 year ago. Over the past 6 months she has had to retire from her position in an accounting firm because of poor job performance. Her spouse notes that she has been more withdrawn from social interactions and that she is no longer able to balance their checking account. Laboratory studies show adequate urea clearance and acceptable anemia and bone and mineral indices. A recent comprehensive neurologic evaluation showed no abnormalities except for evidence of small vessel disease on brain magnetic resonance imaging. Your colleague asks whether renal transplantation or a change in dialysis modality may offer benefit..,Which ONE of the following should you tell your colleague about cognitive function in ESRD patients?,"A. The majority of patients on HD experience deterioration of cognitive function B. Deterioration of cognitive function is not related to depression C. At initiation of dialysis, cognitive performance is poor in comparison with patients in the general population, but it will always improve if a kidney transplantation is performed D. The rate of decline in cognitive function within the first 3 years of dialysis initiation is similar among patients treated with either PD or HD",A,temp nan,End Stage Renal Disease & Dialysis 2778,"A 56-year-old man with a history of bilateral type 1 papillary renal cell carcinoma necessitating bilateral nephrectomy is seen 2 months after initiation of home HD. His tumors were localized and were completely resected. A living-related kidney transplantation is planned after a waiting period of 2 years. His hemoglobin has shown a progressive decrease from 12.3 g/dl to the current value of 7.4 g/dl despite optimization of his iron status with maintenance iron sucrose. He wants to avoid transfusions that may lead to allosensitization. However, after a discussion of benefits and risks, he has opted to forego treatment with erythropoiesis-stimulating agents because of his recent cancer diagnosis even though he does not have active cancer. His medications include iron sucrose 50 mg once weekly and doxercalciferol 7 µg three times per week. Laboratory studies show a hemoglobin of 7.4 g/dl, transferrin saturation of 32%, ferritin of 450 ng/ml, albumin of 4.0 g/dl, calcium of 9.8 mg/dl, phosphorus of 4.9 mg/dl, and PTH of 286 pg/ml..",Which ONE of the following is CORRECT regarding the treatment of his anemia?,"A. There is no association of erythropoietin use and the de novo development of a malignancy B. The iron sucrose at the prescribed dose should be discontinued because it increases the risk of infectious complications C. Adding oral cinacalcet to reduce PTH levels will likely improve hemoglobin levels D. Roxadustat is a possible future alternative agent for the treatment of his anemia E. Competitive prolyl-hydroxylase inhibitors are ineffective in correcting anemia in patients who have undergone bilateral nephrectomy",D,temp nan,End Stage Renal Disease & Dialysis 2779,You are conducting a quality initiative project in your outpatient dialysis clinic with an oxygen saturation monitor. You discover that 10% of patients at your center have an oxygen saturation < 90% for more than one third of the recorded treatment time. You report the findings during a monthly quality assurance meeting of the clinic..,Which ONE of the following should you indicate about the significance of prolonged intradialytic hypoxemia (PIH) to your staff?,"A. PIH is associated with higher all-cause hospitalization and mortality B. On average, oxygen saturation is lowest in the last hour of treatment C. Patients with PIH require lower doses of erythropoiesis-stimulating agents than do patients without PIH D. PIH is more common in younger patients",A,temp nan,End Stage Renal Disease & Dialysis 2780,"A 54-year-old physician with ESRD is evaluated during dialysis rounds. She complains of several weeks of increased pruritus but is otherwise asymptomatic. Her most recent laboratory studies show a corrected calcium of 9.8 mg/dl, serum phosphorus of 7.9 mg/dl, and a PTH of 730 pg/ml. She has been prescribed sevelamer carbonate 2400 mg three times daily with meals, but she takes it inconsistently because it causes nausea and diarrhea. Calcium acetate and lanthanum carbonate were not tolerated because of nausea. She asks about other alternatives. During this discussion, you explain to her that tenapanor, a novel agent that enhances gastrointesti-nal excretion of phosphate, is currently under investigation..",Which ONE of the following should you tell her about tenapanor for the treatment of hyperphosphatemia in ESRD?,"A. Tenapanor reduces serum phosphate con-centration by inhibiting the sodium-dependent phosphate transporter NPT2b located on the luminal membrane of the small intestine B. Tenapanor treatment was associated with adverse events in < 5% of study participants, and the most common complaint in treated patients was nausea C. Tenapanor caused a dose-dependent reduction in the serum phosphate concentration D. Serum calcium levels decreased significantly whereas the serum potassium concentrations remained unchanged",C,temp nan,End Stage Renal Disease & Dialysis 2781,A 67-year-old man with stage 5 CKD complicating chronic transplant rejection and obesity is seen for the onset of early uremic symptoms. He chooses PD and asks about the risk of developing peritonitis soon after having the catheter placed..,Which ONE of the following should you tell him about early peritonitis?,"A. Preoperative prophylactic antibiotics re-duce the risk of early peritonitis B. Early peritonitis is less common in patients who have an embedded catheter placed C. Patients who transition from a failed trans-plantation to PD are not at increased risk for early peritonitis D. Patient weight has no bearing on the likelihood of the development of early peritonitis",A,temp nan,End Stage Renal Disease & Dialysis 2782,"A 68-year-old woman with ESRD due to diabetic nephropathy and AF who has been maintained on PD for 3 years is evaluated 4 weeks after a recent hospitalization for peritonitis due to methicillin-resistant Staphylococcus species. Over the past 2 years, she has had four episodes of PD-related peritonitis. Since discharge, she has had increased leg edema and has had difficulty achieving adequate ultrafiltration. Her medications include intraperitoneal vancomycin and warfarin. On physical examination, her abdomen is firm to palpation and diffusely tender. A peritoneal equilibrium test reveals a dialysate-to-creatinine ratio of 0.41, decreased from 0.63 six months ago. The peritoneal fluid analysis reveals 10 total nucleated cells/mm3 with 40% neutrophils. A gram stain of the peritoneal effluent is negative. A computed tomographic scan shows a thickened peritoneum, bowel wall thickening, and a few dilatated loops of bowel. A diagnosis of encapsulating peritoneal fibrosis is made..",Which ONE of the following placed this patient at increased risk for encapsulating peritoneal fibrosis?,"A. PD vintage for < 5 years B. Absence of documented of high average peritoneal transport characteristics C. Warfarin anticoagulation D. Recurrent and recent bacterial peritonitis",D,temp nan,End Stage Renal Disease & Dialysis 2783,"As Chair of the medical review committee for your ESRD network, you have been tasked with identifying best practices among centers in your region to improve outcomes in PD-related peritonitis..",Which ONE of the following center characteristics is MOST likely to be associated with more favorable PD-related peritonitis outcomes?,"A. Centers with <10 incident PD patients per year B. Centers providing <10 hours of total training time C. Centers with > 29% of dialysis patients treated with PD D. Centers that train patients within the first 10 days after catheter insertion",C,temp nan,End Stage Renal Disease & Dialysis 2784,"A 45-year-old woman with stage 5 CKD is seen in consultation because of progressive azotemia and uremic symptoms after admission for management of anemia. Over the past four weeks, she has had worsening dysguesia, nausea, intermittent vomiting, and intermittent hiccups. On physical examination, her BP is 165/90 mmHg. She has mild pitting lower extremity edema and trace asterixis. Laboratory studies show a hemoglobin of 6.9 g/dl, potassium of 5.7 mEq/L, total CO2 of 15 mmol/L, BUN of 145 mg/dl, serum creatinine of 12 mg/dl, calcium of 8.4 mg/dl, and phosphorus of 7.8 mg/dl. You discuss renal replacement therapy and she understands that dialysis should be initiated now. She is disappointed that she failed to make adequate preparations for dialysis. She had originally hoped to undergo PD, but she had been told by another provider that the catheter requires at least 4 weeks to heal. She has not had prior abdominal surgery and has no obvious contraindications for PD. You begin a discussion about urgent-start PD..",Which ONE of the following should you tell the patient about urgent-start PD?,"A. Abdominal wall complications can be reduced to < 10% by initiating PD with low dwell volumes with gradual titration over the first month of therapy B. Pericatheter leak and subcutaneous leak are the most common mechanical complications of urgent-start PD C. Urgent-start PD should always be initiated as intermittent peritoneal dialysis rather than automated peritoneal dialysis to avoid catheter dysfunction D. Transfer to HD within 1 year occurs in over 30% of patients treated with urgent-start PD",A,temp nan,End Stage Renal Disease & Dialysis 2785,"A 56-year-old man with stage 5 CKD due to hepatitis C–related membranoproliferative glomerulonephritis and cirrhosis is seen in consultation. His cirrhosis has been complicated by ascites and leg edema that have become increasing more difficult to control with diuretics. He has not had encephalopathy or variceal bleeding. He is on the waiting list for a combined liver-kidney transplantation. His Child-Turcotte-Pugh class is B, and his Model for End-stage Liver Disease (MELD) score is 20. You conclude that he will likely require dialysis within the next 2 months, and you begin a discussion about dialysis options. He indicates a preference for PD..",Which ONE of the following should you tell him about PD in the context of his cirrhosis?,"A. Patients who have cirrhosis and ESRD who are treated with PD have a significantly lower mortality in comparison with HD patients who have cirrhosis and ESRD. B. Serum albumin is lower in patients with cirrhosis who undergo PD than in cirrhotic patients treated with HD C. The MELD score is independently associated with all-cause mortality in patients with cirrhosis treated with PD D. PD is contraindicated because he has ascites",A,temp nan,End Stage Renal Disease & Dialysis 2786,"A 23-year-old man with type 1 diabetes mellitus maintained on home HD is evaluated during a monthly care conference. The ESRD developed because of diabetic nephropathy. A kidney trans-plant failed because of chronic rejection, and he began home HD 3 years ago. He had quit smoking before his transplantation, but he recently resumed smoking one pack of cigarettes per day. He occasionally smokes marijuana. He lives in an apartment two blocks away from the dialysis clinic. He has a well-matured arteriovenous fistula (AVF), which has been in place for several years..",Which ONE of the following factors is associated with an INCREASED risk of transition from home HD to in-center dialysis in this patient?,"A. His age B. Active smoking/substance abuse C. History of kidney transplantation D. Use of AVF compared with a central venous catheter E. Residence proximate to his dialysis center",B,temp nan,End Stage Renal Disease & Dialysis 2787,,"Question 2787: Which ONE of the following statements is CORRECT regarding the mortality of patients with ESRD and disorders of glucose metabolism?","A. There is a U-shaped association between hemoglobin A1c levels and mortality B. The lowest mortality among patients in the United States is associated with hemoglobin A1c levels in the 6% to 7% range C. Following similar trends when compared with the general population, the life expectancy of women on dialysis with diabetes exceeds that of men on dialysis with diabetes D. Glucagon-like peptide levels are not associated with mortality in patients with ESRD on long-term HD",A,temp nan,End Stage Renal Disease & Dialysis 2788,A 33-year old woman with ESRD due to diabetic kidney disease is evaluated during routine dialysis rounds. She is interested in trying pro-biotics to help her symptoms of intermittent loose stools. She has read about the role of gut bacteria in generating uremic toxins and asks about the impact of altering the gut microbiome..,Which ONE of the following should you tell her about probiotics and alterations of the microbiome in patients on dialysis?,"A. Oral vancomycin–induced suppression of gut bacteria is responsible for the degradation of tryptophan and tyrosine that leads to the accumulation of uremic byproducts in the serum of dialysis patients B. Oral vancomycin increases the diversity of the gut microbiome in patients with ESRD receiving HD C. Uremic toxins are thought to induce the enzyme transketolase in the human gut, causing functional thiamine deficiency D. Diabetic ESRD patients who received pro-biotics containing lactobacillus were found to have decreased fasting serum glucose and serum insulin, along with increased insulin sensitivity",D,temp nan,End Stage Renal Disease & Dialysis 2789,"A 55-year-woman with ESRD due to diabetes mellitus and obesity on maintenance HD is evaluated for exquisitely painful lesions that developed over the preceding 3 days on her thighs and lower abdomen. A skin biopsy demonstrates changes consistent with calcific uremic arteriolopathy (CUA), or calciphylaxis. She is not on warfarin anticoagulation. She is subsequently enrolled in a study that requires baseline measurements of serum concentrations of matrix Gla protein (MGP)..",Which ONE of the following is CORRECT with respect to her levels of MGP compared with ESRD patients without calcific uremic arteriolopathy?,"A. Her fraction of total MGP that is carboxylated is more likely to be lower B. She is more likely to have reduced blood levels of uncarboxylated MGP C. She is more likely to have reduced blood levels of carboxylated MGP D. Her MGP levels are not associated with calcific uremic arteriolopathy in the absence of warfarin therapy",A,temp nan,End Stage Renal Disease & Dialysis 2790,"A 65-year-old man with stage 5 CKD is evaluated for increasing uremic symptoms. He has not yet started dialysis. He has sustained a 4-kg weight loss over a period of four weeks and has had dysguesia and anorexia over the past week. He has chosen in-center HD as his initial treatment modality. He has a well-matured AVF in place, which was created 18 months ago. You discuss starting dialysis electively, and the patient asks about incremental initiation of dialysis as opposed to a more traditional three-times-per-week schedule. His urea clearance is 10 ml/min per 1.73 m2..",Which ONE of the following should you tell him about incremental initiation of dialysis and issues related to preservation of residual renal function?,"A. Incremental HD is associated with a significantly slower decline in residual kidney function compared with conventional dialysis B. Incremental HD is associated with in-creased mortality in patients with baseline residual urea clearance > 6ml/min per 1.73 m2 C. Preservation of residual renal function is associated with a mortality benefit in PD but not HD D. Patients who undergo incremental dialysis require lower doses of erythropoiesis-stimulating agents and have significantly higher serum albumin levels",A,temp nan,End Stage Renal Disease & Dialysis 2791,"A 64-year-old woman with ESRD on long-term HD with type 2 diabetes mellitus and AF is evaluated in follow up after a recent diagnosis of calcific uremic arteriolopathy. Her medications are warfarin, sevelamer carbonate, sliding-scale insulin (currently receiving three to four injections per day), glipizide, and cinacalcet (60 mg daily). The physical examination shows tender necrotic lesions on her medial thighs without evidence of secondary infection. She is euvolemic. She has a functional left forearm AVF that does not have skin breakdown or aneurysmal dilatation. The remainder of the examination is normal. She is initially managed with an aggressive regimen of wound management, analgesics, and sodium thiosulfate. Warfarin is discontinued. Laboratory studies show sodium of 138 mEq/L, potassium of 3.8 mEq/L, chloride of 102 mEq/L, total CO2 of 24 mmol/L, calcium of 9.0 mg/dl, phosphorus of 4.3 mg/dl, PTH of 256 pg/ml, glucose of 158 mg/dl, and albumin of 3.8 g/dl. The single pool Kt/V is 1.5. The dialysate calcium concentration is 2 mEq/L..",Which ONE of the following is the MOST appropriate next step in managing this patient’s CUA?,"A. Avoid skin trauma and subcutaneous injections B. Discontinue cinacalcet C. Increase cinacalcet D. Parathyroidectomy E. Transfer to PD",A,temp nan,End Stage Renal Disease & Dialysis 2792,"You see a 58-year-old African-American man with a history of ESRD attributed to diabetic nephropathy on HD with a newly hired advanced practice provider. The patient currently receives epoetin 16,000 units IV with each dialysis treatment and weekly sodium ferric gluconate 62.5 mg once per week. Laboratory studies show a hemoglobin of 11.4 g/dl, ferritin of 136 ng/dl, and transferrin saturation of 27%..",The advanced practice provider asks about outcomes and the impact of using IV iron in combination with erythropoiesis-stimulating agents (ESAs) Which ONE of the following should you tell the advanced practice provider about the anemia-related and non–anemia-related effects of ESAs and iron therapy in patients undergoing HD?,"A. Maintenance IV iron is not associated with a reduction in ESA use B. Epidemiologic data suggest that the trend toward lower use of ESAs is associated with improved all-cause mortality in patients with ESRD C. Maintenance IV iron is not associated with a greater likelihood of achieving hemoglobin levels between 10 and 12 g/dl D. Administration of iron sucrose compared with ferrous gluconate in patients with ESRD is associated with increased infectious mortality E. The modest association of cancer and ESA use is not dose dependent but rather is idiosyncratic",C,temp nan,End Stage Renal Disease & Dialysis 2793,"A 63-year-old white man is discovered to have trace to 1+ hematuria on dipstick urinalysis performed during a routine annual physical examination. This is confirmed on a repeated urinalysis 2 weeks later. Test results for albuminuria were negative. An examination of the urine sediment revealed 5–10 erythrocytes per high power field, 25%–30% of which were dysmorphic. No casts were observed. The urinalysis was normal 1 year ago. No urinary symptoms are present. There is no family history of hematuria. He takes no medication other than a low-dose aspirin daily. He is a lifelong cigarette smoker. The blood pressure and physical examination are normal except for a moderately enlarged, nontender prostate. The serum creatinine level is 0.9 mg/dl. Serum electrolytes, calcium, albumin, and total protein are all normal. A complete blood count is normal. The prostate-specific antigen level is 3 ng/ml. The urine albumin/creatinine ratio averaged 28 mg/g based on three measurements. A kidney ultrasound is normal..",Which ONE of the following tests should be performed next?,"A. Cystoscopy and computed tomographic urogram if the cystoscopy is normal B. A 24-hour urine for albumin excretion C. Serum C3, C4, and an anti–nuclear antibody D. A serum free light chain assay",A,temp nan,Primary Glomerular Disease 2794,,"Question 2794: Which ONE of the following features noted in a kidney biopsy specimen should INCREASE suspicion of low nephron endowment in a non-obese child with a renal biopsy diagnosis of minimal change disease or FSGS?","A. Increased glomerular density (number of glomeruli per cubic millimeter) B. Glomerular hypertrophy C. Global glomerulosclerosis D. Arteriolonephrosclerosis",B,temp nan,Primary Glomerular Disease 2795,"A 22-year-old man develops sudden onset of nephrotic syndrome (serum albumin 1.9 g/dl and urine protein excretion 11 g per day) about a week after an upper respiratory infection. His past medical history includes onset of nephrotic syndrome at age 5 with a complete remission after steroid therapy. Infrequent relapses occurred at ages 7, 9, and 14, each fully responsive to an 8-week course of corticosteroids. A kidney biopsy performed at age 14 showed minimal change disease. A cousin has kidney disease of unknown cause. His physical examination shows marked peripheral edema. The blood pressure is 142/88 mmHg. The serum creatinine level is now 1.1 mg/ dl. A urinalysis shows a trace of blood and 4+ protein. The serum C3 level is normal..",Which ONE of the following is the MOST appropriate next step in management?,"A. Perform genetic studies for a podocytopathy B. Start empiric therapy with cyclosporine C. Repeat kidney biopsy D. Start empiric therapy with oral corticosteroids",D,temp nan,Primary Glomerular Disease 2796,"An 18-year-old Hispanic man experiences nephrotic syndrome (serum albumin 2.9 g/dl and urine protein excretion of 6 g per day). A kidney biopsy reveals a lesion of FSGS, not otherwise specified, by light microscopy with diffuse foot process effacement on electron microscopy. Immunofluorescence microscopy shows mesangial deposits of IgM and C3. There is no family history of kidney disease. His physical examination shows a body mass index of 28 kg/m2, a blood pressure of 146/90 mmHg, and moderate peripheral edema. His serum creatinine level is 1.4 mg/dl. After 8 weeks of prednisone at a dosage of 1 mg/kg per day and angiotensin inhibition with losartan, he remains nephrotic with a stable serum creatinine level. HIV serology is negative..",Which ONE of the following is the MOST appropriate next step in this patient’s evaluation?,"A. Perform genetic testing for a podocytopathy B. Determine the APOL1 risk allele status C. Search for an underlying lymphoma D. Request B7-1 (CD80) stain of the kidney biopsy specimen",A,temp nan,Primary Glomerular Disease 2797,A 38-year-old woman is found to have a lesion of membranous nephropathy on kidney biopsy per-formed to evaluate nephrotic syndrome (serum albumin 3.0 g/dl and urine protein excretion 9 g per day). The biopsy specimen showed characteristic diffuse thickening of the glomerular basement membrane with no associated endocapillary hypercellularity. Electron microscopy showed numerous subepithelial immune complex–type deposits. No enhanced anti–phospholipase A2 receptor (PLA2R) antigen expression was present in the glomeruli. No mesangial deposits were seen by electron microscopy. IgG4 was the dominant IgG subclass in the immune deposits. Kappa and lambda light chains were of equal intensity. The result of ELISA for serum anti–PLA2R antibody was negative. Serologic test results for hepatitis B and C were negative..,Which ONE of the following tests would you order next?,"A. Serum C3 and C4 B. Anti–nuclear antibody and anti–double stranded DNA antibody C. Anti–thrombospondin 7A staining of the renal biopsy specimen D. Serum free light chains",C,temp nan,Primary Glomerular Disease 2798,A 38-year-old man is found to have a lesion of membranous nephropathy when a kidney biopsy is performed to evaluate nephrotic syndrome (serum albumin 3.0 g/dl and urine protein excretion 6 g per day). The results of his physical examination are normal except for minimal peripheral edema. An anti–PLA2R antibody test of serum is positive in low titer (30 RU/ml). A PLA2R antigen is hyperexpressed in glomeruli. The serum creatinine is 1.0 mg/dl. A complete blood count is normal. Results of serologic studies for hepatitis B and C are negative..,Which ONE of the following would you do next?,"A. Evaluate for an underlying malignancy B. Initiate treatment with cyclosporine plus steroids C. Initiate treatment with cyclophosphamide plus steroids D. Initiate treatment with rituximab E. Observe with renin-angiotensin inhibition therapy only",E,temp nan,Primary Glomerular Disease 2799,,"Question 2799: Which ONE of the following statements is CORRECT concerning the use of rituximab for treatment of primary membranous nephropathy?","A. The response to rituximab is conditioned by the prior response to immunosuppressive therapeutic agents B. Combinations of rituximab with low-dose cyclophosphamide can achieve very high remission rates C. Four doses of rituximab at a dosage of 375 mg/m2 per week are superior to a regimen of rituximab dosing based on serial measurement of circulating CD19-positive B cell counts in achieving a reduction in proteinuria D. The concomitant use of corticosteroids is required for rituximab to be effective",B,temp nan,Primary Glomerular Disease 2800,"A 32-year-old Hispanic man with hematuria, impaired kidney function, and proteinuria (serum creatinine 1.9 mg/dl and urine protein excretion of 2.1 g per day) is found to have IgA nephropathy with an Oxford-MEST-C score of M1, E1, S0, T1, C2 on kidney biopsy. Four days later his serum creatinine is 2.8 mg/dl..",Which ONE of the following treatment regimens would be MOST likely to reduce the rate of progression to ESRD?,"A. Aggressive use of renin-angiotensin inhibition B. A 6-month course of low-dose corticosteroids C. A 6-month course of combined mycophenolate mofetil (MMF) plus low-dose steroids D. Treatment with high-dose steroids, cyclo-phosphamide, and plasmapheresis E. Treatment with high-dose steroids only",D,temp nan,Primary Glomerular Disease 2801,,"Question 2801: Which ONE of the following findings would NOT indicate a worse prognosis for a patient with recently diagnosed IgA nephropathy, with a MEST-C Score of M1, E1, S0, T1, C0 and persistent proteinuria of about 1.0 gram per day?","A. Extensive C4d deposition in the mesangium B. Concomitant persistent microscopic hematuria C. Low birth weight D. Low glomerular density (number of glomeruli per cubic millimeter) E. Increased serum C3 and normal C4 levels",E,temp nan,Primary Glomerular Disease 2802,,"Question 2802: For patients ESRD due to IgA Nephropathy, which ONE of the following is MOST likely to unfavorably affect the risk of recurrence of the disease in a renal allograft?","A. A posttransplantation regimen consisting of tacrolimus, MMF, and low-dose corticosteroids B. A posttransplantation regimen consisting of tacrolimus, MMF, and early corticosteroid withdrawal C. An Oxford-MEST-C score in the native kidney at the time of diagnosis of M1, E1, S1, T2, C0 D. Elevated IgA serum levels at the time of transplantation",B,temp nan,Primary Glomerular Disease 2803,"A 68-year-old woman has a diagnosis of C3 glomerulonephritis with a membranoproliferative pattern of injury by light microscopy on kidney biopsy performed because of recent hematuria, proteinuria, and impaired renal function (serum creatinine of 1.6 mg/dl). The serum C3 level is low, and C4 level is normal..",Which ONE of the following studies should be performed?,"A. Genetic analysis for pathogenic mutations of complement factor H, I, and membrane cofactor protein B. Serum levels of soluble C5b–C9 C. Serum free light chains and immunofixation D. Serum level of C3 nephritic factor E. Serum anti–nuclear antibody and anti–double stranded DNA antibody",C,temp nan,Primary Glomerular Disease 2804,"A 63-year-old woman is found to have hematuria, proteinuria, and impaired kidney function (serum creatinine of 1.8 mg/dl). A kidney biopsy shows a membranoproliferative pattern of injury with 10% glomerular involvement with crescents on light microscopy. Immunofluorescence microscopy reveals strong IgG and C3 deposition of about equal intensity. Deposits of kappa and lambda light chains are equal in intensity. No specimen is available for electron microscopy. The serum C3 and C4 levels are normal. Test results for hepatitis C virus infection are negative. The anti–nuclear antibody is 1:40. Serum free light chain levels are normal..",Which ONE of the following studies would be MOST useful to clarify the nature of the disease present in this patient?,"A. Stain biopsy specimen for immunoglobulin sub-class deposition B. Stain biopsy specimen for PLA2R antigen C. Stain biopsy specimen for C4d deposition D. Stain biopsy specimen for DNAJB9 deposition E. Stain biopsy specimen with Congo red",D,temp nan,Primary Glomerular Disease 2805,,"Question 2805: Which ONE of the following is MORE commonly found in the collapsing variant of FSGS compared with the other variants of this lesion?","A. Presence of two APOL1 high-risk alleles B. Hyperexpression of CD80 in glomeruli C. IgM mesangial deposits D. Absence of thrombotic microangiopathy",A,temp nan,Primary Glomerular Disease 2806,"A patient with a lesion of FSGS of uncertain cause and an initial proteinuria level of 3.1 g per day and a serum creatinine level of 1.2 mg/dl undergoes treatment with immunosuppression and angiotensin blockade. The proteinuria declines to 0.2 g per day after eight months of therapy, and the serum creatinine level remains stable..",Which ONE of the following values corresponds to an approximate risk of kidney failure (per 100 patient years) for this patient?,"A. 3.2 per 100 patient years B. 6.6 per 100 patient years C. 9.6 per 100 patient years D. 15.4 per 100 patient years E. 20.0 per 100 patient years",B,temp nan,Primary Glomerular Disease 2807,"A 16-year-old young man of Scottish descent with nephrotic syndrome (serum albumin 3.0 g/dl and urine protein excretion 5.4 g per day) has a lesion of FSGS, not otherwise specified. He fails to respond to a 12-week course of initially high-dose and then tapering doses of oral prednisone. His serum creatinine level is 1.6 mg/dl..",Which ONE of the following is MOST likely to provide an indication for a possible monogenic or digenic cause of his disease?,"A. The histologic variant of FSGS B. The level of renal function C. The level of proteinuria D. His ancestry E. His age",E,temp nan,Primary Glomerular Disease 2808,"An African-American man is found to have a “collapsing” variant of an FSGS lesion on a kidney biopsy performed for the evaluation of nephrotic syndrome. His serum albumin is 1.9 g/dl, and his urine protein excretion is 12 g per day. The serum creatinine concentration is 1.9 mg/dl. HIV serology is negative. Genetic studies show that he carries two high-risk alleles at the APOL1 locus..",Which ONE of the following statements is MOST correct concerning this patient?,"A. The genetic studies strongly indicate that he will be unresponsive to treatment B. Information on his birth weight will be of value in determining his prognosis C. The prognosis for renal survival is significantly worse in comparison with an FSGS lesion not otherwise specified D. He is at low risk for the development of a thrombotic microangiopathy",B,temp nan,Primary Glomerular Disease 2809,,"Question 2809: Which ONE of the following serologic and immunohistologic findings BEST describes a patient with recently diagnosed primary membranous nephropathy with normal kidney function who is in the process of expe-riencing a spontaneous remission of the nephrotic syndrome?","A. Serum anti–PLA2R antibody negative and PLA2R1 antigen positive in glomeruli B. Serum anti–PLA2R antibody negative and PLA2R1 antigen negative in glomeruli C. Serum anti–PLA2R antibody positive and PLA2R1 antigen positive in glomeruli D. Serum anti–PLA2R antibody positive and PLA2R1 antigen negative in glomeruli",A,temp nan,Primary Glomerular Disease 2810,"A 36-year-old woman with multiple episodes of relapsing, steroid-sensitive nephrotic syndrome due to minimal change disease is treated with MMF and low doses of oral corticosteroids because of her fears of the gonadal effects of cyclophosphamide and the cosmetic effects of cyclosporine and high-dose steroids..",Which ONE of the following statements is MOST correct concerning this patient?,"A. A relapse is not likely when the MMF is discontinued B. Tacrolimus is a good alternative treatment because of a low risk of long-term nephrotoxicity C. Rituximab therapy would likely offer better prospects for long-term control of her disease D. She is at high risk for the development of advanced chronic kidney disease and ESRD",C,temp nan,Primary Glomerular Disease 2811,,"Question 2811: Which ONE of the following is of LEAST value for discriminating pure diabetic nephropathy from a nondiabetic glomerular lesion in a patient with type 2 diabetes mellitus and overt proteinuria?","A. Duration of diagnosed diabetes mellitus B. Presence of proliferative retinopathy C. Magnitude of proteinuria D. Presence of glomerular hematuria",C,temp nan,Primary Glomerular Disease 2812,"A 26-year-old white man is undergoing an evaluation to serve as a potential kidney donor to his cousin. He is found to have microscopic hematuria (8–10 dysmorphic red blood cells per high power field). The results of physical examination, blood pressure, serum creatinine, C3 and C4, renal imaging, and urinary albumin excretion are all normal. There is no family history of kidney disease except in his cousin, who has ESRD of unknown cause. A kidney biopsy is performed..",Which ONE of the following is the MOST likely pathologic feature to be found in the kidney biopsy specimen?,"A. C3 glomerulopathy B. IgA nephropathy C. Thin basement membrane nephropathy D. Normal with no abnormality seen",D,temp nan,Primary Glomerular Disease 2813,"A 41-year-old man has a diagnosis of nephrotic syndrome due to minimal change disease by clinical examination and by kidney biopsy, including light, immunofluorescence, and electron microscopy..",Which ONE of the following findings in the kidney biopsy specimen would suggest a high risk for relapse following a remission after an adequate course of steroid therapy?,"A. Lower mesangial proliferation grade B. Absence of glomerular enlargement C. Presence of acute tubular necrosis D. Extent of focal global glomerulosclerosis",A,temp nan,Primary Glomerular Disease 2814,"A 28-year-old man has a diagnosis of nephrotic syndrome due to primary FSGS, not otherwise specified. His serum creatinine level is 1.5 mg/dl, his serum albumin is 2.1 g/dl, and his urine protein excretion is 8.6 g per day..",Which ONE of the following findings on immunofluorescence microscopy would MOST likely indicate a poor response to cortico-steroids and a poor long-term prognosis?,"A. Absence of IgM or C3 deposition B. IgM deposition only C. C3 deposition only D. IgM and C3 deposition",D,temp nan,Primary Glomerular Disease 2815,"A 38-year-old woman undergoes a kidney biopsy for nephrotic syndrome and hematuria. Anti–nuclear antibodies are positive at a titer of 1:80, but serum C3 and C4 levels are normal. An anti–DNA antibody test result is negative. Hepatitis B serologies are negative. The light and electron microscopic studies are most compatible with a lesion of membranous nephropathy. Immuno-fluorescence microscopy shows only scanty IgG deposits and prominent C3 deposition in a granular distribution along the capillary wall..",Which ONE of the following studies should be done next?,"A. Serum anti–PLA2R antibody B. Serum complement alternative pathway activation C. Repeated immunofluorescence study of a pronase digested paraffin block section D. Laser dissection–mass spectrometry analysis of biopsy specimen",C,temp nan,Primary Glomerular Disease 2816,"A 34-year-old woman undergoes evaluation for severe nephrotic syndrome (serum albumin 1.4 g/dl, serum creatinine 1.7 mg/dl, urine protein excretion 14 g per day, and anasarca). Both serum C3 and C4 are normal, hepatitis B serologies are negative, and anti–nuclear antibodies are positive at a titer of 1:40. The result of an ELISA anti–PLA2R antibody titer is 450 RU/ml (normal < 14 RU/ml). The blood pressure is 140/90 mmHg. She is recently married and wishes to become pregnant in the near future..",Which ONE of the following should be done next?,"A. Initiate treatment with cyclophosphamide B. Initiate treatment with rituximab C. Perform a kidney biopsy D. Initiate treatment with prednisone 1 mg/kg per day for at least 6 weeks",B,temp nan,Primary Glomerular Disease 2817,,"Question 2817: Which ONE of the following pathologic features is generally regarded as indicating a poor prognosis in IgA nephropathy, in addition to findings that are incorporated into the OX-FORD-MEST-C scores?","A. Increased C4d mesangial deposition B. Increased glomerular density C. Deposition of IgA exclusively in the mesangium D. Increased IgG codeposition",A,temp nan,Primary Glomerular Disease 2818,"A 72-year-old man is found to have membranoproliferative glomerulonephritis by light microscopy on kidney biopsy performed to evaluate proteinuria, hematuria, and impaired kidney function (serum creatinine 2.1 mg/dl). Immunofluorescence microscopy showed extensive C3 deposition in the mesangium and in subendothe-lial areas, with only a trace of IgG/IgM deposition. Electron microscopy showed extensive mesangial and subendothelial electron dense deposits, without any substructure..",Which ONE of the following tests would you do next?,"A. Serum anti–nuclear antibody and anti–dsDNA antibody B. Serum C3 nephritic factor C. Serum free light chain assay D. Serum cryoglobulins",C,temp nan,Primary Glomerular Disease 2819,A 62-year-old man experiences rapidly progressive glomerulonephritis. A kidney biopsy shows extensive crescents by light microscopy and “pseudo-linear” deposits of IgG by immunofluorescence. No electron microscopy is performed because of an insufficient specimen. The result of ELISA for serum anti–glomerular basement mem-brane antibody is negative..,Which ONE of the following tests should you do next?,"A. Serum anti–DNA antibody B. Serum free light chain assay C. Staining of renal biopsy specimen for DNAJB9 D. Repeat kidney biopsy for electron microscopy",C,temp nan,Primary Glomerular Disease 2820,,"Question 2820: Which ONE of the following indicates a high probability of recurrence of a primary form of an FSGS lesion in an adult patient receiving a living donor transplant?","A. Urine protein < 6 g per day at the time of diagnosis prior to transplant B. Serum albumin < 2.5 g/dl at the time of diagnosis prior to transplant C. Lower eGFR at the time of diagnosis prior to transplant D. Family history of nephrotic syndrome",B,temp nan,Primary Glomerular Disease 2821,"A 12-year-old boy has frequently relapsing, steroid-dependent nephrotic syndrome due to biopsy-proven minimal change disease. Prior therapy with a cyclosporine and oral cyclophosphamide has not reduced the frequency of relapses. He has severe Cushing syndrome from repeated treatment courses of oral corticosteroids. His serum creatinine level is 1.0 mg/dl..",Which ONE of the following treatment regimens would be BEST for this patient?,"A. Oral tacrolimus 0.1 mg/kg per day for 1 year B. Oral MMF 2 g per day for 1 year C. Rituximab 375 mg/m2 weekly for 4 doses D. Azathioprine 2 mg/kg per day for 1 year",C,temp nan,Primary Glomerular Disease 2822,"A 40-year-old woman of Korean descent is referred for evaluation of persistent asymptomatic microscopic hematuria that was first discovered during an assessment for life insurance. She has had 5–25 erythrocytes per high power field on urinalysis on three occasions over the past 6 months. The urine albumin-to-creatinine ratio was found to be elevated at 306 mg/g on one occasion. She does not exercise regularly and has not had any associated febrile illnesses. She does not have a family history of hematuria or kidney disease. The physical examination is normal. A urinalysis shows trace proteinuria and con-firms 5–10 erythrocytes per high power field, of which 10% are dysmorphic. No crystals or bacteria are seen. No acanthocytes or erythrocyte casts are seen. Her serum creatinine level is 0.6 mg/dL. The urine albumin-to-creatinine ratio is 326 mg/g. Which ONE of the following is the MOST likely cause of her hematuria?.","A. IgA nephropathy B. Nutcracker syndrome C. Hypercalciuria D. Urethral tear E. Thin basement membrane nephropathy likely to receive inpatient palliative care services based on data from the United States National Inpatient Sample Data?","A. IgA nephropathy B. Nutcracker syndrome C. Hypercalciuria D. Urethral tear E. Thin basement membrane nephropathy A. Acute respiratory distress syndrome B. Cardiogenic shock C. End-stage liver disease with cirrhosis D. AKI requiring dialysis",A,temp nan,Primary Glomerular Disease 2824,,"Question 2824: The transfusion of longer-term stored packed red blood cells compared with fresher blood has been associated with which ONE of the following?","1. Increased risk of transfusion-related reactions B. No difference in mortality at 90 days C. Decreased number of RRT-free days within 28 days of transfusion D. Increased risk of AKI requiring RRT E. Decreased duration of mechanical ventilation",B,temp nan,Acute Kidney Injury & Critical Care Nephrology 2825,"A 79-year-old man with stage G4 CKD and recently diagnosed colon cancer is seen in consultation after admission to the intensive care unit for septic shock. The patient has a mean arterial pressure of 62 mmHg on norepinephrine and dobutamine infusions. On physical exami-nation, he is cachectic, intubated, and sedated. The lung fields are clear to auscultation. There is 1+ leg edema. The kidney function is at base-line, and serum electrolytes are normal. Urine output is 1.3 ml/kg per hour after administration of 60 mg of intravenous furosemide. The intensivist is interested in initiating enteral feeds as soon as possible. You begin a discussion of the relative merits of this intervention in light of the recent findings from the NUTRIREA-2 study..","Based on data from the NUTRIREA-2 study, enteral feeding is MORE likely to be associ-ated with which ONE of the following?","A. Decreased incidence of systemic bacteremia B. Increased incidence of gastrointestinal complications C. Decreased incidence of hypoglycemia D. Increased incidence of 28-day mortality E. Decreased incidence of 28-day mortality",B,temp nan,Acute Kidney Injury & Critical Care Nephrology 2826,"A 67-year-old woman with a history of stage G2 CKD secondary to diabetic nephropathy is evaluated for AKI complicating septic shock. She recently underwent autologous stem cell trans-plantation for acute myelogenous leukemia. Her serum creatinine level has risen from 1.0 mg/dl to 4.9 mg/dl. She now has fever, hypotension, and suspected septic shock. Over the past 10 hours, the urine output has decreased to 0.2 mg/kg per h. A norepinephrine infusion increases the mean arterial pressure to 65 mmHg. You initiate continuous renal replacement therapy (RRT). An angiotensin II infusion is initiated, and the mean arterial pressure increases to 75 mmHg..",Which ONE of the following is CORRECT about the impact of adding the angiotensin II infusion?,"1. She is at decreased risk for inpatient mortality B. Her duration of renal replacement therapy (RRT) will be shorter C. Her Sequential Organ Failure Assessment (SOFA) score will increase D. Her duration of mechanical ventilation will be shorter",B,temp nan,Acute Kidney Injury & Critical Care Nephrology 2827,"You are caring for a 71-year-old man with stage G3a CKD disease who has recently undergone a right hemicolectomy for colon cancer. Post-operatively, he arrives in the intensive care unit receiving mechanical ventilatory support. He does not require vasopressor support. On physical examination, the blood pressure is 130/82 mmHg. He is alert and follows commands. The heart and lung examination results are unremarkable, and there is trace peripheral edema. His surgical and anesthesia teams initially plan a “restrictive” intravenous fluid regimen designed to provide a net zero fluid balance..",Which ONE of the following is associated with this restrictive fluid strategy compared with a liberal fluid strategy?,"A. A decreased risk of surgical site infections B. An increased risk of AKI requiring RRT C. An increased length of stay in the intensive care unit D. An increased risk of mortality at 1 year",B,temp nan,Acute Kidney Injury & Critical Care Nephrology 2828,"A 63-year-old kidney transplant recipient is seen in consultation for stage 2 AKI complicating urosepsis. She has received 4 L Ringer’s lactate solution over the past 8 hours. She remains intubated and ventilated, and she requires support with norepinephrine and vasopressin. Laboratory studies show sodium 138 mEq/L, potassium 4.9 mEq/L, chloride 98 mEq/L, total CO2 18 mmol/L, BUN 36 mg/dl, and creatinine 2.6 mg/dl (increased from her most recent values, which averaged 1.1 mg/dl). An arterial blood gas shows pH 7.17 and PaCO2 20 mmHg. In the setting of acidosis, AKI, and shock, you recommend an infusion of 4.2% sodium bicarbonate titrated to achieve an arterial pH of7.30..","In this setting, administration of bicarbonate as specified will be associated with which ONE of the following outcomes?","A. Increased risk of mortality B. Increased risk of hypercalcemia C. Decreased risk of progressive AKI requir-ing dialysis D. Decreased risk of intensive care unit-associated delirium E. Fewer vasopressor-free days",C,temp nan,Acute Kidney Injury & Critical Care Nephrology 2829,"You are admitting a 65-year-old kidney trans-plant recipient with presumed urosepsis. The patient is confused, disoriented, and hypotensive, with a blood pressure of 90/55 mmHg. The temperature is 39.1C. The oxygen saturation is 88% on ambient air. The oxygen saturation in-creases to 93% after initiation of oxygen at 4 L/min by nasal cannula. Laboratory studies show serum sodium 132 mEq/L, potassium 3.6 mEq/L, chloride 95 mEq/L, total CO2 18 mmol/L, BUN 18 mg/dl, creatinine 1.4 mg/dl (increased from 1.1 mg/dl 3 months ago), leukocyte count 15,200/µl, hemoglobin 6.8 g/dl, and platelet count 126,000/µl..",Which ONE of the following is the MOST important step in mitigating this patient’s morbidity and mortality in the setting of impending septic shock?,"A. Administration of a bolus of 30 ml/kg 0.9% normal saline solution B. Administration of vasopressors to achieve a mean arterial pressure of 70 mmHg C. Administration of a combination of fluids and vasopressors to achieve a target central venous saturation of 70% D. Early administration of appropriate antibiotics",D,temp nan,Acute Kidney Injury & Critical Care Nephrology 2830,"You are meeting with your hospital’s chief information officer and her team. They have several questions for you regarding the ability of the electronic medical records to improve the care of patients with AKI and also of those at risk for AKI. Although electronic alerts that are not linked to care bundles or action items have not universally led to improved outcomes, subsequent analyses have demonstrated there are still subgroups of patients who may derive a clinical benefit from electronic AKI alerts..",Which ONE of the following individuals is MOST likely to benefit from electronic alerts in the medical record that identify AKI?,"A. A 35-year-old woman with serum creatinine that has increased from 0.7 mg/dl to 2.4 mg/dl over 1 day B. A 76-year-old woman with serum creatinine that has increased from 0.7 mg/dl to 1.0 mg/dl over 3 days C. A 68-year-old woman with serum creatinine that has increased from 0.7 mg/dl to 2.4 mg/dl over 1 day D. A 66-year-old man with serum creatinine that has increased from 0.7 mg/dl to 2.4 mg/dl over 3 days",B,temp nan,Acute Kidney Injury & Critical Care Nephrology 2831,The chief quality officer at your institution asks for your advice about clinical decision support systems and decision-making algorithms developed for AKI..,"Based on the existing literature, implementing such electronic medical record support will result in which ONE of the following improvements in care for patients with and at risk for AKI?","A. A reduction in nephrotoxin exposure and events requiring renal replacement therapy (RRT), but no change in hospital mortality B. A reduction in renal replacement therapy (RRT) events, inpatient mortality, and radiocontrast exposure C. No change in nephrotoxin exposure or renal replacement therapy (RRT) events, but a reduction in inpatient mortality D. Uniform reductions in nephrotoxin expo-sure and inpatient mortality, but no change in renal replacement therapy (RRT) events E. A reduction in nephrotoxin exposure, but no change in renal replacement therapy (RRT) events and inpatient mortality",B,temp nan,Acute Kidney Injury & Critical Care Nephrology 2832,"Several weeks after joining a new practice, you note that you are often consulted late into the course of AKI. You discuss the relative merits of implementing an alert system that activates a prompt recommending nephrology consultation at the time of a clinically significant increase in the serum creatinine level or a decrease in urine output..",Which ONE of the following outcomes has been demonstrated to result from implementation of nephrology consultation prompted by an AKI alert system?,"A. Fewer cases of severe AKI, but no difference in early nephrology consultation or renal recovery B. Fewer cases of severe AKI and increased early nephrology consultation, but no change in renal recovery C. Fewer cases of severe AKI, a higher incidence of renal recovery, and increased early nephrology consultation D. No change in the rate of severe AKI, increased early nephrology consultation, and a higher incidence of renal recovery",C,temp nan,Acute Kidney Injury & Critical Care Nephrology 2833,,"Question 2833: Implementing a Kidney Disease Improving Global Outcomes—based care bundle for post-operative patients with urinary tissue inhibitor of metalloproteinase 2*IGF binding protein factor 7 levels > 0.3 [(ng/ml)2/1000] MOST consistently leads to which ONE of the following patient outcomes?","A. Shorter length of mechanical ventilation B. Decreased rate of stage 2 and 3 AKI C. Decreased requirement for RRT D. Reduced inpatient mortality E. Decreased urine output",B,temp nan,Acute Kidney Injury & Critical Care Nephrology 2834,"A 28-year-old woman with ESRD receives a de-ceased donor kidney transplant. She is given a 100-mg dose of intravenous furosemide after successful anastomosis of the renal vessels. Two hours after furosemide administration, the sub-sequent urine output is noted to be 82 ml..",This patient’s urine output in response to intravenous furosemide (furosemide stress test) BEST predicts which ONE of the following outcomes?,"A. Acute renal allograft rejection B. 1-year mortality C. Delayed graft function after renal transplantation D. 1-year graft survival after renal transplantation",C,temp nan,Acute Kidney Injury & Critical Care Nephrology 2835,,"Question 2835: Which ONE of the following patients has the LOWEST risk of AKI?","A. 56-year-old man with fulminant hepatic failure due to acetaminophen toxicity re-quiring emergent liver transplantation and a high modified Model for End-stage Liver Disease score of 32 B. A 62-year-old man with a stage G3a:A1 CKD (eGFR 46 ml/min per 1.73 m2) scheduled for elective coronary artery by-pass surgery with a GFR that increases by 42 ml/min per 1.73 m2 after a protein load of 1.2 g/kg body weight C. A 38-year-old woman with Child-Pugh class C cirrhosis, baseline serum creatinine of 1.3 mg/dl, receiving a 4-week course of ceftazidime and gentamicin for gram negative bacteremia D. A 70-year-old woman with stage G4 CKD (eGFR 28 ml/min per 1.73 m2), chronic systolic heart failure, with cardiogenic shock because of acute ischemic papillary muscle rupture now requiring an intra-aortic balloon pump and emergency mitral valve replacement",B,temp nan,Acute Kidney Injury & Critical Care Nephrology 2836,"A 28-year-old man in your hospital’s neuro-intensive care unit has just been declared brain dead after a motorcycle accident that was associated with significant head trauma. His family and physicians are interested in determining whether he is a viable candidate for kidney donation. Although you do not have any baseline information about his kidney function, his serum creatinine level has risen from 1.2 mg/dl to 1.4–1.6 ml/dl over the past 2 weeks..",Which ONE of the following biomarkers is MOST likely to predict renal tubular injury on biopsy beyond that predicted by the serum creatinine level?,"A. Urine kidney injury molecule-1 B. Urine neutrophil gelatinase-associated lipocalcin C. Urine liver–fatty acid binding protein D. Urine interleukin-18",B,temp nan,Acute Kidney Injury & Critical Care Nephrology 2837,"A 65-year-old woman with class II New York Heart Association (NYHA) heart failure, hypertension, and type 2 diabetes mellitus is under-going an evaluation for aortic valve replacement. Her baseline serum creatinine level is 2.1 mg/dl and the urine albumin-to-creatinine ratio is 536 mg/g. An echocardiogram shows a normal ejection fraction and moderate left ventricular hypertrophy..","Based on the available data, which ONE of the following statements is MOST accurate re-garding her risk of postoperative AKI?","A. Her baseline serum creatinine is not a risk factor for postoperative AKI B. The transcatheter approach for aortic valve replacement may be associated with a lower risk of AKI compared with a surgical approach C. Albuminuria, but not serum creatinine, predicts the risk of postoperative AKI D. Left ventricular dysfunction, but not NYHA heart failure class, is associated with AKI risk",B,temp nan,Acute Kidney Injury & Critical Care Nephrology 2838,"A 70-year-old man with stage G3b:A3 CKD, diabetes mellitus, and hypertension is seen in consultation before coronary angiography. There is no history of congestive heart failure, and the physical examination is normal except for signs of diabetic retinopathy and neuropathy. An echocardiogram shows normal left ventricular function. You are asked to recommend the best prophylactic regimen to prevent AKI. His eGFR is 43 ml/min per 1.73 m2. The urine albumin-to-creatinine ratio is 978 mg/g..",Which ONE of the following recom-mendations is MOST consistent with the findings of the Prevention of Serious Adverse Events Following Angiography (PRESERVE) trial?,"A. N-acetylcysteine should be administered 2 days before radiocontrast exposure to reduce the risk of dialysis-requiring AKI in this high-risk patient B. Intravenous isotonic bicarbonate infusion is superior to isotonic saline solution in this patient, who does not have congestive heart failure C. Neither N-acetylcysteine compared with placebo nor isotonic bicarbonate therapy compared with saline solution is more likely to reduce the risk of dialysis-requiring AKI after radiocontrast exposure D. He should receive both isotonic bicarbonate and N-acetylcysteine for prophylaxis because he is at extremely high risk for contrast-induced nephropathy",C,temp nan,Acute Kidney Injury & Critical Care Nephrology 2839,"A 60-year-old man with diabetes mellitus, hyper-tension, and chronic obstructive pulmonary dis-ease is admitted to the hospital with pneumonia. Blood cultures grow Pseudomonas aeruginosa. Over the next 48 hours, he experiences acute respiratory distress syndrome, hypotension requiring hemodynamic support with vasopressors, and oliguric stage 3 AKI. It is determined that continuous RRT is the best dialysis option..",Which ONE of the following statements most accurately reflects current evidence regarding the MOST appropriate modality of dialysis for this patient?,"A. Continuous venovenous hemodiafiltration is superior to other modalities of solute removal in gram-negative sepsis B. A polymyxin B adsorptive modality is indicated and has been shown to improve 28-day mortality in patients with endotoxic shock in randomized controlled trials C. Diffusive solute removal is superior to convective solute removal in this clinical setting with both continuous and intermit-tent therapies D. There is no significant difference in out-comes between diffusive or convective modalities of CRRT",D,temp nan,Acute Kidney Injury & Critical Care Nephrology 2840,"A 70-year-old man with stage G3b:A2 CKD, diabetes mellitus, and hypertension is seen in the office after a recent hospitalization for hip surgery. His hospital course was complicated by stage 1 AKI. His baseline serum creatinine level was 1.9 mg/dl. While he was hospitalized, the serum creatinine level peaked at 3.1 mg/dl and eventually fell to the current level of 2.1 mg/dl..",Which ONE of the following is the MOST accurate statement regarding the impact of the AKI episode on his long-term cardiovascular prognosis?,"A. It has no impact on his subsequent risk of cardiovascular events B. It increases his risk of incident congestive heart failure C. It is not associated with subsequent cardiac events because he did not experience dialysis-requiring AKI D. It is not associated with cardiac events because the kidney function almost com-pletely recovered to baseline",B,temp nan,Acute Kidney Injury & Critical Care Nephrology 2841,"A 55-year-old man with diabetes mellitus is evaluated in consultation for AKI complicating a 25% total body surface area burn and septic shock. The BUN and serum creatinine have risen from 46 mg/dl and 1.5 mg/dl on admission to 100 mg/dl and 5.7 mg/dl, respectively. The urine output has fallen to < 200 ml over the past 24 hours. The blood pressure is 108/60 mmHg on vasopressors, and the central venous pressure is 14 cm H2O. The intensivists caring for the patient ask you about high-dose continuous renal replacement therapy (RRT)..",Which ONE of the following should you tell them about dialysis dose and modality based on evidence derived from randomized controlled trials?,"A. Continuous renal replacement therapy (RRT), if chosen, should be provided with effluent flow rate of at least 35 ml/kg per h B. Continuous renal replacement therapy (RRT), if chosen, should be provided with effluent flow rate of at least 20 ml/kg per h C. Continuous venovenous hemofiltration should be chosen over continuous venovenous hemodialysis because it offers a survival advantage in this patient D. Randomized trials support high-volume effluent flow of 70 ml/kg per h in this clinical setting",b,temp nan,Acute Kidney Injury & Critical Care Nephrology 2842,A 72-year-old woman is seen in consultation for AKI complicating acute decompensated heart failure..,Which ONE of the following types of cardiac dysfunction is associated with the HIGHEST risk of mortality in this patient with AKI?,"A. Isolated severe right ventricular systolic dysfunction (right ventricular ejection frac-tion of 25%) B. Diastolic dysfunction with normal left ventricular systolic function C. Mild biventricular systolic dysfunction (right and left ventricular ejection fractions of 50%) D. Modest left ventricular systolic dysfunction (left ventricular ejection fraction of 45%)",A,temp nan,Acute Kidney Injury & Critical Care Nephrology 2843,"A 72-year-old man with pneumonia due to Staphylococcal aureus is seen in consultation for nonoliguric AKI. He has been treated with intravenous vancomycin, and trough levels have ranged between 15 and 20 mg/L. On physical examination, he appears frail and is in mild respiratory distress. The blood pressure is 130/ 80 mmHg, and the temperature is 38.1C. There are crackles and bronchial breath sounds in the left lower lung zone. There is trace peripheral edema. Over the past 3 days, the serum creatinine level has risen from 1.2 mg/dl to 2.4 mg/dl. The urinalysis shows 1+ protein, no blood, and 5 to 10 muddy brown casts per high-power field..",Which ONE of the following should you indicate in your consultation?,"A. Trough levels of 15 to 20 mg/L are not associated with nephrotoxicity B. Only peak vancomycin levels correlate with nephrotoxicity C. No change in vancomycin dosing is warranted because he does not have oliguria D. Vancomycin nephrotoxicity has been associated with trough levels greater than or equal to 15 mg/L",D,temp nan,Acute Kidney Injury & Critical Care Nephrology 2844,"You are asked to see a 79-year-old woman with influenza A complicated by secondary bacterial pneumonia and sepsis for AKI. She has been receiving vancomycin and piperacillin-tazobactam since admission 5 days ago. The serum creatinine has risen from 0.6 mg/dl to 1.7 mg/dl, and the urine volume has decreased to 500 ml per day. On physical examination, she is confused and bedridden. The blood pressure is 110/60 mmHg, the heart rate is 110/min, and the respiratory rate is 22/min. The lung examination reveals diffuse crackles in the lower lung zones bilaterally. The pulse is irregularly irregular. There is no cardiac murmur. There is 1+ pretibial edema. The urinalysis shows 1+ protein and many muddy brown casts. Vancomycin trough levels are 16 to 20 mg/L..",Which ONE of the following statements is the MOST accurate regarding antibiotic-mediated renal toxicity in this patient?,"A. Vancomycin nephrotoxicity is unlikely because trough levels are not greater than 25 mg/L B. Vancomycin/piperacillin-tazobactam nephrotoxicity is unlikely in the absence of leukocyte casts C. The combination of vancomycin and piperacillin-tazobactam is associated with a higher risk of AKI than either antibiotic alone D. Urgent dialysis is indicated to reduce vancomycin levels and prevent further toxicity",C,temp nan,Acute Kidney Injury & Critical Care Nephrology 2845,"A 67-year-old man with type 2 diabetes mellitus and hypertension is admitted to the coronary intensive care unit with acute decompensated heart failure. He is seen in consultation because of a rise in his serum creatinine from 1.0 mg/dl on admission to 1.3 mg/dl 3 days later. He has received intermittent boluses of intravenous furo-semide, with a net diuresis of 3 kg, which resulted in an improvement in his congestive symptoms. The serum N-terminal pro-B-type natriuretic peptide (NT-proBNP) level declined 35% from the admission level, which was initially elevated threefold above the reference range..",Which ONE of the following statements BEST characterizes the association between kidney and cardiac dysfunction in this patient?,"A. The reduction in the NT-proBNP level of > 30% is associated with improved sur-vival independent of the change in kidney function B. The benefits of lowering NT-proBNP with diuresis are negated by the risks of con-comitant worsening of kidney function C. Neither the degree of change in NT-proBNP level nor the percentage change in creati-nine is of prognostic significance in this patient D. The weight reduction of 3 kg is the most important predictor of improved outcomes in this patient",A,temp nan,Acute Kidney Injury & Critical Care Nephrology 2846,"A 76-year-old woman with chronic systolic heart failure is admitted with acute decompen-sated heart failure. Her home medication regimen included bumetanide 1 mg once daily and lisinopril. She is initially treated with increased doses of bumetanide and lisinopril. The serum creatinine rises from 1.6 mg/dl on admission to 2.1 mg/dl and is associated with onset of transient oliguria. The doses of bumetanide and lisinopril are reduced, and the serum creat-inine subsequently falls to 1.6 mg/dl on the third hospital day, with resolution of oliguria. She is discharged on an increased dose of bumetanide (1 mg twice daily) and her usual dose of lisinopril..",Which ONE of the following statements MOST accurately reflects her short-term prognosis?,"A. She is not at a risk for readmission because she experienced stage 2 but not stage 3 AKI B. Although she experienced transient azote-mia that returned to baseline within 3 days, she still is at a higher risk for readmission than are patients without AKI C. Given that her kidney function almost completely recovered to baseline, she is no longer at increased risk for adverse outcomes after discharge D. Transient oliguria is more important than rise of creatinine is predicting the risk of readmission",B,temp nan,Acute Kidney Injury & Critical Care Nephrology 2847,"A 70-year-old woman with advanced CKD due to diabetic nephropathy is evaluated before initia-tion of dialysis. She required coronary artery bypass surgery 2 months ago, which was com-plicated by stage 2 AKI. While hospitalized, the serum creatinine level rose from 2.0 mg/dl to 4.0 mg/dl. Although the creatinine at discharge had improved slightly to 3.5 mg/dl, she subsequently had progressive CKD. She now has subtle uremic symptoms and a mild degree of volume overload. On physical examination, she is in no distress. The blood pressure is 130/80 mmHg. There is 1+ leg edema. The remainder of the examination is unremarkable. Laboratory studies show serum sodium 138 mEq/L, potassium 5.4 mEq/L, chloride 102 mEq/L, total CO2 19 mmol/L, BUN 92 mg/dl, creatinine 8.5 mg/dl, calcium 9.2 mg/dl, and phosphorus 5.9 mg/dl. The hemoglobin is 10.2 g/dl. Plans are under way to start dialysis in the outpatient setting next week. She has an arteriovenous fistula that is ready for use..",Which ONE of the following statements MOST accurately characterizes the prognostic importance of AKI before initiation of long-term dialysis?,"A. Her history of AKI in the predialysis period does not affect her mortality B. The episode of AKI will not affect her survival on dialysis because she has an arteriovenous fistula C. Given her history of AKI before initiation of dialysis, she should initiate dialysis in a hospitalized setting D. Her pre-ESRD episode of AKI is associ-ated with a 30% greater risk of mortality at 1 year in comparison with patients without AKI before incident dialysis",D,temp nan,Acute Kidney Injury & Critical Care Nephrology 2848,A 65-year-old man with stage G3a CKD is seen in consultation before coronary artery bypass grafting surgery..,Which ONE of the following should you tell his surgeon with regard to on-pump versus off-pump cardiopulmonary bypass and his risk of AKI?,"A. Off-pump surgery in eligible patients is associated with a reduced risk of AKI during the first postoperative month B. Off-pump cardiac surgery is associated with long-term preservation of renal function at 1 year C. On-pump surgery poses no additional risk than off-pump surgery when the time on pump is < 90 minutes D. On-pump surgery is preferred because it provides greater renal blood flow during the surgery",A,temp nan,Acute Kidney Injury & Critical Care Nephrology 2849,"A 44-year-old man with metastatic melanoma is referred for AKI. He started combination therapy with nivolumab (an antibody-inhibiting programmed cell death protein 1 [PD-1]) and ipilimumab (a cytotoxic T-lymphocyte-associated protein 4 [CTLA-4] inhibitor) 2 months ago. Over the past 6 weeks, the serum creatinine level has risen from 1.3 mg/dl to 2.8 mg/dl. Serum complement levels are normal. A complete blood count shows leukocyte count 6500/µl, hemoglobin 12.9 g/dl, and platelets 427,000/µl. The absolute eosinophil count is elevated at 2300/µl (reference range < 400/µl). The urinalysis shows 5 to 10 leukocytes per high-power field and occasional leukocyte casts. The urine protein-to-creatinine ratio is 3204 mg/g, and the urine albumin-to-creatinine ratio is 2986 mg/g..",Which ONE of the following lesions is the MOST likely to be found on kidney biopsy?,"A. Acute tubulointerstitial nephritis with podocyte injury B. Renal-limited thrombotic microangiopathy C. Immune complex–mediated glomerulonephritis D. Minimal change glomerulopathy with acute tubular necrosis E. Secondary membranous nephropathy",A,temp nan,Acute Kidney Injury & Critical Care Nephrology 2850,"A 60-year-old man is admitted to neurosurgical intensive care unit after experiencing a hemorrhagic stroke. A ventriculostomy drain is placed because of increased intracranial pressure. His course is complicated by contrast–induced AKI, which progresses to oliguric AKI after onset of pneumonia and sepsis. On physical examination, he is intubated and sedated. The blood pressure 110/50 mmHg. The chest examination shows crackles and decreased breath sounds in the right midlung zone. There is 2+ leg edema. Laboratory studies show sodium 130 mEq/L, potassium 6.2 mEq/L, chloride 94 mEq/L, total CO2 16 mmol/L, BUN 82 mg/dl, and creatinine 6.1 mg/dl (increased from 1.2 mg/dl 8 days ago). The leukocyte count is 14,700/µl, the hemoglobin is 9.2 g/dl, and the platelet count is 203,000/µl..",Which ONE of the following is the BEST choice of renal replacement therapy (RRT) for this patient?,"A. Intermittent hemodiafiltration B. Intermittent hemodialysis C. Continuous renal replacement therapy (RRT) D. Sustained low-efficiency dialysis",C,temp nan,Acute Kidney Injury & Critical Care Nephrology 2851,"A 75-year-old woman with metastatic melanoma with cutaneous and pulmonary involvement is admitted to the intensive care unit for management of symptomatic hyponatremia. She started treatment with ipilimumab 10 weeks ago and experienced bifrontal headache with progressive confusion over the past week. On physical examination, she is lethargic. The blood pressure is 118/60 mmHg, and the heart rate is 96/min. There is no cutaneous or mucosal hyperpigmen-tation. The heart and lung examination results are unremarkable, and there is no peripheral edema or focal neurologic deficits. The deep tendon reflexes demonstrate a delayed relaxation phase. Laboratory studies show serum sodium 116 mEq/L, potassium 4.3 mEq/L, chloride 82 mEq/L, total CO2 24 mmol/L, BUN 18 mg/dl, creatinine 1.1 mg/dl, and osmolality 242 mOsm/kg. The urine osmolality is 426 mOsm/kg, and the urine sodium is 62 mEq/L. Magnetic resonance imaging of the brain with contrast material shows diffuse pituitary enlargement without focal mass lesions..",Which ONE of the following is the MOST likely cause of the hyponatremia?,"A. Ipilimumab-related hypophysitis with secondary adrenal insufficiency B. Primary adrenal insufficiency because of ipilimumab-related autoimmune adrenalitis C. Metastatic melanoma invading the pituitary gland D. Primary polydipsia",A,temp nan,Acute Kidney Injury & Critical Care Nephrology 2852,"A 70-year-old woman with stage G3b:A2 CKD due to diabetic nephropathy receives a diagnosis of metastatic renal cell carcinoma. She begins treatment with sorafinib, a tyrosine kinase inhibitor targeting the vascular endothelial growth factor receptor. Three weeks into therapy, she experiences edema, hypertension with a blood pressure of 180/100 mmHg, and increased proteinuria. Medications are amlodipine, lisinopril, chlorthalidone, and rosuvastatin. Laboratory studies show that the serum creatinine level has risen from a baseline level of 1.6 mg/dl to 2.3 mg/dl. A complete blood count with differential and serum complement levels are normal. The urinalysis shows fine granular casts and oval fat bodies. The urine albumin-to-creatinine ratio is 6400 mg/g, increased from 106 mg/g 1 month ago..",Which ONE of the following is the MOST likely diagnosis?,"A. Focal and segmental glomerulosclerosis plus thrombotic microangiopathy B. Renal vein thrombosis C. Progression of diabetic nephropathy D. Immune complex glomerulonephritis",A,temp nan,Acute Kidney Injury & Critical Care Nephrology 2853,"As the medical director of a dialysis unit you are reviewing the medical records of a 56-year-old patient with diabetes mellitus, hypertension, and a history of intravenous drug abuse who is going to start in-center hemodialysis at your dialysis center. Laboratory studies reveal negative hepatitis B surface antigen, a negative hepatitis B e antigen, a positive hepatitis B surface antibody, a positive hepatitis B core antibody, and a negative IgM hepatitis B core antibody..",Which ONE of the following statements is TRUE regarding his need for isolation and hepatitis B status?,"A. The patient does not need to be isolated because he likely has chronic hepatitis B infection B. The patient does not need to be isolated because he has developed immunity as a result of immunization C. The patient does not need to be isolated because the anti-hepatitis B core antibody is likely a false positive result D. The patient does not need to be isolated because he has developed immunity as the result of a prior hepatitis B infection E. The patient does not need to be isolated because he is recovering from an acute hepatitis B infection",D,temp nan,Infection Control & Prevention in Outpatient Hemodialysis Facilities 2854,"A 55-year-old man who has been receiving maintenance hemodialysis for 5 years is under-going evaluation for kidney transplantation. Se-rum liver chemistries are normal. Serologic studies show a positive hepatitis C antibody, a positive hepatitis B surface antibody, a positive hepatitis B total core antibody, a negative hepatitis B surface antigen, and a negative hepatitis B e antigen. The hepatitis C RNA level by polymerase chain reaction is 26,000,000 U/ml..",Which ONE of the following interpretations is CORRECT?,"A. He has both active hepatitis B and hepatitis C infection B. He has hepatitis C infection and has been vaccinated against hepatitis B C. He has hepatitis C infection and has previously been infected with hepatitis B D. He has immunity to both hepatitis B and hepatitis C because of prior infection",C,temp nan,Infection Control & Prevention in Outpatient Hemodialysis Facilities 2855,A 70-year old man with advanced chronic kidney disease due to IgA nephropathy is evaluated 1 week before the planned initiation of hemodialysis. He was born in South Korea and moved to the United States 25 years ago. A 5-tuberculin-units purified protein derivative (PPD) tuberculin skin test shows 15 mm of induration. He does not recall receiving Bacillus Calmette-Guerin (BCG) vaccination..,Which ONE of the following is the next BEST step in this patient’s management?,"A. Perform a 10-tuberculin-units PPD tuberculin skin test B. Order an interferon-γ release assay C. Order a chest radiograph D. Inform the patient that this test result is likely a false positive because of prior BCG vaccination E. Initiate isoniazid prophylaxis at 300 mg daily",C,temp nan,Infection Control & Prevention in Outpatient Hemodialysis Facilities 2856,"A 68-year-old man visiting from an outside dialysis center is evaluated during routine dialysis rounds. He has been receiving hemodialysis for the past 3 months after a kidney transplantation failed because of chronic rejection. His immuno-suppression has been tapered to prednisone 5 mg daily. He has had low-grade fever, malaise, and a pruritic rash that is intermittently painful over the chest, abdomen, and thighs for 1 week. On physical examination, his temperature is 37.9 C. Other vital signs are normal. Many clusters of dry and crusted skin lesions are noted on the left side of the chest. Numerous vesicles, pustules, and scabs in various stages of eruption are seen over his lower abdominal wall and proximal thighs. The face, neck, upper extremities, and skin overlying a left brachiobasilic fistula are spared. His vaccination record is unavailable..","In addition to standard precautions, which ONE of the following is the MOST appropriate management?","A. Cover the lesions and administer dialysis at the outpatient facility in a regular chair B. Cover the lesions and administer dialysis at the outpatient facility in an isolation room under contact precautions C. Transfer the patient to a tertiary care hospital after outpatient hemodialysis treatment D. Immediately transfer the patient to a hospital for both airborne and contact isolation",D,temp nan,Infection Control & Prevention in Outpatient Hemodialysis Facilities 2857,"Since the launch of the Nephrologists Trans-forming Dialysis Safety by the American Society of Nephrology with the support of the United States Centers for Disease Control and Prevention, you and your interdisciplinary team have made infection control a priority at your dialysis facility. The “Days Since Infection” Poster has been downloaded from the American Society of Nephrology website and has been prominently displayed in your unit. Before weekly dialysis rounds, you decide to inconspicuously observe dialysis staff performing hand hygiene..",In which ONE of the following situations is hand washing with soap and water preferred for dialysis staff over the use of alcohol-based hand rubs?,"A. After inadvertently touching a hemodialysis machine B. After documenting notes on a portable computer C. After contact with a patient with a recent diagnosis of Clostridium difficile–induced diarrhea D. Before preparation of medications for par-enteral administration E. Before changing the dressing covering the exit site of a central venous catheter (CVC)",C,temp nan,Infection Control & Prevention in Outpatient Hemodialysis Facilities 2858,"A hemodialysis center has received notice from a state health agency regarding several infection control deficiencies, including poor compliance with recommended hand hygiene procedures that were observed during a recent on-site audit..",Which ONE of the following strategies is MOST likely to improve adherence of staff with recommended hand hygiene practices?,"A. Encourage the staff to wash their hands with soap and water before and after each contact with a patient B. Promote the frequent use of gloves during the dialysis procedure because hand hygiene is not necessary if gloves are worn during patient care C. Make alcohol-based hand rub readily avail-able near dialysis stations, and observe hand hygiene opportunities monthly, pro-viding staff with feedback regarding their performance D. Minimize the time needed for hand hygiene by training staff to apply a small amount of an alcohol-based hand rub for 10 seconds",C,temp nan,Infection Control & Prevention in Outpatient Hemodialysis Facilities 2859,An investigation is initiated at a hemodialysis clinic after it is discovered that six patients have acquired hepatitis C over a period of 1 year. Nucleotide sequencing analysis by the Centers for Disease Control and Prevention revealed high homology and close clustering of hepatitis C virus quasispecies among these patients. Multiple lapses in infection control practices at the dialysis center were identified..,Which ONE of the following infection control gaps has frequently been associated with outbreaks of hepatitis C virus infection in hemodialysis units?,"A. Dialyzing hepatitis C-positive patients in the main area of the unit instead of dialyzing the patient using a dedicated room, machine, and equipment B. Failure of personnel to change gloves and perform hand hygiene when moving be-tween patients, between patients and potentially contaminated surfaces, and between machines C. Poor compliance by staff with recommended use of gowns when caring for patients with open skin wounds with drainage not contained by dressings D. Failure to adequately reprocess reusable dialyzers",B,temp nan,Infection Control & Prevention in Outpatient Hemodialysis Facilities 2860,"Outbreaks of hepatitis C and, less frequently, hepatitis B and bacterial bloodstream infections in outpatient hemodialysis centers have often been due to poor adherence to infection control practices. Inappropriate handling and administration of parenteral medications in hemodialysis clinics pose an additional risk for the transmission of such pathogens..",Which ONE of the following medication injection safety measures minimizes the risk of blood-borne pathogen transmission in outpatient hemodialysis facilities?,"A. Use a mobile cart to transport injectable medications to multiple patients B. Prepare medications at the patient’s dialysis station whenever possible C. Avoid administration of medications from the same syringe to more than one patient D. When using single-dose vials of medication, refrigerate vials containing residual (excess) medication before pooling for administration to subsequent patients.",C,temp nan,Infection Control & Prevention in Outpatient Hemodialysis Facilities 2861,You are making rounds with two first-year nephrology fellows at an outpatient dialysis clinic. The fellows ask you about the removal of endotoxins during water purification..,Which ONE of the following components of water purification should you instruct them is responsible for the removal of the majority of endotoxins before the final pass through endotoxin ultrafilters prior to feeding the distribution loop?,"A. Water softener resin B. Brine tank C. Carbon medium D. Reverse osmosis membrane",D,temp nan,Infection Control & Prevention in Outpatient Hemodialysis Facilities 2862,As medical director of a hemodialysis facility you are reviewing four possible occurrences with the care team that would necessitate disinfection of the internal hemodialysis components before the machine is used for the subsequent treatment..,Which ONE of the circumstances would warrant disinfection of the internal hydraulic components of the hemodialysis machine between treatments?,"A. An air leak B. A blood leak due to a ruptured dialyzer membrane C. Completion of hemodialysis in a patient with hepatitis C virus infection D. Completion of hemodialysis in a patient with HIV infection",B,temp nan,Infection Control & Prevention in Outpatient Hemodialysis Facilities 2863,"A 62-year-old man who initiated hemodialysis is evaluated during weekly rounds. The charge nurse has placed the patient in isolation because the hepatitis B surface antigen returned positive on admission laboratory studies. A review of his record indicates that he had no prior history of hepatitis. Serologic studies performed 3 months ago as part of an evaluation for transplantation showed that the hepatitis B surface antigen, hepatitis B surface antibody, and hepatitis B core IgM were all negative. He is up to date on pneumococcal vaccination, received influenza vaccination several weeks ago, and received his first dose of hepatitis B vaccine 5 days ago. On physical examination, his vital signs are normal. He has no pallor or jaundice. The remainder of his physical examination results are normal. Laboratory studies show normal liver chemistries, nor-mal prothrombin time, normal albumin, negative hepatitis B surface antibody, negative hepatitis B core IgM, and negative total hepatitis B core antibody..",Which ONE of the following is the BEST explanation of the positive hepatitis B surface antigen serologic result?,"A. Acquired immunity from vaccination against hepatitis B virus B. Prior hepatitis B virus infection C. Acute hepatitis B infection D. Recent exposure to hepatitis B vaccine",D,temp nan,Infection Control & Prevention in Outpatient Hemodialysis Facilities 2864,"In your role as medical director of a dialysis facility you decide to observe dialysis staff during a shift change. A newly hired technician is observed taking a patient off dialysis. The technician is observed returning a patient’s blood, followed by clamping of the arteriovenous lines. The lines are then disconnected from the dialysis machine. The technician places a nonsterile pad over the arterial site and removes the arterial needle while wearing nonsterile gloves. After hemostasis is achieved, a nonsterile gauze pad is placed on the arterial site. This same procedure is repeated for the removal of the venous needle. He then moves to the next patient, dons a new set of gloves, and repeats the same takeoff procedure..",Which ONE of the following represents a breach in proper infection control practices?,"A. Placement of a nonsterile gauze pad at the cannulation site B. Failure to apply antibiotic ointment at the arterial or venous sites cannulation sites before placing the dressing C. Failure to perform proper hand hygiene between patient encounters D. Failure to use sterile gloves during decannulation",C,temp nan,Infection Control & Prevention in Outpatient Hemodialysis Facilities 2865,A recent continuous quality improvement analysis at your dialysis unit revealed that the frequency of CVC infections has recently risen to > 1 per 1000 catheter days (equivalent to > 3 per 100 patient-months). Current procedures include cleansing of exit sites with povidine-iodine and use of sterile gauze to cover the exit at each treatment. An antibiotic lock solution is currently being used in one patient with a history of recurrent catheter-related bloodstream infections (CRBSIs)..,Which ONE of the following interventions is the next BEST step to reduce the frequency of CRBSIs in your unit?,"A. Change sterile gauze to a transparent semi-permeable dressing after each treatment B. Use an alcohol-based chlorhexidine (.0.5%) solution for cleansing the exit site skin, and scrub the hub with 70% alcohol C. Use needle-free connectors for all CVC hubs D. Use gentamicin-citrate lock solution in all patients E. Use an alteplase lock solution in all patients",B,temp nan,Infection Control & Prevention in Outpatient Hemodialysis Facilities 2866,"One afternoon, you enter the treatment area of your dialysis facility and notice fresh blood (about 30 ml) on the floor from a patient who experienced post procedure bleeding from the access site. After appropriate measures are taken, the bleeding has subsided and the patient is clinically stable. You review the policy for cleaning and disinfection of blood spills as the staff proceed to clean the treatment area..",Which ONE of the following is the MOST appropriate method for removing blood on the floor?,"A. Single application of soap and water B. Double application of soap and water C. Single application of tuberculocidal disinfectant D. Double application of tuberculocidal disinfectant E. Soap and water followed by tuberculocidal disinfectant",D,temp nan,Infection Control & Prevention in Outpatient Hemodialysis Facilities 2867,"An outpatient hemodialysis facility’s medical director became concerned upon receipt of the updated National Healthcare Safety Network Facility Rate Table. The table indicated that the facility’s bloodstream catheter infection rate was significantly higher than the national average. The interdisciplinary team, under the leadership of the medical director, reviewed and analyzed data, including the results of CDC Infection-Related Audit Tools through the facility’s Quality Assurance Performance Improvement program (QAPI). Deficiencies were found in the areas of hand hygiene, catheter connection and disconnection, dialysis station routine disinfection, and catheter exit site care. The medical director recommended corrective actions to improve the facility culture of safety rather than addressing each process individually..",Which ONE of the following BEST describes how medical directors can influence the culture of safety in dialysis facilities?,"A. Develop action plans to improve infection control practices, explaining the rationale of each plan with provision of feedback on the plan’s impact B. Provide educational materials to both nephrology physicians and facility staff directed toward best infection prevention and control practices C. Develop new policies and guidelines that are easier to understand compared with those currently being used D. Ensure that error-prone staff members undergo disciplinary actions and re-education E. Designate an individual among the facility staff who is charged with assuring proper infection prevention and control techniques throughout the facility",A,temp nan,Infection Control & Prevention in Outpatient Hemodialysis Facilities 2868,"During a Quality Assessment and Performance Improvement (QAPI) meeting, the clinical manager informs the medical director and dialysis team that the clinic has experienced an increased rate of bloodstream infections (BSI) and a high rate of CVC access use over the past 6 months..",Which ONE of the following BEST characterizes the expected role of medical director with regard to this clinical issue according to the United States Center for Medicare and Medicaid Services Conditions for Coverage for ESRD facilities?,"A. Medical directors are responsible for the oversight of all care-related activities, including the high BSI and CVC rates at that facility B. Medical directors manage the interdisciplinary team (IDT) for all patients, and the IDT evaluates the causes of high BSI and CVC rates C. Medical directors should serve as the attending physician for all patients at their designated facility D. The nurse manager is the clinician who oversees the QAPI program that would address the high infection rate",A,temp nan,Infection Control & Prevention in Outpatient Hemodialysis Facilities 2869,"You are employed as a full-time nephrologist by a regional for-profit health care (HC) system that owns three local hospitals. The management team of this HC entity plans to open an outpatient dialysis facility in close proximity to their hospitals. You are asked to provide expertise in the design of the dialysis facility. Specifically, they ask your opinion about hemodialyzer reuse at the facility..",Which ONE of the following should you tell the management team about hemodialyzer reuse?,"A. Infection risk with hemodialyzer reuse is equal to that of a single-use dialyzer B. Gram-negative bloodstream infections have been reported with hemodialyzer reuse C. Dialyzer reprocessing of reuse dialyzers is an easy-to-perform single-step procedure with the latest automated reprocessing machines D. The current Association for the Advancement of Medical Instrumentation (AAMI) guidelines mandate a single specific step-by-step procedure for hemodialyzer reuse to minimize risks",B,temp nan,Infection Control & Prevention in Outpatient Hemodialysis Facilities 2870,Your dialysis facility technician and nurse manager contact you for recommendations after cultures from the last outlet of the distribution loop reveal a bacterial level of 300 colony forming units/ml. Hemodialysis is actively being performed during the receipt of the culture report..,Which ONE of the following is the MOST appropriate management?,"A. Prescribe prophylactic antibiotics for the patients receiving hemodialysis B. Immediately discontinue dialysis and order blood cultures C. Decrease dialysate flow and blood flow rates to minimize diffusive exposure D. Assess whether continuing dialysis is less detrimental than withholding treatment E. Change to a different hemodialysis machine with new dialyzer and blood line",D,temp nan,Infection Control & Prevention in Outpatient Hemodialysis Facilities 2871,"A 54-year-old woman receives maintenance hemodialysis treatments through a left internal jugular tunneled catheter. She has exhausted all other options for permanent dialysis access. Upon arrival for a scheduled hemodialysis treatment, she reports having fever and chills but no additional symptoms. On physical examination, the patient is in no acute distress. Her temperature is 38.6C, her blood pressure is 140/86 mmHg, her heart rate 96/min, and her respiratory rate is 14/min. The catheter exit site is unremarkable, and the patient is alert and otherwise clinically stable. No peripheral veins are identified for peripheral blood cultures..",Which ONE of the following is the next BEST step in this patient’s management?,"A. Obtain two sets of blood cultures from the hemodialysis catheter before administering antibiotics B. Obtain one set of blood cultures from the hemodialysis catheter and one set of blood cultures from the hemodialysis circuit, both before antibiotic administration C. Cancel the scheduled dialysis treatment and send the patient to the emergency room for evaluation and management D. Administer antibiotics immediately, then obtain two sets of blood cultures from the hemodialysis circuit–one from the arterial port and another from the venous port",B,temp nan,Infection Control & Prevention in Outpatient Hemodialysis Facilities 2872,"A 47-year-old woman has ESRD resulting from lupus nephritis. One month ago, she transitioned to in-center hemodialysis from peritoneal dialysis because of ultrafiltration failure. Four days ago, she experienced a fever to 38.7C, chills, and cough during a scheduled hemodialysis treatment. She did not have dyspnea or hypotension. Your nephrology partner requested blood cultures and ordered intravenous vancomycin. A chest radiograph showed a left lower lobe in-filtrate. The oxygen saturation was 98% on ambient air, and the leukocyte count was 9600/µL. Two days later, the blood cultures return, showing methicillin-sensitive Staphylococcus aureus in both bottles that is sensitive to vancomycin, gentamicin, cefazolin, daptomycin, and linezolid. The patient has improved and is now afebrile. She has no known drug allergies..",Which ONE of the following is the MOST appropriate antibiotic regimen for this patient?,"A. Continue vancomycin for at least 4 weeks and monitor trough levels B. Add gentamicin after every dialysis treatment for synergy C. Replace vancomycin with cefazolin to complete at least 4 weeks of antibiotics D. Replace vancomycin with daptomycin to complete 4 weeks of antibiotics E. Discontinue vancomycin and start oral linezolid to complete 4 weeks of antibiotics",C,temp nan,Infection Control & Prevention in Outpatient Hemodialysis Facilities 2873,"A patient from your dialysis center with recently diagnosed vancomycin-resistant Staphylococcus aureus bacteremia is ready for hospital discharge. The patient has received 1 week of antibiotic therapy, and infectious disease consultants have recommended outpatient parenteral anti-infective therapy for several more weeks with daptomycin plus ceftaroline through a separate internal jugular vein access. There is one separate isolation room at your facility that is currently used to treat several patients with hepatitis B virus infection. As a result, the isolation room is not available for this patient..","In addition to strict adherence to standard infection control practices, which ONE of the following additional infection control measures should you recommend for this patient?","A. Dialyze the patient at a station with as few adjacent stations as possible B. Inform the hospital to cancel the discharge because the patient needs inpatient isolation until she completes the antibiotic course C. Recommend hand hygiene with soap and water for dialysis staff after contact with this patient D. Recommend that staff wear reusable gowns when caring for the patient at all times",A,temp nan,Infection Control & Prevention in Outpatient Hemodialysis Facilities 2874,,"Question 2874: Which ONE of the following statements regarding antibiotic use in outpatient hemodialysis units in the United States is CORRECT?","A. Patients with CVCs have equivalent rates of intravenous antibiotic use compared with patients with arteriovenous fistulas and grafts B. First-dose antimicrobial therapy in hemodialysis units accounts for the highest proportion of inappropriate intravenous antibiotic use C. Over 60% of patients using long-term hemodialysis receive at least one dose of intravenous antibiotics each year D. Vancomycin is the most commonly pre-scribed intravenous antibiotic",D,temp nan,Infection Control & Prevention in Outpatient Hemodialysis Facilities 2875,"A 40-year-old man receiving maintenance hemodialysis is found to have methicillin-resistant Staphylococcus bacteremia after presenting to the dialysis unit with fever. He has a mature arteriovenous fistula that is cannulated by staff using the “buttonhole” technique. Examination of the arteriovenous fistula shows no erythema, tenderness, or purulent discharge from the cannulation sites. The results of his physical examination are otherwise normal. An echocardiogram and magnetic resonance images of the spine are normal. He is treated with vancomycin for 4 weeks, with resolution of his fever. He had a similar episode 6 months ago..",Which ONE of the following is the BEST strategy to prevent recurrent bloodstream infections in this man?,"A. Culture his nares and treat with intranasal mupirocin if the culture grows Staphylococcus aureus B. Evaluate the processes of care in the dialysis facility such as wearing masks, washing hands, and using antiseptics and gloves when accessing the fistula C. Switch to the “rope ladder” for needle placement in the fistula D. Focus on patient hygiene such as showering and washing the fistula arm with soap and water before each dialysis treatment E. Obtain surveillance blood cultures from the fistula at monthly intervals, and treat when the results are positive",C,temp nan,Infection Control & Prevention in Outpatient Hemodialysis Facilities 2876,The medical director of a dialysis unit has oversight of the infection prevention and control program. In putting together the infection prevention and control plan you highlight the essential components and resource requirements of the program during a monthly Quality Assessment and Performance Improvement meeting..,Which ONE of the following is an essential component of the infection prevention and control program within a dialysis facility?,"A. Infection surveillance and use of infection rate data to drive prevention B. One full-time staff person who is in charge of the infection control program C. Hiring an epidemiologist to investigate blood-stream infections D. Use common medication carts to deliver medications to patients",A,temp nan,Infection Control & Prevention in Outpatient Hemodialysis Facilities 2877,Water supplies may be contaminated or disrupted during a natural or civil emergency. Medical directors are often called to determine best options for the outpatient hemodialysis facility when these occur..,Which ONE of the following statements is TRUE about managing potentially contaminated dialysis water supplies in the event of a natural disaster?,"A. A tanker truck that has been repurposed from hauling vegetable oil for transporting water is an acceptable source of water for hemodialysis pretreatment systems B. Water transported by tanker trucks to dialysis facilities should not be chlorinated before the beginning of the haul C. Water from a well deeper than 30 meters is considered a safe water supply for hemodialysis after a hurricane or flooding D. During a “Boil-Water Advisory,” dialysis water treatment systems that rely on de-ionization units can be used without additional modifications",A,temp nan,Infection Control & Prevention in Outpatient Hemodialysis Facilities 2878,"An outpatient hemodialysis facility receives an immediate jeopardy citation from a Medicare surveyor after repeated violations are discovered in infection control involving repeated cross-contamination of medical supplies and improper disinfection of treatment surfaces. The facility has previously been cited for poor infection control. Interviews with numerous facility staff, including the medical director, suggest a widespread lack of individual accountability and a tendency to blame others for problems in the unit..",Which ONE of the following issues BEST describes the fundamental problem at this facility?,"A. Outdated infection control policies B. Inadequate staffing C. Poor patient and staff understanding of hand hygiene D. High community incidence of healthcare-acquired infections E. Lack of an effective culture of safety",E,temp nan,Infection Control & Prevention in Outpatient Hemodialysis Facilities 2879,"During a review of cultures of dialysate at your hemodialysis center, you note a bacterial con-centration of > 50 colony-forming units/ml (CFU/ml)..","In addition to ongoing retesting, which ONE of the following is the MOST appropriate next step in management?","A. No additional measures are required B. Withhold dialysis treatments at the center until the bacterial counts are < 25 CFU/ml C. Perform chemical disinfection D. Replace the reverse osmosis membrane",C,temp nan,Infection Control & Prevention in Outpatient Hemodialysis Facilities 2880,"A 70-year-old man receiving maintenance hemodialysis is seen on dialysis rounds. Pruritic nodules have developed on his trunk and extremities. His primary care physician prescribed a topical corticosteroid cream that has helped with the pruritus, but the nodules continue to reappear. A physical examination shows linearly arranged 3- to 5-mm hyperpigmented nodules with excoriation. His partner has had similar symptoms. You suspect that the lesions are due to bedbug bites..","In addition to symptomatic treatment with topical corticosteroids and oral antihistamines, which ONE of the following is the MOST appropriate management?","A. Dialyze the patient at the far end of the room, and restrict the chair solely for that patient’s use B. Implement no additional environmental measures at the dialysis facilities C. Make a mandatory assignment of the patient to an isolation room D. Use a single 8- to 10-hour application of permethrin 5% cream on the entire body",A,temp nan,Infection Control & Prevention in Outpatient Hemodialysis Facilities 2881,As the medical director you receive a call from an attending nephrologist at your dialysis unit. This nephrologist reports that a 10-year-old patient has received a diagnosis of an active infestation of pediculosis capitis..,"In addition to thorough bathing and heat washing of the patient’s affected clothes and linens and use of a topical pediculicide, which ONE of the following is the MOST appropriate management?","A. Use extra care in the terminal cleaning of the dialysis station, with special attention to the dialysis chair after the patient has completed the dialysis treatment B. Examine all patients in the facility for head lice because lice may survive for weeks on dialysis equipment and chairs C. Institute daily hair washing with a shampoo/conditioner or conditioner before and after application of a topical pediculicide D. Prescribe lindane shampoo for the affected patient",A,temp nan,Infection Control & Prevention in Outpatient Hemodialysis Facilities 2882,"You are paged by a nurse from a hemodialysis facility you are covering about an 81-year-old woman with ESRD due to polycystic kidney disease, along with mild cognitive impairment. Her spouse has noted that she has not been eating well for the past 3 days. She has been more confused than usual and has complained of “feeling hot.” The patient has no other specific complaints and is oriented to place and person. She is not taking any new medications. Her temperature is 37.3C, her pulse is 96/min and regular, and her blood pressure is 102/80 mmHg. The patient is in no respiratory distress but is mildly tachypneic, with a respiratory rate of 22/min. The oxygen saturation is 92% on ambient air. Her weight is equal to the estimated dry weight. The nurse hears crackles at both lung bases. There is no leg edema. The arteriovenous fistula has no erythema or fluctuance..",Which ONE of the following is the next BEST step in this patient’s management?,"A. Review the patient’s baseline vital signs B. Order a 500-ml bolus of intravenous normal saline C. Lower the patient’s target weight by 1 kg D. Order empiric vancomycin and ceftriaxone",A,temp nan,Infection Control & Prevention in Outpatient Hemodialysis Facilities 2951,"A 65-year-old woman with type 2 diabetes mellitus, obesity, and stage G3a:A1 CKD asks about dietary modifications to reduce her risk of CKD progression. She currently follows a 2000-calorie diet that is low in sodium, fat, and sugar. She drinks several diet sodas per day..",Which ONE of the following dietary interventions is MOST likely to reduce her risk of CKD progression?,"A. Discontinuation of caffeinated beverages B. Discontinuation of milk-containing beverages C. Consumption of .2 liters of water per day D. Discontinuation of beverages containing artificial sweeteners",D,temp nan,Chronic Kidney Disease and Progression 2952,"A 35-year-old woman of African ancestry who is 12 weeks pregnant with her first child is seen in consultation because she is concerned about the potential risks of APOL1 risk variants. Her husband has focal segmental glomerulosclerosis (FSGS) and carries two APOL1 risk variants for CKD. Which ONE of the following should you tell her about the potential impact of APOL1 risk variants on her pregnancy and her child’s risk of kidney disease? A. Her risk of preeclampsia is further increased if she carries two APOL1 risk variants B..","Her child has a 50% chance for the development of FSGS C. About 13% of African Americans carry two high-risk APOL1 variants, and the pene-trance of associated CKD is ,40%","A. Her risk of preeclampsia is further increased if she carries two APOL1 risk variants B. Her child has a 50% chance for the development of FSGS C. About 13% of African Americans carry two high-risk APOL1 variants, and the pene-trance of D. Inheritance of other genetic variants in addition to the APOL1 risk variants is required for kidney disease to develop in the child",C,temp nan,Chronic Kidney Disease and Progression 2953,You have been assigned to speak at a regional internal medicine meeting about CKD. You begin your talk with a discussion of the burden of kidney disease in the United States and globally..,Which ONE of the following should you tell the audience?,"A. Over the past decade, the total number of people with CKD in the United States has remained stable B. Disability-adjusted life years attributed to CKD have increased in the United States, whereas disability-adjusted life years for cardiovascular disease and cancer have decreased C. Deaths due to CKD have remained stable within the United States but have increased worldwide D. About 5% of the population in the United States has stage 1-5 CKD",B,temp nan,Chronic Kidney Disease and Progression 2954,"An 82-year-old Korean War Veteran returns for follow-up of stage G3b:A1 CKD attributed to hypertension. He has not had a kidney biopsy. He has a history of gout. He served in the U.S. Navy and worked for over 10 years as a welder in the manufacture of naval ships. He has never had his blood lead levels measured. You raise the possibility of lead exposure as a possible con-tributing factor in his kidney disease. The serum creatinine level is 1.8 mg/dl, the urine albumin-to-creatinine ratio is 20 mg/g, and the urine protein-to-creatinine ratio is 130 mg/g..",Which ONE of the following should you tell him regarding lead exposure and kidney disease?,"A. Only very high whole blood lead levels (.1000 mg/L) are associated with lead nephropathy B. Prior lead exposure can cause gout but is not associated with CKD C. Even mild elevations in blood lead levels among adults have been associated with an increased risk of CKD D. His CKD is not due to lead because he does not have overt proteinuria",C,temp nan,Chronic Kidney Disease and Progression 2955,,"Question 2955: Which ONE of the following statements regarding the public health impact of diabetes mellitus in the United States is CORRECT?","A. Approximately one out of two cases of CKD is attributed to diabetes B. Population efforts to reduce diabetes are unlikely to lower the CKD burden in the United States C. The prevalence of CKD is twofold higher among adults with diabetes than in those without diabetes D. More timely and effective implementation of CKD care has been shown to reduce adverse kidney disease outcomes in adults with diabetes",D,temp nan,Chronic Kidney Disease and Progression 2956,"In the year 2000, the incidence rate of ESRD among Native Americans was 805 per million population, a rate fourfold higher than whites in the United States. By the year 2016, ESRD incidence among Native Americans declined substantially and approached the incidence rates observed in white individuals in the United States..",Which ONE of the following factors has been shown to be associated with this sharp decline in ESRD incidence among Native Americans over the past 16 years?,"A. A population-based approach to diabetes care based in the community and the primary clinical setting B. Transfer of diabetes care traditionally as-signed to primary care providers to nephrologists C. The use of new glucose-lowering agents D. Prescription of fenofibrate in patients with CKD and serum triglyceride levels between 150 and 499 mg/dl",A,temp nan,Chronic Kidney Disease and Progression 2957,"A 68-year-old African American man returns for follow-up of type 2 diabetes mellitus and albu-minuria. You inform him that his BP is elevated at 152/90 mmHg. He indicates that he frequently forgets to take his antihypertensive medications. Laboratory studies show a serum creatinine level of 1.0 mg/dl and a urine albumin-to-creatinine ratio of 22 mg/g. The hemoglobin A1c is 9%. His best friend, who is African American, had pro-gressive CKD because of diabetic nephropathy and required initiation of dialysis at age 69. He frequently accompanies his friend to the dialysis facility and has noticed that most of the patients served by the center are African American. He feels helpless and believes that dialysis is in-evitable. He is skeptical about the benefits of glucose lowering and hypertension control..",Which ONE of the following should you tell him about his risk for and therapy of kidney disease?,"A. Tight control of BP and glucose are more effective in white than black adults with type 2 diabetes for prevention of CKD B. Regardless of race, control of glucose levels and BP among adults with type 2 diabetes mellitus slows progression of CKD C. His cumulative lifetime risk of ESRD is .80% D. If he experiences diabetic nephropathy, he can expect more rapid progression of CKD if he has two APOL1 high-risk variants",B,temp nan,Chronic Kidney Disease and Progression 2958,"A 65-year-old man with CKD due to diabetic nephropathy is evaluated after a recent decline in GFR from 33 to 28 ml/min per 1.73 m2 over the past year. One week ago, his endocrinologist discontinued metformin and started sitagliptin at a reduced dose (25 mg daily). The patient is concerned about the safety of this medication in the context of his xCKD..",Which ONE of the following should you tell him about the use of incretin-based therapies such as sitagliptin in the setting of CKD?,"A. Incretin-based therapies increase the risk of hypoglycemia B. Incretin-based therapies are associated with an increased risk of mortality when used in adults with non–dialysis-dependent CKD C. Incretin-based therapies are not associated with an increased risk of hypoglycemia or mortality among adults with non–dialysis-dependent CKD when compared with other glucose-lowering agents D. Incretin-based therapies increase the rate of CKD progression",C,temp nan,Chronic Kidney Disease and Progression 2959,"A 72-year-old woman with type 2 diabetes mellitus and stage G3b:A3 CKD with an esti-mated GFR (eGFR) of 39 ml/min per 1.73 m2 and a urine albumin-to-creatinine ratio of 400 mg/g 1 year ago returns for follow-up. Laboratory stud-ies on the day of the clinic visit show an eGFR of 29 ml/min per 1.73 m2 and a urine albumin-to-creatinine ratio of 250 mg/g. She takes losartan 100 mg daily, and her BP is 128/65 mmHg. She tells you that her endocrinologist recently started empagliflozin to reduce her risk of cardiovascular disease. She wants to know what effects this drug will have on her kidney function..",Which ONE of the following is CORRECT regarding the effects of empagliflozin on kidney function?,"A. The annual decline in eGFR over the next several years is anticipated to be more rapid B. The risk of kidney stones is increased with the use of empagliflozin C. The risk of ESRD is increased with the use of empagliflozin D. The initial decline in eGFR will be accom-panied by a slower rate of eGFR decline over time",D,temp nan,Chronic Kidney Disease and Progression 2960,"A 55-year-old man is evaluated in follow-up care for stage G3b:A2 CKD. He is accompanied by his brother, who tells you that the patient uses marijuana on occasion. He is concerned that this use will worsen his brother’s kidney function..",Which ONE of the following should you tell them about the effects of marijuana on CKD incidence and progression?,"A. Daily but not occasional marijuana use is associated with CKD incidence and progression B. Daily use of marijuana is associated with a faster decline in kidney function C. Marijuana use is associated with increased urine albumin excretion D. Marijuana use is not associated with CKD incidence or progression",D,temp nan,Chronic Kidney Disease and Progression 2961,"The director of a community health center is updating the center’s anti-drug and anti-tobacco campaign. She asks your opinion whether it is correct to highlight cigarette smoking, cocaine, heroin, and methamphetamine use as risk factors for kidney disease..",Which ONE of the following should you tell her?,"A. Yes, cigarette smoking, cocaine, metham-phetamine, and heroin use are all associated with an increased risk of kidney failure B. No, because only cocaine use is associated with kidney disease C. No, because the risk of kidney disease consequent to illicit drug use is restricted to patients with diabetes and hypertension D. No, because methamphetamine and heroin use have no impact on CKD progression",A,temp nan,Chronic Kidney Disease and Progression 2962,A 49-year-old woman with stage G3b:A3 CKD due to diabetic nephropathy is evaluated in follow-up care. You review her laboratory studies with her and begin to explain the significance of a fourfold increase in the urine-to-creatinine ratio over the past 3 years. She notes that her BP has been better controlled after an increase in her enalapril dose 3 months ago. She would like to repeat the albumin-to-creatinine ratio today..,Which ONE of the following should you tell her about the significance of the albumin-to-creatinine ratio and its measurement?,"A. Declines in the albumin-to-creatinine ratio are not associated with a reduced risk of ESRD B. The recommended frequency of albumin-to-creatinine ratios is every 3 months C. Currently, progression of CKD is clinically categorized only by changes in the albumin-to-creatinine ratio D. A fourfold increase in the albumin-to-creatinine ratio may indicate a higher risk of ESRD",D,temp nan,Chronic Kidney Disease and Progression 2963,"A 55-year-old woman with type 2 diabetes mellitus and stage G3a:A3 CKD is referred for worsening albuminuria. Over the past year, the urine albumin-to-creatinine ratio has increased from 212 mg/g to 362 mg/g. Her medications include losartan 100 mg daily and simvastatin 20 mg daily. Her BP is at goal, with a resting BP of 128/75 mmHg in clinic measured with an auto-mated device after she rested for 5 minutes, which is consistent with previous measurements over the past 2 years. She has no edema. A 24-hour urine revealed a daily urinary sodium excretion of 248 mmol. Laboratory studies show a serum sodium of 138 mEq/L, potassium of 4.9 mEq/L, chloride of 100 mEq/L, total CO2 of 24 mmol/L, BUN of 18 mg/dl, creatinine of 1.3 mg/dl, calcium of 9.0 mg/dl, phosphorus of 2.6 mg/dl, 25-hydroxyvitamin D level of 25 ng/ml, and parathyroid hormone of 56 pg/ml..","In addition to continued losartan, which ONE of the following is the MOST appropriate management of the increased albuminuria?","A. Cholecalciferol 1000 units daily B. A 2-gram sodium-restricted diet C. Paricalcitol 1 mg daily D. Lisinopril 5 mg daily",B,temp nan,Chronic Kidney Disease and Progression 2964,"A 65-year-man with morbid obesity, type 2 diabetes mellitus, and Barrett esophagus is re-ferred for evaluation and management of stage G4:A2 CKD. His medications are chlorthalidone 25 mg daily, losartan 100 mg daily, amlodipine 10 mg daily, and omeprazole 40 mg daily. His resting sitting BP is 125/68 mmHg. Laboratory studies show an eGFR of 28 ml/min per 1.73 m2 and a urine albumin-to-creatinine ratio of 450 mg/g. The urine sediment shows a few hyalofatty casts but is otherwise unremarkable. His eGFR has not changed over the past 12 months. He asks if anything can be done to slow his CKD pro-gression. He read a recent press release about proton pump inhibitors and CKD progression, and he asks if he should stop or change this medication..",Which ONE of the following should you tell him about proton pump inhibitors in the context of his CKD?,"A. His proton pump inhibitor should be stop-ped because it is a major risk factor for CKD progression B. The proton pump inhibitor should be stop-ped because it increases the risk of hyper-magnesemia in advanced CKD C. The proton pump inhibitor should not be discontinued if the medication is required to treat his Barrett esophagus and there is no clinical evidence of harm D. Studies have shown strong and consistent associations of proton pump inhibitors with CKD risk and progression",C,temp nan,Chronic Kidney Disease and Progression 2965,"You are seeing a 65-year-old man with a newly hired advanced-practice provider in your practi-ce’s CKD clinic. The patient has stage G4:A2 CKD. He has no complaints and denies fatigue, shortness of breath, or chest pain. His medications include chlorthalidone, losartan, pravastatin, and aspirin. The BP is 128/67 mmHg. The remainder of the physical examination is unremarkable. Laboratory studies reveal an eGFR of 22 ml/min per 1.73 m2, which is stable when compared with the eGFR measured 4 months ago. The urine albumin-to-creatinine ratio is 45 mg/g. The hemo-globin is 8.9 g/dl. The iron saturation is 32%. The advanced-practice provider asks about adding an erythropoiesis-stimulating agent..",Which ONE of the following should you tell the advanced-practice provider regarding initiation of erythropoiesis-stimulating agents in this patient?,"A. An erythropoiesis-stimulating agent will not slow his CKD progression B. An erythropoiesis-stimulating agent will not improve his functionality and exercise tolerance C. Erythropoiesis-stimulating agent use will not reduce his need for blood transfusions D. An erythropoiesis-stimulating agent should be held when hemoglobin levels exceed 10.0 g/dl",A,temp nan,Chronic Kidney Disease and Progression 2966,"A 22-year-old man with a remote history of acute kidney disease during childhood is referred to you for evaluation. He has no hypertension or diabetes mellitus and takes no prescribed or over-the-counter medications. His past medical history is notable for a hospitalization when he was a toddler for severe pneumonia, which was complicated by “a kidney problem,” and he thinks he may have received dialysis a few times. He has otherwise been healthy, but he does not have any more information on his past illness. The eGFR is 98 ml/min per 1.73 m2 by the CKD-Epi equation and the urine albumin-to-creatinine ratio is 17 mg/g..",Which ONE of the following should you tell him regarding his lifetime risk of ESRD?,"A. His lifetime risk of ESRD may be increased by more than fourfold because of his child-hood history of kidney injury B. His risk is not increased because his serum creatinine is now normal C. The childhood illness is not relevant for this patient D. Adults with childhood CKD have a higher risk of ESRD than do adults without a childhood CKD history, but initiation of dialysis in adults with a history of childhood CKD occurs at a later age",A,temp nan,Chronic Kidney Disease and Progression 2967,"A 31-year-old man from Costa Rica is referred to you by a colleague for evaluation of an elevated serum creatinine level. He has no history of diabetes mellitus or hypertension. His body mass index is 17.9 kg/m2, and his BP is 122/82 mmHg. The remainder of the examination is normal. Laboratory studies show a serum creatinine of 1.7 mg/dl and a urine albumin-to-creatinine ratio of 37 mg/g. Serum complement levels are nor-mal. The urine sediment shows a few fine granular casts. You suspect the patient may have Mesoamerican nephropathy..",Which ONE of the following biopsy findings is MOST consistent with the diagnosis of Mesoamerican nephropathy?,"A. Tubulointerstitial nephritis with infiltrates predominantly of T lymphocytes B. Mesangial and subendothelial immune deposits C. Collapse and sclerosis of the entire glomer-ular tuft D. Hypercellular crescent formation with fibri-noid necrosis",A,temp nan,Chronic Kidney Disease and Progression 2968,"A 58-year-old man with stage G3a:A3 CKD secondary to diabetic nephropathy is evaluated during a routine follow-up visit. His medications are lisinopril 40 mg once daily, atorvastatin 40 mg once daily, and metformin 500 mg twice daily. On physical examination, the BP is 128/85 mmHg. The remainder of the examination is unremarkable. Home BP readings average 128/80 mmHg. Laboratory studies show a hemo-globin A1c of 7.1% and an eGFR of 48 ml/min per 1.73 m2. The urine albumin-to-creatinine ratio is 420 mg/g..",The addition of which ONE of the following interventions is the MOST likely to delay progression of his CKD?,"A. Losartan B. Atrasentan C. An SGLT-2 inhibitor D. Liraglutide",C,temp nan,Chronic Kidney Disease and Progression 2969,"A 57-year-old man with type 2 diabetes and hypertension is evaluated in follow-up care for stage G3a:A3 CKD. He was recently prescribed empagliflozin by his primary care physician after his hemoglobin A1c level was found to be elevated at 7.9%. He is asymptomatic. The BP is 128/82 mmHg. He has no edema. Laboratory data show an eGFR of 54 ml/min per 1.73 m2 and UACR of 350 mg/g. He is concerned about the adverse effects of empagliflozin, especially in the setting of preexisting kidney disease..",Which ONE of the following should you tell him about potential adverse events related to treatment with sodium glucose cotransporter-2 (SGLT-2) inhibitors?,"A. Amputation has been reported with all SGLT-2 inhibitors B. The risk of genital infections is higher C. There is no risk of acute kidney injury D. The risk of hypoglycemia is increased",B,temp nan,Chronic Kidney Disease and Progression 2970,,"Question 2970: Which ONE of the following BEST describes current evidence regarding the use of hypo-glycemic agents in CKD?","A. Dulaglutide is superior in reducing CKD progression and albuminuria compared with insulin glargine B. Incretin-based therapies delay the progres-sion of CKD C. Only individuals with stage G4 CKD de-rive cardiovascular benefit from SGLT-2 inhibitors D. The risk of metformin-associated lactic acidosis is dependent on the stage of CKD",D,temp nan,Chronic Kidney Disease and Progression 2971,"An 81-year-old man with stage G4:A3 CKD is evaluated during a routine follow-up visit. He has long-standing hypertension, type 2 diabetes mel-litus, and hyperlipidemia. He is asymptomatic. On physical examination, the BP is 132/84 mmHg. The pulse is found to be irregularly irregular, with a rate of 74/min. The remainder of the physical examination is normal. An elec-trocardiogram shows atrial fibrillation, which was not noted in a previous electrocardiogram ob-tained 2 weeks ago. Laboratory studies show a serum creatinine level of 2.6 mg/dl and a urine albumin-to-creatinine ratio of 680 mg/g. You discuss the benefits and risks of anticoagulation..",Which ONE of the following should you tell him regarding the potential benefits and risks he will derive from anticoagulation for atrial fibrillation?,"A. It will not increase his risk of bleeding B. It will lower his risk of ischemic stroke C. It will lower his mortality risk D. Direct oral anticoagulants, but not warfarin, have been proved to improve clinical out-comes in stage 4 CKD",C,temp nan,Chronic Kidney Disease and Progression 2972,"A 66-year-old woman with a history of atrial fibrillation, hypertension, hyperlipidemia, gastro-esophageal reflux disease, and depression is re-ferred for evaluation of CKD. Her medications include amiodarone, fluoxetine, furosemide, pan-toprazole, and atorvastatin. On physical exami-nation, the BP is 116/78 mmHg. The pulse is irregularly irregular. There is trace pedal edema. The remainder of the examination is normal. Laboratory studies show a hemoglobin of 12.1 g/dl, sodium of 142 mEq/L, potassium of 3.6 mEq/L, chloride of 102 mEq/L, total CO2 of 29 mmol/L, BUN of 63 mg/dl, and creatinine of 1.5 mg/dl (eGFR 35 ml/min per 1.73 m2). An electrocardiogram shows normal sinus rhythm with a prolonged QTc interval of 520 milliseconds..",Which ONE of her medications is LEAST likely to be directly contributing to the in-creased QTc interval?,"A. Furosemide B. Pantoprazole C. Citalopram D. Amiodarone",B,temp nan,Chronic Kidney Disease and Progression 2973,"A 58-year-old African American woman with stage G4:A3 CKD is evaluated during a routine follow-up visit. Her primary care physician re-cently diagnosed atrial fibrillation and started anticoagulation with warfarin. An echocardiogram showed a left ventricular ejection fraction of 50%, an elevated left ventricular mass index, increased relative wall thickness, elevated pulmonary artery pressure, and increased left atrial diameter..",Which ONE of the following findings on her echocardiogram is associated with either a >30% decline in eGFR or ESRD?,"A. Increased pulmonary artery systolic pressure B. The left ventricular ejection fraction of 50% C. Increased left ventricular mass index D. Increased relative wall thickness",C,temp nan,Chronic Kidney Disease and Progression 2974,"A 42-year-old woman with stage G3a:A2 CKD, morbid obesity, type 2 diabetes mellitus, and hypertension is seen in consultation before planned bariatric surgery. Her eGFR is 59 ml/min per 1.73 m2. The urine albumin-tocreatinine ratio is 190 mg/g. She asks about the potential benefits and risks of undergoing bariatric surgery..",Which ONE of the following should you tell her about her the benefits and risks of bariatric surgery?,"A. It is not associated with an increased risk of acute kidney injury B. It reduces the risk of incident CKD and ESRD C. It does not have an impact on urinary protein excretion D. It has no impact on resolution or reduction of hypertension",B,temp nan,Chronic Kidney Disease and Progression 2975,"A 56-year-old man with hypertension is seen in consultation for CKD. His eGFR is 44 ml/min per 1.73 m2, decreased from 50 ml/min per 1.73 m2 3 years ago. On physical examination, the BP is 118/70 mmHg, and the body mass index is 34 kg/m2. The remainder of the examination is normal. His most recent laboratory studies show a total cholesterol of 188 mg/dl, low-density lipoprotein-cholesterol of 110 mg/dl, calcium of 9.8 mg/dl, phosphorus of 3.6 mg/dl, hemoglobin A1c of 5.2%, and high-sensitivity C-reactive protein of 4.2 mg/L (reference range ,3 mg/L)..",Which ONE of the following interventions has the BEST potential to improve his cardiovas-cular and kidney health?,"A. Diet and exercise B. Lorcaserin C. Bariatric surgery D. Further BP reduction",A,temp nan,Chronic Kidney Disease and Progression 2976,"A 63-year-old woman with a history of hyper-tension, obesity, and depression is seen in follow-up care for stage G4:A2 CKD. She was recently found to have iron deficiency anemia. The results of colonoscopy and esophagogastroduodeno-scopy were normal. Laboratory studies show a hemoglobin of 8.6 g/dl and a transferrin satu-ration of 10%. You discuss the risks and benefits of available iron supplements..",Which ONE of the following should you tell her?,"A. The 1-year risk of infections and cardiac-related adverse events is increased with intravenous ferric carboxymaltose com-pared with oral iron therapy B. Oral iron supplementation is unlikely to improve her hemoglobin level C. Oral iron supplementation with ferric citrate leads to higher rates of adverse events when compared with placebo D. Compared with oral iron therapy, the in-cidence of adverse events is similar with intravenous ferric carboxymaltose",D,temp nan,Chronic Kidney Disease and Progression 2977,"You are asked to see a 70-year-old man for the management of stage G3b:A3 CKD and associ-ated complications. He has type 2 diabetes mellitus, hypertension, and coronary artery dis-ease, for which he underwent coronary artery bypass surgery 5 years ago. A recent echocardio-gram shows a left ventricular ejection fraction of 35%. An electrocardiogram shows normal sinus rhythm with normal intervals (QRS in-terval 110 milliseconds). Laboratory studies show a serum sodium of 137 mEq/L, potassium of 3.8 mEq/L, chloride of 105 mEq/L, total CO2 of 21 mmol/L, BUN of 34 mg/dl, creatinine of 1.5 mg/dl, and an eGFR of 46 ml/min per 1.73 m2 (comparedwithaneGFRof48ml/minper 1.73 m2 1 year ago and 51 ml/min per 1.73 m2 2 years ago)..",Which of ONE the following is MOST likely to improve his survival?,"A. Cardiac resynchronization therapy B. Placement of an implantable cardioverter defibrillator C. Addition of mineralocorticoid receptor antagonis D. Addition of sodium bicarbonate targeting a total CO2 level of 23 to 28 mmol/L",C,temp nan,Chronic Kidney Disease and Progression 2978,"A 57-year-old man with type 2 diabetes, hyper-tension, peripheral artery disease, and stage G2:A3 CKD is evaluated in follow-up care. His medications include lisinopril, metformin, cana-gliflozin, pravastatin, and aspirin. The physical examination reveals evidence of diabetic retinop-athy and neuropathy. The eGFR, using the CKD-EPI creatinine-based equation, has consistently ranged from 70 to 75 ml/min per 1.73 m2 over the past year. The urine albumin-to-creatinine ratio is 650 mg/g..",Which ONE of the following factors is LEAST likely to place him at increased risk for amputation?,"A. The current level of eGFR B. Increased albuminuria C. The presence of neuropathy D. Canagliflozin therapy",A,temp nan,Chronic Kidney Disease and Progression 2979,You have been asked by your colleagues to review new methods of cardiovascular risk prediction in your CKD practice. You review the literature and present your findings..,Which ONE of the following should you report to your colleagues?,"A. Coronary flow reserve declines with kidney disease progression, and cardiovascular risk in patients with low coronary flow reserve is amplified in those with advanced CKD B. Elevations in N-terminal pro–brain natriuretic peptide (NT-proBNP) are not associated with an increased risk of subsequent cardiac events C. The addition of echocardiographic variables to clinical variables and NT-proBNP improves cardiovascular risk prediction D. Coronary artery calcification in CKD is associated with an increased risk of adverse cardiovascular events",D,temp nan,Chronic Kidney Disease and Progression 2980,An 82-year-old man with stage G4:A2 CKD is brought in by his family for evaluation of CKD. A recent MRI without gadolinium contrast material performed during an evaluation after a fall showed abnormal white matter volume. His family is interested in understanding the significance of these findings and its relationship to kidney function..,Which ONE of the following statements about cognitive function and MRI data in CKD is CORRECT?,"A. Albuminuria and lower eGFR are associated with worse global cognitive function B. Both higher UACR and lower eGFR are associated with abnormal white matter volume C. Both higher UACR and lower eGFR are associated with higher grey matter volume D. Increased albuminuria is not associated with poorer processing speed and working memory",B,temp nan,Chronic Kidney Disease and Progression 5804," A 52-year-old woman with ESRD due to IgA nephropathy has been on the transplantation wait-ing list for 2 years. She has tolerated peritoneal dialysis well without complications and has not had any recent hospitalizations. Her blood type is B, and the calculated panel of reactive antibodies (cPRA) is 80% with 2 years of dialysis time. The expected waiting times in her region are 2 years, 3 years, 4 years, and 4.5 years for blood types AB, A, B, and O, respectively. The waiting time for a high kidney donor profile index (KDPI) (> 85%) donor is approximately 6 to 12 months shorter. She asks about options that could reduce her waiting time. She declines consent for Public Health Service-Increased Risk (PHS-IR) donor kidney or a hepatitis C–positive kidney..",Which ONE of the following is the BEST available treatment option to reduce her waiting time?,"1. Consent for high KDPI kidney offers 2. Testing for anti-A antibody titers to assess suitability for non-A1 donor kidneys 3. Desensitization with rituximab and intravenous immunoglobulin (IVIG) 4. Immunoadsorption therapy to remove lymphocytotoxic antibodies",B,temp nan,Transplantation 5805," A 69-year-old woman with ESRD due to type 2 diabetes mellitus is to receive a deceased-donor transplant after 4 years on the transplantation waiting list. Shortly after placement on the waiting list, she had a 1-week hospitalization for catheter-associated sepsis. Later that year, she underwent emergent cholecystectomy and spent 8 days in the hospital. She has had periods of “inactive status” while awaiting transplantation because of deconditioning, including a hospital stay for a hip fracture after a fall within the past year. She is unable to work, lives at home, and cares for most of her personal needs requiring only occasional assistance, yielding a Karnofsky score of 60. A recent timed “get up and go” test was 15 seconds (11–20 seconds is consistent with frail elderly or disabled status). Her cardiovascular test results are satisfactory, and she is otherwise considered an acceptable candidate for transplantation..",Which ONE of this patient’s characteristics is associated with INCREASED mortality after transplantation?,"1. Hospital admissions during the first year of waitlisting 2. A serious fall in the year before transplantation 3. A Karnofsky score of 60 4. A “timed get up and go” test result indicative of frailty",A,temp nan,Transplantation 5806," A 71-year-old man with ESRD due to diabetic nephropathy maintained on peritoneal dialysis for the past 2 years is evaluated for kidney trans-plantation. He has coronary artery disease that required three-vessel coronary bypass surgery 5 years ago. His left ventricular function is normal. A recent stress myocardial perfusion scan revealed no ischemia. He does not have peripheral vascular disease and is not frail on formal assessment. He is approved for kidney transplantation and is placed on the active waitlist. His expected posttransplantation survival (EPTS) score is 93. He cannot identify potential living donors. In his region, he would be predicted to receive a KDPI 35-85 kidney with 4 years of dialysis time and a KPDI > 85 kidney with 3 years of dialysis time..","When considering his survival with or without transplantation and potential organ acceptance offers for him, which ONE of the following should you recommend?","1. He should not be consented for KDPI .85 kidneys because of the increased risk associated with high KDPI kidneys transplanted to individuals with high EPTS scores 2. He should not be placed on the waiting list because he is not expected to have a survival benefit from transplantation 3. He should be encouraged to consent for KDPI > 85 kidneys because of the increased survival associated with high KDPI kidneys compared with remaining on the waitlist for a KDPI 35-85 kidney 4. He should be considered only for KDPI > 85 kidneys, given his high EPTS",C,temp nan,Transplantation 5807," A 52-year-old man with autosomal dominant poly-cystic kidney disease and ESRD has been active on the kidney transplantation waitlist for 5 years. He receives a kidney offer and is informed “the kidney appears excellent” but is considered a PHS-IR donor. The patient contacts you and asks whether he should accept the organ..","A discussion begins about the benefits and risks of receiving a PHS-IR donor kidney versus continued waiting for a non–PHS-IR donor kidney When considering the acceptance of a PHS-IR donor compared with continued waiting for a donor that is NOT at increased risk, which ONE of the following should you tell him?","1. There is no survival advantage in accepting the PHS-IR kidney versus accepting the next non-IR offer 2. It is likely that the PHS-IR kidney is of higher expected function/longevity than the next non-IR offer will be 3. The risk for hepatitis C virus (HCV) infection is about 5% and for HIV is about 1% 4. Although graft function is historically inferior from PHS-IR donors than from non-IR donors, this difference is now mitigated by newer antiviral treatments",B,temp nan,Transplantation 5808," A 67-year-old man with ESRD due to obstructive uropathy who initiated hemodialysis 3 months ago is placed on the transplantation waiting list. He has no identified living donors, and his waiting time for a deceased donor is estimated to be about 5 years. He wishes to explore all possible transplantation options, including consideration of an HCV-infected (HCV +/nucleic acid testing [NAT] + donor. He is seronegative for HCV..",Which ONE of the following should you tell him regarding kidney transplantation with HCV1 donor kidneys?,"1. If untreated, HCV Ab1/NAT2 kidney donors carry a risk of viral transmission that is > 90% 2. Direct-acting antiviral therapy for the clearance (cure) of HCV is less effective in the kidney transplantation population than in the general population 3. The median waiting time to transplantation is reduced to a similar extent for those who consent to receive a kidney from an HCV Ab + donor as those who consent to receive a high KDPI kidney 4. Transplantation of HCV Ab + /NAT + kidneys followed by direct-acting antiviral therapy is cost effective compared with continued waiting during dialysis and subsequent receipt of an HCV2 deceased-donor kidney",D,temp nan,Transplantation 5809," A 59-year-old woman wishes to donate her kidney to her niece, a 32-year-old woman with ESRD due to IgA nephropathy. She has no past medical or surgical history. Her BMI is 25 kg/m2, her blood pressure is 132/80 mmHg, and her measured GFR is 95 ml/min per 1.73 m2..",Which ONE of the following INCREASES her risk of ESRD after donation?,"1. Female sex 2. Relationship to the recipient 3. Age > 50 years 4. BMI 25 kg/m2",C,temp nan,Transplantation 5810," A deceased-donor kidney from a donor with anoxic brain injury and brain death is to be shipped from Virginia to Arizona for a patient with a cPRA of 98%. The expected cold ischemia time is 26 hours, increasing concern for delayed graft function once transplanted..",Which ONE of the following interventions has been shown to REDUCE the risk of delayed graft function?,"1. Hypothermic machine perfusion (HMP) of the explanted kidney 2. Dopamine infusion of the donor before procurement 3. Remote ischemic conditioning of the recipient (thigh occlusion) 4. Complement inhibition of the recipient at the time of transplant",A,temp nan,Transplantation 5811," A patient is scheduled to receive a deceased-donor kidney procured in their local area with an expected cold ischemia time (the time from procurement to initiation of transplant vascular anastomosis) of 8 hours, a time far lower than the national average. The kidney is placed on an HMP pump..","In the setting of low cold ischemia time, which ONE of the following is an advantage of HMP pumped kidneys compared with standard cold storage preservation with University of Wisconsin solution?","1. Reduced incidence of delayed graft function 2. Reduced incidence of acute rejection 3. Improved graft survival 4. Reduced mortality",A,temp nan,Transplantation 5812," A 59-year-old African American woman with hypertensive nephrosclerosis is evaluated 3 months after living unrelated kidney transplantation. She is maintained with tacrolimus 4 mg twice daily, myco-phenolate, and prednisone. Her 12-hour tacrolimus trough levels vary between 5 and 12 ng/ml. She experiences of insomnia, tremor, and headaches. You believe her symptoms may be related to tacrolimus, and you suggest transitioning her to once-daily LCP tacrolimus (LCPT) at a dose of 6 mg daily..","Which ONE of the following is TRUE regarding the pharmacokinetics of LCPT compared with immediate release, twice-daily tacrolimus?","1. Lower C maximum (peak concentration) and lower AUC (area under the curve, total exposure 2. Similar C maximum but lower AUC 3. Higher C maximum but lower AUC 4. Lower C maximum but higher AUC",D,temp nan,Transplantation 5813," A 38-year-old man with reflux nephropathy and tophaceous gout is evaluated 6 months after a zero-HLA matched living kidney donor transplantation. He is taking tacrolimus, mycophenolate mofetil, prednisone, and allopurinol. He has a BMI of 24 kg/m2, and his serum creatinine has ranged between 1.1 and 1.4 mg/dl. He has had no complications except for persistent neutropenia. Trimethoprim-sulfamethoxazole and valganciclovir were held, and the dose of mycophenolate mofetil was progressively lowered to 250 mg twice daily. Attempts to taper allopurinol failed because of severe flares of gout. He has required filgrastim injections weekly for the past 3 weeks. He is cytomegalovirus (CMV) IgG negative, and his donor was CMV IgG positive. He has not had CMV viremia by CMV polymerase chain reaction (PCR) assays every 2 weeks for the past 2 months. He has no fever and is asymptomatic..",Which ONE of the following modifications of maintenance immunosuppression is the MOST appropriate treatment option for this patient?,"1. Discontinue tacrolimus, start belatacept 2. Discontinue mycophenolate, start azathioprine 3. Discontinue tacrolimus, start everolimus 4. Discontinue mycophenolate, start everolimus",D,temp nan,Transplantation 5814," A 50-year-old woman with ESRD due to polycystic kidney disease has a cPRA of 50% and has a strongly positive crossmatch with her husband, her only potential living donor. Her blood type is A, and her husband’s blood type is O. She has been on the kidney transplantation waiting list for 2 years. The mean waiting time to transplantation in her region is 3 years..",Which ONE of the following is MOST likely to result in transplantation within the next 6 to 12 months?,"1. Desensitization with plasma exchange, IVIG, and rituximab followed by living donor transplantation 2. Entrance into a paired exchange program 3. Continued waiting for a deceased-donor kidney 4. Desensitization with IVIG and rituximab while awaiting deceased-donor transplantation",B,temp nan,Transplantation 5815," A 45-year-old woman with ESRD due to type 2 diabetes mellitus has a cPRA of 88% as a result of prior blood transfusions. Her husband is evaluated and approved as a living kidney donor. Her blood type is O, and he has blood type B. Her anti-B IgG titer is 1:16. She does not have any preformed donor-specific HLA antibodies to her husband. Given the challenge of finding an appropriate match in a paired exchange network because of her high cPRA and blood type, and the likely success with plasmapheresis in achieving a negative crossmatch with her husband, an ABO-incompatible transplantation is performed after desensitization. The desensitization protocol includes four plasmapheresis sessions, IVIG after each treatment, and rituximab at the time of transplantation..","Compared with ABO-compatible trans-plantation, which ONE of following outcomes at 1 year is expected in this patient?","1. Decreased risk of antibody-mediated rejection 2. Decreased risk of bleeding complications 3. Decreased risk of infections 4. Increased risk of uncensored graft loss",D,temp nan,Transplantation 5816," A 55-year-old woman who underwent kidney transplantation 18 months ago undergoes allograft biopsy because of a sustained rise in her serum creatinine from 1.4 mg/dl to 1.9 mg/dl over a 3-month period. She has a de novo donor-specific antibody to HLA A27 (3200 mean fluorescence intensity). The biopsy report describes double contours of the glomerular basement membrane, rare foci of tubulitis with < 4 cells per high power field, interstitial inflammation involving 25% of the total cortex, areas of interstitial fibrosis with 25% sclerosis of the cortical parenchyma, 2+ peritubular capillaritis with staining for the complement degradation product C4d, but no large vessel inflammation or hyalinosis..",Which ONE of the following is the histologic diagnosis?,"1. Grade IA chronic active T cell–mediated rejection 2. Acute mixed T cell–mediated and antibody-mediated rejection 3. Acute T cell–mediated rejection 4. Chronic active antibody-mediated rejection",D,temp nan,Transplantation 5817," A 24-year-old man receives a diagnosis of biopsy-proven active antibody-mediated rejection after a period of medication nonadherence. The biopsy specimen is notable for interstitial edema without clear evidence of fibrosis..",Which ONE of the following treatment regimens has the BEST evidence to support its use for the treatment of antibody-mediated rejection?,"1. Plasmapheresis and IVIG 2. Plasmapheresis, IVIG, and rituximab 3. Plasmapheresis, IVIG, and bortezomib 4. IVIG and rituximab",A,temp nan,Transplantation 5818," A 46-year-old woman is scheduled to receive a 2-haplotype matched kidney from her younger sister. During her preoperative visit, she asks about the success rates of clinical trials of tolerance induction for someone like her with such a very well-matched kidney donor because she hopes to avoid long-term immunosuppression..",Which ONE of the following should you tell her about tolerance induction?,"1. Clinical tolerance has never been successfully induced in human kidney transplant recipients 2. Clinical tolerance has been induced in zero antigen mismatched living donor recipients, although it is durable in only about one-third of patients 3. Demonstration of CD4(+) CD25(high) FOXP3(+) T regulatory cells in peripheral blood fails to predict transplantation tolerance 4. B cells play no role in the induction and maintenance of clinical tolerance",B,temp nan,Transplantation 5819," A 39-year-old man is evaluated during a routine office visit 3 months after living related donor kidney transplantation. The primary disease that led to ESRD was primary focal segmental glomerulosclerosis (FSGS). The urine protein-to-creatinine ratio is found to have risen to 1500 mg/g. A biopsy specimen of the allograft shows recurrent FSGS. The patient asks you about his prognosis..",Which ONE of the following should you tell him about his findings?,"1. Younger age at the time of transplantation is associated with a lower risk of recurrent FSGS 2. Recurrent glomerular disease significantly reduces the rate of death-censored graft survival to about 10% at 5 years 3. Subnephrotic-range proteinuria after kidney transplantation does not affect allograft survival 4. Proteinuria after kidney transplantation in-creases the risk of graft failure but not death 5. Recurrent disease is more likely to develop after living related donor transplantation compared with deceased-donor kidney transplantation",E,temp nan,Transplantation 5820," A 69-year-old woman is assessed preoperatively before a scheduled living related donor kidney transplantation next month (her daughter is the donor). She has required resection of two squamous cell carcinomas from her face, both of which were < 1 cm. Review of the pathology report indicates that the margins were clear. Her last skin cancer excision was 3 years ago. She has had ongoing dermatology follow-up examinations annually. Examination of the skin shows no new skin lesions. She asks about the significance of her history of skin cancer with respect to her outcomes after transplantation..",Which ONE of the following should you tell her about skin cancer and kidney transplantation?,"1. Pretransplantation skin cancer has no impact on the risk of graft failure 2. Pretransplantation skin cancer has no impact on the risk of solid organ malignancy de-velopment after transplantation 3. Pretransplantation skin cancer has no impact on the risk of mortality after transplantation 4. Pretransplantation skin cancer increases the risk of posttransplantation lymphoproliferative disorder",D,temp nan,Transplantation 5821," A 22-year-old woman with ESRD due to renal dysplasia receives a deceased-donor kidney transplant. She is Epstein Barr virus (EBV) seronegative, and her donor is Epstein Barr seropositive (EBV D + /R -). Four months after transplantation, the result of a screening EBV PCR is positive, with a viral load of 4250 copies/ml. She feels well and has no symptoms. She is concerned about this finding and asks about its significance..","In addition to the associated increased risk of posttransplantation lymphoproliferative disorder, which ONE of the following should you tell her?","1. Persistent high-grade EBV viremia is a risk factor for cancer other than posttransplantation lymphoproliferative disorder 2. Patients with EBV viremia during the first 6 months after transplantation have equivalent allograft survival compared with EBV-negative patients 3. Patients with late-onset EBV viremia after transplantation are at increased risk of death compared with EBV-negative patients 4. The risk of EBV-associated complications can clearly be mitigated with antiviral therapy",A,temp nan,Transplantation 5822,,"Question 5822 Which ONE of the following is a risk factor for post-transplantation malignancy?","1. Younger age 2. Female sex 3. Use of two or more immunosuppressive agents 4. Black race",C,temp nan,Transplantation 5823," A 48-year-old woman has ESRD due to FSGS. She is wait-listed for deceased donor transplantation and is seen for a routine re-evaluation. Her BMI is 36 kg/m2. She expresses concern about her risk for the development of diabetes mellitus after transplantation and asks about steroid elimination (no corticosteroids after an intraoperative bolus)..",Which ONE of the following should you tell her about steroid elimination after the intra-operative steroid bolus following transplantation and the risk of diabetes mellitus?,"1. It will increase her risk of rejection 2. It will reduce her risk of becoming diabetic 3. It will adversely affect her risk of early graft failure and mortality 4. It will adversely affect her achieved eGFR 5. It will increase her risk of recurrent FSGS",A,temp nan,Transplantation 5824," A 49-year-old man with type 1 diabetes and ESRD is referred for kidney transplantation. He has hypertension, dyslipidemia, and no history of cardiovascular disease. His electrocardiogram shows mild left ventricular hypertrophy. A trans-thoracic echocardiogram reveals left ventricular hypertrophy and mild diastolic dysfunction with-out overt pulmonary hypertension..",Which ONE of the following statements about pre-transplantation cardiovascular screening tests for this patient is most CORRECT?,"1. Trial data clearly indicate the need for left and right heart catheterization 2. An exercise treadmill stress test is sufficient to screen for asymptomatic coronary artery disease 3. Most transplantation programs submit trans-plantation candidates to a rigorous cardio-vascular evaluation, although the evidence basis for this approach is lacking 4. Cardiac positron emission tomography myocardial perfusion imaging has replaced the need for coronary angiography for such patients",C,temp nan,Transplantation 5825," A 63-year-old woman with IgA nephropathy is evaluated 2 weeks after receiving a preemptive deceased-donor kidney transplant. She has known hypertension and dyslipidemia. She has previously experienced myalgias while taking atorvastatin at a dose of 80 mg daily, prompting her to stop statin therapy about 1 month before transplantation. You discuss re-initiation of statin therapy. She asks about the potential impact of statin therapy on her outcomes..",Which ONE of the following should you tell her about statin therapy and outcomes after kidney transplantation?,"1. Statin therapy after transplantation is associated with improved survival after transplantation 2. Statin therapy after transplantation reduces the risk of major adverse cardiovascular events without affecting mortality 3. Statin therapy is ineffective after transplantation because of the impact of immunosuppression on lipid levels 4. There is no relationship between statin dose and outcomes after transplantation",A,temp nan,Transplantation 5826," A 47-year-old woman with ESRD due to IgA nephropathy is evaluated for kidney transplantation. She was previously treated with a 6-month course of corticosteroids and is concerned about her future risk of fracture. Her medications include lisinopril for hypertension and omeprazole for gastroesophageal reflux. Her laboratory studies show a serum parathyroid hormone level (PTH) of 172 pg/ml (reference range 12–88 pg/ml), an alkaline phosphatase level of 128 U/L (reference range 38–126 U/L), a 25-hydroxy-vitamin D level of 30 ng/ml, and a procollagen type-1 N-terminal propeptide level of 80 µg/L (reference range 16–96 µg/L). A dual energy x-ray absorptiometry scan shows normal bone density with a Z score of 2.0 and a T score of 1.2..",Which ONE of this patient’s clinical parameters is associated with the GREATEST risk of fracture after transplantation?,"1. The procollagen type-1 N-terminal propeptide level 2. Omeprazole therapy 3. The serum PTH level 4. The results of dual energy x-ray absorptiometry 5. The serum alkaline phosphatase level",B,temp nan,Transplantation 5827," An endocrinologist consults with you about your patient, a 62-year-old man who received a living donor kidney transplant 8 months ago who has persistent hyperparathyroidism. Recent laboratory studies showed a serum creatinine level of 1.2 mg/dl, calcium of 11.8 mg/dl, phosphorus of 1.7 mg/dl, PTH of 387 pg/ml, and a 25-hydroxy-vitamin D level of 32 ng/ml. A nonobstructing stone measuring 8 mm is seen on an ultrasonogram of the transplanted kidney. You and your endocrine colleague discuss the relative merits of sub-total parathyroidectomy versus cinacalcet therapy..",Which ONE of the following should you tell your endocrine colleague about subtotal parathyroidectomy versus therapy with cinacalcet in renal transplant recipients?,"1. Cinacalcet is more effective in increasing serum phosphorus levels than subtotal parathyroidectomy 2. Cinacalcet is more effective than subtotal parathyroidectomy in lowering PTH levels 3. Subtotal parathyroidectomy more effectively controls hypercalcemia when compared with cinacalcet 4. Cinacalcet is associated with a higher risk of hypocalcemia than subtotal parathyroidectomy 5. Subtotal parathyroidectomy is associated with improved patient survival compared with cinacalcet",C,temp nan,Transplantation 5828,,"Question 5828 Which ONE of the following statements about the treatment of post-transplantation osteoporosis is most CORRECT?","1. Raloxifene improves bone mineral density in transplant recipients 2. Teriparatide reduces the risk of fracture after kidney transplantation 3. Denosumab is a safe and effective treatment for osteoporosis after transplantation 4. Bisphosphonate therapy is associated with improvements in bone mineral densitometry scores after transplantation 5. Vitamin D analogues clearly reduce the risk of fracture after transplantation",D,temp nan,Transplantation 5829," A 58-year-old woman with ESRD of unknown cause is referred for transplantation. She was recently found to have HCV infection (genotype 1a). The HCV RNA PCR level is 200,000 IU/ml. She has no evidence of cirrhosis. She asks whether she should be treated for hepatitis C before or after kidney transplantation..",Which ONE of the following should you tell her about the treatment of hepatitis C in relation to kidney transplantation?,"1. Hepatitis C should be treated before trans-plantation to minimize the risk of liver failure after transplantation 2. Hepatitis C treatment should be deferred until after transplantation of a hepatitis C + graft to reduce waiting time 3. Hepatitis C should be treated before trans-plantation to abrogate the risk of coinfection with a different strain of virus that may pose the risk of drug resistance 4. Hepatitis C should be treated before trans-plantation to minimize the risk of rejection 5. Regardless of whether hepatitis C is treated before or after transplantation, she is at risk of anemia because of the requirement for ribavirin therapy",B,temp nan,Transplantation 5830," A 39-year-old man with ESRD of unknown cause is referred for transplantation. He has HIV infection that is well controlled by highly active antiretroviral therapy. His CD4 count is normal. He has never had an AIDS-defining diagnosis, and his CD4 count is > 400 cells/µL. He asks about the impact of his HIV infection on his outcomes after transplantation..",Which ONE of the following should you tell him about outcomes following kidney transplantation in people with well controlled HIV infection?,"1. HIV positive patients have a higher risk of graft loss than HIV2 recipients 2. HIV positive kidney transplant recipients have the same risk of mortality as HIV2 recipients 3. Concurrent use of protease inhibitors with tacrolimus poses an increased risk of nephrotoxicity without affecting the risk of graft failure 4. The risk of opportunistic infections is higher in HIV positive kidney transplant recipients compared with HIV negative recipients 5. Concurrent HCV positivity will have no additional impact on outcome after transplantation",A,temp nan,Transplantation 5831,,"Question 5831 Which ONE of the following statements about therapy for inhibition of mammalian target of rapamycin is most CORRECT?","1. It is associated with a lower risk of BK virus allograft nephropathy after kidney transplantation compared with calcineurin inhibitor therapy 2. It is associated with a lower risk of CMV infection after kidney transplantation com-pared with calcineurin inhibitor therapy 3. It is associated with improved graft survival after transplantation compared with tacrolimus therapy 4. It is associated with a lower risk of acute rejection compared with tacrolimus-based immunotherapy 5. It has no impact on wound healing",B,temp nan,Transplantation 5832," A 29-year-old woman with type 1 diabetes mellitus since age 2 years and stage G5 CKD is referred for transplantation. In addition to mild uremic symptoms, she has longstanding brittle diabetes mellitus with hypoglycemic unawareness. She asks about pancreas transplantation. Her brother has volunteered to be her living donor..",Which ONE of the following should you tell her about pancreas transplantation?,"1. Living donor kidney transplantation followed by a pancreas after kidney transplantation will likely provide the best long-term renal outcomes 2. Pancreas transplantation alone is associated with inferior outcomes compared with simultaneous kidney pancreas transplantation 3. Pancreas transplantation is associated with a lower risk of infection compared with kidney transplantation alone 4. Her risk of BK viremia will be lower if she receives a pancreas in addition to a kidney transplant",A,temp nan,Transplantation 5833," A 48-year-old man with ESRD of unknown cause is referred for transplantation. He was found to have blood and protein in his urine when he underwent a medical evaluation for the Navy and was consequently denied entry at age 18. For the past 20 years, he has been intermittently treated for hypertension, although his adherence to medical care has been sporadic. He is now highly motivated to receive a transplant. He initiated hemodialysis about 5 months ago after acutely presenting with uremia. His evaluation at that time showed that he had atrophic kidneys and a urine protein-to-creatinine of 3200 mg/g..",Which ONE of the following statements about this patient’s risk for recurrent glomerular disease after transplantation is most CORRECT?,"1. Kidney biopsy should be performed now to help assess his risk for recurrent glomerular disease 2. He should be told that approximately 15% of transplants fail because of recurrent glomerular disease 3. His risk of recurrent glomerular disease will be lower if he receives a living related compared with a deceased donor transplant 4. His allograft prognosis will be worse if he receives a living donor versus a deceased donor transplant because of the risk of recurrent glomerular disease",B,temp nan,Transplantation 5684," You receive a request to evaluate a 22-year-old college student who was found to have a BP of 158/99 mmHg at the dentist’s office. Because of her final examination schedule, she cannot come to your office to be seen for another month..",What is the next best step in the management of this patient?,"1. Initiate amlodipine 5 mg daily so that you can assess if she needs additional therapy when she presents for her visit 2. Recommend that she obtain kiosk-based BP measurements at her local supermarket until she is able to come into the clinic to be evaluated 3. Recommend that she purchase a validated home BP monitor and provide her with guidance on appropriate measurement technique 4. Initiate losartan-hydrochlorothiazide 25 to 12.5 mg daily so that you can assess if she needs additional therapy when she presents for her visit",C,temp nan,Hypertension 5685," You have been following a 68-year-old man for several years who has a history of stable proteinuric CKD Stage IV, hypertension, heart failure with preserved ejection fraction, and hypothyroidism. His BP has been at or below his goal in the office for the past 6 years on valsartan 320 mg daily, chlorthalidone 25 mg daily, and amlodipine 5 mg daily. He presents to clinic noting that he recently purchased a home BP monitor, which has been showing BPs consistently 10 to 15 mmHg above his goal..",Which of the following is the best response to this patient’s home readings?,"1. Ignore the readings, given that his BPs are consistently well controlled in the office 2. Suggest that he stop monitoring his BP at home because you suspect he is not able to use appropriate measurement technique 3. Add spironolactone for improved BP control because of the presence of masked uncontrolled hypertension 4. Recommend 24-hour ambulatory BP monitoring (ABPM) to confirm the presence of masked uncontrolled hypertension",D,temp nan,Hypertension 5686," A new patient presents for evaluation of asymptomatic labile BPs at home ranging from the 80s/30s mmHg to 190s/110s mmHg using his new cuffless watch, which was advertised by his favorite technology magazine as being the top new device on the market for monitoring BP..",Which of the following is the best approach to this patient’s BP measurement?,"1. Recommend that he undergo 24-hour ABPM 2. Encourage him to continue to monitor his BP at home using this device because BP monitors cannot be marketed unless they are accurate 3. Recommend that he undergo evaluation for autonomic dysfunction 4. Recommend that he obtain a kiosk-based BP measurement at his local supermarket until he is able to come into the clinic to be evaluated",A,temp nan,Hypertension 5687," A 44-year-old man is referred to you for a hypertension evaluation. Your office assistant has recorded two seated BPs averaging 142/91 mmHg..",Which of the following questions is most likely to be helpful when considering a secondary component in the BP elevation?,"1. How many steps a day do you typically walk? 2. How many meals do you eat at home in a typical week? 3. What medications or drugs, prescription or otherwise, are you using? 4. Have you recently traveled to a third-world country?",C,temp nan,Hypertension 5688," A 53-year-old untreated woman is referred from a local hospital health fair when she was noted to have a high BP and an elevated random glucose value. In your office today her BP averages 158/ 78 mmHg, and her body mass index (BMI) is 27.3 kg/m2..","She does not have a primary care doctor, and cannot recall a prior BP value because she has felt fine and “did not see the need to check it.” Which of the following items in your initial evaluation of her case is most likely to identify nonhypertensive-related cardiovascular risk factors important for you to know in selecting therapy for her BP elevation?","1. Low white blood cell count 2. A urine dipstick registering ++ protein 3. Electrocardiogram showing right bundle branch block 4. Elevated C-reactive protein",B,temp nan,Hypertension 5689," In evaluating a 61-year-old patient new to your practice, you observe that he has elevated BP readings averaging 163/94 mmHg in your office, and similar values at home. You learn that he had a heart attack with a percutaneous coronary intervention 8 months ago. Shortly after that, he lost his job and health care benefits and disappeared from care for a time. He has recently gotten a new job and is now insured again..","When you explain to him what you are thinking about his BP, which of the following statements would most accurately summarize his current situation?","1. “Now that you have had a heart attack, we can relax a bit because the damage is behind you, and we will spend the next months working on your weight and your physical activity level” 2. “Now that you have had a heart attack, we need to be cautious about not lowering your lower BP number (diastolic) too much because we don’t want to cause another heart attack” 3. “Now that you have had a heart attack, your body is showing us that it is vulnerable to things like an elevated BP, and we really need to work hard to get this under control soon” 4. “Now that you have had a heart attack, we believe that the upper BP number (systolic) should be 100 + your age in years, and your current upper number of 163 mmHg is about where it should be”",C,temp nan,Hypertension 5690," You are following a 73-year-old man with stage 4 CKD, diabetes, and hypertension. Office and home readings have always varied substantially from reading to reading. The patient states that his BP always goes up because of traffic on the way to clinic, and he is not sure his home BP cuff works correctly. You obtain ABPM, which shows marked variability, but most readings are within target range..",Which of the following is likely to be true regarding his BP variability based on current data?,"1. Because most of his ABPM readings are within range, his BP is considered well controlled 2. Although short-term BP variability as measured by ABPM is worrisome, long-term BP variability using office or home readings has not been shown to be associated with the risk of cardiovascular events or mortality 3. Although long-term BP variability using office or home readings is worrisome, short-term BP variability as measured by ABPM has not been shown to be associated with the risk of cardiovascular events or mortality 4. He is at increased risk for a major adverse cardiovascular event or death",D,temp nan,Hypertension 5691," A 48-year-old woman is referred for difficult-to-control hypertension. The following drugs have been tried and stopped because of side effects: amlodipine (edema), metoprolol (excessive fatigue), chlorthalidone (severe hypokalemia). Her BP is improved but still elevated at 152/96 mmHg on losartan 100 mg. Her family history is notable for her father, who has severe hypertension, diabetes, and CKD; her mother is healthy and normotensive. She has no history of pulmonary edema, and the results of physical examination are unremarkable. She is afraid to try spironolactone because she read about the risk of hyperkalemia..",What is the next best step?,"1. Reassure her and start spironolactone with careful monitoring of potassium 2. Start clonidine or hydralazine because she may tolerate them better 3. Retry a lower dose of chlorthalidone and monitor the potassium 4. Evaluate her for primary aldosteronism",D,temp nan,Hypertension 5692," A 77-year-old woman with a history of hypertension for the past 15 years comes to clinic for review. She is currently treated with amlodipine and losartan, and her clinic BP today is 132/85 mmHg. She has a family history of dementia, and had a previous magnetic resonance imaging of her brain to investigate a transient ischemic attack 3 years ago, which showed early white matter changes without focal infarction. She feels very well and has excellent functional status, without symptoms of cognitive impairment. However, given her family history, she asks if there is anything additional that can be done to reduce her risk of dementia..","Regarding the effect of tight BP control on the development of cognitive impairment, which of the following is correct?","1. Tighter BP control (target systolic BP < 120 mmHg) has been associated with reduced progression of white matter changes, without changes in overall brain volume 2. Intensive BP control was associated with reduced risk of mild cognitive impairment at 5 years follow-up 3. Tighter BP control was associated with higher rates of falls, as compared with standard control, in patients with underlying cerebrovascular disease 4. Intensive BP control was associated with reduced cerebral perfusion in those with small-vessel cerebrovascular disease",B,temp nan,Hypertension 5693," A 32-year-old woman is found to have a BP of 148/92 mmHg on screening BP at an insurance medical examination. She is overweight and sedentary, with a high-salt diet. You recommend lifestyle changes as the initial step. Four months later, she returns to clinic. She is now on a low-salt diet and has lost 15 pounds through diet and exercise. Her BP today is 142/88 mmHg. You recommend that she continue her lifestyle changes, but also start taking antihypertensive medication..","When choosing a first-line agent for treatment of hyper-tension, which of the following is correct?","1. Dihydropyridine and nondihydropyridine calcium channel blockers were similarly effective in reducing complications related to hypertension 2. Chlorthalidone and hydrochlorothiazide had similar effects on rates of myocardial infarction, heart failure, and stroke, and have similar adverse effect profiles 3. Treatment with thiazide diuretics has been associated with better BP control and reductions in cardiovascular outcomes, as compared with other first-line agents for management of hypertension 4. Compared with other first-line agents, treatment with angiotensin-converting enzyme (ACE) inhibitors led to the greatest reduction in rates of stroke and hospitalization for heart failure",C,temp nan,Hypertension 5694," A 54-year-old man is referred to your clinic for assessment of hypertension. He has been taking valsartan for 6 months, and his BP remains above goal at 144/92 mmHg. His 24-hour ABPM reveals similar ambulatory BP readings. You discuss with him the need to add another agent to control his BP..","The patient refuses and instead wishes to switch to a different medication because this one is “not working.” Which of the following is correct regarding combination therapy for the treatment of hypertension?","1. As compared with monotherapy, initiation of combination antihypertensive therapy is associated with reduced mortality at 3-year follow-up. 2. In African patients with uncontrolled hypertension, combination therapy with perindopril/hydrochlorothiazide led to im-proved BP control as compared with perindopril/amlodipine 3. Two-drug combination therapies are preferred to triple therapy because higher rates of adverse events are seen with triple therapy combinations 4. Observational data suggest that the majority of patients started on a single agent for treatment of hypertension end up on a combination agent by 3 years follow-up",A,temp nan,Hypertension 5695," A 72-year-old woman with hypertension comes to clinic for BP follow-up. She is currently treated with amlodipine and losartan, and her clinic BP today is 136/78 mmHg. Her serum creatinine is 0.8 (estimated GFR 74 mL/min per 1.73 m2), and spot urine is negative for microalbuminuria. She lives independently and feels very well, with excellent functional status. Based on the Systolic Blood Pressure Intervention Trial (SPRINT) and the 2017 American College of Cardiology / American Heart Association (ACC/AHA) high BP guideline, you recommend adding a third medication to lower her BP to < 130/80 mmHg, change her losartan to losartan/hydrochlorothiazide, and schedule follow-up in 2 weeks. At her follow-up visit, BP is 128/72 mmHg, and she feels well. Her potassium has declined from 4.6 to 3.8 mEq/L. However, her serum creatinine has increased to 1.1 mg/dL, with a decline in her estimated GFR by 30% to 50 mL/min per 1.73 m2..","Regarding the effect of tight BP control on kidney function, which of the following is correct?","1. Intensive BP control (target systolic BP < 120 mmHg) that leads to acute kidney injury in patients without prior CKD is likely to progress to ESRD 2. Intensive BP control in SPRINT was associated with a small but persistent decline in GFR compared with standard BP control 3. Intensive BP control was associated with higher levels of kidney injury biomarkers, indicating permanent kidney damage 4. Intensive BP control was associated with reduced renal endpoints of ESRD or 50% decline in GFR",B,temp nan,Hypertension 5696," A 60-year-old man is referred after two sets of BP measurements were noted to be elevated at a BP screening. He has smoked one pack per day for 40 years but is otherwise well. His BMI is 32 kg/m2, and BP 136/86 mmHg in both arms, but the results of his examination are otherwise negative. His total cholesterol is 220 mg/dL, with HDL 40 mg/dL and LDL 160 mg/dL. Electrolytes are normal, and the result of urinalysis is negative..","Regarding the initial approach to treatment recommendations, which of the following is correct?","1. He should be instructed on lifestyle modification alone with follow-up in 3 months 2. He should be started on lifestyle modification and medication with follow-up 3. He should be instructed on lifestyle modification with the decision on medication dependent on assessment of 10-year cardiovascular disease event risk 4. He should be started on a combination medication to bring him quickly to his target BP",C,temp nan,Hypertension 5697," A 53-year-old African American man is referred for treatment of uncontrolled hypertension. He has CKD stage 3 with estimated GFR 45 mL/min, a urine protein/creatinine ratio of 1 g/g, and a strong family history of hypertension and ESRD. His BP is 159/92 mmHg, with a heart rate of 55 beats/min. Medications include chlorthalidone 50 mg daily, losartan 100 mg daily, and amlodipine 10 mg daily..",What is the best next action to adjust his regimen to control his BP?,"1. Chlorthalidone should be discontinued because it is less effective at this GFR 2. An ACE inhibitor should be added for additional anti-proteinuric effect 3. A β-blocker is the most appropriate next agent 4. Spironolactone is the most appropriate next agent",D,temp nan,Hypertension 5698," A 59-year-old man is referred to you for severe hypertension and multiple drug intolerances. He is taking hydralazine 25 mg four times daily, furosemide 40 mg twice daily, lisinopril 40 mg daily, and labetalol 200 mg three times daily. In clinic, his BP is 164/ 92 mmHg. By his report, he often forgets doses of medication during the day..",Which of the following is true regarding suboptimal adherence to antihypertensive medication?,"1. Around 25% of patients show nonadherence or suboptimal adherence to antihypertensive medications 2. Using electronic health record medication management tools plus nurse-led patient education improves medication adherence 3. Smartphone apps that provide reminder alerts, adherence reports, and peer support may lead to improved BP control 4. Smartphone apps that provide reminder alerts, adherence reports, and peer support may improve medication adherence",D,temp nan,Hypertension 5699," You are asked to give a public health presentation on the global burden of hypertension at an international conference..",Which of the following statements is correct regarding recent trends in deaths and disability-adjusted life years (DALYs) or years of ill health globally due to hypertension?,"1. Number of hypertension-related deaths increased by about 50% between years 2007 and 2017 2. Number of hypertension-related DALYs reduced by about 15% between years 2007 and 2017 3. Both overall number and age-standardized rates of deaths and disability related to hypertension continued to rise between years 2007 and 2017 4. Number of deaths and disability related to hypertension continued to rise while age-standardized rates declined between years 2007 and 2017",D,temp nan,Hypertension 5700," Based on the recent guidelines for hypertension diagnosis, a colleague has observed an almost doubling of younger patients (< 40 years old) diagnosed with hypertension. He is concerned whether or not these patients are actually at risk of complications from hypertension..",Which of the following should you tell your colleague about risk of cardiovascular disease in young newly diagnosed hypertensive patients?,"1. Patients with BP of ≥ 130/80 mmHg before the age of 40 years are at increased risk of cardiovascular disease 2. Patients with BP of ≥ 130/80 mmHg before the age of 40 years are not at increased risk of cardiovascular disease 3. Only men with BP of ≥ 130/80 mmHg before the age of 40 years are at increased risk of cardiovascular disease 4. Only women with BP of ≥ 130/80 mmHg before the age of 40 years are at increased risk of cardiovascular disease",A,temp nan,Hypertension 5701," The 2017 ACC/AHA Blood Pressure Guideline revised the definition of hypertension and are expected to influence the prevalence of hypertension in the community. As part of a planning process to allocate resources to manage hypertension within your hospital, a colleague in hospital leadership asks you to explain the impact of these guidelines..",Which of the following best reflects the current estimates of the impact of the 2017 ACC/AHA Blood Pressure Guideline?,"1. The number of people with hypertension will increase by only < 2% of the adult US population 2. Hypertension prevalence will increase by almost fivefold in older people (> 65 years) 3. Hypertension prevalence will increase by about 30%, and every newly diagnosed patient will receive recommendation for antihypertensive medication 4. Recommendation for antihypertensive medication will in-crease by only < 2% of the adult US population",D,temp nan,Hypertension 5702," A 52-year-old man with hypertension and diabetes is seen during a routine clinic visit. His office BP is 149/97 mmHg, and he is on antihypertensive medications. He also reports a history of severe periodontitis. You advise him to escalate antihypertensive drug therapy..","Based on recent studies on BP and periodontitis, which of the following is correct?","1. Antihypertensives may be less effective in the presence of periodontitis 2. Severe periodontitis is associated with a doubling of incidence of hypertension 3. Periodontitis does not predict BP control 4. Treating the patient’s periodontitis will not alter his BP control",A,temp nan,Hypertension 5703," A 71-year-old white woman presents with office BP of 85/57 mmHg. She reports having had high BP in her 40s and 50s. She recently read a report in the newspaper about people with problematic BP developing dementia..","Based on a recent study on chronicity of high BP and dementia, which of the following should you tell her regarding risk of dementia in patients of her age?","1. Low BP in patients her age is not linked to dementia 2. Low BP in patients her age is linked to dementia if patients have a history of high BP 3. Low BP in patients her age is linked to dementia only if patients are African Americans and have a history of high BP 4. Low BP in patients her age is linked to dementia only if patients are genetically susceptible to dementia and have a history of high BP",B,temp nan,Hypertension 5704," A 48-year-old man with suspected masked hypertension recently completed 24-hour ABPM. His 24-hour ambulatory BP was 145/ 98 mmHg. He does not smoke, leads a fairly active life, and does not have a family history of hypertension. He works for an insurance company and works more than 55 hours per week. He wants to know if he should be concerned about his long work hours..",Which of the following should you tell him?,"1. Assure him that long working hours have no relationship with hypertension 2. Assure him that long working hours are associated with hypertension only if someone has a family history of hypertension 3. Assure him that long working hours are associated with hypertension only if someone leads a sedentary lifestyle 4. Assure him that long working hours are associated with both masked hypertension and sustained hypertension",D,temp nan,Hypertension 5705," At an international conference on hypertension and cardiovascular disease, you hear a speaker discuss the prevalence of hypertension and its complications in a developing country. She also describes the patterns observed across differing levels of income and education..",Which of the following statements is true regarding the association between socioeconomic status and hypertension?,"1. Across all economic regions, hypertension is more common among individuals of lower socioeconomic status 2. Hypertension generally correlates with the prevalence of obesity and physical inactivity 3. Individuals of lower socioeconomic class and education generally have lower risk of both hypertension and cardiovascular disease 4. Control of hypertension is higher among those of lower socioeconomic class",B,temp nan,Hypertension 5706," A 47-year-old patient comes for his annual physical with his primary care physician, and his BP is 140/92 mmHg. He is a nonsmoker, his BMI is 24 kg/m2, and he has no family history of early-onset hypertension or cardiovascular disease. He works in advertising and reports that his job requires long hours and frequent travel. He reports he has “no time to exercise” and relaxes by having one or two alcoholic drinks each evening. You begin to discuss lifestyle changes with him as an initial step to address his hyper-tension. He asks you if there are any reliable data linking alcohol intake and “working too hard” to hypertension..",Which of the following is true?,"1. Definitive studies have shown that light or moderate alcohol intake lowers the risk of hypertension 2. Observational data suggest that working > 48 hours weekly is associated with increased risk of hypertension compared with working < 35 hours weekly 3. Observational data suggest that working > 48 hours weekly is associated with increased risk of masked hypertension but not sustained hypertension 4. A study of genetic variants that predict alcohol intake suggests that alcohol intake is negatively associated with hypertension risk",B,temp nan,Hypertension 5707," A 19-year-old woman presents with sudden-onset anxiety and complains her heart is “fluttering.” Physical examination reveals tachycardia and BP of 157/93. Urinary analysis reveals increased metanephrine excretion, and a retroperitoneal mass is identified with subsequent CT imaging..","Assuming there is a reduced fractional excretion of sodium, which of the following is directly contributing to the sodium retention occurring in this patient?","1. Renal tubular α-adrenergic receptors 2. Juxtaglomerular α -adrenergic receptors 3. Preganglionic β-adrenergic receptors 4. Vascular α -adrenergic receptors 5. Cardiac β -adrenergic receptors",A,temp nan,Hypertension 5708," A novel antihypertensive therapy is developed using a viral vector engineered to target the juxtaglomerular apparatus. Preliminary studies in healthy subjects found a significant reduction in plasma renin activity, reduced circulating (pro)renin levels, reduced urinary excretion of prostaglandin E-2 metabolites, and a modest (8 mmHg) reduction in BP. Plasma osmolality and electrolytes were unaltered..",Which of the following is the most likely mechanism by which this novel therapy is eliciting an antihypertensive effect?,"1. Decreased expression of sodium-potassium-chloride cotransporter 2. Decreased expression of the (pro)renin receptor 3. Increased expression of ACE 4. Increased expression of angiotensin II receptor type 1",B,temp nan,Hypertension 5709," A 46-year-old man presents to his primary care provider for an annual visit. His BMI has gradually increased over the past 5 years from 33.4 to 37.6, and his BP increased from 124/83 to 138/85 mmHg. Other aspects of his physical examination and history are unremarkable..",Which of the following best describes the components of the AHA’s DASH diet that would be beneficial in combating the gradual increase of BP in this patient?,"1. Increased sodium delivery to the collecting duct epithelial sodium channel to stimulate potassium excretion 2. Decreased concentration of bicarbonate in the tubular filtrate 3. Decreased pH in the tubular lumen increasing sodium-hydroxy exchanger activity 4. Increased activation of sodium transporters throughout the nephron",A,temp nan,Hypertension 5710," A 53-year-old man presents to his primary care provider for an annual visit. His BP values have gradually increased from 124/83 to 148/85 mmHg over the past 8 years. Other aspects of his physical examination and history are unremarkable..","Based on recent mechanistic studies, which of the following is most likely to be true regarding circulating extracellular vesicles from this patient?","1. Extracellular vesicles (EVs) do not influence vascular tone 2. EVs from this patient will cause direct vasoconstriction as compared with EVs from a normotensive individual 3. EVs from normotensive patients cause direct vasodilation as compared with EVs from this patient 4. EVs from this patient will reduce acetylcholine-induced vasodilation as compared with EVs from a normotensive individual 5. Prior destruction of EVs from this patient will reduce acetylcholine-induced vasodilation",E,temp nan,Hypertension 5711," A 28-year-old man presents to his primary care provider because of fatigue, his spouse’s concerns with snoring, and BP of 136/85 mmHg. A sleep study is performed, and obstructive sleep apnea is confirmed. The patient’s BP is reduced to 125/83 mmHg during 3-month follow-up with appropriate use of continuous positive airway pressure (CPAP)..",Which of the following best describes the physiologic mechanism resulting in reduced BP in this patient after regular use of CPAP?,"1. Increased circulating norepinephrine concentration 2. Increased plasma angiotensin II concentrations 3. Increased urinary aldosterone concentration 4. Decreased activity of renal nerves 5. Decreased urinary sodium excretion",D,temp nan,Hypertension 5712," You are evaluating a 39-year-old man with resistant hyper-tension and severe obstructive sleep apnea (OSA). He is taking four medications including chlorthalidone, amlodipine, losartan, and spironolactone. His BP remains at 149/89 mmHg. He has marked difficulty wearing a CPAP mask. He has seen an article online and asks if renal denervation might help his hypertension or sleep apnea..","Which of the following is correct regarding renal denervation, hypertension, and sleep apnea, based on current data?","1. Because hypertension is the result of OSA, there may be a benefit of renal denervation on hypertension, but there is no reason to suggest that renal denervation would affect OSA itself 2. The studies on renal denervation excluded patients with OSA, so a benefit of denervation on OSA patients cannot be predicted 3. Renal denervation may decrease hypertension and the se-verity of OSA 4. Improved compliance with CPAP would have a greater effect on his BP than renal denervation",C,temp nan,Hypertension 5713," You are giving a talk on diet and hypertension at a local chapter of the National Kidney Foundation. A member of the audience says, “I read on the internet that all kidney patients should eat as little potassium as possible..","Does potassium have any effect on BP?” Which of the following is likely to be true regarding potassium intake and BP?","1. No correlations have been observed between potassium in-take and CKD progression 2. Hypertension correlates better with the dietary sodium/potassium ratio than with sodium intake alone 3. Potassium restriction increases natriuresis in animal studies, especially in CKD 4. Potassium restriction increases sodium delivery to the distal tubule",B,temp nan,Hypertension 5714," A 36-year-old woman presents with a 2-week history of increasing lower limb edema and weight gain. On examination, her BP is 126/88 mmHg, her weight is 67 kg (from usual weight of 58 kg), and she has periorbital, upper limb, and lower limb edema. Spot urine protein/creatinine ratio is 12.4 g/g. Her creatinine is 0.8 mg/dL, and albumin 2.2 g/dL. She undergoes kidney biopsy, and the specimen shows a normal appearance on light microscopy but diffuse foot process effacement on electron microscopy..","Regarding the management of minimal change disease, which of the following is correct?","1. When compared with corticosteroids as first-line therapy, treatment with tacrolimus is associated with similar rates of remission and relapse 2. Alternate-day corticosteroid therapy is associated with reduced rates of remission compared with daily corticosteroid dosing 3. Rituximab has been shown to be beneficial in children who fail to respond to steroid or calcineurin inhibitor therapy 4. When compared with a tacrolimus-based treatment protocol, treatment with rituximab was associated with a greater cumulative steroid dose",A,temp nan,Primary and Secondary Glomerular Diseases 5715," Focal and segmental glomerulosclerosis (FSGS) is a histologic pattern of injury that is due to several different pathophysiologic processes. Renal biopsy is required for diagnosis of FSGS, and particular features on the histologic and ultra-structural examinations may differentiate between the causes and predict prognosis..","With regard to the pathophysiology of FSGS, which of the following is correct?","1. The variant of FSGS lesion (as described in the Columbia classification) seen on light microscopy is helpful in differentiating between primary and genetic forms of FSGS 2. Abnormalities in genes encoding glomerular basement membrane (GBM) collagen are associated with primary FSGS 3. In secondary FSGS, glomerular hypertension causes podocyte injury, leading to podocyte detachment and loss 4. The degree of foot process effacement seen on electron microscopy is used to differentiate between primary and genetic FSGS",C,temp nan,Primary and Secondary Glomerular Diseases 5716," A 48-year-old man is referred to you for a second opinion. He received a diagnosis of FSGS 6 months earlier, having presented with nephrotic syndrome. His renal biopsy specimen shows FSGS, with 15% interstitial fibrosis and tubular atrophy, and electron microscopy showed diffuse foot process effacement. He was initially treated with prednisone 1 mg/kg, but tacrolimus was added as a steroid-sparing agent after 2 months because of new-onset diabetes mellitus. His proteinuria has improved from 12 g/24 hours at the time of diagnosis to 6 g/24 hours currently. His serum creatinine is 1.8 mg/dL. He wishes to discuss his treatment options with you..","With regard to the treatment of FSGS, which of the following is correct?","1. Randomized controlled trials have demonstrated the efficacy of rituximab in the treatment of FSGS 2. Up to 60% of patients will experience a relapse after initial treatment of FSGS with corticosteroids 3. In retrospective studies, up to 30% of adults with steroid-resistant FSGS had a potentially causative genetic mutation identified 4. Recent studies have shown benefit for sodium-glucose transporter-2 (SGLT2) inhibitors in management of secondary FSGS",C,temp nan,Primary and Secondary Glomerular Diseases 5717," A 76-year-old man presents with recent-onset nephrotic syn-drome. He has 5 g of proteinuria/24 hours, his albumin is 2.8 g/dL, and creatinine is 1.5 mg/dL. He underwent renal biopsy, which was consistent with membranous nephropathy (MN), and showed negative staining for phospholipase A2 receptor (PLA2R) in the glomerular capillary walls. Anti-PLA2R serology is also negative. He undergoes a thorough evaluation for underlying malignancy, which gives negative results. He wishes to be evaluated for other causes of MN..",Which of the following is correct with regard to PLA2R-negative MN?,"1. Staining for exostosin 1/exostosin 2 (EXT1/EXT2) on renal biopsy specimens is associated with underlying malignancy 2. Neural epidermal growth factor like 1 protein (NELL-1)–associated MN is a rare cause of primary MN 3. Thrombospondin type 1 domain containing 7A (THSD7A)–associated MN is associated with high rates of underlying malignancy 4. Semaphorin 3B is a common cause of PLA2R-negative MN in the elderly",C,temp nan,Primary and Secondary Glomerular Diseases 5718," A 56-year-old woman is referred from her primary care provider for assessment. She recently experienced weight gain and ankle edema. She was found to have 3+ proteinuria and increased creatinine to 2.4 mg/dL, from a prior baseline of 0.8 mg/dL. Her 24-hour urine collection shows 10 g proteinuria, and her PLA2R antibody is positive at 150 RU/mL..","When considering a treatment plan for this patient, which of the following is correct?","1. Renal biopsy could be deferred in this patient 2. Monitor serologies monthly and treat if PLA2R fails to decrease by > 25% in 3 months 3. In the MENTOR study, patients treated with rituximab and cyclosporine had similar estimated GFR (eGFR) at 24-month follow-up 4. In the MENTOR study, those treated with rituximab had higher rates of remission at 24-month-follow up",D,temp nan,Primary and Secondary Glomerular Diseases 5719," A 28-year-old patient is found to have dipstick hematuria during a routine physical. He is referred to nephrology for review, and his evaluation reveals BP of 148/92 mmHg, urinalysis shows 2+ heme and 1+ protein. Creatinine is 1.4 mg/dL, and his urine protein/creatinine ratio is 1680 mg/g. He undergoes renal biopsy, which shows a membranoproliferative pattern of injury, with evidence of diffuse proliferative glomerulonephritis. On immunofluorescence, C3 staining is dominant, and electron microscopy shows electron-dense deposits in the subepithelial and subendothelial space..",Which of the following is correct regarding this patients’ glomerular disease?,"1. On immunofluorescence, costaining with IgG and C3 is seen in dense deposit disease 2. C3 glomerulonephritis commonly presents with crescentic glomerulonephritis 3. Abnormalities of genes encoding the complement system are the most common cause of C3 glomerulopathy 4. C3 nephritic factors are seen in many patients with monoclonal gammopathy–associated C3 glomerulopathy",D,temp nan,Primary and Secondary Glomerular Diseases 5720," A 48-year-old woman comes to you for a second opinion about management of her recently diagnosed dense deposit disease. She has been prescribed lisinopril for proteinuria, but after 6 months of follow-up, she continues to have 2.5 g proteinuria/24 hours, and her creatinine has increased from 1.2 mg/dL to 1.5 mg/dL. She also has a history of prediabetes and is very reluctant to take steroids unless there is a strong indication to do so..","Regarding treatment of C3 glomerulopathy (C3G), which of the following is correct?","1. Complement blockade with eculizumab has shown benefit in the majority of patients with C3G 2. Treatment with mycophenolate mofetil and steroids has been associated with improved rates of remission of C3G in observational studies 3. In patients with monoclonal gammopathy–associated C3 glomerulopathy, treatment with immunosuppression shows similar outcomes to clone-based therapy 4. Use of angiotensin-converting enzyme inhibitors may be associated with reduced levels of complement activation in C3G by blocking cleavage of C3",B,temp nan,Primary and Secondary Glomerular Diseases 5721," A 38-year-old woman is referred for listing for renal trans-plantation. She experienced severe pre-eclampsia during her first pregnancy 4 years ago, with AKI that required dialysis at the time of delivery. She was left with CKD stage IV, and despite control of her hypertension, she has progressive CKD with eGFR 20 mL/min per 1.73 m2. She has never had a renal biopsy or specific treatment for her kidney disease..",Which of the following is correct regarding complement-mediated thrombotic microangiopathy (TMA)?,"1. Renal cortical necrosis can be an underappreciated presentation of pregnancy-associated TMA 2. Pregnancy-associated TMA is rarely associated with abnormalities in genes encoding complement or complement regulatory proteins 3. All subtypes of complement-mediated TMA have high recurrence rates after transplantation 4. Eculizumab can be safely stopped in transplant recipients who have stable renal function at 6 months after transplantation",A,temp nan,Primary and Secondary Glomerular Diseases 5722," A 47-year-old man recently underwent renal biopsy for workup of microscopic hematuria and proteinuria, with urinary protein excretion of 1.5 g daily. His biopsy specimen showed mesangial and endocapillary proliferation, with 20% tubular atrophy/interstitial fibrosis, and IgA deposition on immunofluorescence. He wishes to discuss the risks and benefits of immunosuppression for treatment of his IgA nephropathy..",Which of the following is correct regarding treatment of IgA nephropathy?,"1. In a randomized controlled trial, immunosuppression comprising cyclophosphamide and steroids, followed by azathioprine and steroids, delayed progression of CKD 2. Treatment with budesonide led to reduced proteinuria and improved eGFR in patients with patients with IgA nephropathy 3. Treatment with steroids for IgA nephropathy was well tolerated and associated with lower rates of progression of CKD 4. IgA nephropathy commonly recurs after renal transplantation",D,temp nan,Primary and Secondary Glomerular Diseases 5723," A 63-year-old man presents with a 6-week history of systemic illness including malaise, weight loss, low-grade fever, and upper respiratory symptoms. He is found to have AKI with creatinine 4.5 mg/dL and new-onset hematuria and proteinuria. ANCA is positive, with elevated anti-proteinase 3 (PR3) titer. Renal biopsy shows pauci-immune crescentic glomerulonephritis, with 20%glomerulosclerosis. He has been prescribed pulse corticosteroids and wishes to discuss his treatment options for additional therapy..",Which of the following statements is correct regarding in-duction therapy for AAV?,"1. The MYCYC trial showed that induction therapy with mycophenolate and steroids was inferior to cyclophosphamide and steroids for remission induction at 6-month follow-up 2. The PEXIVAS trial showed that in cases of moderate to severe renal impairment, plasma exchange was associated with reduced rates of ESKD 3. In the PEXIVAS trial, reduced corticosteroid dosing was associated with no difference in risk of ESKD, but with reduced rates of infectious complications 4. The addition of a small molecule C5a inhibitor, avacopan, to induction therapy was associated with higher rates of infectious complications as compared with standard treatment with corticosteroids and rituximab/cyclophosphamide",C,temp nan,Primary and Secondary Glomerular Diseases 5724," A 37-year-old woman received a diagnosis of AAV 6 months ago and was treated with rituximab and prednisone for induction therapy. Her ANCA titer became negative within 3 months of initiating therapy, and her peripheral B cell count was zero at most recent check. She has been on tapering dose steroids but remains on prednisone 5 mg daily. She has been reading about the risks of immunosuppression and wants to discuss the benefits of aza-thioprine versus rituximab..","Regarding remission maintenance in AAV, which of the following is correct?","1. In a clinical trial comparing it with maintenance therapy with azathioprine, rituximab was superior at maintaining remission of AAV 2. A ritixumab dosing strategy based on repopulation of peripheral B cells and/or changes in ANCA titer led to fewer relapses and longer relapse-free survival than regular scheduled 6-monthly dosing of rituximab 3. In a study examining the duration of treatment with oral immunosuppressants to maintain remission, there were no differences in rates of ESKD associated with the use of azathioprine and prednisone for 24 versus 48 months 4. Recent observational studies suggest that elderly patients tolerate corticosteroids well, but the use of cyclophosphamide for induction therapy is associated with higher rates of infectious complications",A,temp nan,Primary and Secondary Glomerular Diseases 5725," You have been asked to give a presentation to your colleagues in primary care on lupus nephritis, with emphasis on the presentation of lupus nephritis, interpretation of the serologic and histologic evaluation, and recommendations for follow-up testing..","In the evaluation of lupus nephritis, which of the following is correct?","1. Observational studies suggest that lupus nephritis is being diagnosed at a younger age 2. The time from diagnosis of SLE to the development of lupus nephritis is shorter in African Americans and other minority groups 3. In patients with proliferative lupus nephritis who undergo repeated biopsy, histologic findings correlate closely with clinical response to treatment 4. Anti-PLA2R antibodies are occasionally identified in patients with class V lupus nephritis",B,temp nan,Primary and Secondary Glomerular Diseases 5726," A 28-year-old woman presents to her primary care provider with joint pain, fatigue, and a malar rash. Her laboratory evaluation shows positive ANA and anti–double stranded DNA antibodies; urinalysis shows 1+ blood and 2+ protein; serum creatinine is 0.8 mg/dL. Her biopsy specimen reveals class IV lupus nephritis, and you plan to start immunosuppressive therapy. You recommend starting mycophenolate mofetil and steroids, but she asks about alternative options and about recent developments in treatment..",Which of the following is correct regarding the treatment of lupus nephritis?,"1. The LUNAR trial showed improved outcomes from the addition of rituximab to mycophenolate mofetil and steroids 2. Post hoc analyses suggest that patients who achieve full depletion of peripheral B cells may gain greater benefit from rituximab 3. Calcineurin inhibitors have shown little benefit in the treatment of lupus nephritis 4. The addition of voclosporin to treatment with mycophenolate and steroids was associated with rates of remission, and adverse events that were similar to those of mycophenolate and steroids alone",B,temp nan,Primary and Secondary Glomerular Diseases 5727," A 55-year-old man is referred for evaluation of CKD, proteinuria, and new hematuria. He is HIV positive and has been taking antiretroviral therapy, including tenofovir disoproxil fumarate, for > 20 years with undetectable viral load. He received a diagnosis of type 2 diabetes and mild hypertension 10 years ago. He has 10 – 20 RBCs and 5 – 10 WBCs per high powered field on urine microscopic examination, 2 grams of protein on 24-hour collection, and creatinine 2.2 mg/dL (eGFR 40 mL/min per 1.73 m2)..",Which of the following statements is correct?,"1. His antiviral medications should be left unchanged because they have been effective in controlling HIV for over 20 years 2. Kidney biopsy will most likely show HIV-associated nephropathy 3. A stone should be excluded because tenofovir disoproxil fumarate commonly causes nephrolithiasis 4. He is at risk for immune complex glomerulonephritis that may or may not be related to his HIV",D,temp nan,Primary and Secondary Glomerular Diseases 5728," A 64-year-old woman who is new to the medical system recently received a diagnosis of hepatitis C virus (HCV) infection. She was referred to you because CKD screening performed as part of her HCV evaluation demonstrated creatinine 1.9 mg/dL, 2+ proteinuria, and 2+ hematuria. She feels well and has virtually no symptoms. She has not yet been prescribed any therapy..",Which of the following statements is accurate?,"1. A direct-acting antiviral (DAA) regimen, such as sofosbuvir and velpatasvir, and rituximab should be started concurrently because she has an active glomerulonephritis 2. She should be treated with rituximab first, followed by any DAA 3. She should be treated with a DAA but not sofosbuvir because her eGFR is < 30 mL/min per 1.73 m2 4. She should be treated with a pangenotypic DAA irrespective of her eGFR, and rituximab should not be used, at least initially",D,temp nan,Primary and Secondary Glomerular Diseases 5729," You are consulted for 3+ proteinuria and AKI in a 39-year-old man who is admitted for severe unilateral flank pain. He is an active user of both IV cocaine and heroin, and he is known to be hepatitis B virus positive but has refused treatment in the past. On examination he is very uncomfortable but has no point tenderness over either flank. Urinalysis shows 2+ hematuria; leukocyte esterase is negative, with sediment showing nondysmorphic RBCs and few WBCs. Serum creatinine is 1.2 mg/dL. A 24-hour urine collection demonstrated 12 grams protein. Abdominal imaging shows normal kidneys, no obstruction, and no stone..",What is the highest priority “next intervention”?,"1. Perform a kidney biopsy to diagnose a glomerular cause 2. Treat for presumed urinary tract infection 3. Arrange for CT venography to exclude renal vein thrombosis 4. Treat with lamuvidine for presumed HBV-associated MN",C,temp nan,Primary and Secondary Glomerular Diseases 5730," Your patient is a 49-year-old HIV+ man with a 10-year history of hypertension, diabetes, and stage 4 CKD. He asks you about the risks and benefits of transplantation..",Which of the following statements is true regarding transplantation?,"1. He would do better with transplantation than with dialysis, but time on the waiting list will be much higher than if he were HIV negative 2. He would do better with dialysis because the immuno-suppressive medications required for transplantation would probably activate HIV replication 3. He should do fine with transplantation because the risks of transplantation are identical to those of HIV-negative patients, including the risk of rejection 4. He would do better with transplantation than with dialysis and should be referred for evaluation",D,temp nan,Primary and Secondary Glomerular Diseases 5731," A 55-year old woman is referred for evaluation of creatinine of 1.8 mg/dL with negative urinalysis results. By her report she has been in very good health except for “borderline diabetes” treated with metformin and diet. Creatinine was noted to be elevated in the setting of a viral illness before she underwent a chest CT-scan. Prior creatinine was approximately 1 year ago and was within normal range. She has no history of retinopathy. She has her urine checked at least twice yearly, and she thinks the results were always normal. Biopsy demonstrates GBM thickening, severe mesangial expansion, and nodular sclerosis in addition to mild acute tubular necrosis (ATN). The creatinine is down to 1.4 mg/dL by the time the biopsy is performed..",Which of the following statements is likely to be true regarding the case presentation?,"1. The patient is an inaccurate historian, and the diabetes was likely more severe and with accompanying retinopathy 2. Her primary care physician is not screening adequately for microalbuminuria, which always accompanies histologic changes of diabetic nephropathy 3. The biopsy was mislabeled and should be repeated 4. In addition to mild ATN from the viral syndrome, she has diabetic nephropathy with histologic changes that preceded clinical evidence such as eGFR decline or albuminuria",D,temp nan,Primary and Secondary Glomerular Diseases 5732," A 23-year-old man who is studying for his master’s degree in microbiology self-refers for evaluation of kidney function be-cause of a recent diagnosis of type 1 diabetes. He asks you to explain glomerular hyperfiltration. Which of the following statements is FALSE regarding hyperfiltration?.",Select one:,"1. Glomerular hyperfiltration occurs within 1 to 5 years in most individuals with type 1 diabetes and is associated with structural lesions that predict a decline in GFR 2. Glomerular hyperfiltration predicts cardiovascular mortality in type 2 diabetes 3. The sodium glucose cotransporters increase sodium delivery to the macula densa, which decreases GFR 4. Hyperglycemia and hyperaminoacidemia are initiating factors in altering glomerular hemodynamics",C,temp nan,Primary and Secondary Glomerular Diseases 5733," A 48-year-old man is referred for management of diabetic nephropathy. He has hypertension, hyperlipidemia, and atherosclerotic cardiovascular disease. His HbA1C is 8.5%, microalbumin/creatinine ratio is 480 mg/g, and eGFR 48 mL/min per 1.73 m2. He is taking metformin, lisinopril, hydrochlorothiazide, and atorvastatin..",Which of the following is NOT associated with SGLT2 inhibitors?,"1. Increased risk of genital fungal infections 2. Decreased risk of progression to ESKD 3. Increased risk of diabetic ketoacidosis 4. Increased cardiovascular mortality risk",D,temp nan,Primary and Secondary Glomerular Diseases 5734," You are following up a 50-year-old woman with longstanding hypertension, stage 3 CKD, and a recent diagnosis of diabetes. She is taking metformin, and her HbA1C is slightly above target. Her eGFR is 40 mL/min per 1.73 m2..",Which of the following treatment options is most correct?,"1. Stop the metformin and start an SGLT2 inhibitor 2. Continue the metformin and start an SGLT2 inhibitor 3. Stop the metformin and start insulin and an SGLT2 inhibitor 4. Continue the metformin and start glucagon-like peptide-1 receptor agonist",B,temp nan,Primary and Secondary Glomerular Diseases 5735," A 71-year-old man with a history of monoclonal gammopathy of undetermined significance is referred for nephrology evaluation. His last bone marrow biopsy was 3 years prior and showed 10% kappa-restricted plasma cells. He has had slowly progressive CKD, with a recent serum creatinine of 1.8 mg/dL, which was 1.4 mg/dL 5 years before his current evaluation. He is also noted to have urine study results showing glycosuria with normoglycemia and a serum bicarbonate of 17 meq/L..","Based on the clinical presentation, what would be the most likely histologic diagnosis if the patient underwent kidney biopsy?","1. Light chain proximal tubulopathy 2. Light chain deposition disease 3. AL (light chain) amyloidosis 4. Light chain cast nephropathy",A,temp nan,Primary and Secondary Glomerular Diseases 5736," A 59-year-old man with no known history of kidney disease experiences AKI, with a serum creatinine of 2.7 mg/dL and eGFR of 24 mL/min per 1.73m2. He has a urine protein/creatinine ratio of 2.2 g/g. He has no hematuria. A serologic workup is performed and includes a negative antibody testing for HIV, hepatitis C, hepatitis B, and serum anti-PLA2R. Serum protein electrophoresis, serum immunofixation, serum kappa-lambda free light chain assay, urine protein electrophoresis. and urine immunofixation tests are pending. He undergoes kidney biopsy, which shows membranoproliferative glomerulonephritis on light microscopy and immunofluorescence microscopy showing staining for IgG (3+), kappa (2+), lambda (2+). and C3 (3+). Electron microscopy is pending, but an immunohistochemical stain for DnaJ homolog subfamily B member 9 (DNAJB9) is positive..",Which of the following statements is true about DNAJB9-positive fibrillary glomerulonephritis?,"1. The majority of patients with DNAJB9-positive fibrillary glomerulonephritis have an underlying lymphoproliferative disorder 2. All kidney biopsy specimens should undergo pronase digestion of paraffin-embedded tissue in order to evaluate for the presence of “masked” immunoglobulin deposits 3. Congo red staining is required to rule out AL (light chain) amyloidosis 4. DNAJB9 positive fibrillary glomerulonephritis is not an monoclonal gammopathy of renal significance (MGRS)-associated lesion, and therefore no hematologic workup is recommended",D,temp nan,Primary and Secondary Glomerular Diseases 5737," A 73-year-old man with no known history of kidney disease presents to the hospital with abdominal fullness, a new rash, and decreased urine output. On examination, he is found to have nonblanching palpable purpura on his upper and lower extremities and new lower extremity edema. He also exhibits splenomegaly and has palpable bilateral cervical lymphadenopathy on examination. He reports occasional fevers and night sweats. His serum creatinine on admission labs is 3.4 mg/dL (eGFR 17 mL/min/1.73 per m2).His urinalysis results are significant for 3+ protein and 3+ blood, with 20 to 50 red blood cells per high powered field on urine microscopy, including the presence of dysmorphic red blood cells. Antinuclear antibody, anti-myeloperoxidase and anti–serine proteinase 3 antibody test results are negative, as are antibodies for HIV, hepatitis B, and hepatitis C. He undergoes kidney biopsy..",What is the most likely diagnosis?,"1. Proliferative glomerulonephritis with monoclonal immunoglobulin deposits (PGNMID-Ig) 2. Pauci-immune, necrotizing, and crescentic glomerulonephritis 3. Type I cryoglobulinemic glomerulonephritis 4. Type II cryoglobulinemic glomerulonephritis",C,temp nan,Primary and Secondary Glomerular Diseases 5738," A 67-year-old man with metastatic melanoma was recently prescribed treatment with nivolumab. Two months after initiation of therapy, he experienced an acute rise in serum creatinine from 1 mg/dL to 2.8 mg/dL. Renal biopsy revealed severe tubulointerstitial nephritis that was attributed to the immune checkpoint inhibitor..",What is the best management strategy?,"1. Continue nivolumab and initiate glucocorticoids 2. Hold nivolumab and monitor kidney function 3. Hold nivolumab and initiate glucocorticoids 4. Change nivolumab to ipilimumab 5. Immune checkpoint inhibitors should be permanently discontinued",C,temp nan,Primary and Secondary Glomerular Diseases 5739," A 64-year-old-woman is prescribed the tyrosine kinase inhibitor sunitinib for renal cell cancer. Three months after initiation of treatment, she is noted to be newly hypertensive with a BP of 160/92 mmHg. Urinalysis reveals 3+ proteinuria, and a sub-sequent 24-hour urine collection yielded 3.7 g protein. A urinalysis from 4 months prior was unremarkable. Her serum creatinine has increased from 0. 9 mg/dL at baseline to 1.3 mg/dL, and her serum albumin is 3.1 g/dL. She undergoes renal biopsy for diagnostic evaluation..",What is the most likely finding?,"1. Thrombotic microangiopathy 2. Collapsing FSGS 3. Membranous nephropathy 4. AA amyloidosis",B,temp nan,Primary and Secondary Glomerular Diseases 5740," A 76-year-old man receives a diagnosis of multiple myeloma and painful osteolytic bone lesions. His eGFR is 25 mL/min. He is prescribed pamidronate and subsequently experiences nephrotic-range proteinuria. The proteinuria improves after holding further pamidronate..",What is the next best step in the management of his bone disease?,"1. Initiate zoledronate 2. Initiate denosumab 3. Rechallenge patient with pamidronate 4. Antiresorptive therapy should be permanently discontinued",B,temp nan,Primary and Secondary Glomerular Diseases 5741," A 34-year-old woman presents to the emergency room with necrotic skin ulcers on her nose and ears, arthralgias, and myalgias. Initial laboratory results are notable for neutropenia and hematuria. A serologic evaluation reveals she has positive titers for both PR3-ANCA and MPO-ANCA..",Double-positive MPO and PR3-ANCA is associated with which of the following?,"1. Progression to end-stage renal disease 2. Development of pulmonary hemorrhage 3. Increased risk of mortality 4. Drug-induced AAV",D,temp nan,Primary and Secondary Glomerular Diseases 5742," A 35-year-old man with hypertension is referred for evaluation of decreased GFR. He has one healthy brother; his father has diabetes, hypercholesterolemia, and coronary artery disease; and his mother was treated for breast cancer. His maternal uncle experienced end-stage kidney disease at age 40. His maternal grandmother at age 76 years has a history of mild hearing loss and stage 3a CKD. There is no history of consanguinity. His only medication is hydrochlorothiazide. On examination his BP is 145/88 mmHg, his pulse rate is 70/min, and his body mass index is 29 kg/m2. There is no peripheral edema, and there are no actively inflamed joints. Laboratory studies show creatinine 2.0 mg/dL, BUN 55 mg/dL, sodium 140 meq/L, potassium of 5.0 meq/L, total CO2 21 meq/L, hemoglobin 126 g/L, leukocytes 7.8 e9/L, platelets 292 e9/L, hemoglobin A1C 6.5%. His ANA, HIV, ANCA and hepatitis serology, and complement levels are all normal or negative. Urinalysis has 2+ blood, negative protein, and negative nitrites. Urinary albumin/creatinine ratio was 160 mg/g. His creatinine 1 year ago was 1.8 mg/dL..",What is the MOST likely mode of inheritance of this condition?,"1. Autosomal dominant 2. Autosomal recessive 3. X-linked recessive 4. De novo somatic mutation",C,temp nan,Primary and Secondary Glomerular Diseases 5743," A 27-year-old man presents with persistent subnephrotic proteinuria without hematuria or reduced GFR. Marked podocyte deposition of globotriaosylceramide (GB3) is observed..",Which model best demonstrates a pathophysiologic pathway of Fabry disease?,"1. GLA mutation, α-galactosidase A deficiency, GB3 accumulation 2. GLA mutation, β-galactosidase A deficiency, GB3 accumulation 3. GLA mutation, α-galactosidase A deficiency, lyso-GB3 accumulation 4. GLA mutation, β-galactosidase A deficiency, lyso-GB3 accumulation 5. Silencing of X-linked genes, Gb3 accumulation, podocyturia",C,temp nan,Primary and Secondary Glomerular Diseases 5744," A 46-year-old woman presents with fatigue, weakness, and dyspnea in the setting of viral illness for 2 weeks. Her medical history is remarkable for longstanding Raynaud syndrome and migraine headaches. Medications include topiramate and ibuprofen. Her serum phosphate is 0.3 mmol/L (1 mg/d). Other laboratory values are notable for serum bicarbonate of 16 mEq/L..",What is the next best test to evaluate the role of renal phosphate wasting in this patient’s hypophosphatemia?,"1. Serum parathyroid hormone (PTH) and fibroblast growth factor-23 (FGF23) concentrations 2. Serum protein electrophoresis and light chains 3. Fractional excretion of phosphate 4. Tubular maximal reabsorption of phosphate/GFR in a second morning urine after an overnight fast",D,temp nan,"Disorders of Divalent Ions, Renal Bone Disease and Nephrolithiasis" 5745," You are following a 54-year-old woman with polycystic kidney disease and eGFR 15 mL/min per 1.73m2. Her son is a potential kidney donor but is a soldier currently deployed in Afghanistan. Your patient does not want him to donate a kidney until his active service is over because she is afraid his remaining kidney will be injured. Both the patient and her son wish to avoid dialysis at all costs. Her creatinine is 3.9 mg/dL. Her serum phosphate is 5.9 mg/dL. She is not on any phosphate binders. She asks you to explain the science behind dietary phosphate restriction so she can adjust her diet..",Which of the following is true regarding intestinal phosphate absorption?,"1. It does not matter whether the source of phosphate is processed food, dairy products or vegetables as long as the total amount of phosphate is the same. 2. Unlike processed foods, the bioavailability of phosphate in whole grains is very high. 3. When dietary phosphate is high, the passive paracellular route of phosphate absorption is less important in determining the serum phosphate concentration than the closely regulated transport-mediated transcellular pathway. 4. The expression of the sodium driven phosphate transporter NaPi-IIb increases when dietary phosphate is low.",D,temp nan,"Disorders of Divalent Ions, Renal Bone Disease and Nephrolithiasis" 5746," You are asked to evaluate hypophosphatemia in a 39-year-old woman who has ESRD due to IgA nephropathy, and recently received a deceased donor kidney allograft. Anti-rejection medications include prednisone, mycophenolate and tacrolimus. She is also on trimethoprim/sulfamethoxazole and omeprazole..",Which of the following statement is true regarding this patient’s phosphate balance?,"1. Her medications have no effect on phosphate balance 2. Her hypophosphatemia is most likely unrelated to her kidney disease or transplant status 3. Her phosphate concentration is maintained by intestinal absorption, internal shifts and renal excretion 4. A phosphate sensing molecule has been identified in humans and underlies post-transplantation hypophosphatemia",C,temp nan,"Disorders of Divalent Ions, Renal Bone Disease and Nephrolithiasis" 5747," A 70-year-old diabetic man is evaluated in the emergency department because of 2 days of paresthesia and muscle cramps. His medical history is notable for adenocarcinoma of the prostate since 2017, treated initially with docetaxel chemotherapy, androgen deprivation therapy, and cholecalciferol followed by denosumab because of tumor progression and bone metastatic disease; denosumab 120 mg subcutaneously every 4 weeks was started earlier. Two weeks before his presentation to the emergency department, he was hospitalized with urinary tract infection and AKI secondary to obstruction. He was treated with parenteral ceftriaxone for 7 days and discharged with a long-term urinary indwelling catheter after receiving denosumab. Laboratory data at the time of the discharge showed hemoglobin of 10.0 g/dl, serum sodium of 138 mEq/L, potassium of 4.0 mEq/L, glucose of 80 mg/dl, albumin of 3.4 g/dl, BUN of 60 mg/dl, creatinine of 2.1 mg/dl, serum calcium of 7.0 mg/dL, phosphorus of 2.4 mg/dl, PTH of 90 pg/ml (reference range 12–72 pg/ml), and 25-hydroxyvitamin D level of 12 ng/ml (reference range 20–100 ng/ml)..",What is the most likely diagnosis?,"1. Denosumab-induced hypocalcemia 2. Denosumab-induced worsening of kidney function 3. Denosumab-induced hyperphosphatemia 4. Ceftriaxone-induced interstitial nephritis with consequent worsening of kidney function",A,temp nan,"Disorders of Divalent Ions, Renal Bone Disease and Nephrolithiasis" 5748," A 27-year-old man presents for evaluation of recurrent nephrolithiasis. He has had four stones since he was 17 years old, requiring lithotripsy twice. His father and brother also have kidney stones. A 24-hour urine demonstrates a volume of 3.2 L and abnormally high urine calcium..",Which of the following proteins regulate calcium excretion and have been associated with increased stone risk?,"1. Polycystin 2 2. Claudin 3. Klotho 4. FGF23",B,temp nan,"Disorders of Divalent Ions, Renal Bone Disease and Nephrolithiasis" 5749," A 6-year-old girl was referred to a renal pediatric service because of polydipsia, polyuria, and the finding of nephrocalcinosis on abdominal ultrasonography. Laboratory findings were notable for eGFR of 55 ml/min per 1.73 m2, hypomagnesemia (1.25 mg/dl; reference range, 1.6–2.6 mg/dl), elevated fractional excretion of magnesium (Fe Mg, 11%; reference range, < 4%), hypercalciuria (13 mg/kg per day; reference range, < 4 mg/kg per day), hypocitraturia (95 mg/day; reference range, 300–600 mg/day), and elevated serum PTH in serum (120 pg/ml; reference range, 10–65 pg/ml). She had received a diagnosis of myopia magna and macular coloboma at the age of 4 years. Her parents were cousins, and her 20-year-old brother had recently received a diagnosis of advanced chronic kidney disease (CKD)..",Which ONE of the following genetic abnormalities best explains this clinical condition?,"1. A loss of function mutation in the calcium-sensing receptor gene 2. A gain of function mutation in the calcium-sensing receptor gene 3. A loss of function mutation in the CLDN19 gene 4. A loss of function mutation in the CLDN16 gene",C,temp nan,"Disorders of Divalent Ions, Renal Bone Disease and Nephrolithiasis" 5750," A 40 year-old man with a history of well-controlled HIV infection presented to his routine evaluation with a 2-month history of irritability and polyuria. He had no history of opportunistic infection, his viral load was undetectable, and his CD4+ T cell count was 600 (reference range, 410–1590 × 106/L). His medications were abacavir/lamivudine/dolutegravir. Laboratory workup disclosed creatinine of 2.5 mg/dl and albumin-corrected serum calcium of 15.1 mg/dl (confirmed twice). He denied use of illicit drugs, vitamin supplements, calcium, or diuretics. Computed tomography of the chest, abdomen, and pelvis showed nodular soft tissue densities in both buttocks. The patient confirmed a history of cosmetic injections with polymethylmethacrylate for HIV lipodystrophy 3 months before the onset of symptoms. Serum PTH was 5 pg/ml (reference range, 12–72 pg/ml)..","At this point, which ONE of the following would you recommend next?","1. Measuring PTH-related protein 2. Measuring 1,25 dihydroxyvitamin D 3. Measuring serum and urine protein electrophoreses with immunofixation 4. Measuring 25 dihydroxyvitamin D",B,temp nan,"Disorders of Divalent Ions, Renal Bone Disease and Nephrolithiasis" 5751," A 45-year-old man with ESKD who had been using maintenance hemodialysis for 10 years received a diagnosis of severe and refractory hyperparathyroidism and was referred to your service for a parathyroidectomy. Laboratory values were calcium 11 mg/dl, phosphorus 7.5 mg/dl, PTH 1300 pg/ml (reference range, 12–72 pg/ml), and alkaline phosphatase 450 U/ml (reference range, 36–92 U/ml), serum albumin 3.2 g/dl. Six hours after surgery, his serum calcium dropped to 7.2 mg/dl..",Which ONE of the following is most likely a risk factor for his hypocalcemia?,"1. Hyperphosphatemia 2. Hypoalbuminemia 3. Hypercalcemia 4. High serum alkaline phosphatase",D,temp nan,"Disorders of Divalent Ions, Renal Bone Disease and Nephrolithiasis" 5752," A 46-year-old woman is being evaluated in clinic for stage 4 CKD and is found to have mild hyperphosphatemia with a phosphorus of 4.5 mg/dl..","On the basis of results from the COMBINE trial, which ONE of the following statements is correct?","1. Addition of nicotinamide to lanthanum carbonate is more effective than lanthanum carbonate alone for lowering se-rum PTH in patients with stage 4 CKD 2. Lanthanum carbonate therapy can be expected to lower serum FGF23 concentrations by at least 30% in patients with stage 4 CKD 3. Lanthanum carbonate plus nicotinamide should be considered as a first-line therapy for reducing serum phosphorus in patients with stage 4 CKD 4. Treatment with lanthanum carbonate either alone or in combination with nicotinamide has little effect on serum PTH in patients with stage 4 CKD",D,temp nan,"Disorders of Divalent Ions, Renal Bone Disease and Nephrolithiasis" 5753," A 62-year-old man with a history of hypertension, peripheral vascular disease, and ESKD requiring long-term maintenance hemodialysis therapy is seen during routine dialysis rounds. His medication regimen includes cinacalcet 30 mg, calcium acetate 2001 mg with meals, and paricalcitol 4 µg three times per week. Recent laboratory results show a serum calcium of 10.8 mg/dl, serum phosphorus of 5.0 mg/dl, and serum PTH of 475 pg/ml..",Which ONE of the following medication changes would be MOST appropriate at this time?,"1. Change calcium acetate to sevelamer carbonate 2. Increase paricalcitol dose to 6 µg thrice weekly 3. Reduce calcium acetate to 1334 mg with meals 4. Increase cinacalcet dose to 60 mg daily",A,temp nan,"Disorders of Divalent Ions, Renal Bone Disease and Nephrolithiasis" 5754," It has been demonstrated that vitamin D, phosphate, and calcium are all important regulators of parathyroid gland biology..",Which ONE of the following statements is correct?,"1. Paricalcitol lowers PTH production by enhancing the binding of calcium to the calcium-sensing receptor 2. Cinacalcet blocks the binding of PTH to its receptors in bone 3. Cinacalcet stimulates the degradation of circulating PTH 4. Phosphate increases PTH production by reducing binding of calcium to the calcium-sensing receptor",D,temp nan,"Disorders of Divalent Ions, Renal Bone Disease and Nephrolithiasis" 5755," In a recent large epidemiologic study that analyzed data collected from patients with CKD over several decades, the association between serum PTH and several key clinical outcomes was studied..",Which ONE of the following best describes the observed associations with high concentrations of PTH in univariate and multivariate analyses?,"1. Increased risk of fracture, no increased risk of all-cause mortality 2. Increased risk of fracture, increased risk of vascular events, increased risk of mortality 3. No increased risk of fracture, increased risk of vascular events, no increased risk of all-cause mortality 4. No increased risk of fracture, increased risk of vascular events, increased risk of all-cause mortality",B,temp nan,"Disorders of Divalent Ions, Renal Bone Disease and Nephrolithiasis" 5756,,"Question 5756 For which ONE of these dialysis patients would a bone biopsy be most likely to help make a correct diagnosis?","1. A 58-year-old woman with pruritis and recent distal radius fracture, PTH level of 750 ng/ml, serum phosphate level of 5.9 mg/dL, serum calcium level of 10.3 mg/dL. 2. A 40-year-old man who had noticed pain in his thigh when he stands and had previously been treated with aluminum hydroxide for severe hyperphosphatemia. Radiographs of the hands show subperiosteal resorption. PTH is 290 ng/L; serum phosphate is 5.4 mg/dl. 3. A 64-year-old woman with diabetes who underwent a parathyroidectomy 3 years ago and who fractured her hip after a fall when she tripped over a root Her serum calcium is 8.9 mg/dl, phosphate is 4.1 mg/dL, PTH is 56 pg/ml, and bone alkaline phosphatase is low (15 µg/L). 4. A 50-year-old man with a stress fracture of his femoral neck. He has a history of obesity and underwent a gastric bypass procedure 10 years ago. His laboratory studies show PTH of 450 ng/L, calcium of 7.1 mg/dl, phosphate of 3.5 mg/dl, 25-hydroxy-vitamin D level of 12 ng/ml, and alkaline phosphatase of 300 IU/L.",B,temp nan,"Disorders of Divalent Ions, Renal Bone Disease and Nephrolithiasis" 5757," A 53-year-old woman with stage 4 CKD is diagnosed with osteoporosis after a radial fracture. She is already taking phosphate binders and is taking calcitriol for treatment of secondary hyperparathyroidism. Her primary care physician suggests that she start taking a bisphosphonate, and when she comes to see you, she asks about the effect of these agents on bone formation..","In patients treated with bisphosphonates, which of the following is correct?","1. The bone formation rate does not change because bisphosphonates decrease the bone resorption rate 2. The bone formation rate decreases to levels that are average for normal people 3. The bone formation rate gradually decreases over approximately 6 months and then remains at a rate that is below normal 4. The bone formation rate increases because of stimulation of PTH",C,temp nan,"Disorders of Divalent Ions, Renal Bone Disease and Nephrolithiasis" 5758," Low levels of klotho have been associated with cardiovascular disease in CKD, and treatment with klotho is being investigated in preclinical models of cardiovascular disease..",What action of soluble klotho raises the concern that supplementation could have a negative effect on bone?,"1. Klotho can act as a coreceptor for FGF23 on the parathyroid cells and reduce PTH 2. In early stages of CKD, klotho could act as a coreceptor for FGF23 on renal tubular cells to increase the excretion of phosphate 3. Klotho decreases calcium reabsorption in the distal tubule, causing high urine calcium 4. Klotho inhibits Wnt signaling by blocking the frizzled receptor, and this reduces the number of osteoblasts",D,temp nan,"Disorders of Divalent Ions, Renal Bone Disease and Nephrolithiasis" 5759," The term “bone turnover” can be expressed as the bone formation rate if there is coupling between resorption and formation..",In which of these clinical situations is it most likely that the bone formation rate and the bone resorption rate will be approximately the same?,"1. A patient with high PTH and increased calcium and alkaline phosphatase 2. A patient who has been treated with glucocorticoids, who has some metabolic acidosis and weight loss 3. A patient who has experienced multiple myeloma and has evidence of skeletal lesions on magnetic resonance imaging 4. A patient who received a bisphosphonate injection 3 weeks before the biopsy",A,temp nan,"Disorders of Divalent Ions, Renal Bone Disease and Nephrolithiasis" 5760," A 49-year-old biophysicist with polycystic kidney disease has been receiving hemodialysis for 4 years while awaiting trans-plantation. He has an abdominal plain radiograph during an emergency department evaluation for abdominal pain. The abdominal pain turns out to be related to constipation, and he is discharged from the emergency department..","He reads the radiology report online, which notes “extensive calcification observed in the tunica media of the aorta.” He asks you, “What caused this problem?” Which of the following statements is most accurate regarding the development of vascular calcification?","1. Secondary calciprotein particles (CPP-II) form as kidney function declines and as result of decreases in expression of fetuin-A and ƴ-linolenic acid—rich protein, and induce osteoblast differentiation of vascular smooth muscle cells. 2. The removal of extracellular vesicles from serum increases the calcification of vascular smooth muscle cells in both uremic and nonuremic individuals. 3. Increased phosphate concentration is necessary and sufficient for vascular calcification. 4. Patients receiving hemodialysis invariably experience vascular calcification.",A,temp nan,"Disorders of Divalent Ions, Renal Bone Disease and Nephrolithiasis" 5761," A 58-year-old hemodialysis patient with longstanding type 1 diabetes and hypertension and on the deceased donor waiting list for a kidney pancreas transplant has a lateral abdominal radiograph as part of evaluation for back pain..","The radiologist reports “significant abdominal aortic calcification of 12/24 using the Kauppila scoring method.” Which of the following statements is the most accurate?","1. Abdominal aortic calcification is associated with increased cardiovascular risk and mortality, but this association is quickly lost after transplantation 2. Abdominal aortic calcification is associated with increased cardiovascular risk but has not been shown to associate with increased mortality 3. A low Kauppila score associates with a high cardiovascular risk 4. Abdominal aortic calcification has been associated with increased risk of cardiovascular events and increased mortality, which persists after transplantation",D,temp nan,"Disorders of Divalent Ions, Renal Bone Disease and Nephrolithiasis" 5762," A 75-year-old man has ESKD from longstanding diabetes and hypertension and is receiving hemodialysis three times weekly, 4 hours. His dialysis prescription is as follows: 4 hours, optiflux 180 dialyzer, blood flow rate 400 ml/min, dialysate flow rate 800 ml/min. Dialysis bath includes: sodium 137 mEq/L, potassium 2 mEq/L, calcium 2.25 mEq/L, magnesium 1 mEq/L..","Based on current data, which ONE of the following changes to dialysis regimen would improve the serum calcification propensity as measured by increased T50?","1. Change sodium to 135 mEq/L 2. Change potassium to 1 mEq/L 3. Change magnesium to 2 mg/dl 4. Increase dialysis time to target higher Kt/V",C,temp nan,"Disorders of Divalent Ions, Renal Bone Disease and Nephrolithiasis" 5763," You have been following up a 59-year-old man with alcohol use disorder, chronic pancreatitis with malabsorption, diabetes, and ESRD who is receiving hemodialysis. He commonly misses dialysis because of transportation issues. He frequently skips medications because of financial constraints. He has widespread vascular calcification on imaging..",Which of the following is least likely to prevent or delay progression of vascular calcification?,"1. Magnesium supplement 2. Vitamin K supplement 3. Calcium-containing phosphate binder 4. Calcimimetic agents",C,temp nan,"Disorders of Divalent Ions, Renal Bone Disease and Nephrolithiasis" 5764," A 79-year-old woman with ESRD secondary to diabetes and hypertension fractured her hip while ambulating to a chair after dialysis. She receives dialysis three times weekly. Her most recent PTH is 89 mg/dl, phosphorus is 4.9 md/dl (reference range 1–9 mg/dl), and calcium is 8.8 mg/dl..",Which statement below is incorrect?,"1. In her age group, the risk of fracture is higher for ESRD compared with non-ESRD patients 2. The mortality risk for ESRD patients is higher after the fracture compared with mortality risk in nonESRD patients after fracture 3. Factors that increase the fracture risk of ESRD patients include age and CKD with mineral and bone disorder but not osteoporosis 4. She is more likely to have a prolonged hospitalization after the fracture",C,temp nan,"Disorders of Divalent Ions, Renal Bone Disease and Nephrolithiasis" 5765," A 61-year-old woman has stage 3 CKD related to class 4 lupus nephritis treated successfully with Cytoxan and glucocorticoids when she was 30 years old. She has mild hypertension and is taking lisinopril 20 mg daily, atorvastatin, and a multivitamin. PTH, phosphorus, and alkaline phosphatase have always been in normal range. She has never been screened for osteoporosis because her nephrologist told her the results of common screening methods would be difficult to interpret. Her current primary care physician asks you for advice regarding screening..",Which of the following statements is most accurate?,"1. There is little utility to screening by dual energy x-ray absorptiometry (DXA) because she likely has renal osteodystrophy, and DXA would not be sufficiently predictive of fracture risk 2. There is little utility to screening because the treatments for osteoporosis are contraindicated by her reduced eGFR 3. Bone density screening would not be helpful because patients with lupus will have too much arthritis to get accurate density measurements 4. She should be screened by DXA every 12 to 24 months",D,temp nan,"Disorders of Divalent Ions, Renal Bone Disease and Nephrolithiasis" 5766," A 72-year-old man has stage 4 CKD related to renovascular disease. He has coronary artery disease and has had a ST-elevation myocardial infarction, has undergone multiple angioplasties, has peripheral vascular disease, and has severe chronic obstructive pulmonary disease (COPD). He has been taking glucocorticoids for years for COPD. He has had several fractures related to falls and also has a nontraumatic vertebral fracture. He has been taking calcitriol for many years, but this was stopped when his PTH was noted to be < 100 pg/ml. Follow-up laboratory studies after vitamin D receptor activators was stopped show a low bone-specific alkaline phosphatase and PTH 120 pg/ml. Vitamin D levels are normal. Phosphorus is normal, and he is not using phosphate binders. Calcium is within normal range at 9.8 mg/dl..",Which of the following agents could be used for his osteoporosis?,"1. Pamidronate 2. Denosumab 3. Teriparetide 4. Romosozumab",C,temp nan,"Disorders of Divalent Ions, Renal Bone Disease and Nephrolithiasis" 5767," A 60-year-old woman with stage 5 CKD and an eGFR 12 ml/min per 1.73 m2 is planned for hemodialysis and is noted to have osteoporosis on a routine DXA with a left femoral neck T score of −2.8. Her medical history. includes diabetes, obesity, hypertension, and a 20-pack/year smoking history, Her phosphate is 4.6 mg/dL, PTH is 530 pg/mL, bone-specific alkaline phosphatase is high, and vitamin D and calcium are normal..",Which of the following agents is best for her osteoporosis?,"1. Alendronate 2. Denosumab 3. Teriparetide 4. Abaloparatide",B,temp nan,"Disorders of Divalent Ions, Renal Bone Disease and Nephrolithiasis" 5768," A 66-year-old woman with ESRD treated with maintenance hemodialysis is evaluated for severely painful induration of her anterior abdominal wall. She has chronic atrial fibrillation treated with metoprolol and warfarin. Skin biopsy of an indurated lesion showed arteriolar calcification and thrombosis of subcutaneous adipose tissue. Fine stippled calcification was noted surrounding the perieccrine glands..",Which ONE of the following statements most correctly describes this patient’s prognosis?,"1. The presence of arteriolar calcification suggests advanced lesion stage and worse prognosis 2. The presence of arteriolar thrombosis indicates advanced lesion stage and worse prognosis 3. The 1-year mortality rate for patients with this condition is < 10% 4. The location of skin lesions influences prognosis 5. Ulcerated lesions have a similar prognosis to nonulcerated lesions",D,temp nan,"Disorders of Divalent Ions, Renal Bone Disease and Nephrolithiasis" 5769," A 52-year-old white woman with ESRD secondary to diabetes mellitus treated with long-term ambulatory peritoneal hemodialysis receives a diagnosis of calcific uremic arteriolopathy. The bilateral skin lesions are on her thighs and are severely painful. She has a history of subtotal parathyroidectomy performed many years ago. Her laboratory data include serum calcium 7.8 mg/dL, serum phosphate 6.4 mg/dL, serum PTH 27 pg/ml, and serum albumin 3.2 g/dL. Her current medications include calcium carbonate and calcitriol..","In addition to discontinuing calcium carbonate and calcitriol, which ONE of the following is the most appropriate treatment of mineral bone abnormalities in this patient?","1. Add cinacalcet 2. Urgent surgical parathyroidectomy 3. Add aluminum hydroxide 4. Add sevelamer",D,temp nan,"Disorders of Divalent Ions, Renal Bone Disease and Nephrolithiasis" 5770," A 59-year-old woman is referred for evaluation of recurrent stones sustained over the past 15 years. She has a 10-year history of diabetes, hypertension, and hypercholesterolemia, treated with metformin, lisinopril, and atorvastatin. She describes a steady increase in weight since her last pregnancy 30 years ago. She works as a computer programmer and notes little to no exercise except for walking from her car to her office. She tries to drink fluids during the day but notes that she often leaves a bottle close to full at the end of the day. Her body mass index (BMI) is 33, with a waist circumference of 90 cm; her BP at home is 140/82 mmHg..",Which of the following does not contribute to her risk of stone recurrence?,"1. Insufficient urine volume 2. Physical inactivity 3. High BMI 4. Diabetes",B,temp nan,"Disorders of Divalent Ions, Renal Bone Disease and Nephrolithiasis" 5771," A 24-year-old man presents to an emergency department with sudden-onset left flank pain, nausea, and vomiting after spending 4 days rafting on the Colorado River in July. He hasahistoryofkidneystones2yearsand4years ago,both evaluated with computed tomography (CT) urograms but not requiring intervention. Since that time he believes he has passed multiple small stones that required no intervention. His BP is 145/85 mmHg, and his heart rate is 110 beats/min. He is afebrile . He is extremely uncomfortable, but the results of abdominal examination are benign. Laboratory analysis shows creatinine 1.0 mg/dl and normal compre-hensive medical panel results. The results of a complete blood count are normal, with no leukocytosis. Urinalysis shows 3+ heme and no protein. Pain is adequately relieved with intravenous ketorolac..",What is the best initial imaging test?,"1. CT scan with intravenous contrast material 2. Radiograph of abdomen (kidneys, ureters, bladder) 3. CT urogram 4. Renal ultrasonography",D,temp nan,"Disorders of Divalent Ions, Renal Bone Disease and Nephrolithiasis" 5772," A 48-year-old woman is referred for evaluation of recurrent nephrolithiasis. She has a history of recurrent stones since age 18 years, which she addresses with copious fluid intake. A CT scan performed to evaluate hematuria shows “possible” medullary sponge kidney. Her medical history is otherwise remarkable for osteopenia detected by screening DXA scan..",Which of the following is the most likely abnormality detected on 24-hour urine metabolic study?,"1. High calcium 2. Low urine pH 3. High uric acid 4. High cystine",A,temp nan,"Disorders of Divalent Ions, Renal Bone Disease and Nephrolithiasis" 5773," A 45-year-old woman is referred for evaluation of recurrent stones. Her BMI is 45. She has diabetes and hypertension. She states that she is considering bariatric surgery for weight loss but is concerned because she heard that it will increase her risk of stones, even though she had previously been told that obesity was part of the reason for her prior stone risk..",Which of the following is true regarding bariatric surgery?,"1. Because the stones are calcium oxalate, the risk can be minimized by restricting dietary calcium and oxalate 2. The risk of stones related to bariatric surgery is lower than the risk conferred by obesity 3. The risk conferred by bariatric surgery can be minimized by restricting dietary fat, increasing dietary calcium or taking a calcium supplement with meals, and maintaining high urinary volume of > 2.5 L/d 4. The risk of stones is the same in all types of bariatric surgery",C,temp nan,"Disorders of Divalent Ions, Renal Bone Disease and Nephrolithiasis" 5774," A 69-year-old woman has ESKD related to longstanding diabetes and hypertension. She has paroxysmal atrial fibrillation, coronary artery disease, and severe left ventricular hypertrophy by echocardiography. Dialysis regimen is 4.0 hours, three times weekly. Kt/Vurea is 1.5 per session, usual weight gain is 3 to 4 L. Predialysis laboratory values consistently demonstrated [K] 6.5 to 7.0 mEq/L, [Na] 125 to 129 mEq/L, [HCO3] 18 to 24 mEq/L [Cl] 99 to 105, [phosphorus] 7 to 8 mg/dl. She dialyzes with an arteriovenous fistula with Qb approximately 400 ml/min. Venous pressure is 100 mmHg. She shows up for every treatment but commonly signs off early because of back pain after sitting in the chair for too long. She lives alone and tries to eat “healthy for her heart.” Dietician has discussed diet many times..",Which of the following is the best management strategy for her hyperkalemia?,"1. Continue dialysate [K] 2 mEq/L, reinforce dietary restriction, check for recirculation, add if these measures do not yield a decrease in the serum [K], add a potassium-binding resin on non-dialysis days 2. Increase dialysis time to 4.5 hours, continue dialysate [K] 2mEq/L 3. Dietary restriction, change dialysate to [K] 1 mEq/L 4. Dietary restriction and continue current regimen",A,temp nan,End-Stage Kidney Disease 5775," A 79-year-old man with ESKD related to hypertension and Apolipoprotein L1 gene is on dialysis: 3 days, 4 hours, QB/Qd 400 to 600 ml/min. Dialysate [Ca] is 2.5 mEq/L. Laboratory results show Kt/Vurea 1.4, serum [Ca] 8.4 mg/dl, [Phos] 5.9 mg/dl, parathyroid hormone 679 mg. His BP tends to drop 1 to 2 hours into treatment, which limits his ability to achieve dry weight..",Changing his dialysate [Ca] to 2.75 would do which of the following?,"1. Cause parathyroid hormone to increase further 2. Cause no change in total or ionized [Ca] 3. Decrease frequency of episodes of intradialytic hypotension 4. Have no effect on laboratory values or dialysis stability",C,temp nan,End-Stage Kidney Disease 5776," A 60-year-old woman with ESKD related to diabetes and received a living-related transplant that failed after 15 years. She is being evaluated for a second transplant from her partner. She is dialyzed three times weekly for 3.5 hours. Her estimated dry weight (EDW) is 64 kg, and she typically gains 4 to 5 L between dialysis sessions. Although she is generally hemodynamically stable and appears to tolerate aggressive ultrafiltration (UF), the dialysis nurses are reluctant to increase her ultrafiltration rate (UFR), so she leaves above her EDW. She has presented several times to the emergency department in pulmonary edema, always on the weekend. She continues to make urine..",What is the best approach to prevent episodes of pulmonary edema?,"1. Decrease her EDW to 62 2. Given that she tolerates it, double her UFR to make sure she always leaves at EDW 3. Add a loop diuretic on non-dialysis days and continue to limit her UFR during dialysis to < 13 ml/kg per h",C,temp nan,End-Stage Kidney Disease 5777," An 82-year-old woman has stage 5 CKD related to longstanding diabetes and hypertension plus NSAID use. She also has mild to moderate aortic stenosis and a mildly reduced ejection fraction, cerebral vascular disease having undergone carotid endarterectomy, and mild to moderate peripheral vascular disease that her vascular surgeon is closely following. She has poor manual dexterity and is perceived not to be a good candidate for peritoneal dialysis (PD). She walks a quarter of a mile daily in an effort to stay mobile and healthy and is very invested in her grandchildren and great-grandchildren, of whom she has many. She and her family recognize the difficulties of hemodialysis (HD) but still want to try for at least a while. She is eating well and has not lost weight..",What is the single best approach to preparing her for dialysis?,"1. Have her referred immediately for an arteriovenous fistula so it may mature in time for dialysis 2. Try to convince her to do active medical management/conservative care rather than dialysis because she is a poor candidate for dialysis 3. Refer her to interventional radiology/nephrology for a tunneled dialysis catheter so that it is ready to use when she needs it 4. Follow her closely for uremic symptoms; when she decides to start HD, admit her for nonurgent placement of tunneled dialysis catheter and initiate HD in the hospital",D,temp nan,End-Stage Kidney Disease 5778," A 49-year-old woman with longstanding lithium use has estimated GFR (eGFR) 8 ml/min per 1.73 m2. She has well-compensated bipolar disease and is now off lithium. She makes a significant amount of urine daily, and she is otherwise healthy. She is terrified about starting dialysis. She has turned down PD numerous times because she does not wish to infuse fluid into her belly. She has several friends who have promised her a kidney if she would agree to transplantation. She is an electrical engineer and is the chief executive officer of a startup company that markets software internet-based video technology, and she does not believe she has time for either PD or three-times-weekly HD..",Which of the following is the best single approach to renal re-placement therapy?,"1. Wait until she becomes uremic and admit her for standard three-times-weekly dialysis 2. Refer her for transplantation evaluation and hope for the best 3. Refer her to palliative care to help with conservative therapy 4. Measure her creatinine clearance and discuss with her the possibility of starting dialysis 2 days per week",D,temp nan,End-Stage Kidney Disease 5779," A 54-year-old businessman started on automated PD (APD) 1 year ago. After the initial adjustment, he has finally adapted to his current APD regimen and enjoys his busy work-life sched-ule. His current APD regimen is 1.5% dextrose solution with a total fill volume of 8 L over 8 hours. He is currently euvolemic with no uremic symptoms and has a good appetite. He is also able to maintain a daily urine output of about 800 ml/day. However, his latest Kt/Vurea is calculated to be 1.6, with BUN 58 mg/dl and creatinine 9.0 mg/dl. His albumin is 3.0 g/L, and hemoglobin is 10.2 g/dl..","Which of the following is the most appropriate option for the management of his APD, following the recent International Society for Peritoneal Dialysis (ISPD) guideline?","1. Increase dwell treatment time to 10 L over 10 hours 2. Add a day dwell of icodextrin 3. Switch him to HD 4. Continue current APD regimen and reassess again later",D,temp nan,End-Stage Kidney Disease 5780," A 60-year-old man has longstanding type 2 diabetes and CKD approaching end stage with an eGFR 9 ml/min. He is not keen for HD and has opted for PD at home..",Which of the following is a better option for an access for his peritoneal dialysis?,"1. Straight single-cuff Tenchkoff catheter 2. Straight double-cuff Tenchkoff catheter 3. Coiled single-cuff Tenchkoff catheter 4. Coiled double-cuff Tenchkoff catheter",B,temp nan,End-Stage Kidney Disease 5781," A 45-year-old woman with type 2 diabetes, a history of ischemic heart disease with prior non-ST-segment elevation myocardial infarction, and hypertension has been on PD for 4 years. On routine follow-up, her BP is 140/80 mmHg on lisinopril 10 mg once daily and carvedilol 12 mg twice daily. Her creatinine is 5.0 mg/dl, and her albumin is 2.7 g/dl. She takes atorvastatin 40 mg once daily, and her lipid profile is as follows: total cholesterol 300 mg/dl, LDL 180 mg/dl, and HDL 48 mg/dl. She has had four episodes of peritonitis over the past 4 years on PD..","Based on recent studies and KDIGO guidelines, which of the following would be expected to improve her long-term outcome?","1. Increasing her statin dose to target a lower LDL 2. Review the causes of her episodes of peritonitis to reduce risk of recurrence 3. Escalate her antihypertensives to target BP at < 130/ 80 mmHg 4. Stop the angiotensin-converting enzyme inhibitor to im-prove her residual renal function (RRF)",B,temp nan,End-Stage Kidney Disease 5782," A 60-year-old man with stage 5 CKD due to longstanding type 2 diabetes, eGFR 6 ml/min, and a urine output of approximately 800 ml/day presented with fluid overload and uremic symptoms of nausea, fatigue, and anorexia. He has repeatedly defaulted outpatient follow-up in the past 6 months owing to poor acceptance of disease trajectory. On physical examination, his BP was 160/82, and pulse oxygen saturation was 92% on room air. He had bibasal crackles on lung examination and 2+ ankle edema. His serum creatinine was 13 mg/dl, and serum potassium was 5.2 mmol/L. After lengthy discussion, he opted to initiate PD..",Which one of the following is the BEST option for preserving his RRF?,"1. Add oral loop diuretics 2. Start with twice daily PD exchanges, then gradually step up 3. Use low-glucose degradation product and neutral pH bio-compatible PD solutions 4. Start him with automated PD",C,temp nan,End-Stage Kidney Disease 5783," A 69-year-old man with ESKD due to diabetes, and a history of ischemic heart disease, presents to clinic complaining of in-creased exertional dyspnea over the past 2 weeks. He started continuous ambulatory PD (CAPD) 3 years ago. His current CAPD regimen consists of three exchanges using 2 L of 1.5% standard PD solution. He does not record his UF volume and admits that he sometimes shortens dwell time to 4 hours. On physical examination, BP is 125/86 mmHg, body weight is 80 kg, and there is no pedal edema. Weekly Kt/Vurea is 1.6, and 24-hour urine collection showed residual urine of 547 ml. His serum albumin was 29 g/dl. Peritoneal equilibration tests demonstrate dialysate-to-plasma creatinine ratio at 4 hours was 0.47, identifying him as having low transporter status..","Bioimpedance spectroscopy indicated that volume of overhydration was + 3.6 L. In addition to reinforcing dietary and PD compliance, which of the following strategies will improve this patient’s fluid overload?","1. Switch to automated PD 2. Arrange serial bioimpedance spectroscopy 3. Change to three bags of 1.5% 2-L low-glucose degradation product neutral pH solution 4. Add furosemide 250 mg daily",D,temp nan,End-Stage Kidney Disease 5784," You are asked to review the rate of CAPD peritonitis for a quality improvement project at your local center..","Based on the ISPD 2016 guideline, at which of the following levels of culture-negative peritonitis, among all the peritonitis episodes, should a review of sampling and culture methods be carried out?","1. More than 5% 2. More than 10% 3. More than 15% 4. More than 20%",C,temp nan,End-Stage Kidney Disease 5785," A 54-year-old man who has been on PD for 2 years was admit-ted with abdominal pain and cloudy peritoneal fluid. A diagnosis of CAPD peritonitis is made, and intraperitoneal cefazolin and ceftazdime were started as first-line antibiotics. He is now afebrile, but his peritoneal fluid remains cloudy. His peritoneal fluid culture has grown out Enterococcus species, and sensitivity results are pending..",Which of the following may be the most suitable next choice of antibiotics?,"1. Continue intraperitoneal cefazolin, given that he is now afebrile 2. Add intraperitoneal gentamicin 3. Add intraperitoneal linezolid 4. Switch to oral amoxicillin",D,temp nan,End-Stage Kidney Disease 5786," A 77-year-old woman with heart failure with preserved ejection fraction, type 2 diabetes, and CKD stage 5, and with eGFR 13 ml/min, comes to your clinic for follow-up. She has had three admissions for decompensated heart failure in the prior 4 months and reports worsening lower limb edema despite escalating diuretics. Her BUN has risen to 104 mg/dl, and her most recent creatinine is 3.5 mg/dl. Her energy has declined over the past month and she wishes to discuss her options for starting dialysis..",Which of the following is correct with regard to initiation of PD?,"1. Patients who start on “full-dose” PD have similar preservation of RRF as those who start with incremental PD 2. APD is associated with a lower risk of technique failure than CAPD 3. Biocompatible PD solutions have been associated with lower rates of PD peritonitis 4. Use of icodextrin-based PD solutions are associated with more rapid loss of RRF",B,temp nan,End-Stage Kidney Disease 5787," As part of a quality improvement initiative at your PD unit, you have arranged to meet with the surgical team to review protocols for PD catheter insertion at your hospital. Your surgeons have traditionally placed PD catheters using an open surgical technique and are considering a switch to laparoscopic technique. In advance of this meeting, you will circulate a document summarizing recent studies in this area, and include discussion points to review during the meeting..","In planning your PD new catheter placement protocol, which one of the following statements is correct?","1. Embedded PD catheters are associated with higher rates of exit site infections 2. Laparoscopic placement of PD catheters is recommended because it has been clearly associated with superior out-comes when compared with open surgical placement 3. A recent randomized controlled trial showed that straight PD catheters have higher rates of dysfunction than coiled PD catheters 4. No differences have been found in rates of complications such as peritonitis and exit site infections when straight and coiled PD catheters have been compared",D,temp nan,End-Stage Kidney Disease 5788," A 54-year-old man with ESKD secondary to IgA nephropathy started on automated PD 3 months ago. Despite escalating doses of antihypertensives, his BP control has been worsening over the past few months, and his home readings currently average at 168/96 mmHg. He has well-maintained RRF and makes > 600 ml urine per day. His physical examination result is unremarkable and without overt signs of fluid overload. When you suggest changing his PD prescription to increase his UF, he replies that he does not feel over-loaded and prefers to increase his medication further..","When managing BP and volume status in PD patients, which of the following is correct?","1. Bioimpedance studies suggest that fluid overload is more common in HD patients than PD patients 2. Risk factors for fluid overload (as measured by bioimpedance spectroscopy) in PD patients include hypoalbuminemia and diabetes 3. Management of fluid status using bioimpedance data has been shown to improve outcomes for patients on PD 4. In patients initiating PD, higher BPs have been associated with increased mortality rates",B,temp nan,End-Stage Kidney Disease 5789," A 63-year-old woman with ESKD secondary to ANCA vasculitis has been on PD for 1 year. She is currently listed for renal transplantation, with 2 years of wait time. When she initiated PD, she had excellent RRF, but over the past few months, her urine output has begun to decline. She is keen to stay on PD until she receives a transplant, and she understands the importance of RRF to successful PD. She asks what can be done to protect her remaining kidney function..","In considering strategies to maintain RRF in PD patients, which of the following is correct?","1. Use of angiotensin-converting enzyme inhibitor and angiotensin receptor blocker has been associated with lower rates of cardiovascular mortality but faster loss of RRF 2. Exposure to PD fluids containing high concentrations of glucose has been associated with faster loss of RRF 3. Bioimpedance studies suggest that maintaining mild volume expansion may protect against loss of RRF in PD patients 4. Studies have not shown a benefit of biocompatible PD solutions over conventional PD solutions for preservation of RRF",B,temp nan,End-Stage Kidney Disease 5790," A 63-year-old man with ESKD secondary to polycystic kidney disease has been on PD for 4 years, He was recently admitted with chest pain and received a diagnosis of non-ST-segment elevation myocardial infarction, He has been discharged on ap-propriate secondary prevention and comes to your office for a postadmission visit. When reviewing his case, you see that his home BPs average at 134/88 mmHg, and his recent LDL was 64 mmol/L..","When considering this patient’s risk factors for cardiovascular outcomes and mortality, which of the following is correct?","1. Hyperlipidemia has been associated with increased mortality in PD patients 2. Monitoring of home BP has been associated with improved clinical outcomes 3. Prospective studies have failed to show a correlation be-tween RRF and mortality in PD patients 4. During the past decade, mortality rates for patients on PD declined more quickly than rates for HD patients",D,temp nan,End-Stage Kidney Disease 5791," A 60-year-old man with ESKD has been on home HD (HHD) using the NxStage System One at home for 1 year. He uses a left forearm fistula. He has been clinically stable during and after HD. His BP is well maintained throughout dialysis. * Medical history: IgA nephropathy, hypertension, no diabetes or coronary artery disease. * Dialysis prescription: dry weight 72 kg, 3 hours 5 days per week, total weekly UF volume 8 L. Calculated standard Kt/Vurea per week 2.3. * Dialysate composition: [Na] 140 mEq/L, [K] 2 mEq/L, chloride 101 mEq/L, [Ca] 3 mEq/L, lactate 45 mEq/L. * Laboratory tests are done before HD monthly; this time, la-boratory tests after HD were done as well. * Before HD: creatine 6.2 mg/dl (548 µmol/L), Na 136 mEq/L, K 5.4 mEq/L, Cl mEq/L 98, total carbon dioxide (tCO2) 24 mEq/L, anion gap 14. * After HD: creatinine 3.5 (309 µmol/L), Na 137 mEq/L, K 3.6 mEq/L, Cl 99 mEq/L, tCO2 20 mEq/L, anion gap 18. * Normal liver function test results..",Why is the serum bicarbonate lower after HD than before HD?,"1. Possible laboratory error 2. Drug overdose 3. Ketoacidosis 4. Lactic acidosis related to organ ischemia or liver disease 5. The patient has lactic acidosis as a result of the presence of lactate as a buffer in the dialysate",E,temp nan,End-Stage Kidney Disease 5792," A 70-year-old man on HHD has used the NxStage System One for 4 years. He has been feeling tired, has poor appetite, and has lost 2 kg over the past year. He is anuric. Dialysis prescrip-tion: 2.5 hours 5 days per week, Dialysate volume 30 L, calculated standard Kt/Vurea per week 1.9, dry weight 110 kg. Week-ly UF 13 L. Laboratory test results before HD: creatinine 16.2 mg/dl (1432 mmol/L), Na 136 mEq/L, K 5.9 mEq/L, Cl 98 mEq/L, tCO2 22 mEq/L, calcium 9.6 mg/dl (2.4 mmol/L), phosphate 7.5 mg/dl (2.5 mmol/L), albumin 3.3 g/dl (33 mg/L). He is on calcium carbonate phosphate binders, which he does not tolerate well. He is on three antihypertensive medications. You are concerned about possibly inadequate dialysis dose as the cause of the patient’s problem. Therefore, you have selected as a goal a weekly standard Kt/Vurea of 2.4..",Which of these prescriptions or modalities would best address the clinical issues described here?,"1. Convert to nocturnal HD 7 hours 5 times per week at the same dialysate volume of 30 L 2. Dialyze for 4 hours five times per week 3. Convert to in-center HD 4 hours three times per week 4. Convert to PD",A,temp nan,End-Stage Kidney Disease 5793," A 56-year-old man received his second kidney transplant 9 years ago. His kidney function has been declining related to transplant glomerulopathy. His medical history includes diabetes mellitus, hypertension, and coronary artery disease. He had two previous myocardial infarctions and one episode of congestive heart failure, leading to hospitalization. He was told that he will need dialysis within the next few months. On an echocardiogram he has eccentric left ventricular hypertrophy, and his left ventricular ejection fraction is 33%. He is highly presensitized, and therefore a new kidney transplant is not expected soon. The patient is asking about his dialysis options..",Which of the following is true regarding nocturnal HHD?,"1. BP control is likely to be slightly worse compared with three-times-weekly in-center HD 2. The Frequent Hemodialysis Network extended trial conclusively demonstrated that nocturnal HD results in lower mortality 3. He may not need to use phosphate binders with nocturnal HD 4. His left ventricular hypertrophy will likely worsen on intensive HD",C,temp nan,End-Stage Kidney Disease 5794," A 42-year-old woman has advanced CKD level 5, approaching renal replacement therapy. She does not a have a live donor for a pre-emptive kidney transplantation. She is considering some of the home dialysis options, specifically HHD. She is asking what type of dialysis other patients choose in the United States..",What is the use of HHD in the United States?,"1. The approximate number of prevalent patients in the United States is PD 60,000 and HHD 10,000 2. The incidence rate of HHD is 5% 3. The prevalence rate of HHD is 8% 4. There was a significant increase in HHD uptake during the COVID-19 pandemic",A,temp nan,End-Stage Kidney Disease 5795," A 34-year-old man with ESKD related to IgA nephropathy is awaiting deceased donor kidney transplantation. He teaches physiology at a local community college. He has been on conventional in-center HD for 2 years. However, he used to be a triathlete and is interested in intensive HHD to allow him to resume competition. He has read about HHD and asks you about the benefits..",Which of the following benefits are associated with HHD compared with PD and transplantation?,"1. HHD has been associated with improved survival compared with both living and deceased donor transplantation 2. HHD is associated with improved survival compared with living but not deceased donor transplantation 3. HHD is associated with improved survival compared with deceased donor but not living donor transplantation 4. HHD has been associated with improved survival com-pared with PD in patients who have been on in-center HD for longer than 1 year",D,temp nan,End-Stage Kidney Disease 5796," You are asked to give a talk to the chief medical officer of your hospital in anticipation of possible changes in the use of home kidney replacement therapies as a result of the COVID-19 pandemic..",Which of the follow is true regarding HHD?,"1. Use of PD but not HHD, increased during the COVID-19 pandemic 2. Use of HHD, but not PD, increased during the COVID-19 pandemic 3. Pre-emptive transplantation, but neither HHD nor PD in-creased during the COVID-19 pandemic 4. The use of all home therapies increased during the COVID-19 pandemic",A,temp nan,End-Stage Kidney Disease 5797," A 59-year-old patient with stage 5 CKD requests a second opinion with you to discuss the possibility of home dialysis..","According to recent data, which of the following is the most significant barrier to patients switching to home dialysis?","1. The amount of medical and procedural supervision avail-able to the patient 2. Relative cost of therapies 3. The utility of the standard Kt/Vurea in determining the prescription 4. The relative effect of home therapies on phosphate",A,temp nan,End-Stage Kidney Disease 5798," While rounding at the in-center dialysis unit you meet with a 75-year-old man with a history of diabetes, coronary artery disease, and peripheral vascular disease after having undergone a left above-the-knee amputation, who has been on dialysis for 6 years. Although he previously lived independently, he has been living in a skilled nursing facility for the past year. This is his first dialysis after being discharged from hospitalization with volume overload related to poor UF in the outpatient setting resulting from hypotension, and he required intubation and a stay in the intensive care unit. He met with palliative care in the hospital and completed some advance care planning, including an out-of-hospital “do not intubate and do not resuscitate” form. He ex-plains to you that should he continue to be unable to tolerate UF as an out patient, he would not want to go back to the hospital and would want to discontinue dialysis. He asks how long he would live if dialysis were discontinued..",Which of the following is the most appropriate response?,"1. “This is a very serious decision; I would like for you to meet with our social worker to discuss your mood and screen you for depression” 2. “Prognosis varies; however, most experience tells us that on average patients may live about 7 days without dialysis; This may be slightly more or less depending on other medical conditions that someone has, and how much residual renal function one has” 3. “Prognosis is unpredictable, but I would expect you could live for several weeks without dialysis” 4. “That's an excellent question, but unfortunately we do not know enough about prognosis under such circumstances; I will refer you to outpatient palliative care to discuss this decision further”",B,temp nan,End-Stage Kidney Disease 5799," A 72-year-old woman with a medical history of ESKD on maintenance HD, diabetes with neuropathy, hypertension on losartan, and atrial fibrillation on Eliquis is brought to the emergency department by her daughter after being found con-fused at home, unable to recognize family, and appearing disheveled. She has been compliant with her dialysis treatments, and her last treatment was 2 days prior. Although she has neuropathy, it has been controlled on a long-term stable low dose of gabapentin (100 mg three times per week), and she has no history of falls. She has had no fever, chills, or sick contacts. The daughter notes that she was having severe back pain last week, for which she saw her primary care doctor, who prescribed baclofen 10 mg daily. The result of blood work is unrevealing, and CT of the head does not show acute cerebrovascular events..",Which one of the following is the most likely causal agent of her altered mental status?,"1. Gabapentin 2. Baclofen 3. Eliquis 4. Losartan",B,temp nan,End-Stage Kidney Disease 5800," As medical director of an outpatient HD unit, you form a committee to decrease 30-day readmissions in your patients. The committee decides to start to focus on identifying patients who are at the highest risk for multiple readmissions. After re-viewing the available evidence regarding “high users” or those with frequent readmissions, you plan a focus on high-risk patient groups to reduce readmission rates..",Which of the following characteristics is most predictive of multiple readmissions in patients receiving HD?,"1. Age > 65 years 2. Community dwelling 3. Number of hospitalizations in the past year 4. Recent admission for a noncardiac cause",C,temp nan,End-Stage Kidney Disease 5801," You are consulting in the outpatient office on a new patient. He is a 65-year-old man with CKD stage 4 (eGFR 20 ml/min per 1.73 m2) from diabetic nephropathy. His wife accompanies him to the office. He notes unintentional weight loss of more than 10 pounds over the last year, complains of poor appetite and exhaustion, and upon evaluation he has slow gait speed and ambulates with a cane. Additionally, his wife notes that he requires help with some activities such as grocery shopping, cooking, and keeping track of medications. They are interested in transplantation referral and a home modality for dialysis..",Which of the following is the patient most likely to experience during his clinical course?,"1. He is at decreased risk for hospitalizations or emergency department visits 2. He should be referred for renal transplantation and is at low risk of post-transplantation complications 3. He is at increased risk of having technique failure with a home modality, such as PD 4. He should be initiated on dialysis earlier to help treat the symptoms he is currently experiencing",C,temp nan,End-Stage Kidney Disease 5802," An 85-year-old woman with a history of CKD stage 5 with an eGFR of 8 ml/min per 1.73 m2, dementia, diabetes, peripheral vascular disease, and cardiac disease with a remote history of myocardial infarction is referred to you for evaluation of treatment options for her advanced kidney disease. Her daughter accompanies her to the visit and notes that the patient has re-quired 24-hour care for the past several years and spends most of her days in a chair or bed at home. The daughter re-ports that she had previously discussed dialysis with her mother, and her mother was not interested in pursuing it. Her daughter asks your advice on conservative management of ESKD..","With regard to patients choosing conservative management of ESKD, which of the following is correct?","1. In the last 30 days of life, those who choose conservative management use fewer hospice services 2. In patients of all ages, those who choose dialysis live longer than those choosing conservative management 3. The family members and caregivers of those pursuing conservative management report lower quality of life 4. Those who pursue conservative management spend less time in hospital",D,temp nan,End-Stage Kidney Disease 5803," A 47-year-old woman with ESKD secondary to diabetes mellitus is being discharged after a 3-day inpatient admission for fluid overload and pulmonary edema. This is her third hospitalization within the past 6 months, and she asks whether she would have fewer readmissions if she were on PD..","With regard to readmissions for patients on renal replacement therapy, which of the following is correct?","1. Readmission rates are higher for patients on HD than those on PD 2. In patients who are on HD, the frequency of dialysis after an admission for pulmonary edema affects the rates of readmission 3. The 30-day readmission rate for patients on HD is 45% 4. Hospital admissions early after dialysis initiation do not significantly influence the likelihood of receiving a renal transplant in the second year on dialysis",B,temp nan,End-Stage Kidney Disease 5474," A 25-year-old woman with no significant medical his-tory participated in a full marathon for the first time. After she completed the race in 5 hours, she experienced seizures and was transferred to the emergency department. According to her friend, she had been drinking water frequently at the water stations during the marathon..","Vital signs: Glasgow Coma Scale (GCS): E2V2M2 (Eye response, opens eyes in response to pain; Verbal response, makes sounds; Motor response, extension to painful stimuli); BP 130/76 mm Hg (supine position), pulse rate 70 beats/min, respiratory rate 18/min, SpO2 95% (room air), temperature 36.5 degree C, weight 63 kg (+3 kg more than usual) Blood: total protein 7.3 g/dl, albumin 4.2 g/dl, BUN 12 mg/dl, creatinine 0.7 mg/dl, uric acid 3.0 mg/dl, Na 121 mmol/L, K 4.0 mmol/L, Cl 87 mmol/L, glucose 250 mg/dl, osmolality 260 mOsm/kg H2O Urine: osmolality 450 mOsm/kg H2O Which ONE of the following is the MOST likely cause of this patient’s seizures?","A. Dehydration B. Exercise-associated hyponatremia C. Use of nonsteroidal anti-inflammatory drugs during marathon training D. Hyperglycemia E. Volume depletion",B,temp nan,Electrolytes and Acid-Base Disorders 5475," A 21-year-old woman with no significant medical history participated in a rave party on a Saturday night. Her friend recommended the use of ecstasy. The patient was dancing and drinking several cocktails, and she gradually became confused and drowsy. Unfortunately, her friend thought that her drowsiness was due to too much alcohol, and she was left unattended. Subsequently, when she became unresponsive, she was transported to the emergency department. Vital signs: GCS: E1V1M1 (Eye response, does not open eyes; Verbal response, makes no sounds; Motor response, makes no movements); BP 100/60 mm Hg (supine position), pulse rate 120 beats/min, respiratory rate 12/min, SpO2 85% (room air), temperature 36.0 degree C, weight 60 kg (+3 kg more than usual) Blood: total protein 7.5 g/dl, albumin 4.5 g/dl, BUN 25 mg/dl, creatinine 1.1 mg/dl, uric acid 3.0 mg/dl, Na+ 120 mmol/L, K+ 4.5 mmol/L, Cl- 85 mmol/L, glucose 100 mg/dl, osmolality 250 mOsm/kg Urine: Na+ 120 mmol/L, K+ 30 mmol/L, osmolality 500 mOsm/kg H2O Computed tomography of brain: cerebral edema She received a diagnosis of cerebral edema caused by acute hyponatremia resulting from ecstasy..",Which ONE of the following is the MOST appropriate initial treatment for this patient?,"A. Loop diuretic (furosemide) IV (20 mg) B. 3% NaCl bolus IV (100 mL over 10 minutes up to x 3) C. 3% NaCl infusion (100 ml/h) D. Tolvaptan (15 mg via nasogastric tube) E. Urea (30 g via nasogastric tube)",B,temp nan,Electrolytes and Acid-Base Disorders 5476," A 47-year-old man who was a heavy drinker had received a diagnosis of liver dysfunction for the past 5 years. He drank eight to 10 beers per day, and lived alone, reportedly in poor circumstances. He visited the clinic when his family noticed gait instability. Vital signs: GCS: E4V5M6 (Eye response, opens eyes spontaneously; Verbal response, oriented, converses normally; Motor response, obeys commands); BP 120/ 60 mm Hg (supine position), pulse rate 70 beats/min, respiratory rate 14/min, SpO2 98% (room air), temperature 36.5 degree C, weight 59 kg (+2 kg compared with a month before), conjunctiva icteric Blood: total protein 6.3 g/dl, albumin 3.0 g/dl, BUN 6 mg/dl, creatinine 0.5 mg/dl, uric acid 5.5 mg/dl, Na+ 120 mmol/L, K+ 2.8 mmol/L, Cl- 70 mmol/L, glucose 80 mg/dl, osmolality 260 mOsm/kg H2O Urine: Na+ 18 mmol/L, K+ 18 mmol/L, osmolality 100 mOsm/kg H2O He received a diagnosis of chronic hyponatremia with mild symptoms (gait instability) due to beer potomania, and his serum [Na+] was corrected. After 24 hours, his serum [Na+] increased to 132 mmol/L, his K was 3.1 mmol/L, and he had no other symptoms..",Which ONE of the following is the MOST appropriate next step in management?,"1. Continue current treatment until his serum [Na+] reaches 135 mmol/L 2. B. Continue the current treatment until all symptoms are eliminated 3. Increase K+ replacement for his hypokalemia 4. Terminate the current treatment and observe the patient carefully 5. Terminate the current treatment and re-lower the serum [Na+] with 5% dextrose in water and desmopressin",E,temp nan,Electrolytes and Acid-Base Disorders 5477," A 38-year-old woman with no significant medical his-tory experienced severe nausea, vomiting, and diarrhea for 2 days. She was referred to the emergency department because of weakness and difficulty in food intake. Vital signs: GCS: E4V5M6 (Eye response, opens eyes spontaneously; Verbal response, oriented, converses normally; Motor response, obeys commands); BP 90/50 mm Hg (supine position), pulse rate 120 beats/min, respiratory rate 16/min, SpO2 99% (room air), temperature 37.8 degree C, body weight 55 kg (-4 kg). Blood: total protein 7.0 g/dl, albumin 3.8 g/dl, BUN 50 mg/dl, creatinine 1.2 mg/dl, uric acid 8.8 mg/dl, Na 128 mmol/L, K 3.5 mmol/L, Cl 90 mmol/L, glucose 80 mg/dl, osmolality 270 mOsm/kg H2O Urine: Na+ 15 mmol/L, K+ 15 mmol/L, osmolality 450 mOsm/kg H2O Volume depletion due to severe vomiting and diarrhea was diagnosed; normal saline was administered to restore her extracellular fluid volume. Subsequently, her condition improved (BP 110/70 mm Hg [supine position]), but serum [Na+] increased to 140 mmol/L within a day..",Which ONE of the following is the MOST likely cause of this patient’s serum [Na+] increase?,"1. Extracellular fluid volume replacement by normal saline infusion leading to increased free water excretion (aquaresis) 2. Fluid restriction due to inability to consume fluids orally 3. Increased water losses from sweating 4. Na+ repletion by normal saline infusion 5. Remission of nausea and vomiting, eliminating non-osmotic arginine vasopressin secretion",A,temp nan,Electrolytes and Acid-Base Disorders 5478," An 88-year-old man with syndrome of inappropriate antidiuretic hormone secretion caused by small-cell lung cancer was being treated with fluid restriction and urea, but his serum [Na+] did not improve sufficiently. Moreover, his diet-related quality of life was reduced by this treatment. Tolvaptan administration (15 mg/d) led to an appropriate improvement in serum [Na+] within the upper limit of correction. While initiating tolvaptan, the doctor repeatedly instructed the patient to drink water whenever he felt thirsty. At his initial follow-up visit, the patient reported he was drinking well, with good appetite and an improved diet-related quality of life. However, 2 weeks later, he visited a clinic because of lethargy. Vital signs: GCS: E4V5M6 (Eye response, opens eyes spontaneously; Verbal response, oriented, converses normally; Motor response, obeys commands); BP 100/ 60 mm Hg (supine position), pulse rate 65 beats/min, respiratory rate 14/min, SpO2 95% (room air), temperature 36.2 degree C, weight 70 kg (+2 kg) Blood: total protein 6.3 g/dl, albumin 3.0 g/dl, BUN 40 mg/dl, creatinine 0.9 mg/dl, uric acid 5.0 mg/dl, Na+ 160 mmol/L, K+ 4.5 mmol/L, Cl- 120 mmol/L, glucose 100 mg/dl, osmolality 350 mOsm/kg H2O Urine: Na+ 15 mmol/L, K+ 30 mmol/L, osmolality 120 mOsm/kg H2O Hypernatremia was diagnosed and 5% dextrose in water was administered..",Which ONE of the following is the MOST likely cause of this patient’s hypernatremia?,"1. Aquaresis produced by tolvaptan 2. Decreased thirst in response to hypertonicity in the elderly 3. Loss of appetite 4. Mistakenly continued fluid restriction 5. Partial improvement of syndrome of inappropriate antidiuretic hormone secretion because of treatment of small-cell lung cancer",B,temp nan,Electrolytes and Acid-Base Disorders 5479," A 59-year-old man with a medical history of type 2 diabetes mellitus, hypertension, coronary artery dis-ease, and bipolar disorder taking lithium is admitted to the intensive care unit with pneumonia and septic shock. On examination, the patient is sedated, receiving mechanical ventilation, with an FiO2 40%, positive end-expiratory pressure 8 cmH2O, BP 118/79 mm Hg, heart rate 91 beats/min. The results of heart and lung examination are unremarkable. There is no jugular venous distension, or peripheral edema. For the past 24 hours, his urine output was 1300 ml. Laboratory evaluation shows serum sodium 155 mmol/L, potassium 3.7 mmol/L, chloride 123 mmol/L, total CO2 25 mmol/L, BUN 24 mg/dl, serum creatinine 1.3 mg/dl (baseline creatinine 1 mg/dl), blood glucose 170 mg/dl, and urine osmolarity 629 mOsm/kg..",Which ONE of the following answers is MOST likely the correct cause of the patient’s hyponatremia?,"1. Hypernatremia is iatrogenic and requires intravenous or oral free water supplementation 2. Hypernatremia is secondary to lithium-induced partial nephrogenic diabetes insipidus 3. Hypernatremia is secondary to osmotic diuresis from hyperglycemia 4. Hypernatremia is secondary to lithium-induced complete nephrogenic diabetes insipidus",A,temp nan,Electrolytes and Acid-Base Disorders 5480," An 86-year-old woman with a medical history of hypertension, multi-infarct dementia, and atrial fibrillation presents from a nursing home with mental status changes to the emergency department. On presentation, her BP is 106/58 mm Hg, heart rate is 90 beats/min (irregular), respiratory rate is 22/min, oxygen saturation is 97% and weight is 60 kg. On examination she has dry mucous membranes, she is oriented to name only, her lung and heart examination results are unremarkable, and there is no leg edema. Laboratory evaluation shows serum sodium 176 mmol/L, potassium 3.5 mmol/L, chloride 129 mmol/L, bicarbonate 35 mmol/L, creatinine 1.5 mg/dl (base-line creatinine 1.2 mg/dl, and urine osmolarity 655 mOsm/kg..",Which is the correct statement regarding the rate of correction and amount of patient’s free water deficit?,"1. Assuming minimal ongoing free water losses, the infusion of 2 liters of free water would decrease the serum sodium concentration to approximately 155 mmol/L 2. Assuming 45% total body water content in this elderly woman, the free water deficit is 3.5 L 3. A change in the serum sodium concentration of 1 mmol/L/h is likely safe and should not cause neurologic sequelae. 4. Assuming minimal ongoing free water losses, the infusion of 4 liters of free water would decrease the serum sodium concentration to approximately 154 mmol/L",D,temp nan,Electrolytes and Acid-Base Disorders 5481," A 44-year-old man with a medical history of pulmonary sarcoidosis has his condition evaluated for increased thirst and urination of 2 weeks duration. He takes maintenance 10 mg of prednisone. He has no other contributory medical history and does not take other medications. On examination, BP is 125/74 mm Hg, heart rate is 72 beats/min, and respiratory rate is 15/min. The lungs and heart examination results are unremarkable. Laboratory evaluation shows serum sodium 147 mmol/L, potassium 3.8 mmol/L, chloride 110 mmol/L, BUN 16 mg/dl, creatinine 1.1 mg/dl, calcium 8.7 mg/dl, albumin 4 g/dl, urine sodium 20 mmol/L, and urine osmolarity 115 mOsm/kg. The urine output exceeds 6 liters per day..",What is the correct statement regarding the cause of and diagnostic workup for hypotonic polyuria?,"1. The indirect water deprivation test will accurately differentiate between central diabetes insipidus and primary polydipsia 2. Hypertonic saline infusion test and resultant copeptin release will accurately differentiate between central diabetes insipidus and primary polydipsia 3. In the setting of sarcoidosis, nephrogenic diabetes insipidus due to hypercalcemia is the most likely cause of hypernatremia 4. Copeptin levels after a water deprivation test will accurately differentiate between central diabetes insipidus and primary polydipsia",B,temp nan,Electrolytes and Acid-Base Disorders 5482," A 40-year-old woman with a medical history of schizophrenia presents to the emergency department with tonic-clonic seizures. She is intubated on arrival. On examination, BP is 179/88 mm Hg, and heart rate 137 beats/min. Laboratory evaluation shows serum sodium 183 mmol/L, serum osmolality 363 mOsm/kg, potassium 3.6 mmol/L, chloride 128 mmol/L, blood glucose 115 mg/dl, BUN 22 mg/dl, creatinine 1.0 mg/dl. The patient’s family reports that she might have ingested 500 ml of triple-strength Japanese seasoning soy sauce 3 to 4 hours before seizure onset..",Which of the following statements regarding the management of an acute salt intoxication is correct?,"1. Goal serum sodium correction should not exceed more than 10 mmol/L in 24 hours or 0.5 mmol/L per hour to avoid the development of cerebral edema 2. Cerebral edema is unlikely to occur in this setting of acute hypernatremia, and rapid correction of serum Na to baseline should be undertaken 3. Acute hypernatremia allows for the formation of idiogenic osmoles counterbalancing the osmotic gradient between extracellular and intracellular compartments 4. Administration of free water at high rates is sufficient to treat salt intoxication, and extracorporeal removal can cause too rapid a decrease in the serum sodium concentration",B,temp nan,Electrolytes and Acid-Base Disorders 5483," A 55-year-old man with a medical history of hypertension, type 2 diabetes mellitus, hyperlipidemia, and ESKD, receiving hemodialysis, is seen for a nephrology consultation in the neurologic intensive care unit, where he has been admitted for subarachnoid hemorrhage. The patient is intubated, is using minimal ventilatory settings, and does not require any sedation. His GCS score is 7. An external ventricular drain is placed, and the patient is given hypertonic saline infusion. BP 135/75, heart rate 67 beats/min. Laboratory evaluation shows sodium 147 mmol/L, potassium 6.0 mmol/L, chloride 108 mmol/l, total CO2 21 mmol/L, BUN 88 mg/dl, and creatinine 10.4 mg/dl. His last dialysis was performed 3 days ago. The patient is full code, and the family would like to pursue full medical and surgical management at this point..",What is the optimal decision in regard to the renal replacement therapy for this patient?,"1. Perform intermittent hemodialysis with regular blood and dialysate flows because the patient is starting to experience electrolyte abnormalities, and rising BUN might contribute to worsening mental status 2. Begin continuous venovenous hemodiafiltration with isotonic bicarbonate replacement fluid and discontinue hypertonic saline infusion 3. Begin continuous venovenous hemodiafiltration with standard sodium concentration dialysate and replacement fluid; at the same time continue hypertonic saline infusion to maintain permissive hypernatremia 4. Hold off renal replacement therapy at this time and manage the electrolyte abnormalities medically",C,temp nan,Electrolytes and Acid-Base Disorders 5484," A 22-year old woman is referred for hypokalemia. The patient’s history indicates progressive weakness over the past 3 weeks and weight gain despite exercise and following a healthy balanced diet. The patient does not describe having diarrhea or vomiting, and she does not have a history of using over-the-counter (OTC) medications or herbal supplements. Her body mass index (BMI) is 19 kg/m2, and BP is 120/80 mm Hg. The results of physical examination are unremarkable. Laboratory evaluation shows plasma sodium 133 mEq/L, plasma potassium 2.7 mEq/L, plasma chloride 83 mEq/L, plasma bicarbonate 34 mEq/L, plasma BUN 6 mg/dl, plasma creatinine 0.6 mg/dl, arterial blood gases pH 7.46 and PCO2 44 mm Hg, urine electrolytes (mEq/L) [Na+] 70, urine [K+}, urine [Cl-] <15, urine pH8..",What is the most likely cause of this patient’s hypokalemia?,"1. Early (“continuous”) vomiting 2. Magnesium deficiency 3. Long-term laxative use 4. Remote diuretic use",A,temp nan,Electrolytes and Acid-Base Disorders 5485," A 32-year-old man with a history of cystic fibrosis presents to the emergency department with progressive confusion and is unable to provide any medical history. His family informs that over the past few days, he had been painting a new roof in the heat of summer. On medication review, it is learned that the patient takes topiramate for migraine prophylaxis and inhaled tobramycin for Pseudomonas aeruginosa colonization. History is limited because of altered mental status. On examination, BP is 90/60 mm Hg and pulse is 100 beats/min. Mucous membranes are dry, and poor skin turgor is noted. Additional laboratory studies show plasma sodium 135 mEq/L, plasma potassium 2.2 mEq/L, plasma chloride 86 mEq/L, plasma bicarbonate 40 mEq/L, plasma BUN 56 mg/dl, plasma creatinine 2.3 mg/dl, plasma uric acid 12.2 mg/dl, plasma creatine kinase 676 U/L..",What is the most likely pathophysiologic explanation for this patient's severe hypokalemia?,"1. Inhibition of carbonic anhydrase in the proximal tubule 2. Activation of calcium-sensing receptor in the distal tubule 3. Metabolic alkalosis–induced transcellular shift of potassium out of the cells 4. Loss of potassium in the sweat glands from perspiration with extracellular fluid volume depletion and secondary hyperaldosteronism 5. Urinary loss of bicarbonate by the intercalated cells",D,temp nan,Electrolytes and Acid-Base Disorders 5486," A 36-year-old man is referred to nephrology clinic for evaluation of hypokalemia. The patient described progressive fatigue and leg cramps over the past year since initiating intermittent fasting and adhering to a vegetarian diet. He takes magnesium supplements for the leg cramps but denies using other OTC medications, herbal supplements, or illicit drugs. The patient is sexually active and is taking pre-exposure prophylaxis, emtricitabine, and tenofovir to prevent HIV. He is afebrile, BP is 120/80 mm Hg, pulse is 80 beats/min, and respiratory rate is 16/min. The results of physical examination are unremarkable. Laboratory data: Plasma sodium 138 mEq/L, potassium 3.4 mEq/L, chloride 114 mEq/L, bicarbonate 15 mEq/L, BUN 28 mg/dl, creatinine 0.6 mg/dl, glucose 92 mg/dl, uric acid 2.3 mg/dl, serum phosphorus 1.9 mg/dl. Urine studies: pH 6, 1+ protein, 2+ glucose, no erythrocytes or leukocytes. The patient was admitted, and an intravenous hypotonic saline solution (0.45% NaCl) containing 75 mEq sodium bicarbonate was initiated at a rate of 1 mEq/kg per h. Spot urine and simultaneous plasma electrolytes were obtained to measure fractional excretion of HCO3-..",What would you predict as the expected trend and values for pertinent laboratory data shown in the next series of choices?,"1. Plasma K+ ↓, Plasma HCO3- ↑, Urine pH ↑"", Fractional excretion of [HCO3-] 2% 2. Plasma K+ ↑, Plasma HCO3- ↑, Urine pH ↔, Fractional excretion of [HCO3-] 15% 3. Plasma K+ ↑, Plasma HCO3- ↓, Urine pH ↓, Fractional excretion of [HCO3-] 2% 4. Plasma K+ ↓, Plasma HCO3-↑, Urine pH ↑, Fractional excretion of [HCO3-] 15%",D,temp nan,Electrolytes and Acid-Base Disorders 5487," A 22-year-old woman is seen in a nephrology clinic for electrolyte abnormalities. The patient described having excessive muscle cramps after ballet lessons and indicates that she has been vigorously training for an upcoming dance competition. She denies current use of OTC medications, herbal supplements, or diuretics, and she specifically denies vomiting or diarrhea. There is a history of “idiopathic edema” for which she received hydrochlorothiazide 12.5 mg/d, but she categorically denies current use. Her sister has a history of Hashimoto thyroiditis, and her mother had a history of stroke. Her BP in the clinic is 110/60 mm Hg, and her BMI is 19 kg/m2. The results of physical examination are unremarkable. Blood chemistry: Plasma sodium 138 mEq/L, potassium 2.7 mEq/L, chloride 95 mEq/L, bicarbonate 32 mEq/L, creatinine 0.6 mg/dl, magnesium 1.3 mg/dl, aldosterone 20 ng/dl (high). Spot urine studies: K+/Cr ratio 32 mEq/g, Ca+2/Cr ratio 0.4mg/g (high), fractional excretion of Mg+2 3% (high)..",What is the most likely cause of hypokalemia?,"1. Abuse of laxatives 2. Surreptitious diuretic use 3. Herbal medicine containing glycyrrhizic acid 4. Remote vomiting 5. OTC thyroid supplementation",B,temp nan,Electrolytes and Acid-Base Disorders 5488," A 27-year-old man presents to a nephrology clinic for consultation. The patient was seen in the emergency department of a local hospital with profound muscle weakness, and a [K+] of 2.3 mEq/L was reported. The patient’s condition improved, and the discharge plasma [K+] was 4.1 mEq/L. This same sequence of events occurred 3 weeks later, and he is referred by the emergency department physician for evaluation of the recurrent hypokalemia. He has been describing increased salt-craving and frequent muscle cramps. His medical history is significant for depression treated with selective serotonin reuptake inhibitor antidepressants. He denies the use of any OTC medications or herbal supplements. He has been taking magnesium oxide 400 mg four times a day and potassium chloride 20 mEq two times a day for several years. The patient is afebrile, with BP 100/70 mm Hg, pulse 78 beats/min, and respiratory rate 16/min. He appears euvolemic on examination. No sensory deficits are noted, and muscle strength in the upper and lower extremities is 2/4. Electrocardiogram reveals “U” waves in limb leads II and V1. The plasma electrolytes at the time of this visit: Plasma sodium 139 mEq/L, potassium 2.7 mEq/L, chloride 95 mEq/L, bicarbonate 29 mEq/L, creatinine 0.7 mg/dl, magnesium 1.1 mEq/L. The patient is admitted to the hospital for diagnostic studies and treatment of hypokalemia. Spot urine studies obtained after stabilization are as follows: K+/Cr ratio 25 mEq/g, Ca+2/Cr ratio 0.04 mg/g, fractional excretion of Mg+2 5% (high)..",What is the most likely cause of hypokalemia and hypomagnesemia?,"1. Mutation in CLDN16 gene encoding claudin-16 2. Mutation in SLC12A3 gene encoding NCC in the distal tubule 3. Mutation in KCNJ1 gene encoding ROMK in collecting duct 4. Mutation in SLC12A1 gene encoding for NKCC2 in thick ascending limb of Henle",B,temp nan,Electrolytes and Acid-Base Disorders 5489," You are following up a 71-year-old man for CKD. He has long-standing diabetes and hypertension. His eGFR is 32 mL/min per 1.73, and his microalbumin-to-creatinine ratio is 1.7 grams. His serum potassium is 5.8 mEq/L and takes lisinopril 10 mg daily..",Which of the following is the best approach to the management of his hyperkalemia?,"1. Leave his regimen unchanged as recent studies have shown that low potassium confers higher cardiovascular risk and mortality than does high potassium 2. Avoid potassium-binding agents as they all have gastrointestinal side effects and are risky for long-term use 3. Initiate patiromer to allow continued treatment with renin-angiotensin system inhibitors. 4. Add fludrocortisone to reduce potassium in this patient",C,temp nan,Electrolytes and Acid-Base Disorders 5490," A 16-year-old boy is referred for evaluation of hyperkalemia detected during emergency department evaluation of muscle weakness. He notes frequent episodes of muscle weakness since childhood, associated with physical exertion or exposure to cold. Episodes of weakness have worsened as he has gotten older and are associated with muscular pain after episodes. During the most recent episode his potassium was 6.5 mEq/L and was associated with electrocardiographic changes, with normal chloride, sodium, and bicarbonate. His condition was subsequently evaluated by his pediatrician, who noted normal potassium of 4.5 mEq/L and normal renin and aldosterone concentrations. The patient notes that his late father used to describe the exact same problem, as do his uncle and cousin. In addition, his younger sister is starting to have the same symptoms..",What is the most likely underlying cause of hyperkalemia?,"1. A loss of function mutation of the gene encoding the epithelial sodium channel 2. A mutation of WNK (with no lysine) kinase resulting in increased expression of the sodium-chloride (NCCT) cotransporter and inhibition of potassium channels in principal cells 3. A mutation in the SCNA4 gene for the alpha subunit of the skeletal muscle cell sodium channel 4. A mutation in genes encoding proteins that modulate WNK activity, including Kelch-like 3 and Cullin 3",C,temp nan,Electrolytes and Acid-Base Disorders 5491," You are invited to speak at a symposium sponsored by the Federal Emergency Management Agency (FEMA) on emergency preparedness in urban centers. A FEMA council member has heard that hyperkalemia is a major cause of death of victims with crush injury sustained as a result of being buried under rubble in earthquakes. She asks you how to best determine hyperkalemia, particularly when there are multiple victims and time to is limited..",What is your response?,"1. Measuring potassium in serum is faster than measuring potassium in whole blood 2. A deep learning algorithm using an electrocardiogram is able to exclude hyperkalemia with acceptable accuracy 3. Serum potassium is usually lower than plasma potassium 4. Measuring potassium in whole blood is not as accurate as measuring potassium in serum or plasma",B,temp nan,Electrolytes and Acid-Base Disorders 5492," A 79-year-old woman with ESKD routinely presents on Tuesday mornings with predialysis potassium 6.8 mEq/L. She undergoes dialysis three times weekly for 4 hours via a fistula. Her Kt/V is 1.7..",Which of the following is most likely to be an evidence-based approach to her hyperkalemia?,"1. Sodium zirconium cyclosilicate should not be chosen as it is effective in reducing potassium in non-dialysis CKD patients but not in dialysis patients 2. Sodium zirconium cyclosilicate should be chosen for this patient rather than patiromer as it is less likely to cause edema 3. Sodium zirconium cyclosilicate also removes hydrogen ions and may increase serum bicarbonate 4. Sodium zirconium cyclosilicate works quickly and has been shown to provide added benefit to insulin and glucose in the treatment of acute severe hyperkalemia",C,temp nan,Electrolytes and Acid-Base Disorders 5493," A 74-year-old man with type II diabetes mellitus and heart failure with reduced ejection fraction presents to the emergency department with a 1-week history of fatigue, vomiting, headache, and abdominal pain. He is found to have AKI, with creatinine increased to 2.3 mg/dl from baseline 1.2 mg/dl. Other laboratory results show Na+ 134 mEq/L, K+ 4.0 mEq/L, Cl- 98 mEq/L, HCO3- 15 mmol/L, BUN 68 mmol/L, glucose 196 mmol/L. Lactate is 1.5 mEq/L. On examination, he is in hemodynamically stable condition, with a mildly tender abdomen, but the results of physical examination are otherwise unremarkable. For his diabetes, he takes insulin and another oral antidiabetic medication that he cannot recall the name of. He reports that his blood sugars have been very well controlled, such that his insulin was recently reduced by 50%..","In considering this case, which of the following statements is most accurate?","1. Metformin is the most likely culprit agent for acidosis in this case 2. The recent reduction in insulin dosing may be the precipitant for this clinical presentation 3. The patient’s near-normal blood glucose excludes ketoacidosis as a cause of this presentation 4. This patient should be given an intravenous infusion of isotonic bicarbonate",B,temp nan,Electrolytes and Acid-Base Disorders 5494," The internal medicine residency director asks you to lead a teaching session for medicine residents on drugs and the kidney. She references a patient recently admitted with zidovudine-induced metabolic acidosis and asks that you include an overview of drug-induced acidosis..","With regard to drug-induced metabolic acidosis, which of the following is correct?","1. Topiramate is associated with urine crystals and with crystalline stone formation 2. Valproate is highly protein bound and is not amenable to removal with hemodialysis 3. Concomitant use of antibiotics has been associated with increased risk of pyroglutamic acidosis from acetaminophen use 4. Linezolid causes a non-anion gap by blocking carbonic anhydrase",C,temp nan,Electrolytes and Acid-Base Disorders 5495," A 37-year-old male patient with CKD stage IV comes for clinic follow-up. At last review, [HCO3] had decreased to 19 mEq/L, and NaHCO3 650 mg twice daily was initiated. Today, he reports abdominal bloating and some nausea after taking the medications. His BP is well controlled at 122/68 mm Hg, and on examination he has trace lower limb edema. He has read that NaHCO3 may promote fluid retention and is reluctant to continue taking it. He asks for an explanation why it is necessary..",Which of the following is correct with regard to treatment of metabolic acidosis in CKD?,"1. A meta-analysis of treatment with NaHCO3 for metabolic acidosis did not show any increase in requirement for antihypertensive or diuretic agents 2. Increased ammoniagenesis resulting from acidosis leads to increases in endothelin-1 (ET-1) levels 3. In renal transplant recipients, metabolic acidosis has been associated with increased risk of graft failure, but not cardiovascular events 4. Alkali therapy has been shown to reduce urine ET-1 and aldosterone levels",D,temp nan,Electrolytes and Acid-Base Disorders 5496," A 54-year-old woman with CKD stage III, eGFR 45 mL/min, secondary to IgA nephropathy comes to clinic for review. She also has type II diabetes mellitus, obesity with BMI 35 kg/m2, and hypertension. Her laboratory results show BUN 24 mmol/L, creatinine 1.8 mg/dl, Na+ 135 mEq/L, K+ 4.3 mEq/L, C-l 102 mEq/L, HCO3 23 mEq/L. Urine microalbumin/creatinine ratio is 200 mg/g. BP is 122/74 mm Hg while she takes valsartan 160 mg daily, and for her diabetes, she takes dapagliflozin and insulin and has an HbA1c of 6.8%. She is endeavoring to make lifestyle changes and lose weight to improve her overall health but asks if you have other suggestions that may reduce the risk of CKD progression..","With regard to acidosis in early-stage CKD, which of the following is correct?","1. Switching to a diet of alkali-producing fruits and vegetables may delay progression to hypobicarbonatemic acidosis 2. In observational studies, up to 40% of patients with CKD stage III have metabolic acidosis 3. Decreases in urine citrate suggest H+ retention in patients with CKD and have been shown to trend closely with changes in serum [HCO3-] 4. Inhibitors of the renin angiotensin system have been shown to be protective against the development of acidosis",A,temp nan,Electrolytes and Acid-Base Disorders 5497," A 66-year-old woman with a history of hypertension, CKD stage IV, and dyslipidemia is seen at renal clinic. On review of her laboratory results, total CO2 has decreased to 20 mEq/L. You advise her to increase her intake of alkali-generating fruits and vegetables, but she is reluctant to make dietary changes. As an alternative, you suggest that she start oral bicarbonate therapy..","Regarding the benefits of alkali therapy in CKD, which of the following is correct?","1. In patients with CKD stage III to stage V, treatment of metabolic acidosis has been associated with reduction in progression to ESKD 2. Bicarbonate therapy has not been associated with improvements in mortality in randomized controlled trials (RCTs) 3. Patients with uncontrolled hypertension and advanced heart failure have been enrolled in RCTs investigating the use of NaHCO3 in CKD 4. In an RCT, treatment with veverimer was associated with improved [HCO3-] but no significant improvement in physical function scores",A,temp nan,Electrolytes and Acid-Base Disorders 5498," A 22-year-old woman presents to her primary care physician with weakness and fatigue that has been worsening for the past few months. Her medical history is significant for mild depression. She does not smoke cigarettes daily but drinks alcohol on weekends. She is currently not sexually active. She denies recreational drug use. Medications include a daily multivitamin. Family history includes hypertension and diabetes in her mother and father. Physical examination: weight 52 kg, pulse 84 beats/min, BP 110/70 mm Hg, temperature 37 degreeC. Dentition is normal, no pallor, no icterus. Lungs are clear bilaterally. Heart reveals normal S1 and S2, 2/6 ejection systolic murmur at the left sternal border. Abdomen soft, bowel sounds present. Extremities have no edema, strong peripheral pulses. Neurologic examination reveals no focal problems. Leukocyte count 6000/µl; Hemoglobin 12.9 g/dl; Platelets 189,000/µl; Serum Sodium 142 mEq/L; Potassium 2.9 mEq/L; Chloride 98 mEq/L; Total CO2 38 mEq/L; BUN 8 mg/dl; Creatinine 0.6 mg/dl; Random glucose 98 mg/dl; Urine toxicology screen results are negative. Electrocardiogram reveals sinus rhythm. Urine electrolytes: spot urine K+ 50 mEq/L, Cl- 8 mEq/L, Na+ 52 mEq/L. Plasma renin activity is elevated. plasma aldosterone concentration is elevated..","On the basis of these findings, of the following, what is the most likely cause of the patient’s hypokalemic metabolic alkalosis?","1. Glucocorticoid-remediable hyperaldosteronism 2. Surreptitious vomiting 3. Gitelman syndrome 4. Diuretic abuse",B,temp nan,Electrolytes and Acid-Base Disorders 5501," A 65-year-old man presents with the sudden onset of severe weakness and fever developing over the previous 24 hours. The patient had been followed up by his primary care physician before admission, and earlier laboratory values and BP have all been normal. Relevant medical history is that the patient has had difficulty urinating, with urinary retention and nocturia. Previous workup has revealed benign prostatic hypertrophy. Early on the day of admission the patient awoke feeling weak and warm and experienced burning on urination. His temperature was elevated, and he came to the emergency department. Physical examination shows a patient who is pale and confused. Weight 55 kg, pulse 120 beats/min, BP 70/50 mm Hg, temperature 40 degrees C. Lungs are clear bilaterally. Heart reveals normal S1 and S2, abdomen is soft with some supra-pubic tenderness and right flank pain tenderness. Peripheral pulses are noted to be weak, extremities are cold, and skin is mottled. Neurologic examination is noteworthy for a depressed mental status. Initial laboratory values: Leukocyte count 12,000/µl; Hemoglobin 13.3 g/dl; Platelets 120,000/µl; Serum Sodium 138 mEq/L; Potassium 4.0 mEq/L; Chloride 103 mEq/L; Total CO2 6 mEq/L; BUN 45 mg/dl; Creatinine 2.0 mg/dl;mRandom glucose 98 mg/dl. Workup reveals arterial blood gases with PO2 90 mm Hg, pH 7.06, PCO2 18 mm Hg, [HCO3-] 5 mEq/L, and serum lactate 20 mEq/L. The patient is given broad-spectrum antibiotics and because of the low pH [HCO3-] is administered (5% dextrose in half normal saline with 75 mEq/L NaHCO3) Over the next 2 days, the patient’s hemodynamics stabilize, and arterial blood gases show PO2 92 mm Hg, pH 7.47, PCO2 48 mm Hg, [HCO3-] 34 mEq/L. Urine pH is 8..","On the basis of the patient’s course, which of the following is true regarding the patient’s metabolic alkalosis?","1. It is due to Gitelman syndrome that was not apparent on admission 2. It is due to primary hyperaldosteronism that was not apparent on admission 3. It was generated by metabolism of retained lactate to bicarbonate and is now maintained by volume contraction 4. It was generated by metabolism of retained lactate to bicarbonate and should spontaneously correct",D,temp nan,Electrolytes and Acid-Base Disorders 5502," A 45-year-old woman presents with new onset of hypertension but is otherwise healthy. Physical examination results are is noteworthy for a BP of 160/105 mm Hg and are otherwise normal, with no evidence of volume depletion or edema. Initial lab results: Leukocyte count: 5,000/µl; Hemoglobin 13.9 g/dl; Platelets 230,000/µl; Serum Sodium 145 mEq/L; Potassium 2.5 mEq/L; Chloride 97 mEq/L; Total CO2 34 mEq/L; BUN 8 mg/dl; Creatinine 0.6 mg/dl; Random glucose 98 mg/dl A random plasma renin is low, and a plasma aldosterone is high..","On the basis of these findings, what is the most likely diagnosis?","1. Gitelman syndrome 2. Renovascular hypertension 3. Primary hyperaldosteronism 4. Liddle disease",C,temp nan,Electrolytes and Acid-Base Disorders 5503," A 35-year old man presents with new onset of hyper-tension but is otherwise healthy. History was noncontributory with the exception of extensive use of chewing tobacco. The results of physical examination are noteworthy for a BP of 155/110 mm Hg and are otherwise normal, with no evidence of volume depletion or edema. Laboratory values: Leukocyte count 5,500/µl; Hemoglobin 14.2 g/dl; Platelets 200,000/µl; Serum Sodium 145 mEq/L; Potassium 2.3 mEq/L; Chloride 96 mEq/L; Total CO2 35 mEq/L; BUN 8 mg/dl; Creatinine 0.6 mg/dl; Random glucose 98 mg/dl Random plasma renin and aldosterone are low. Cortisol levels are normal..","On the basis of these findings, what is the most likely diagnosis?","1. Surreptitious diuretic abuse 2. Syndrome of apparent mineralocorticoid excess due to glycyrrhizic acid 3. Primary hyperaldosteronism 4. Villous adenoma",B,temp nan,Electrolytes and Acid-Base Disorders 5504," Your hospital CEO is concerned about the high cost and length of stay of patients who experience AKI. She asks you about possible measures to decrease length of stay among AKI patients..","Of the following, what single measure has been recently shown to decrease AKI-related length of stay?","1. Fewer measurements of serum creatinine, particularly after procedures 2. Use of cystatin C rather than serum creatinine 3. Implementation of hospital-wide clinical support system that alerts clinicians that the patient has AKI 4. Use of artificial intelligence–driven algorithms to deter-mine dosage of antibiotics",C,temp nan,Acute Kidney Injury & Critical Care Nephrology 5505," At a board meeting for a local community hospital, the chief of cardiothoracic surgery asks you if there is any way to identify patients undergoing kidney “angina” similar to cardiac patients..","Based on published data, which of the following biomarkers has the best chance of identifying incipient acute tubular necrosis in a patient undergoing cardiac bypass?","A. Neutrophil gelatinase-associated lipocalin (NGAL) B. Kidney injury molecule (KIM)-1 C. (Urinary tissue inhibitor of metalloproteinases-2) • (insulin-like growth factor-binding protein 7) (TIMP-2) • (IGFBP7) D. n-acetyl-beta-d-glucosaminidase (NAG)",C,temp nan,Acute Kidney Injury & Critical Care Nephrology 5506," In your role as an internal medicine consultant at a community hospital, you are asked to see a patient for preoperative clearance for open reduction and internal fixation of a fractured hip. The patient is a 69-year-old white man who was admitted overnight to a surgical intensive care unit (ICU). He has a medical history of type 2 diabetes mellitus and cardio-vascular disease, and he underwent angioplasty 4 years prior. He is a smoker. On examination he is morbidly obese and is normotensive. He has a serum creatinine of 0.82 mg/dl and a microalbumin-to-creatinine ratio of ~800 mg/g..",Which clinical feature is characteristic of patients who experienced AKI in recently published ASSESS-AKI?,"A. Smoking status B. Obesity C. Proteinuria D. Ethnicity",C,temp nan,Acute Kidney Injury & Critical Care Nephrology 5507," You are consulting on a 49-year-old woman who was recently discharged from the hospital after an admission for sepsis and AKI in the setting of diverticulitis. She has a medical his-tory of mild asthma and seasonal allergies but has been otherwise healthy. Her baseline creatinine before admission was 0.7 mg/dl. She presented to hospital with 4 days of severe abdominal pain and oligoanuria. She was hypotensive on presentation and was treated with continuous venovenous hemo-filtration for 2 days. Her creatinine was 1.9 mg/dl by the time of discharge. She asks you, what are the chances of her having CKD and ever needing dialysis again..",Which of the following best predicts her risk of CKD?,"A. Having had kidney replacement therapy (KRT) for 2 days B. Oligoanuria on presentation to the hospital C. Creatinine at the time of discharge D. Creatinine and urine protein 3 months after discharge",D,temp nan,Acute Kidney Injury & Critical Care Nephrology 5508," You are consulting on a 62-year-old woman who had AKI in the setting of upper gastrointestinal bleeding from a perforated peptic ulcer. Her creatinine increased from a baseline of 0.7 to 2.3 mg/dl and is 1.8 mg/dl before discharge. Her creatinine 1 year later is 1.5 mg/dl, with estimated GFR of 37 ml/min per 1.73 m2..",Which of the following is true regarding the effect of AKI on mortality?,"A. Mortality is increased with severe (i.e., stage 3 AKI) but not stages 1 and 2 B. The increased long-term mortality associated with AKI is due to dialysis complications C. Mortality is increased in a graded fashion with worsening AKI D. Mortality is increased only in KRT-requiring AKI",C,temp nan,Acute Kidney Injury & Critical Care Nephrology 5509," A 24-year-old man presents to the emergency department with septic shock from Escherichia coli bacteremia related to urinary source. His vital signs are as follows: temperature 38 degree C, heart rate 119 beats/min, BP 82/43 mm Hg, respirations 19/min. Physical examination results are significant for ill appearance, tachycardia, and cool peripheries. His laboratory results are as follows: creatinine 1.1 mg/dl, sodium 134 mmol/L, calcium 7.2 mg/dl, potassium 4.5 mmol/L, albumin 3 g/dl, chloride 98 mmol/L, arterial pH 7.29, bicarbonate 14 mEq/L, pCO2 30 mm Hg, BUN 22 mg/dl..",What is the preferred resuscitative fluid based on recent studies?,"A. 5% dextrose in water with 150-mEq sodium bicarbonate B. 0.9% sodium chloride C. 0.45% sodium chloride D. Lactated Ringer’s",C,temp nan,Acute Kidney Injury & Critical Care Nephrology 5510," A 75-year-old woman with diabetes, hypertension, current tobacco use, and peripheral artery disease is admitted to the hospital with chest pain. Cardiology is recommending a cardiac catheterization because of her risk stratification. She is scheduled to undergo the procedure tomorrow..",What strategies can you implement to prevent AKI?,"A. Check urine sodium and risk-stratify for risk of contrast-induced-AKI B. Delay cardiac catheterization due to high risk of having CI-AKI C. Give 0.9% sodium chloride before and after cardiac catheterization D. Give D5 with 150-mEq sodium bicarbonate before cardiac catheterization",C,temp nan,Acute Kidney Injury & Critical Care Nephrology 5511," You are chief quality officer for your institution. It has been brought to your attention that there is a rising incidence of AKI after cardiac surgeries, which has led to longer hospital length of stay, increased complications, and mortality. After reviewing the data, you want to initiate possible interventions that can lower the risk but also be cost effective..",What strategy has been suggested by the PrevAKI randomized controlled trial to prevent AKI in this setting and be cost effective?,"A. Work with Information Technology on developing E-alerts for surgeons alerting them to patients at high risk of AKI B. Implement the KDIGO AKI bundle with use of (TIMP-2) • (IGFBP7) C. Develop smart order sets with automatic preoperative and postoperative hydration with intravenous 0.9% saline solution D. Provide education to surgeons on the nephrotoxicity of loop diuretics",B,temp nan,Acute Kidney Injury & Critical Care Nephrology 5512," A 74-year-old man without known CKD disease undergoes transcatheter aortic valve replacement because of symptomatic severe aortic stenosis. The procedure is uneventful. On postoperative day 2, his serum creatinine is 1.2 mg/dl (baseline 0.8 mg/dl) and he is asymptomatic. The next day his serum creatinine is 1.0 mg/dl and he is discharged with a clinical appointment scheduled for the following week..",Which ONE of the following statements is correct concerning transcatheter aortic valve replacement and AKI?,"A. AKI after transcatheter aortic valve replacement is rare, occurring in <10% of patients B. The volume of iodinated contrast media used during the procedure is associated with the development of AKI C. At 30 days after hospital discharge, fewer than a third of the patients have not fully recovered kidney function and still have acute kidney disease D. Increased length of stay was observed in AKI patients; however, long-term follow-up demonstrated similar 7- year mortality in AKI versus non-AKI patients E. AKI stages 1 and 2 occur with equal frequency in patients undergoing transcatheter aortic valve replacement, whereas <10% of patients experience AKI stage 3",C,temp nan,Acute Kidney Injury & Critical Care Nephrology 5513," A 59-year-old woman presents to the emergency room with chest pain and promptly receives a diagnosis of ST-elevation myocardial infarction. Her clinical condition quickly deteriorates, she experiences a mean arterial pressure < 60 mm Hg, she has cold extremities, and she is confused..","Her urine output 24 hours after admission is 200 ml, an equivalent of 0.12 ml/kg per h. Which ONE of the following statements is correct regarding AKI development in cardiogenic shock?","A. Using a serum creatinine criterion is more sensitive than urine output criterion in the diagnosis of AKI in cardio-genic shock B. In patients with cardiogenic shock who undergo coronary angiography, the amount of iodinated contrast medium is not associated with the development of AKI C. AKI stage 1 patients have increased risk of death when compared with non-AKI patients D. More than 20% of patients with cardiogenic shock require KRT E. In patients with cardiogenic shock, those with higher values of central venous pressure and lower values of mean arterial pressure are more prone to the development of AKI",E,temp nan,Acute Kidney Injury & Critical Care Nephrology 5514," You are consulted by the Department of Cardiovascular Surgery at your institution because the department wants to incorporate new technologies in their practice that could reduce complications after cardiac surgeries. Which ONE of the following statements regarding prevention of cardiovascular surgery-associated AKI is MOST accurate? A. The urinary test (TIMP-2) • (IGFBP7) can be applied in patients undergoing cardiopulmonary bypass as a screening tool to select those at high risk for AKI who might benefit from an AKI bundle; this strategy has been demonstrated to reduce the prevalence of severe AKI B..","Current guidelines recommend the use of (TIMP-2) • (IGFBP7) in patients undergoing cardiac surgery to enable early detection of those in whom AKI will develop C The use of cell cycle arrest biomarker neutrophil gelatinase-associated lipocalin (NGAL) can discriminate cardiac surgery patients at higher risk for AKI who might benefit from measures such as early dialysis initiation","A. The urinary test (TIMP-2) • (IGFBP7) can be applied in patients undergoing cardiopulmonary bypass as a screening tool to select those at high risk for AKI who might benefit from an AKI bundle; this strategy has been demonstrated to reduce the prevalence of severe AKI B. Current guidelines recommend the use of (TIMP-2) • (IGFBP7) in patients undergoing cardiac surgery to enable early detection of those in whom AKI will develop D. In patients without CKD undergoing elective cardiac surgery, a measured renal functional reserve cutoff value of < 30 ml/min per 1.73 m2 is applied to discriminate patients at risk for the development of AKI E. Despite growing evidence of its potential role in preventing cardiac surgery-associated AKI, the urinary test (TIMP-2) • (IGFBP7) still lacks approval by the US Food and Drug Administration",A,temp nan,Acute Kidney Injury & Critical Care Nephrology 5515," You have been assigned by the head of the Emergency Department to create a flowchart for diuretic use in acute heart failure. Which ONE of the following statements represents the approach endorsed by current guidelines?.","A. Furosemide dose should not exceed 300 mg in 24 hours B If decongestive goals are not met with the use of furosemide, the next pharmacologic step consists of adding a thiazide or thiazide-like agent such as metolazone or chlorthalidone","A. Furosemide dose should not exceed 300 mg in 24 hours C. In patients with residual interstitial congestion (rales, pitting edema, pleural effusion) and a predominant phenotype of tissue edema, the use of the selective V2 antagonist tolvaptan or of a sodium-glucose linked transporter-2 inhibitor may promote aquaresis, decreasing intravascular compartment osmolality and increasing plasma refilling D. After 6 hours of the first dose of loop diuretic if urine output is <100 ml/hour, the dose should be increased by 30% E. When continuous infusion of furosemide is used, a bolus loading dose should be avoided to reduce the risk of ototoxicity",B,temp nan,Acute Kidney Injury & Critical Care Nephrology 5516," A 38-year-old-man with untreated hepatitis C virus infection and known cirrhosis, diabetes, and hypertension presents with oliguric AKI with creatinine of 1.4 mg/dl (baseline is 0.8 mg/dl), altered mental status, hypotension, and fever..",Which ONE of the following would EXCLUDE the diagnosis of hepatorenal syndrome outlined by the International Club of Ascites in patients with cirrhosis and ascites?,"A. Serum creatinine <1.5 mg/dl B. Treated bacterial sepsis C. Untreated spontaneous bacterial peritonitis with septic shock D. Fractional urinary sodium excretion <1% E. No improvement in serum creatinine after ascitic drainage",C,temp nan,Acute Kidney Injury & Critical Care Nephrology 5517," You have been treating a 48-year-old man with severe hypertension, metabolic syndrome, gout with lisinopril, chlorthalidone, and amlodipine. You are aware that he has had several admissions for alcohol-related pancreatitis and recurrent hepatitis and blood pressure has been steadily decreasing. He calls to tell you that his weight is increasing but his BP is “at target” even though he no longer takes his medication..",Which ONE of the following statements is incorrect about the pathogenesis of the hepatorenal syndrome?,"A. Plasma angiotensin concentrations are increased B. Visceral sympathetic nervous system activity is increased C. Portal blood flow is reduced D. Systemic BP pressure is reduced E. Plasma vasopressin concentrations are increased",C,temp nan,Acute Kidney Injury & Critical Care Nephrology 5518," A 94-year-old man is admitted with cirrhosis, ascites, and a 40-pound weight loss in the setting of hepatocellular cancer. He is oliguric, though not anuric, and his creatinine is 1.4. mg/dl. You do not have access to a prior baseline creatinine yet. He is severely anemic, and his phosphorus is 2.8 mg/dl. He has marked asterixis, and you are asked if this is due to hepatic encephalopathy, or could it possibly be uremia. Your medical student wishes to know the best way to measure the kidney function..",Which ONE of the following statements is correct about using serum creatinine to assess renal function patients with hepatorenal syndrome?,"A. Estimated GFR measurements are an accurate measure of renal function B. The proportion of renal tubular creatinine secretion is increased C. Cystatin C is a better marker for renal function than serum creatinine D. Increasing bilirubin increases serum creatinine measurement E. Muscle wasting increases creatinine production",B,temp nan,Acute Kidney Injury & Critical Care Nephrology 5519," A 49-year-old man has a long-standing history of ethanol-related cirrhosis. He has marked ascites and requires repeated paracentesis. He presents with upper gastrointestinal bleeding in a setting of known esophageal varices. He experiences AKI, and his creatinine remains elevated despite control of bleeding and restoration of hemodynamic parameters..",Which ONE of the following treatments has the greatest impact on survival of patients with hepatorenal syndrome?,"A. Continuous renal replacement therapy B. Octreotide and midodrine C. Terlipressin and albumin D. Treatment with molecular adsorbent recirculating system (MARS) E. Liver transplantation",E,temp nan,Acute Kidney Injury & Critical Care Nephrology 5520," A 64-year-old man presents to his primary care practitioner with a history of weight loss, poor appetite, increasing lower limb edema, and worsening fatigue. Routine laboratory studies reveal Hb 7.5g/dl, BUN 97 mg/dl, creatinine 4.65 mg/dl, calcium 12.5 mmol/L, and albumin 2.6 g/dl. The patient is admitted to the hospital for further evaluation and is found to have a circulating IgG kappa paraprotein. He under-goes renal biopsy, and the results of analysis of the biopsy sample are consistent with a light chain crystalline cast nephropathy. Over the subsequent days, his AKI progresses, and he experiences oligoanuric AKI..",Which ONE of the following is the most appropriate initial treatment for a patient with dialysis-dependent light chain crystalline cast nephropathy?,"A. A course of plasmapheresis B. Peritoneal dialysis C. Clone directed therapy of the paraproteinemia D. A course of high-cutoff hemodialysis E. Hematopoietic stem cell transplantation",C,temp nan,Acute Kidney Injury & Critical Care Nephrology 5521," A 45-year-old woman is referred to you for assessment of recurrent renal calculi and progressive CKD. She undergoes renal biopsy for further evaluation. The result of analysis of the biopsy specimen is notable for moderate tubulointerstitial fibrosis, and the specimen shows reddish-brown crystals on hematoxylin and eosin staining that are birefringent under polarized light..","Which ONE of the following is the most appropriate treatment for a patient with crystalline nephropathy due to 2,8-dihydroxyadeninuria?","A. Intravenous fluids and loop diuretic B. Infusion of sodium bicarbonate to alkalinize the urine C. Oral potassium citrate D. Low-purine diet and allopurinol E. Oral pyridoxine supplementation",D,temp nan,Acute Kidney Injury & Critical Care Nephrology 5522," A 42-year-old woman is admitted to gynecology with a tubulo-ovarian abscess complicating recent surgery for an ovarian tumor and is treated conservatively with clarithromycin. Four days into her clarithromycin course, she experiences dermatologic manifestations suggestive of Stevens-Johnson syn-drome and concurrently experiences acute sepsis (although she is not in septic shock) and AKI. Laboratory studies show serum sodium 138 mEq/L, potassium 6.0 mEq/L (no hyperkalemic wave changes in electrocardiography), chloride 117 mEq/L, serum HCO3- 18 mmol/L, BUN 57 mg/dl, and creatinine 3.1 mg/dl (raised from 1.2 mg/dl on admission). Over the previous 12 hours, her urine output was 0.47 ml/kg per hour. The result of urinalysis is unremarkable. Fractional excretion of sodium is 0.9%. The patient is transferred to the ICU because of imminent concerns that she may require KRT. She has remained hemodynamically stable when transferred to ICU, and laboratory studies are repeated on her arrival in the ICU 3 hours later, with sodium 139 mEq/L, potassium 5.9 mEq/L, chloride 117 mEq/L, total serum HCO3- 18 mmol/L, BUN 57 mg/dl, and creatinine 3.0 mg/dl..",Which ONE of the following is the BEST option regarding treatment of this patient at this point in the KRT initiation strategy perspective?,"A. There are current indications to commence KRT immediately B. KRT should ideally be commenced within the next 12 hours C. Continue supportive medical management and step down back to ward-level care; patient is unlikely to require KRT D. Close and frequent monitoring of clinical status and lab-oratory parameters in the ICU and initiate KRT if indication arises",D,temp nan,Acute Kidney Injury & Critical Care Nephrology 5523," A 61-year-old man is admitted to the ICU for postoperative recovery after a right-sided total hip replacement, which was completed as an emergency procedure after the patient sustained a traumatic fall at home. The patient had clinical deterioration in the ICU, where he became hemodynamically unstable, with mean arterial pressure (MAP) <65 mm Hg, and required inotropic support, which improved his MAP. Infection and inflammatory markers were only mildly raised since his ICU admission, and he did not have signs of pyrexia. No acute postsurgical wound complications were found. However, the patient experienced severe AKI, and urine output for the past 12 hours was 0.20 ml/kg per hour. Laboratory testing revealed serum sodium 132 mEq/L, potassium 6.3 mEq/L (no hyperkalemic wave changes in electrocardiography), chloride 110 mEq/L, total HCO3- 16 mmol/L, BUN 62 mg/dl, and creatinine 4.1 mg/dl (raised from 1.5 mg/dl on admission). The result of urinalysis was unremarkable. Fractional excretion of sodium was 0.8%. KRT initiation was recommended..","Which ONE of the following is the BEST option for KRT initiation for this man, considering the most recent 2019 Kidney Disease Improving Global Outcomes “Controversies in AKI” Conference Report suggestions?","A. Continuous kidney replacement therapy (CKRT) (24 hours/day) B. Intermittent hemodialysis (IHD) (4 hours x 3 days/week) C. Prolonged intermittent kidney replacement therapy (PIKRT) (8 hours/day) D. Peritoneal dialysis (commencing on short cycle times of 1–2 hours)",A,temp nan,Acute Kidney Injury & Critical Care Nephrology 5524," A 56-year-old woman who is morbidly obese, with a medical history of nonalcoholic fatty liver disease and primary hyperparathyroidism with parathyroidectomy a year ago, is admitted to the ICU with symptoms consistent with acute pancreatitis, where supportive medical management with intravenous fluids, analgesia, and nutritional support were administered. Unfortunately, she has deteriorated clinically and experiences AKI. Urine output for the past 12 hours was 0.25 ml/kg per hour. Laboratory testing revealed sodium 141 mEq/L, potassium 6.4 mEq/L (no hyperkalemic wave changes in electrocardiography), chloride 111 mEq/L, total HCO3- 15 mmol/L, BUN 60 mg/dl, and creatinine 4.0 mg/dl (increased from 1.3 mg/dl on admission). The result of urinalysis is unremarkable, and fractional excretion of sodium is 0.9%. KRT initiation is recommended. Catheter access through ultrasound guidance is successfully obtained in the femoral site. Laboratory test results for clotting profile have been within normal ranges throughout..",Which ONE of the following is currently the BEST option for anticoagulation strategy in this patient during KRT?,"A. Regional citrate anticoagulation (RCA) B. Low molecular weight heparin (LMWH) C. Unfractionated heparin D. No anticoagulation required",A,temp nan,Acute Kidney Injury & Critical Care Nephrology 5525," A 55-year-old man presents to the emergency department with acute shortness of breath; a nasal swab for PCR for SARS-CoV-2 yields positive results. His medical history includes CKD, diabetes, and hypertension. Serum creatinine is 1.6 mg/dl, which is the patient’s baseline..",Which ONE of the following statements is most accurate for this patient?,"A. Risk of AKI peaks within 24 hours after admission to the emergency department B. AKI occurrence is highly predictive of deteriorating respiratory status C. If patient is given invasive ventilation, he is more likely to need KRT D. Diabetes is the single most important risk factor for AKI",C,temp nan,Acute Kidney Injury & Critical Care Nephrology 5526," You have been consulted for renal replacement therapy management in a 45-year-old woman with severe COVID-19 admitted to the ICU. Her medical history includes obesity and dyslipidemia. The patient is intubated, has severe acute respiratory distress syndrome, and is currently anuric. Potassium is 5.7 mEq/L, and BUN is 122 mg/dl..",Which ONE of the following statements is most accurate for this patient?,"A. Intermittent hemodialysis has been associated with worse renal outcome compared with continuous techniques B. Available data suggest that emergency start of peritoneal dialysis may associate with higher mortality, but could still be useful when limited dialytic resources are available C. Citrate anticoagulation is more efficient than any form of heparin therapy in prolonging circuit half-life D. Circuit clotting rate may be higher than 30% even when anticoagulation is started",D,temp nan,Acute Kidney Injury & Critical Care Nephrology 5527," A 32-year-old woman comes to the emergency department after a positive result from nasal swab PCR testing for SARS-CoV-2. The patient has no respiratory symptoms, a chest x-ray shows scattered hazy opacities, and a urinalysis shows no glucose, 10 to 15 red blood cells, 3 to 5 white blood cells, 2+ proteins, and no ketones. She reports no urinary symptoms, her last menstruation ended 6 days ago, and her serum creatinine is 0.7 mg/dl..",Which ONE of the following statements is most accurate for this patient?,"A. Urine abnormalities are uncommon in patients with COVID-19 without AKI and are highly predictive of glomerulonephritis B. Nephrotic-range proteinuria can be secondary to COVID-19 in patients who have high-risk APOL-1 alleles C. COVID-19 patients have higher risk of glomerulonephritis, including IgA nephritis and anti-GBM nephritis D. Diffuse microangiopathy is a frequent finding in autopsy and biopsy reports from COVID-19 patients",B,temp nan,Acute Kidney Injury & Critical Care Nephrology 5528," A 72-year-old man experiences severe dyspnea and cough 3 days after the onset of transient fever and myalgias and is diagnosed with COVID-19. He has a medical history of well-controlled diabetes and hypertension and has stage 3 CKD, with serum creatinine of 1.6 mg/dl, GFR of 45 ml/min per 1.73 m2, and estimated protein excretion of ~2.4 g/day. On presentation to the emergency department, his serum creatinine is 4.3 mg/dl, potassium is 5.9 mEq/L, and urine output is scant. His oxygenation declines overnight, and he requires invasive mechanical ventilation..",Which ONE of the following has the greatest effect on mortality?,"A. AKI B. Diabetes C. Hypertension D. Proteinuria",A,temp nan,Acute Kidney Injury & Critical Care Nephrology 5529," A 45-year-old man with IgA nephropathy, obesity, and smoking history presents with positive COVID-19 results, dyspnea, fever, and cough. He is hypotensive in the emergency department and is given vasopressors. His baseline creatinine is 1.4 mg/dl; it is 2.9 mg/dl on presentation and increases progressively. He is given intermittent hemodialysis on day 5. His wife asks you whether he will require dialysis when he leaves the hospital..",What percentage of patients require maintenance dialysis on discharge after being initiated in hospital for Covid-AKI?,"A. 10% to 15% B. 25% to 35% C. 60% to 70%",B,temp nan,Acute Kidney Injury & Critical Care Nephrology 5530," A 71-year-old woman with a history of stage IV non–small cell lung cancer presents to an urgent care center for assessment and management of AKI. She has received three cycles of cisplatin 75 mg/m2, pemetrexed 500 mg/m2, and the checkpoint inhibitor nivolumab 10 mg followed by one cycle of nivolumab alone. She presented for her fifth cycle of nivolumab when she was noted to have elevated serum creatinine. The patient's baseline serum creatinine after the last cycle of cisplatin and pemetrexed was 1.2 mg/dl. Her medical history is remarkable for hypothyroidism and dyspepsia. Her medications include levothyroxine 75 µg daily and omeprazole 20 mg twice a day. On admission, the patient reports some dysphagia to liquids but does not describe any nausea, vomiting, or diarrhea. She does not describe any lightheadedness or dizziness. Her BP is 146/80 mm Hg, pulse is 76, respiratory rate is 12/min. Her examination results are unremarkable. Urinalysis results are bland except for > 50 white blood cells per high-power field. Urinary sodium is 56 mEq/L, and urinary creatinine is 26 mg/dl. Her basic metabolic profile is as follows: sodium 136 mEq/L, potassium 4.4 mEq/L, chloride 105 mEq/L, carbon dioxide 20 mEq/L, BUN 32 mg/dl, creatinine 4.1 mg/dl, glucose 94 mg/dl, and calcium 8.4 mg/dl..",What is the MOST likely cause of AKI in this patient?,"A. Cisplatin nephrotoxicity B. Pemetrexed nephrotoxicity C. Prerenal cause resulting from volume depletion D. Immunotherapy related to acute interstitial nephritis",D,temp nan,Acute Kidney Injury & Critical Care Nephrology 5531," A 63-year-old woman with a history of metastatic breast cancer, recently treated with the VEGF inhibitor bevacizumab 10 mg/kg every 3 weeks, received seven cycles, but it was discontinued 1 month ago because of progression of disease. The patient was given a cyclin-dependent kinase 4/6 inhibitor, abemaciclib 100 mg twice daily at that time, and now presents for routine follow-up care. She has no complaints and takes no other oral medications; however, she has been receiving denosumab 120 mg subcutaneously every 4 weeks for management of bone metastases. Her baseline serum creatinine was 0.8 mg/dl 1 month ago. The patient’s vital signs are within normal limits, and examination results are unremarkable. Her laboratory results are as follows: sodium 141 mEq/L, potassium 4.2 mEq/L, chlorine 108 mEq/L, bicarbonate 28 mEq/L, BUN 18 mg/dl, creatinine 1.5 mg/dl, glucose 105 mg/dl, urinalysis is bland, and random urine protein/creatinine ratio is 0.3..",What should be the next step in the treatment of this patient?,"A. Kidney biopsy to rule out bevacizumab toxicity B. Stop denosumab C. Obtain cystatin C level, continue abemaciclib, and repeat laboratory studies in 1 week D. Stop abemaciclib",C,temp nan,Acute Kidney Injury & Critical Care Nephrology 5532," A 73-year-old woman who underwent hematopoietic stem cell transplantation 3 1/2 years ago for treatment of acute myelogenous leukemia presents to the clinic with new-onset edema. She reports the development of edema approximately 1 month ago, affecting her lower extremities up to her thighs and upper extremity involvement as well. Patient does not describe any nausea, vomiting, or diarrhea. She has been treated with tacrolimus and prednisone for management of liver graft-versus-host disease (GVHD) recently, but GVHD has resolved, her steroids were stopped, and the tacrolimus dose was tapered down to 0.5 mg daily with undetectable through levels about 6 weeks ago. Her medications include furosemide 20 mg daily, tacrolimus 0.5 mg daily, acyclovir 400 mg twice a day, and sulfamethoxazole-trimethoprim 800/160 mg three times a week. The patient’s BP is 138/72 mm Hg, pulse 82/min, and respiratory rate 12/min. The examination is remarkable for anasarca. Her laboratory results are as follows: sodium 137 mEq/L, potassium 4.4 mEq/L, chlorine 105 mEq/L, bicarbonate 27 mEq/L, BUN 18 mg/dl, creatinine 0.8 mg/dl, glucose 125 mg/dl, total protein 5.1 mg/dl, aspartate aminotransferase 52 mg/dl, alanine aminotransferase 34 mg/dl, albumin 2.0 mg/dl, white blood cells 5,400/µl, Hgb 9.8 g/dl, platelets 271,000/µl, urinalysis is bland except for 41 protein, spot urinary protein 4200 mg/dl, spot urinary creatinine 239 mg/dl..",What is the MOST likely cause of this patient’s nephrotic syndrome?,"A. Membranous nephropathy secondary to kidney GVHD B. Transplantation-associated thrombotic microangiopathy C. Sinusoidal obstruction syndrome D. Sulfamethoxazole-trimethoprim",A,temp nan,Acute Kidney Injury & Critical Care Nephrology 5533," You are asked to see a 49-year-old man for AKI. He has a history of non-Hodgkin B cell lymphoma and was recently admitted for chimeric antigen receptor (CAR) T therapy. Two days after receiving CAR T therapy, he experienced a temperature of 39 degrees C and “total body muscle aches.” His creatinine increased from his baseline of 0.8 mg/dl to a current value of 2.1 mg/dl. He is oliguric but not anuric. On examination he is mildly confused and agitated and has mild basilar rales..","Of the following, what are you MOST likely to see on analysis of urine sediment?","A. Pigmented granular casts B. Dysmorphic red blood cells and red blood cell casts C. White blood cells and white blood cell casts D. Oxalate crystals",A,temp nan,Acute Kidney Injury & Critical Care Nephrology 5564," You are evaluating a 34-year-old man who has a serum creatinine of 2.8 mg/dl. Using this creatinine, the Modification of Diet in Renal Disease study equation estimates a GFR of 34 ml/min per 1.73 m2 if he is Black, and 27 ml/min per 1.73 m2 if he is not Black..","He asks “Does my race make a difference?” What would be the effect of removing the race component from currently used GFR estimating equations?","A. Increase in the proportion of Blacks meeting criteria for a diagnosis of CKD B. Increase in the proportion of Blacks meeting GFR based eligibility for living kidney donation C. Increase in the proportion of Blacks eligible for specific medications (e.g., metformin, SGLT2 inhibitors) D. Decrease in the prevalence of CKD stages 4 and 5 among Blacks",A,temp nan,Chronic Kidney Disease 5565," A 49-year-old man with IgA nephropathy, eGFR 49 ml/min per 1.73 m2, and albumin-creatinine ratio (ACR) 750 mg/g asks you what his chances are of needing dialysis. Analyses from the Chronic Kidney Disease Prognosis Consortium (CKD-PC), a large international consortium of cohort studies, demonstrated which ONE of the following? A..","Lower eGFR predicts ESKD and mortality among individuals with CKD, but not in the general population","A. Lower eGFR predicts ESKD and mortality among individuals with CKD, but not in the general B. Greater urinary albumin excretion predicts ESKD and mortality among individuals with CKD, but not in the general population C. Lower eGFR and greater urinary albumin excretion predict ESKD and mortality among individuals with diabetes, but not among individuals without diabetes D. Lower eGFR and greater urinary albumin excretion predict ESKD and mortality among individuals with and without diabetes",D,temp nan,Chronic Kidney Disease 5566," A meta-analysis of observational studies found that an increase in urinary albumin excretion over 2 years was associated with a greater likelihood of subsequent ESKD. The authors concluded that change in urinary albumin excretion could potentially be used as a surrogate end point for clinical trials of interventions to prevent ESKD..",An accompanying meta-analysis of randomized controlled trials also found which ONE of the following?,"A. Results differed significantly in studies using total protein excretion rather than albumin excretion B. Urinary albumin excretion is a valid surrogate end point C. Associations of change in albumin-creatinine ratio/protein-creatinine ratio (ACR/PCR) with all-cause and cardiovascular mortality were stronger than associations with ESKD in studies of individuals with existing CKD D. A change in urinary albumin excretion was found to be much less strongly associated with ESKD in patients treated with renin-angiotensin-aldosterone system (RAAS) inhibitors than in those treated with other methods of intensive blood pressure control",A,temp nan,Chronic Kidney Disease 5567," A meta-analysis of observational studies found 0.75 ml/min per 1.73 m2 per year lower rate of GFR decline (i.e., less steep decline) over a 2-year baseline period was associated with a 30% lower risk of ESKD in participants with baseline eGFR greater than or equal to 60 ml/min per 1.73 m2 (adjusted hazard ratio (HR) = 0.70; 95% CI, 0.68 to 0.72) and a 29% lower risk of ESKD in those with a baseline eGFR < 60 ml/min per 1.73 m2 (adjusted HR = 0.71; 95% CI, 0.68 to 0.74)..",Which ONE of the following is true?,"A. The absolute difference in terms of incidence of ESKD (% of participants) with a treatment which causes a less steep decline in GFR would be expected to be similar for patient populations at low risk of ESKD and high risk of ESKD B. A meta-analysis of randomized trials confirmed the utility of a decrease in eGFR slope as a potential surrogate end point for clinical trials of interventions to prevent ESKD C. A very large patient population (> 10,000 participants) would be required to demonstrate benefit of an intervention for a study with a 0.75 ml/min per 1.73 m2 per year lower GFR decline over 2 years caused by the intervention D. Using a shorter baseline period to calculate GFR slope does not affect the ability to predict ESKD",B,temp nan,Chronic Kidney Disease 5568," A 67-year-old man with CKD stage G3A3 secondary to IgA nephropathy, and history of myocardial infarction is evaluated during a routine office visit. His urinary ACR is 500 mg/g and he is receiving a stable dose of angiotensin-converting enzyme inhibitor. He is concerned about his risk for ESKD and wishes to delay dialysis as long as possible. He recently learned about the emerging new therapies for kidney disease treatments on a blog..",Which ONE of the following treatment is supported by current evidence?,"A. Atrasentan 0.75 mg/d B. Dapagliflozin 10 mg/d C. Semaglutide 0.25 mg once a week D. Finerenone 20 mg/d",B,temp nan,Chronic Kidney Disease 5569," A 69-year-old man with stage 3 CKD has uric acid level of 5.9 mg/dl..",Which ONE of the following statements is correct regarding the effect of allopurinol among CKD patients who are at risk of kidney disease progression but with normouricemia?,"A. Allopurinol has no effect on uric acid lowering and no effect on CKD progression B. Allopurinol lowers uric acid level and slows the rate of GFR decline C. Allopurinol lowers uric acid level but does not slow the rate of GFR decline D. Allopurinol lowers uric acid level and lowers the risk of ESKD",C,temp nan,Chronic Kidney Disease 5570," A 64-year-old woman with stage G3A3 CKD attributed to type 2 diabetes mellitus and hypertension is evaluated in follow-up. Her medications are metformin 500 mg twice daily, empagliflozin 10 mg daily, irbesartan 75 mg daily, and pravastatin 20 mg daily. On physical examination, the BP is 146/95 mm Hg, and the body mass index is 35 kg/m2. Laboratory results show HbA1c of 9%. She would like to discuss additional therapeutic options to optimize her glycemic control. You discuss the option of adding a glucagon-like peptide 1 receptor agonist (GLP1RA)..",Which ONE of the following statements regarding evidence from randomized controlled trials of combining SGLT2i and GLP1RA compared with individual treatment is NOT correct?,"A. Combining SGLT2i and GLP1RA improves glyce-mic control B. Combining SGLT2i and GLP1RA results in better weight control C. Combining SGLT2i and GLP1RA has good safety profile in outpatient settings D. Combining SGLT2i and GLP1RA lowers the risk of cardiovascular and renal adverse events",D,temp nan,Chronic Kidney Disease 5571," A 64-year-old man with CKD stage G4A1 comes to see you in the clinic after referral from his primary care physician. He asks you what he can do to slow his progression of CKD..",Which of the following are associated with slower progression of CKD?,"A. Fragmented and less sleep duration B. Higher dietary animal protein intake C. High urine NGAL levels D. Use of allopurinol in patients with normal uric acid E. Increased intake of fruits and vegetables",E,temp nan,Chronic Kidney Disease 5572," A 49-year-old man is admitted with COVID-19, pneumonia, and AKI. Within 48 hours of admission, he is intubated and started on continuous venovenous hemofiltration. His family asks you what the chances are that he will require long term dialysis..","In patients admitted with severe COVID-19 illness and AKI requiring dialysis, which ONE of the following was associated by multivariate analysis with requirement of dialysis at time of discharge from hospital?","A. History of CKD B. Requirement of mechanical ventilation C. Requirement of pressor support D. History of cardiovascular disease",A,temp nan,Chronic Kidney Disease 5573," A 79-year-old man has CKD stage 4 in the setting of poorly controlled diabetes. He has coronary artery disease and is status post myocardial infarction with reduced ejection fraction. He has severe peripheral vascular disease, is status post right-sided below-the-knee amputation, and has a gangrenous toe on his left foot..",Which adverse event is he MOST LIKELY to experience as result of SGLT2 inhibitor use?,"A. Volume overload B. Myocardial infarction C. Lower extremity amputation D. Dialysis",C,temp nan,Chronic Kidney Disease 5574," You are asked to evaluate a 56-year-old man with CKD. He has had diabetes and hypertension for 25 years. His medications include metoprolol, amlodipine, and metformin. On examination, his BP is 146/92 mm Hg but is otherwise unremarkable. His laboratory results show a urine ACR of 2000 mg/g, serum creatinine of 3.9 mg/dl, and eGFR of 17 ml/min per 1.73 m2. As you are discussing options for renal replacement therapy, he asks about whether his kidney disease could have been detected earlier. Screening for albuminuria is suggested for patients with diabetes, hypertension, GFR < 60 ml/min per 1.73 m2, and cardiovascular disease..",What proportion of patients with diabetes in the United States undergoes annual urine ACR screening?,"A. 5%–15% B. 35%–55% C. 65%–85% D. 90%–100%",B,temp nan,Chronic Kidney Disease 5575," Screening for albuminuria has been recommended for individuals with hypertension by the 2017 American College of Cardiology/American Heart Association Hypertension Guidelines. The Board of Trustees at your local hospital asks you if most medical practices are compliant with these guidelines..",What proportion of patients with hypertension and no diabetes in the United States undergoes ACR screening?,"A. < 10% B. 20%–30% C. 40%–50% D. 60%–70%",A,temp nan,Chronic Kidney Disease 5576," You are asked to consult on a 63-year-old man with eGFR 58 ml/min per 1.73 m2 after a nephrectomy for renal cell carcinoma. He asks you what his chances are of ending up on dialysis..",Which of the following CKD stages is NOT considered very high risk for CKD progression?,"A. CKD G3aA3 B. CKD G3bA1 C. CKD G3bA2 D. CKD G4A1",B,temp nan,Chronic Kidney Disease 5577," You are interviewed by a local news station after a demonstration about inequality in health care occurred outside your hospital. You are asked whether the estimation of kidney function should be the same for all Americans regardless of race..",Which effect would replacing the CKD-EPI 2009 equation with the CKD-EPI 2021 equation have on CKD prevalence in the United States?,"A. Prevalence would increase among White adults and decrease among Black adults B. Prevalence would decrease among White adults and increase among Black adults C. Prevalence would be stable among White adults and decrease among Black adults D. Prevalence would be stable among White adults and increase among Black adults",B,temp nan,Chronic Kidney Disease 5578," A large group practice is incorporating risk prediction algorithms to their electronic medical record. Currently, all patients are screened at their initial visit with a serum creatinine/eGFR and urine ACR. The group wishes to identify patients who have a normal eGFR but are at risk of developing kidney disease such that they can be followed more closely..",Which of the following risk tools would be appropriate to use in a population with GFR > 60 ml/min per 1.73 m2?,"A. 4-variable kidney failure risk equation B. 8-variable kidney failure risk equation C. Advanced CKD risk tool D. Incident CKD risk tool",D,temp nan,Chronic Kidney Disease 5579," You are evaluating a 69-year-old patient with diabetes, hypertension, an eGFR of 52 ml/min per 1.73 m2, and albuminuria of 1800 mg/g who undergoes a kidney biopsy..",Which of the following features on kidney biopsy was NOT associated with risk of CKD progression in the Boston Kidney Biopsy Cohort?,"A. Interstitial fibrosis/tubular atrophy B. Arterial sclerosis/arteriolar sclerosis C. Global glomerulosclerosis D. Fibrinoid necrosis",D,temp nan,Chronic Kidney Disease 5580," You are rounding with a medical student who asks what new imaging technologies are available in the evaluation of kidney disease..",Which of the following is an incorrect pairing of imaging modality and detectable morphology or functional assessment?,"A. Cationic ferritin-enhanced (CFE)-magnetic resonance imaging (MRI): glomerular number B. Point-of-care ultrasonography (POCUS): inflammation C. Diffusion-weighted MRI: fibrosis D. Blood oxygenation level–dependent (BOLD) MRI: cortical tissue hypoxia",B,temp nan,Chronic Kidney Disease 5581," You are asked to evaluate a 39-year-old woman with nephrotic range proteinuria and normal eGFR. Laboratory tests PLA2R antibody, ANCA, antinuclear antibodies, anti-dsDNA, complement C3/C4, HIV, hepatitis C virus, and hepatitis B virus are all negative. She does not wish to undergo biopsy. She asks if there are additional laboratory tests that can be pursued..",Which of the following has not been identified as an autoantigen in primary membranous nephropathy?,"A. EXT1/EXT2 B. TNFR1 C. Sema 3B D. NELL-1",B,temp nan,Chronic Kidney Disease 5582," You are asked to evaluate a 21-year-old man who presents with multiple cysts in both kidneys. His eGFR is normal and he does not have proteinuria or hematuria. He does not have a family history of known polycystic kidney disease or ESKD. His parents, who have accompanied him to the visit, ask whether a kidney biopsy or genetic screening would be most appropriate..",Genetic testing is expected to have the highest diagnostic yield in which of the following kidney dis-ease phenotypes?,"A. Diabetic kidney disease B. Glomerulopathy C. Tubulointerstitial disease D. Congenital or cystic kidney disease",D,temp nan,Chronic Kidney Disease 5583," You are evaluating a 29-year-old man who presented with 4 g of albuminuria and normal eGFR. He mentions that an older sibling has a kidney transplant and two cousins have albuminuria. His father died from an accident when the patient was very young. His mother does not visit doctors often but has never been told there was a problem with her kidneys. You discuss genetic screening with him and he asks you what you are most likely to find..","Apart from APOL1, variants in which gene family are the most common genetic finding in populations with FSGS?","A. COL4A B. UMOD C. NPHP D. Cubilin",A,temp nan,Chronic Kidney Disease 5584," A 65-year-old woman with high BP and obesity presents to the CKD clinic. She has stage 4 CKD A2 and denies any complaints of headaches, edema, and shortness of breath. Her medications include torsemide, losartan, atorvastatin, and aspirin. Her BP is 155/68 mm Hg when sitting comfortably but is not performed in a standardized method with an oscillometric BP device. A resident assisting you in clinic suggests addition of amlodipine but the patient refuses. She is insistent that her home BP is better and she has always had “white-coat hypertension”. To solve the problem, you suggest ambulatory BP monitoring (ABPM)..",Which ONE of the following is true regarding BP monitoring?,"1. Most guidelines (including Kidney Disease: Improving Global Outcomes) recommend treatment of hypertension on the basis of office-measured nonstandardized BPs in adults with CKD B. ABPM profiles reveal white-coat hypertension in the majority of patients C. White-coat hypertension is associated with increased risk of kidney outcomes, cardiovascular outcomes, and mortality D. The presence of masked uncontrolled hypertension, identified by ABPM, is associated with higher risk of composite cardiovascular and renal outcomes compared with participants with con-trolled BP",D,temp nan,Chronic Kidney Disease 5585," A 72-year-old man with type 2 diabetes mellitus, high BP, CKD 4 A3 with estimated eGFR 24 ml/min per 1.73 m2 presents to your CKD clinic for follow-up. He is maintained on losartan 100 mg, amlodipine 5 mg, atorvastatin 80 mg, and aspirin. His automated BP measured in clinic is 116/68 mm Hg. He appears perturbed as he was recently told by another practitioner to discontinue losartan given advanced CKD but is quite hesitant to make any medication changes..",Which ONE of the following statements is true?,"A. Renin-angiotensin system (RAS) inhibitors should be stopped with eGFR < 30 ml/min per 1.73 m2 owing to risk of hyperkalemia B. Excessively tight BP control may be associated with worsening GFR, and his medications should be titrated to a target BP of > 130/80 mm Hg C. Compared with continuing RAS inhibition, stop-ping this therapy is associated with a higher absolute risk of death and major adverse cardio-vascular events in patients with CKD 4 D. In a study comparing new users of dihydropyridine calcium channel blockers (DHP-CCB) versus RAS inhibitors in subjects with CKD G4–5 not on dialysis, there was a significantly lower risk of major adverse cardiovascular event and mortality in patients taking DHP-CCB",C,temp nan,Chronic Kidney Disease 5586," A 60-year-old woman with diabetes mellitus, CKD 3B/A3, urinary ACR 500 mg/g, GFR 32 ml/min per 1.73 m2, hypertension, coronary artery disease, and heart failure with preserved ejection fraction presents to the clinic for a new consultation. She is maintained on lisinopril 40 mg, metformin 500 mg twice a day, atorvastatin 40 mg, and carvedilol 25 mg twice a day. Her BP checked in office is 110/66 mm Hg and she offers no new complaints..",Which of the following statements regarding SGLT2 inhibition is true?,"A. SGLT2 inhibitors should not be initiated at GFR < 35 ml/min per 1.73 m2 B. SGLT2 inhibition does not confer any additional kidney benefit in patients already on angiotensin-converting enzyme inhibitor therapy C. Metformin should be dose reduced or stopped if SGLT2 inhibitor is added D. In the DAPA-CKD trial, the renal benefits were consistent across the spectrum of eGFR, albuminuria, and the presence/absence of diabetes mellitus",D,temp nan,Chronic Kidney Disease 5587," A 51-year-old man with CKD 3B secondary to IgA nephropathy, eGFR 40 ml/min, and urine ACR 560 mg/g attends for follow-up. He also has diabetes mellitus, is being treated with semaglutide with a recent HbA1c of 7%, and has a family history of colon cancer. He has not yet had a first colonoscopy. His BP is at goal on full dose angiotensin II receptor blocker and chlorthalidone 50 mg every day..",Which of the following is correct in considering treatment with an SGLT2 inhibitor in this patient?,"A. SGLT2 inhibitors have not been shown to be safe or effective in patients with glomerular disease such as IgA nephropathy B. He is at high risk of hypoglycemia unless his semaglutide is dose reduced or stopped C. He can safely continue his SGLT2 inhibitor while he takes his bowel preparation for colonoscopy D. Chlorthalidone dose may need to be reduced with the initiation of an SGLT2 inhibitor",D,temp nan,Chronic Kidney Disease 5588," A 56-year-old woman with CKD 4, eGFR 25 ml/min per 1.73 m2, ACR 480 mg/g, hypertension, and heart failure with preserved ejection fraction presents to the CKD clinic for follow-up. Despite taking oral iron, her hemoglobin has been trending lower in recent months, and in today’s laboratory results her hemoglobin is 9.2 g/dl. She reports some fatigue and reduced exercise tolerance, but has been reluctant to start erythropoiesis-stimulating agents because of needle phobia. She has read about new oral agents being developed for treatment of anemia of CKD..",Which ONE of the following statements regarding anemia management in non-dialysis-dependent CKD is true?,"A. Hypoxia-inducible factor-prolyl hydroxylase inhibitors (HIF-PHIs) lead to increase in ferritin and hepcidin B. HIF-PHIs lead to decreases in hepcidin and total iron-binding capacity, and increases in ferritin C. In randomized controlled studies, vadadustat and daprodustat were noninferior to darbepoetin in achieving a mean change in hemoglobin from baseline to end of study D. Vadadustat was noninferior to darbepoetin alpha in analysis of the primary safety outcome (time to first major adverse cardiovascular event)",C,temp nan,Chronic Kidney Disease 5589," You are discussing newer therapies to slow CKD progression with your in-patient rounding team. The intern asked about newer agents, namely the nonsteroidal mineralocorticoid receptor (MR) antagonist finerenone..",Which ONE of the following statements is true?,"A. Finerenone is less potent in blocking MR cofactor binding as compared to agents such as spironolactone B. Finerenone is associated with reduction in the urinary ACR from baseline in patients on maximal doses of renin-angiotensin system inhibitors C. Large studies of finerenone have demonstrated improved cardiovascular outcomes, but have not shown benefit in a composite renal outcome, comprising a decrease greater than or equal to 40% in eGFR or renal death D. Finerenone is not associated with hyperkalemia",B,temp nan,Chronic Kidney Disease 5590," A 50-year-old man with CKD 3b/A3, eGFR 45 ml/min per 1.73 m2, ACR 360 mg/g, and hypertension presents to the CKD clinic for follow-up. He is interested in advice regarding lifestyle modifications to slow CKD progression..",Which ONE of the following statements regarding lifestyle modification in management of nondialysis-dependent CKD is true?,"A. Recent meta-analysis found that lower dietary potassium intake was associated with significantly decreased odds of CKD B. Healthy dietary pattern with higher intake of fruits, vegetables, whole grains, and fiber was associated with a reduced risk of hypertension and improved survival but there was no improvement in renal outcomes C. Current smokers have significantly increased odds of CKD compared with former and never smokers D. Higher levels of activity, defined as exercise of at least 30 minutes a day, is associated with lower odds of CKD",D,temp nan,Chronic Kidney Disease 5591," A 46-year-old woman with CKD G3b, eGFR 32 ml/min per 1.73 m2, secondary to diabetic nephropathy comes to the clinic. Her BP remains above goal at 144/92 mm Hg and she has persistent proteinuria of 2.7 g/g on a spot urine PCR. She is currently on valsartan 160 mg daily, chlorthalidone 50 mg daily, and carvedilol 25 mg twice a day. At prior visits, her potassium has been in the high normal range, 5.2–5.5 mEq/L, despite efforts at dietary control. You would like to increase her valsartan, and discuss adding a potassium-binding resin to her regimen..",Which ONE of the following statements regarding novel potassium binders is true?,"A. Sodium Zirconium Cyclosilicate (ZS-9/SZC) is a non-absorbed highly selective inorganic cation, which binds K+ throughout the gastrointestinal tract in exchange for Ca2+ B. Patiromer is associated with increased sodium load C. Sodium Zirconium Cyclosilicate (ZS-9/SZC) has a slow onset of action (within 12 hours) D. Sodium Zirconium Cyclosilicate (ZS-9/SZC) also can help improve acidosis",D,temp nan,Chronic Kidney Disease 5592," A 68-year-old man with diabetes mellitus, CKD 4/A3, urinary ACR 500 mg/g, GFR 28 ml/min per 1.73 m2, hypertension, coronary artery disease, and heart failure with preserved ejection fraction presents to the clinic for follow-up. The patient is maintained on lisinopril 40 mg, atorvastatin 40 mg, andcarvedilol25mg twice a day. His BP checked in office is 118/70 mm Hg and he has no new complaints. Laboratory parameters are stable with the exception of serum bicarbonate level of 19 mEq/ml..",Which of the following statements regarding management of metabolic acidosis is true?,"A. In a study of CKD patients followed for > 6 months, treatment with veverimer was associated with improvements in objective measurements of physical function B. In a study to assess the tolerability of sodium bicarbonate, treatment with 0.8 mEq per lean body weight per day was associated with more adverse events than 0.5 mEq per lean body weight per day and placebo C. Alkali therapy has not been associated with delayed CKD progression D. Studies of alkali therapy have demonstrated the safety and efficacy of sodium bicarbonate even in those with advanced heart failure",A,temp nan,Chronic Kidney Disease 5593," A 34-year-old woman with a history of diabetes and CKD G3 A2 comes to clinic for review. She is currently taking lisinopril 40 mg daily, chlorthalidone 50 mg daily, and amlodipine 10 mg daily. She checks her BP at home twice weekly and brings readings ranging from 126/84 to 142/88 mm Hg. Her clinic reading today is 138/90 mm Hg. You express concern that her BP is above goal and begin to discuss escalation of her medications. She is reluctant, saying “I had a coffee this morning, and I’m always racing to get here on time”..","When considering treating blood pressure to target levels, which of the following is correct?","A. Current Kidney Disease: Improving Global Outcomes guidelines state that BP readings taken in the home or in a clinical setting should be used to guide treatment B. Observational studies have shown a good correlation between BPs obtained in a usual clinical visit versus those measured using a standardized measurement protocol C. Implementation of standardized BP measurements in an outpatient nephrology clinic would not require any significant changes to typical workflow D. Patients should void urine before having their BPs measured",D,temp nan,Chronic Kidney Disease 5594," A 24-year-old man is evaluated 5 years after receiving a deceased donor kidney transplant. He is doing well and is asymptomatic. His medications include tacrolimus, mycophenolate mofetil, prednisone, omeprazole, trimethoprim/sulfamethoxazole, and acyclovir. On physical examination, the BP is 130/84 mm Hg. The allograft is nontender and without bruit. He has no apparent lower limb edema. The remainder of the examination results are normal. Laboratory studies show sodium of 136 mEq/L, potassium of 3.8 mEq /L, chloride of 112 mEq/L, total CO2 of 17 mmol/L, BUN of 30 mg/dl, creatinine of 1.8 mg/dl (increased from 1.4 mg/dl 1 month ago). Anti-HLA antibody testing shows the presence of a DQ7 donor-specific antibody with mean fluorescence intensity of 12,000 (never previously detected before or after transplantation). A 12-hour tacrolimus trough level is 3 ng/ml (previous level was 3.8 ng/ml 1 month ago). The urinalysis shows no glucose and no protein, blood, or ketones. An ultrasound of the kidney transplant shows normal vasculature and no hydronephrosis..","In addition to ongoing close follow-up of this man’s kidney function, which ONE of the following is the MOST appropriate management?","1. Perform a kidney allograft biopsy 2. Assess for donor-derived cell-free DNA 3. Calculate eplet mismatch score 4. Increase tacrolimus",A,temp nan,Transplantation 5595," A 54-year-old woman with a history of type 1 diabetes mellitus is admitted for management of suspected allograft rejection after undergoing living related kidney transplantation 2 years ago. Her medications include insulin as part, tacrolimus, mycophenolate mofetil, trimethoprim-sulfamethoxazole, valganciclovir, and bisoprolol. Physical examination results are unremarkable. The BP is 130/64 mm Hg supine. The allograft is non-tender and does not have a bruit. Laboratory studies show sodium of 143 mEq/L, potassium of 4.9 mEq/L, BUN of 35 mg/dl, creatinine of 1.6 mg/dl (increased from 1.2 mg/dl 2 weeks ago), and glucose of 260 mg/d. The tacrolimus trough is increased at 8 ng/ml. The urinalysis shows specific gravity of 1.015, 1+ glucose, and 21 protein, no blood or ketones. The allograft biopsy specimen shows glomerulitis (g1) and peritubular capillaritis (ptc 1), and C4d staining is negative. There is also evidence of early chronic interstitial fibrosis and vascular injury (ct1, ci1, ah1, cv2)..","In addition to ongoing close follow-up of this woman’s kidney function, which ONE of the following is the MOST appropriate management?","1. Assess for donor-derived cell-free DNA 2. Assess for donor-specific antibodies 3. Introduce an angiotensin-converting enzyme inhibitor 4. Assess for urinary chemokines",B,temp nan,Transplantation 5596," A 72-year-old man is evaluated 1 year after transplantation of a living donor kidney. He is doing well and has no symptoms. His medications include tacrolimus, azathioprine, omeprazole, aspirin, apixaban, trimethoprim/sulfamethoxazole, acebutolol, amiodarone, and amlodipine. There have been no complications during the first year after transplantation; no rejection episode, no urinary tract infection, and no malignancies. The 3-month biopsy specimen was strictly normal. On physical examination, the BP is 145/103 mm Hg, the allograft is nontender, and there is no apparent edema. Lab-oratory studies show sodium of 141 mEq/L, potassium of 3.9 mEq/L, chloride of 116 mEq/L, total CO2 of 20 mmol/L, BUN of 26 mg/dl, creatinine of 1.2 mg/dl (baseline creatinine is 1.1–1.3 mg/dl). Donor-specific antibody screening results are negative at day 0 and 3, 6, and 12 months after transplantation. A 12-hour tacrolimus trough level is 7.5 ng/ml. The urinalysis shows no glucose, protein, blood, or ketones. An ultrasound of the kidney transplant is normal. You decide to not per-form the 1-year protocol allograft biopsy because of increased risk of bleeding complications in a patient using aspirin and oral anticoagulants..",Which ONE of the following would be the MOST appropriate alternative for the allograft protocol biopsy?,"1. Assess for non-HLA antibodies 2. Assess for cytomegalovirus viremia 3. Assess for C-reactive protein 4. Assess for donor-derived cell-free DNA",D,temp nan,Transplantation 5597," A 38-year-old woman is evaluated 3 months after an episode of acute antibody-mediated rejection, which occurred 2 years after receiving a deceased donor kidney transplant. Her medications include tacrolimus, mycophenolate mofetil, prednisone, trimethoprim sulfamethoxazole, valganciclovir, and nomegestrol acetate. The allograft biopsy performed 3 months ago showed glomerulitis (g2), peritubular capillaritis (ptc2), without interstitial inflammation or acute vascular injury (i0, t0, cg0, v0); C4d staining was positive, and there was no evidence of chronic interstitial injury (ct0, ci0, ah0, cv0), and anti-HLA antibody testing showed the presence of an A24 donor-specific antibody with mean fluorescence intensity of 8200. The patient received plasmapheresis, intravenous immunoglobulins, and rituximab. Currently, the patient has no symptoms, the BP is 135/72 mm Hg, there no signs of congestion, and the allograft is nontender. Laboratory studies show sodium of 139 mEq/L, potassium of 4.7 mEq/L, BUN of 45 mg/dl, creatinine of 1.7 mg/dl (levels were 2.1 mg/dl 3 months ago)..","The urinalysis shows specific gravity of 1.015, no glucose, and 1+ protein, no blood or ketones Which ONE of the following would be the MOST appropriate to evaluate response to treatment in com-bination with creatinine and proteinuria?","1. Assess for donor-specific antibodies 2. Assess for tacrolimus trough level 3. Assess for mycophenolate mofetil area under curve 4. Assess for urinary chemokines",A,temp nan,Transplantation 5598," A 22-year-old woman with stage 4 CKD secondary to IgA nephropathy with a current eGFR of 17 ml/min per 1.73 m2 comes to clinic for follow-up. She has a history of hypertension, and her BMI is 37 kg/m2. She is interested in renal transplantation but currently has no potential live kidney donors. You have previously dis-cussed bariatric surgery with her, but she is cautious about considering this. As you discuss options for renal replacement therapy, she asks about the transplantation evaluation process and what factors might affect her ability to get a kidney..","With regard to access to renal transplantation, which ONE of the following is correct?","1. Receiving dialysis at a for-profit dialysis unit is associated with lower rates of renal transplantation referral 2. In an analysis by the Southeast Kidney Consortium, Black and non-Hispanic patients had similar rates of initiation of transplantation evaluation after transplantation referral as other groups 3. Prior bariatric surgery is a contraindication to renal transplantation 4. Since the introduction of the kidney allocation system in 2014, patients with the highest estimated post-transplantation survival (EPTS score) receive priority access to all kidneys for transplantation",A,temp nan,Transplantation 5599," A 62-year-old man with stage 4 CKD, eGFR 16 ml/min per 1.73 m2, comes to clinic for routine follow-up. After reviewing his symptoms, physical examination results, and medications, you discuss referral for transplantation evaluation with him. You explain that because his eGFR is currently < 20 ml/min per 1.73 m2, he can be pre-emptively listed for renal transplantation and begin to accumulate wait time. He asks about deceased versus live kidney donation and expresses reservations about asking a family member to donate..",Which ONE of the following is correct regarding access to live donor renal transplantation?,"1. Educational and support programs targeted at minority transplantation candidates and their families can increase the chance of receiving a live donor kidney 2. Recipients of a prior kidney transplant have increasing rates of pre-emptive listing of retransplantation in recent years 3. Removing the Black race coefficient from GFR estimating equations has been predicted to decrease the time to reach an eGFR of < 20 ml/min per 1.73 m2 by around 6 to 9 months 4. The majority of disparities in transplantation listing seen between White and Black candidates is explained by differences in levels of comorbidity and medical barriers to transplantation",A,temp nan,Transplantation 5600," A 43-year-old female with ESKD secondary to IgA nephropathy has been on the renal transplant wait list for 6 years. She receives an offer of a deceased donor kidney. The 62-year-old donor died due to a motor vehicle accident and had a history of smoking and well-controlled hypertension. Donor testing is positive for anti-HCV antibody, but HCV nucleic acid testing (NAT) is negative. A renal biopsy is acquired at the time of organ harvest..",Which ONE of the following is correct with regard to predicting outcome of deceased donor kidneys?,"1. The presence of macroscopic arteriosclerosis on pre-transplantation biopsy predicts poorer allograft outcomes 2. The histological features on pre-transplantation biopsy are more strongly predictive of graft loss than the presence of donor-specific antibodies 3. Donor HCV antibody positive status is associated with higher rates of kidney discard 4. In an analysis comparing kidneys discarded in the US due to histological features with matched kidneys transplanted in France, the allograft survival rate was 80% at 5 years",D,temp nan,Transplantation 5601," A 55-year-old woman underwent a deceased donor kidney transplantation 1 year ago. At 4 months after trans-plantation, she had a grade IB acute cellular rejection that was treated with corticosteroids. Currently her allograft function is stable, with a creatinine of 1.4 mg/dl, and she has no proteinuria, but new donor-specific antibody is present. A surveillance kidney biopsy was performed, and the biopsy sample showed fibrosis of 20% of the cortex. She has interstitial inflammation involving > 25% of the sclerotic parenchyma and > 25% of the total cortical parenchyma with moderate tubulitis..","Based on the current Banff classification, how should her rejection be classified?","1. Grade IA active T cell–mediated rejection 2. Chronic active antibody-mediated rejection 3. Grade 1A chronic active T cell-mediated rejection 4. No rejection because there is inflammation in areas of scarred cortex",C,temp nan,Transplantation 5602," A 62-year-old man with a history of a living related donor kidney transplantation 4 months ago was seen for routine follow-up. His creatinine was elevated to 2.2 mg/dl, which was up from a baseline of 1.4 mg/dl. He had no donor-specific antibody identified, and the result of testing for BK viremia was negative. His indication biopsy showed a Banff IIA acute cellular rejection. He was treated with corticosteroids with improvement of his creatinine back down to baseline, and examination of a follow-up biopsy specimen showed resolution of inflammation in nonfibrotic cortex..",Which of the following statements is TRUE based on long-term follow-up studies?,"1. The patient is at increased risk for death with a functioning graft 2. Studies of surveillance biopsies performed on transplants have shown that allograft failure is most commonly due to alloimmune injury 3. Given that the biopsy specimen showed resolution of inflammation, this rejection will likely have no effect on his allograft function in the long term 4. The patient is likely to eventually experience chronic antibody-mediated rejection",A,temp nan,Transplantation 5603," A 23-year-old man is listed for a kidney transplantation and fortunately has an unrelated potential living donor who is a 4/6 HLA mismatch and matched at the DR locus. He has questions about kidney paired donation and the role of DR/DQ eplet matching..",Which of the following statements is MOST CORRECT with regard to how you should counsel the patient?,"1. There is no role for eplet matching because he is matched at the DR locus 2. It would be helpful to first look at the matching at the DQ locus; if he is not matched at DQ, eplet matching may be of benefit 3. He should consider eplet matching because he is mismatched at the A locus 4. Eplet matching may not have a role when there is a living kidney donor available",B,temp nan,Transplantation 5604," Your patient is a 45-year-old woman with two prior failed transplants. She underwent a positive crossmatch kidney transplantation 2 years ago and has an increase in creatinine. Her biopsy shows a chronic active antibody-mediated rejection with microvascular inflammation (peritubular capillaritis and glomerulitis) and mild transplant glomerulopathy. No acute T cell–mediated rejection is present. She is interested in learning more about clinical trials with IL-6 antagonists..",Which of the statements is INCORRECT regarding what is known about the use of IL-6 inhibitors for treatment of rejection?,"1. Patients were withdrawn from the initial pilot study with clazakizumab because of complications from diverticular disease 2. IL-6 inhibitors have been shown to improve long-term allograft survival in randomized controlled trials 3. Preliminary studies have shown that IL-6 inhibitors are associated with stabilization of eGFR 4. IL- 6 receptor antagonists are associated with increased systemic IL-6, which can lead to rapid deterioration of allograft function after they are stopped",B,temp nan,Transplantation 5605," A 55-year-old woman with ESKD from autosomal dominant polycystic kidney disease returns to clinic for continued care after a living unrelated kidney transplant from her husband 6 months ago. She has gained 30 pounds and has had more difficulty sleeping, and walking up tired and “groggy.” She is adherent to her maintenance immunosuppression regimen of tacrolimus 2 mg twice a day, mycophenolate mofetil 1000 mg twice a day, and prednisone 5 mg daily. She is also taking atorvastatin 10 mg daily, amlodipine 5 mg daily, and metoprolol 50 mg twice a day. Her in-office BP is 165/90 mm Hg, and repeated BP 15 minutes later is 155/85 mm Hg. Her physical examination results are significant for a protuberant abdomen, palpable kidneys, a well-healed incision in the right lower quadrant, and trace lower extremity edema bilaterally. Creatinine is 1.1 mg/dl, electrolytes are normal, fasting blood glucose level is 98 mg/dl, tacrolimus level is 7, and urine analysis shows trace of blood but no red blood cells, specific gravity of 1.002, negative protein, glucose, and infection..","Based on this presentation, which of the following factors is MOST likely to affect this patient’s long-term survival?","1. Polycystic native kidneys with malignant transformation to renal cell carcinoma 2. Uncontrolled hypertension with concern for pulmonary hypertension 3. Allograft rejection due to subtherapeutic tacrolimus 4. Development of post-transplantation diabetes mellitus, given her 30 pounds of weight gain",B,temp nan,Transplantation 5606," A 35-year-old man with FSGS who received a living related kidney transplant from his father 15 years ago returns to clinic for his annual visit. His post-transplantation course was complicated by antibody-mediated rejection 5 years ago when he lost his job and immuno-suppression coverage lapsed. He was told 6 months ago by his transplantation nephrologist that his kidney function is declining. He does not describe uremic symptoms and is continuing to work full time as a custodian. His serum creatinine has been 3.8 to 4.2 mg/dl (eGFR 18–20 ml/min). The result of panel reactive antibodies is 50%class I and 30% class II. A renal biopsy shows 80% interstitial fibrosis and tubular atrophy and chronic antibody-mediated injury. His current immunosuppression is cyclosporine 100 mg twice a day, azathioprine 100 mg daily, and prednisone 5 mg daily, but he admits to missing his medications a few times during the week because of his work schedule. His girlfriend, who has accompanied him to clinic, asks about the possibility of being a living donor..",Which of the following is the MOST appropriate counseling?,"1. Patient does not require a second transplant at this time, given the absence of uremic symptoms, and medical management should now focus on extending the longevity of the current transplant 2. Patient does require a second transplant but can-not be listed until his eGFR is < 15 ml/min or he requires dialysis 3. Patient does require a second transplant and can be listed but should be weaned off some immuno-suppression and have a short course of dialysis to prove his adherence to medical therapy 4. Patient will require a second transplant and should be listed and maintained on current immunosuppression while his potential living donor is evaluated",D,temp nan,Transplantation 5607," A nephrology consultation is requested a patient admitted with AKI. He is a72-year-oldmanwithkidney transplantation 5 years ago complicated by metastatic melanoma and was started 3 weeks ago on programmed cell death ligand-1 (anti-PD-1) therapy with nivolumab. He has experienced abdominal pain and diarrhea and reports a flare of his psoriasis. On physical examination he is normotensive and has no evidence of orthostatic hypotension..","Of the following statements regarding immune check-point inhibitors, which ONE is NOT TRUE?","1. Checkpoint inhibitors are associated with immune-related manifestations of colitis, thyroiditis, and rash 2. His allograft dysfunction is most likely related to volume depletion and prerenal AKI 3. A renal biopsy will not distinguish acute interstitial nephritis caused by anti-PD-1 agent from transplant rejection 4. Acute rejection rates are roughly 40% to 50% and occur within 3 weeks of treatment initiation",B,temp nan,Transplantation 5608," A 32-year-old woman is being evaluated at your trans-plantation program for being a living kidney donor to her father, who has ESKD presumed secondary to hypertension. She has no significant past medical problems and has a height of 5 feet 6 inches, weight of 146 pounds, and a BMI of 23.7. During her office visit, a BP measurement with an aneroid automatic sphygmomanometer revealed a BP of 136/82mmHg.Herphysicalexaminationresults were unremarkable. She reports no history of hypertension and does not recall that her BP was ever checked..",Which of the following is the best next step in the assessment of her BP?,"1. Her BP is less than 140/90 mm Hg and hence she is acceptable to proceed with her evaluation with no further testing with regard to evaluation for hypertension 2. Her BP is greater than 130/80 mm Hg, you diagnose her with hypertension, and hence she is not a candidate for donation 3. She likely has white coat hypertension and hence can proceed with further evaluation with no further testing with regard to evaluation for hypertension 4. She should complete a 24-hour ambulatory BP monitoring to decide whether or not she has hypertension",D,temp nan,Transplantation 5609," You are seeing a 23-year-old man to evaluate him as a living kidney donor to his mother, who he reports as being attributed to hypertension. He has no significant medical history. On examination, he is noted to have a BP of 120/80 mm Hg and a BMI of 29. His physical examination results are unremarkable. His laboratory test results, including urinalysis, urine albumin-to-creatinine ratio, and urine protein-to-creatinine ratio are normal. You contact his mother’s nephrologist and find out that she her kidney biopsy specimen showed focal and global sclerosis. The recipient’s team had performed a next-generation sequencing renal genetic testing to find that she was heterozygous for abnormal COL4A4 and APOL1 gene renal risk variants..",Which of the following is the next best step in your evaluation of this living donor candidate?,"1. Proceed with further evaluation without any genetic testing because his normal results of urinalysis and urine albumin excretion preclude risk for development of kidney disease because of gene abnormalities found in his mother 2. Proceed with genetic testing without informing him and with an intention to discuss test results only if they come back as positive 3. Discuss potential risks and benefits of genetic testing with him now, and proceed with genetic testing only if he agrees 4. Decline him as a candidate because he is at an unacceptable risk for ESKD after donation",C,temp nan,Transplantation 5610," A 56-year-old woman is being evaluated for living kidney donation to her husband. She has a history of mild depression, for which she takes venlafaxine, which is prescribed by her primary care provider and is the only medication she currently takes. On examination, her BP is 125/78 mm Hg, height 5 feet 2.5 inches, weight 154 pounds, BMI 27.8 and body surface area 1.72 m2. No abnormal findings are noted on her physical examination. On her laboratory testing, she is noted to have a serum creatinine of 0.83 mg/dl with an eGFR of 78.8 ml/min per 1.73 m2 according to the CKD Epidemiology Collaboration (CKD EPI) formula. She had not had a blood test done in many years..",Which of the following is the best next step in her evaluation with respect to kidney function assessment?,"1. Decline her as a candidate because her GFR is ,80 ml/min per 1.73 m2 2. Accept her as a candidate because her age- and gender-adjusted GFR is appropriate for donation 3. Perform a 24-hour urine creatinine collection to further assess her GFR 4. Perform an ultrasound of bilateral kidneys to assess for asymmetry in size",C,temp nan,Transplantation 5611," A 55-year-old woman is being evaluated to be a living kidney donor to her husband. She is a college professor in mathematics and reports no medical history. She has no family members with kidney disease. She does not take any medications. On examination, she has a height of 5 feet 5 inches, weight of 187 pounds, calculated BMI of 31.3, body surface area of 1.93 m2, and BP of 128/78 mm Hg. She has a normal result of physical examination. Her urinalysis is normal, urine albumin- to -creatinine ratio is 7 µg/mg, serum creatinine is 0.68 mg/dl with a CKD EPI eGFR of 98.5 ml/min per 1.73 m2, 24-hour urine creatinine clearance of 120 ml/min per 1.73 m2 on an adequately collected sample; fasting blood glucose is normal..","With respect to her obesity, what is the ONE next best step in her evaluation?","1. Because her BMI is > 30, she should be declined as a living kidney donor because of the unacceptable risk for future ESKD after donation 2. Because her BMI is < 35, she should be accepted as a living kidney donor because there is no increased risk for future ESKD after donation compared with normal weight candidates 3. Counsel her on her higher relative risk for future ESKD after donation compared with normal weight candidates but low overall absolute risk",C,temp nan,Transplantation 5612," A 28-year-old woman is being evaluated for living kidney donation. Her only medical history is hypothyroid-ism, which is well controlled with levothyroxine. On examination, she has a BP of 123/73 mm Hg, height of 5 feet 6 inches, weight of 180 pounds, BMI of 29.2. The results of her entire laboratory testing, social work evaluation, and evaluation by independent living donor advocate were acceptable for donation. Her serum creatinine is 0.62 mg/dl with a CKD EPI eGFR of 124 ml/min per 1.73 m2, 24-hour urine creatinine clearance of 131 ml/min per 1.73 m2 on an adequately collected specimen. Her CT angiogram identified that kidneys were 12.5 cm each, a nonobstructing 4-mm stone was in right kidney, and single renal artery on both sides. She does not describe any history of kidney stones in herself and her family members. A nuclear medicine split renal scan identified right kidney and left kidney with 47% and 53% of split GFR, respectively..",Which ONE of the following statements reflects the BEST next step?,"1. Decline her as a candidate because of the presence of nephrolithiasis owing to increased risk of future ESKD after donation 2. Accept her to donate either right or left kidney and recommend that she follow up with her primary care physician for further evaluation 3. Perform a 24-hour urine stone profile, and if it is normal, accept for right donor nephrectomy 4. Perform a 24-hour urine stone profile, and if it is normal, accept for left donor nephrectomy",C,temp nan,Transplantation 5613," A 60-year-old woman donated 6 months ago to her 65-year-old husband, who has ESKD due to diabetes. She is normotensive with no evidence of diabetes or dyslipidemia. Her preoperative electrocardiogram showed normal sinus rhythm..",Which ONE of the following statements would be MOST appropriate in counseling this woman?,"1. Cardiovascular death is not the main cause of death among living kidney donors 2. She should have annual or biannual stress test to evaluate for coronary artery disease 3. Living kidney donors have an increased risk of death and major cardiovascular events compared with healthy, nondonor control individuals within the first decade of donation 4. Living kidney donors have an increased risk of cardiovascular death and ischemic heart disease compared with healthy, nondonor control individuals after the first decade of donation",D,temp nan,Transplantation 5614," A 39-year-old man with a medical history of ESKD secondary to hypertension and on hemodialysis since November 2012, currently listed active for transplantation, receives a kidney offer from a 22-year-old deceased donor with a history of seizure (noncompliant with medication). Donor history: no surgical history. Social history: smoked one or two cigarettes a month since age 16, drank one or two beers on the weekends. Cause of death: asphyxiation. BMI 28.46, blood type O, Kidney Donor Profile Index 19%, urine toxicology negative, terminal creatinine 1.2 mg/dl, HbA1C 5.8%, cytomegalovirus IgG positive, Epstein-Barr virus IgG negative. Nasopharyngeal SARS-COV-2 PCR negative, bronchoalveolar SARS-COV-2 PCR negative, rectal SARS-COV-2 PCR positive. Upon further history, the donor family reports that he had flulike symptoms 4 weeks before his death. Recipient calculated panel reactive antibody (cPRA) class I 70%, class II 0%, no donor-specific antibody. T cell and B cell flow crossmatch, negative..",Which ONE of the following would be the MOST appropriate approach for this offer?,"1. Decline the offer because of donor’s history of flu-like symptoms 4 weeks before his death 2. Acceptable donor, proceed with transplantation 3. Decline the offer because of donor’s positive SARS-COV-2 in the rectum 4. Decline the offer because of high cPRA 5. Decline the offer until COVID pandemic resolves",B,temp nan,Transplantation 5615," A 60-year-old man with a history of ESKD, status post deceased donor kidney transplantation in 2009, heart failure with preserved ejection fraction, hypertension, hyperlipidemia, type 2 diabetes mellitus, and pericardial window in 2007 presented to the emergency department because of shortness of breath and fever for the previous 2 days. He went to an urgent care facility, where he was found to be hypoxic and positive for SARS-CoV-2 infection and was sent to the emergency department for further workup and management. He is on tacrolimus 2 mg twice a day, mycophenolate mofetil 500 mg twice a day, and prednisone 5 mg daily for immunosuppression. In the emergency department, his vital signs were significant for BP 145/78, heart rate 98 beats/min, temperature 100.8F, oxygen saturation 87% on room air initially but improved to 97% on 2 liters nasal cannula. Laboratory tests showed white blood cell 3.400/µl (lymphocytes 200/µl), hemoglobin 10.1 g/dl, platelets 125,000/µl, C-reactive protein 4.4 mg/dl, D-dimer 2.47 µg/ml. Chest X-ray revealed mild con-gestion and left lower lobe pneumonia..",Which ONE of the following treatments is NOT recommended for treatment of COVID-19 disease in this patient?,"1. Consideration of reduction in immunosuppression 2. Monoclonal antibodies 3. Remdesivir 4. Dexamethasone",B,temp nan,Transplantation 5616," A 69-year-old Hispanic man with ESKD secondary to polycystic kidney disease received a deceased donor kidney transplant 5 years ago. His post-transplantation course was complicated by multiple urinary tract infections and concomitant AKI leading to chronic allograft injury with baseline serum creatinine of 1.8 mg/dl (normal range < 1.3 mg/dl). His medical history includes coronary artery disease and hypertension. He is admitted to the hospital with multifocal pneumonia secondary to SARS-CoV-2 and currently receives supplemental oxygen 3 L/min via nasal cannula to maintain a SpO2 of 94%. His mycophenolate mofetil was held, and he was continued on tacrolimus with target trough levels of 3 to 5 ng/ml and prednisone 5 mg daily. His blood work on admission showed a serum creatinine of 2.5 mg/dl and white blood cell count of 3,500/µl (normal range: 4,800–10,800 /µl) with lymphocyte count of 600/µl (normal range 1,000–4,800/µl. He was started on dexamethasone treatment. Which ONE of the following factors has NOT been associated with increased risk of mortality in transplant recipients with COVID-19? 1. Age 2. Acute kidney injury 3. Cause of kidney disease 4. Lymphopenia 5. History of coronary artery disease 6..",Oxygen requirement,"1. Age 2. Acute kidney injury 3. Cause of kidney disease 4. Lymphopenia 5. History of coronary artery disease",C,temp nan,Transplantation 5617," A 35-year-old African American woman with ESKD secondary to primary focal segmental glomerulosclerosis received a deceased donor kidney transplant 4 years ago. Her post-transplantation course was uncomplicated. Her panel reactive antibody level at the time of transplantation was 80%, and she received antithymocyte globulin for induction. She is currently maintained on tacrolimus with a target trough of 4 to 6 ng/ml, mycophenolate mofetil 1000 mg twice a day, and prednisone 5 mg daily. Her most recent serum creatinine is 1.1 mg/dl (normal range <1.3 mg/dl). She comes today to the transplantation clinic to discuss whether she needs a third dose of COVID-19 vaccine. She received two doses of mRNA-based COVID vaccine; her last dose was6weeksago.HerantispikeSARS-CoV-2 IgG was checked 2 weeks ago and was undetectable..",Which ONE of the following statements is FALSE regarding her lack of antibody response to vaccine?,"1. Her age is a risk factor for not mounting an anti-body response 2. High-dose mycophenolate mofetil may impair the development of an antibody response to vaccination 3. Being a transplantation patient decreases her response to vaccination 4. After a third dose of mRNA vaccine, 25% to 70%of transplant recipients experience antispike SARS-CoV-2 IgG 5. Her induction treatment with antithymocyte globulin does not decrease her response to vaccination",A,temp nan,Transplantation 5618," A 48-year-old woman with a history of ESKD attributed to type 2 diabetes mellitus is being seen in the office for transplantation candidacy assessment. She has been on three times a week in-center hemodialysis for the past 2 years. She is on long- and short-acting insulin, which she manages herself. She reports taking her medications regularly. There is no history of heart disease, but she has had two admissions to the hospital for fluid overload that required emergent dialysis. She reports frequently cutting short her dialysis treatments because of scheduling conflicts with her job. She is single and lives alone. She does not have confirmed post-transplantation caregivers but indicates that her cousin may be able to help. Her clinic BP measured after 5 minutes of rest is 180/100 mm Hg. Routine laboratory studies show a hemoglobin of 9 g/dl, serum potassium of 6.1 mmol/L, serum sodium of 130 mmol/L, serum calcium of 10.2 mg/dl, and serum phosphorus of 7.8 mg/dl..",Which of the following has been shown in studies to be associated with risk for nonadherence after transplantation?,"1. Failure to complete full dialysis treatments for nonmedical reasons 2. Poorly controlled hypertension 3. Poorly controlled hyperphosphatemia on dialysis 4. Hyperkalemia 5. Inadequate social support",C,temp nan,Transplantation 5619," A 73-year-old man with CKD-5 from polycystic kidney disease is in the clinic for a regular follow-up visit. His eGFR from laboratory studies done a week ago was 19 ml/min. He experiences mild fatigue but is otherwise doing well. He is retired but remains active around the house and works regularly in his garden. He has osteoarthritis in his knees and occasionally uses a cane for assistance. His BMI is 35 kg/m2. He has a history of hypertension, which is well controlled with medications, but no other significant comorbidities. He is interested in pursuing a kidney transplantation for his progressive CKD. He has not explored the living donor options yet and expresses a desire to not go for living donor kidney transplantation..",Which ONE the following is CORRECT regarding this kidney transplantation for this patient?,"1. Wait until patient has an eGFR <10 ml/min to refer him for transplantation evaluation 2. Inform patient that his benefits from transplantation are low, given his age and do not refer for transplantation evaluation 3. Refer patient for transplantation evaluation only if he has a likely living donor option 4. Pre-emptive transplantation should be avoided because elderly patients have a higher risk of death than of progression to dialysis 5. He should be referred for transplantation evaluation now",E,temp nan,Transplantation 5620," A 52-year-old man with a history of ESKD from biopsy-proven AL amyloidosis comes for kidney transplantation evaluation. He had presented 3 years ago with AKI along with hypertension, new-onset lower extremity swelling, and weight gain. His urine protein-to-creatinine ratio was 4.5 g/day. A renal biopsy had revealed AL amyloidosis with lambda light chain restriction. He had no cardiac involvement. He was treated with steroids, chemotherapy, proteosome, inhibitors and stem cell transplantation and went into complete remission 18 months ago. He remains in complete remission..",Which ONE of the following is associated with the best kidney transplant survival in this patient?,"1. Treatment of AL amyloidosis with steroids, chemotherapy, and proteosome inhibitors 2. Absence of cardiac involvement 3. Treatment with stem cell transplantation 4. Achievement and duration of complete remission",D,temp nan,Transplantation 5621," A 64-year-old patient is evaluated for a kidney trans-plantation. His comorbidities include type 2 diabetes mellitus for the past 5 years, well-controlled hyper-tension for the past 10 years, and obesity. He also recalls a history of passing three kidney stones, the last one being about 2 years ago. He did get a stone analysis during one of these episodes and was told that these were calcium oxalate stones. The cause of his ESKD is biopsy-proven FSGS, which was believed to be secondary to obesity. He started hemodialysis 1 year ago and is tolerating this well. His height is 70 inches, and his weight is 146 kg, with a BMI of 46. He has tried several weight loss treatments, including dietary modification and exercise, for the past 2 years. However, he has been unable to lose much weight. He is otherwise active, but your institutional cutoff for BMI is 40. He is frustrated that he cannot undergo transplantation because of his weight..",Which ONE of the following is the BEST approach at this time?,"1. Advise him to continue his dietary efforts and return for transplantation evaluation after his BMI is below 40 2. Advise him that because he is doing well on dialysis, he should continue it because the risks of transplantation in his case are too high 3. Refer him for sleeve gastrectomy 4. Start him on orlistat as a weight-reducing agent",C,temp nan,Transplantation 5622," A 72-year-old man with ESKD secondary to diabetic nephropathy is evaluated for a kidney transplantation. He also has a history of hypertension and coronary artery disease, with placement of one stent 3 years ago. His cardiac test results are otherwise unremarkable. He started hemodialysis last month and is tolerating treatments without a problem. He is hepatitis B surface antigen negative, hepatitis B surface anti-body positive, hepatitis C negative, and HIV negative. He does not have any living donors, and the wait time in your center is over 5 years. He is very interested in getting a transplant because of his active lifestyle and desire for travel. He inquiries about receiving a kidney transplant from a hepatitis C–positive donor..",Which ONE of the following is TRUE regarding kidneys from hepatitis C–positive donors?,"1. He is a candidate only for kidney from a donor with positive hepatitis C serology but negative hepatitis C NAT 2. He is not a candidate for hepatitis C–positive donor kidney, given his hepatitis C–negative status 3. Kidneys from hepatitis C–positive donors have been shown to have lower graft survival 4. Accepting a kidney from hepatitis C–positive donor may help shorten the time to kidney transplantation",D,temp nan,Transplantation 5623," A 34-year-old man with a history of ESKD secondary to type 1 diabetes mellitus comes for a pretransplantation evaluation. He has been diabetic since age 13 years and is currently on a basal-bolus regimen of long-acting and ultra-short-acting insulin. He is currently awaiting insurance approval for an insulin pump. His blood sugars fluctuate widely during the day, with a range of 40 to 500 mg/dl. His last HbA1C was 9%. His BMI is 26 kg/m2. He maintains hypoglycemic aware-ness but goes into diabetic ketoacidosis frequently. His secondary complications from diabetes include peripheral neuropathy and retinopathy. He has no known coronary artery disease. He is currently on dialysis three times a week through a left upper extremity arteriovenous fistula. He lives with his parents and is currently on disability. He has several family members who are interested in donating a kidney to him..",Which of the following options has been shown in studies to have the best kidney survival?,"1. Deceased donor kidney transplantation 2. Living donor kidney transplantation 3. Deceased donor kidney transplantation followed by a pancreas after kidney transplantation 4. Simultaneous kidney and pancreas transplantation 5. Living donor kidney transplantation followed by a pancreas after kidney transplantation",E,temp nan,Transplantation 5624,,"Question 5624 The prevalence of hypertension in the adult world population is estimated at:","1. 10%–20% 2. 25%–35% 3. 45%–55% 4. 60%–70%",B,temp nan,Hypertension 5625," The chief medical officer of your multispecialty clinic wishes to improve quality of care using standardized performance measures and wants to benchmark your hypertension control rates. To plan this initiative, you are asked to present an outline of the scale of the problem to be addressed..","With regard to current epidemiological trends in hypertension, which ONE of the following is TRUE?","1. Using the definition of hypertension from the 2017 AHA/ACC guideline, the prevalence of hypertension in the United States is estimated to be 35% 2. According to the most recent US National Health and Nutrition Examination Survey (NHANES), data from 2017 to 2018, rates of hypertension control have decreased 3. Hypertension is estimated to affect 22.4% of adults in the United States aged 40–59 years 4. Despite increasing prevalence, death rates attributed to hypertension have fallen in recent years",B,temp nan,Hypertension 5626," A 62-year-old woman is referred for evaluation of stage 3a chronic kidney disease. She has no history of hypertension or cardiac disease. She is taking thyroid replacement and an aromatase inhibitor. Her BP is 136/84 mm Hg as the average of three readings taken 1 minute apart by automated BP measurement with heart rate 72 bpm. Her body mass index (BMI) is 27.4 kg/m2. Her examination results are otherwise normal, and she appears euvolemic. Laboratory studies show sodium of 137 mEq/L, potassium of 4.0 mEq/L, chloride of 99 mEq/L, total CO2 of 28 mmol/L, BUN of 20 mg/dl, and creatinine of 1.2 mg/dl with eGFR 51 ml/min per 1.73 m2 body surface area. A urinalysis shows no protein, cells, or casts. Based on this reading, she asks if you consider her BP to be elevated and how it should be further assessed..",The US Preventive Services Task Force recommends which of the following approaches to hypertension screening in the general population?,"1. Careful office measurement with return for repeated measurements if BP is elevated 2. Careful office measurement followed by out-of-office measurement if BP is elevated 3. Initial testing with ambulatory BP monitoring 4. Initial testing with home BP monitoring",B,temp nan,Hypertension 5627," Differences in hypertension control rates in patients of different racial and ethnic minorities may be due to socioeconomic and healthcare disparities, or biologic differences in response to treatment..","Based on an NHANES analysis of antihypertensive drug use and BP control rates by race and ethnicity covering data from 2005 to 2012, which of the following statements is accurate regarding racial and ethnic disparities in hypertension treatment and control?","1. Hypertension treatment and control rates improved for all racial and ethnic groups 2. Black patients had lower control rates owing to lower intensity of treatment by their providers 3. Black patients had the lowest hypertension control rates 4. Differences in hypertension control were caused by economic disparities across racial and ethnic groups",A,temp nan,Hypertension 5628," An 83-year-old man returns for evaluation of his primary hypertension, which is treated with two agents. His home readings have been running 135/66 mm Hg, based on the average of 28 home readings taken twice daily over the prior 2 weeks. He lives independently with his spouse and is active in his local community, including tutoring of middle school students twice a week. He is taking full doses of lisinopril 40 mg daily and amlodipine 10 mg daily. His BP is 136/62 mm Hg as the average of three readings taken 1 minute apart by automated BP measurement with heart rate 72 bpm. His BMI is 24.4 kg/m2. His examination results are otherwise normal, and he appears euvolemic..",Which of the following statements is accurate regarding treatment of hypertension in patients aged 80 years and older in SPRINT?,"1. This cohort was not included in SPRINT, so we have no trial data to apply 2. Further post-hoc analysis indicates potential harm from intensive treatment of this subgroup 3. Gait speed was a predictor of benefits from intensive treatment 4. D. Patients with high cognitive function showed benefit, whereas those with low function did not",D,temp nan,Hypertension 5629," A 32-year-old man returns for reevaluation of stage 1 hypertension diagnosed 12 months ago and treated with recommendations for lifestyle modification. He has multiple risk factors for future cardiovascular disease (CVD), including daily smoking, obesity, glucose intolerance, sedentary lifestyle, and family history of premature CVD. He was advised to make lifestyle modifications, including smoking cessation, regular exercise, and weight loss. He has been under stress with his job and has not been able to find time to exercise or change his diet. With the stress he has continued to smoke. He was seen for a 6-month follow-up visit with similar concerns, but despite good intentions, his circumstances have not improved. His BP is 136/78 mm Hg as the average of three readings taken 1 minute apart by automated BP measurement with heart rate 72 bpm. His BMI is 36.4 kg/m2. His examination results are notable for abdominal obesity and are otherwise normal. Because he is younger than 40 years, the ACC/AHA pooled cohorts ASCVD 10-year risk calculator does not apply..",Which of the following statements is accurate concerning approach to stage 1 hypertension in young adults under age 40 years?,"1. Inasmuch as young adults have low 10-year ASCVD risk scores, observation is the best approach 2. Lifestyle modification alone should be encouraged at follow-up visits until the age of 40 years 3. After 6 months of lifestyle changes, all patients with continued stage 1 hypertension should be prescribed medication 4. After 6 months of lifestyle therapy, drug treatment should be considered for patients with a history of premature birth or multiple cardiovascular risk factors",D,temp nan,Hypertension 5630," After the release of the 2017 ACC/AHA BP guideline, several international guidelines have been released. As with the United States guideline, updated guidelines from the European Society of Cardiology/European Society of Hypertension, the World Health Organization, the American Diabetes Association, and the Kidney Disease Improving Global Outcomes Collaborative trend to lower thresholds for treatment and lower BP targets, although some differences persist..",Which of the following hypertension guidelines advises the lowest target BP?,"1. 2017 ACC/AHA blood pressure guideline 2. 2018 ESC/ESH hypertension guideline 3. 2021 WHO hypertension guideline 4. 2022 ADA Standards of medical care in diabetes 5. 2021 KDIGO hypertension guideline",E,temp nan,Hypertension 5631," A 63-year-old woman is prescribed pazopanib for the treatment of renal cell carcinoma. Her in-office BP before starting the pazopanib was 118/74 mm Hg without antihypertensive medication. However, her BP was found to be 164/88 mm Hg 3 weeks into treatment. Routine blood work revealed a stable serum creatinine level and normal complete blood count..",Which of the following is the most likely mechanism for hypertension induced by vascular endothelial growth factor (VEGF) inhibitors?,"1. Increased vascular resistance via impaired angiogenesis 2. Glomerular capillary injury from thrombotic microangiopathy 3. Increase in aldosterone release from the adrenal cortex 4. Stimulation of collecting duct renin",A,temp nan,Hypertension 5632," A 33-year-old man undergoes whole genome single nucleotide polymorphism (SNP) genotyping for a research study. He was given his results and sets up a clinic visit to discuss his risk for hypertension from an SNP in the promoter region of uromodulin..",Which mechanism explains how SNPs in the promoter region of uromodulin may predispose to hypertension?,"1. Decreased activity of Na-K-Cl cotransporter 2 (NKCC) 2. Increased prorenin receptor activity in the collecting duct 3. Endocytosis of transient receptor potential vanilloid 5 (TRPV5) in the distal convoluted tubule 4. Reduced urinary activation of claudin-19",C,temp nan,Hypertension 5633," A 65-year-old woman with a medical history of hypothyroidism and obesity is seen in clinic for an annual evaluation. She has just returned from a 2-week vacation at the beach, where she admits to eating out frequently, often choosing fried and high-salt foods..",Which mechanism of hypertension is associated with a high-salt diet?,"1. Reduction in tubuloglomerular feedback signaling 2. Altered gut microbiota 3. Activation of brain renin-angiotensin system 4. Impaired prorenin receptor binding in the collecting duct",B,temp nan,Hypertension 5634," A 72-year-old man with CKD stage 3, eGFR 40 ml/min per 1.73 m2, prior CABG, and hypertension comes for a follow-up visit. He is currently treated with carvedilol 12.5 mg twice daily, losartan 75 mg daily, and amlodipine 10 mg daily. In clinic, his BP is above goal at 146/ 88 mm Hg. On examination, he has trace lower limb edema, which he attributes to eating potato chips at the weekend. He admits to a high-salt diet and to eating processed and preprepared foods on a daily basis..",Which of the following is true regarding salt-sensitive hypertension?,"1. In a model of salt-sensitive hypertension, spironolactone was less effective for BP control than losartan in rats consuming a high-salt diet 2. High-salt diet promotes the development of Th17 cells, which upregulate renal sodium reabsorption 3. In hypertensive patients with high salt intake, increased dietary potassium intake does not affect BP 4. In micropuncture experiments, SGLT2 inhibition that leads to a decrease in glomerular capillary pressure is invariably associated with an acute decrease in GFR",B,temp nan,Hypertension 5635," A 68-year-old woman is admitted to the medical intensive care unit with respiratory distress of sudden onset. She has been exposed to Covid-19 by a family member. She has received two Covid mRNA vaccines and a booster. She also has a history of well-controlled hypertension and heart failure. Medications include lisinopril 20 mg daily, aspirin 81 mg daily, carvedilol 12.5 mg twice daily, and furosemide 20 mg daily. She uses bilevel positive airway pressure. Her BP is 148/78 mm Hg, heart rate 88 bpm, chest sounds rhonchi bilaterally systolic ejection murmur present, no edema. Family is concerned that patient may require intubation and want the physicians to stop the lisinopril because they are afraid this will worsen her Covid-19 symptoms..",Which ONE of the following is the MOST appropriate next step in management?,"1. Stop lisinopril and switch to losartan 100 mg daily 2. Stop lisinopril and switch to amlodipine 10 mg daily 3. Continue current medications 4. Stop lisinopril because upregulation of ACE2 by lisinopril will exacerbate Covid-19 symptoms",C,temp nan,Hypertension 5636," A 56-year-old man has hypertension and hyperlipidemia. He has no other significant medical problems but has a strong family history of coronary artery disease and is concerned about reducing his risk for CVD. He takes rosuvastatin 5 mg at night, amlodipine 5 mg and losartan 50 mg both in the morning. He has read in the news that if he takes his medications at night he will be able to reduce his risk of CVD substantially..",Which ONE of the following is the MOST appropriate response by the physician?,"1. He should take all his BP medications at night because this will reduce the risk of CVD by 50% 2. He should take all his BP medications in the morning because this will reduce the risk of CVD by 50% 3. He should take all his BP medications with his statin to improve compliance 4. He should continue taking his medications as prescribed and ensure he is compliant",D,temp nan,Hypertension 5637," A 56-year-old woman with hypertension, hyperlipidemia and type 2 diabetes mellitus (DM) comes for a scheduled office visit. She feels well without complaints. She restricts dietary sodium and goes to the gym 3 times per week, doing aerobic exercise for 30 minutes each time. She is unable to lose weight and is frustrated. Her BMI is 36, BP is 144/96 mm Hg, and the rest of her physical examination results are unremarkable..",Which ONE of the following is the MOST accurate regarding weight loss for this patient?,"1. Physical activity and exercise training reduce obesity, and a 2%–3% reduction in weight will lower BP, but reductions of at least 5%–10% are desired within 6 months to yield improvements in the major CVD risk factors 2. Once weight loss is achieved it is usually easy to maintain 3. A 5%–10% reduction in weight is associated with 10 mm Hg reduction in systolic BP (SBP) 4. Weight loss in the first 2 months of pharmacologic therapy is predictive of changes noted at 1 year 5. Medications including orlistat, phentermine/topiramate, naltrexone/bupropion, liraglutide, and semiglutide are ineffective for weight loss",A,temp nan,Hypertension 5638," A 59-year-old man is referred to you for management of difficult to control hypertension. He has type 2 DM, CVD, and CKD stage 4. His estimated GFR is 27 ml/min per 1.73 m2 and 1.4 grams of albuminuria/day, based on microalbumin-to-creatinine ratio. He tells you that he has never been able to control his BP despite taking multiple medication..","Based on the SPRINT data, all of the following are associated with failure to achieve intensive SBP target EXCEPT:","1. Younger age 2. Baseline SBP >140 mm Hg 3. Greater number of antihypertensive medications 4. Macroalbuminuria 5. Preexisting CKD",A,temp nan,Hypertension 5639," A 65-year-old man who is an established patient in your practice wants to continue doing telemedicine visits even though your practice is now back in the office. His medical history is significant for longstanding hypertension with intermittent poor control, stage 3 CKD, and hyperlipidemia..",Which ONE of the following is the MOST accurate regarding use of telemedicine (TM) visits for care of patients with hypertension?,"1. The most effective approach is to use TM based on telemonitoring of BP and tracking of additional vital and nonvital signs with data exchange between patients and a case manager through the web, emails, text messaging, or video consultation, integrated with education on lifestyle, risk factors, and proper use of antihypertensive medications 2. TM is not accredited for the treatment of patients with hypertension 3. Use of TM for treatment of patients with hypertension should be limited to those with well controlled hypertension 4. Use of TM for treatment of patients with hypertension should be limited to those on less than 3 medications",A,temp nan,Hypertension 5640," A 44-year-old woman has recently received a diagnosis of hypertension. She has no other comorbidities. Her BP has been poorly controlled, and it has been difficult to ensure compliance with medications. Her BP is 168/ 98 mm Hg, and she has no edema. The rest of her physical examination results are unremarkable. The results of a workup for secondary causes of hypertension have been negative. In addition to lifestyle modification, you urge her to monitor her BP at home twice daily and transmit her readings every week. She thinks this is a lot of extra work and wants to know if this is worth it and whether it will improve her BP..",Which ONE of the following is the MOST accurate regarding use of home BP monitoring for care of patients with hypertension?,"1. A recent study of physician perspectives showed high level of implementation of home blood pressure monitoring in routine care. 2. Use of home BP monitoring with use of pharmacist/healthcare provider feedback has been shown to be effective but more costly 3. Use of home BP monitoring with use of pharmacist/healthcare provider feedback has been shown to be ineffective and more costly 4. Use of home BP monitoring with transmission regularly for feedback to healthcare provider and adjustment of medications has been shown to significantly improve BP",D,temp nan,Hypertension 5641," A 38-year-old man with hypertension and hyperlipidemia has issues with compliance and forgets to take his medications on occasion. His BP control has been variable. He has no other health issues. He is complaint with lifestyle modifications..","He is currently taking losartan 50 mg daily, hydrochlorothiazide 12.5 mg daily, and amlodipine 2.5 mg at 1 Which ONE of the following is the MOST appropriate next step in management?","1. Switch to triple combination therapy to improve BP control and compliance 2. Increase amlodipine to 5 mg daily and continue other medications 3. Add spironolactone 12.5 mg daily 4. Add carvedilol 12.5 mg twice daily 5. No change in therapy",A,temp nan,Hypertension 5642," A 63-year-old woman is followed up in your clinic for hypertension. After 3 years clinic visits, her office BP remains poorly controlled and largely unchanged at 160/80 mm Hg despite escalation of her antihypertensive regimen, which now includes maximal doses of candesartan, amlodipine, and hydrochlorothiazide. Owing to concerns over medication adherence, you proceed with directly observed therapy to assess her BP control. Two hours after direct observation of her taking her medications, her office BP is 130/80 mm Hg..",Which of the following is the best description of this patient’s condition?,"1. Resistant hypertension 2. Pseudoresistant hypertension 3. Refractory hypertension 4. Secondary hypertension",B,temp nan,Hypertension 5643," A 45-year-old man is referred to your clinic for poorly controlled BP despite treatment with perindopril, amlodipine, and chlorthalidone. His office BP is 155/ 85 mm Hg; however, he states that his home blood pressure readings are much better at 125/80 mm Hg. He has been told in the past that he has white-coat syndrome..",Which of the following is the best approach to evaluate for the white-coat effect?,"1. Ask the patient to maintain a daily log of home BP readings 2. Repeat the resting BP 10 minutes later while the patient is left unattended 3. Calibrate the home BP cuff against the office BP cuff 4. Arrange for a 24-hour ambulatory BP monitor",D,temp nan,Hypertension 5644," A 50-year-old otherwise healthy woman is referred to you for hypertension (170/100 mm Hg) plus spontaneous hypokalemia (3.1 mmol/L). Plasma renin activity is < 1 ng/ml per hour, and plasma aldosterone concentration is 35 ng/dl. Dynamic confirmatory testing via oral sodium loading confirms the diagnosis of primary aldosteronism. A CT scan of the adrenal gland shows a left adrenal nodule..",What is best next step in the treatment of this patient?,"1. Order an adrenal MRI 2. Order adrenal vein sampling 3. Treat with lifelong spironolactone 4. Proceed with surgical adrenalectomy",B,temp nan,Hypertension 5645," A 70-year-old man who is a former smoker with coronary artery disease is referred to you from his primary care provider for increasing BP over the past year. Before this, his BP was well controlled with metoprolol and lisinopril. He has now required the additions of indapamide and amlodipine to maintain his BP in a normal range (currently 130/80 mm Hg). His kidney function remains normal. Out of concern for renovascular disease, his primary care provider obtained CT angiography, which demonstrated a 75% stenosis of the right proximal renal artery. The primary care provider wants your opinion on how to proceed given this finding..",What would be the MOST appropriate treatment approach at this stage?,"1. Order duplex ultrasonography 2. Arrange for renal arteriography 3. Proceed with percutaneous transluminal renal angioplasty and stent placement 4. Maintain current treatment",D,temp nan,Hypertension 5646," A 51-year-old woman presents in referral to your clinic with elevated BP along with periodic episodes of headache, palpitations, and diaphoresis. Her office BP is 135/85 mm Hg on candesartan and amlodipine. Based on these symptoms, you order plasma free metanephrines, for which the metanephrine level is normal but the normetanephrine level is 30% above the upper limit of normal..",What is the BEST next step to evaluate for pheochromocytoma/paraganglioma in this patient?,"1. Repeat plasma free metanephrines 2. CT scan of the adrenal glands 3. MIBG scintigraphy 4. Clonidine suppression testing",A,temp nan,Hypertension 5647," A 63-year-old man with stable nonproteinuric stage 4 CKD (eGFR 25 ml/min per 1.73 m2) returns to see you in clinic with an elevated office BP of 150/90 mm Hg. He also has new mild edema in his feet and ankles. He is currently treated with perindopril and amlodipine and maintains a low-sodium diet. You would like to prescribe him a diuretic..",Which of the following is FALSE regarding diuretics in this scenario?,"1. Loop diuretics are effective at lowering BP and reducing fluid overload in advanced CKD 2. Thiazide diuretics are ineffective in patients with eGFR < 30 ml/min per 1.73 m2 3. Potassium-sparing diuretics should be used only with caution 4. Close monitoring of serum electrolytes and kidney function should be performed after starting or adjusting the dose of diuretics",B,temp nan,Hypertension 5648," A 58-year-old woman with longstanding hypertension returns to see you. Her BP is 155/95 mm Hg despite maximal doses of amlodipine, candesartan, and chlorthalidone. Her kidney function remains normal. Evaluation for secondary causes of hypertension has been unrevealing, and she appears to be compliant with her medications and your lifestyle recommendations. You would like to add an additional antihypertensive medication to help control her BP..",Which ONE antihypertensive medication would be BEST to add-on to her current regimen?,"1. Doxazosin 2. Hydralazine 3. Labetalol 4. Spironolactone",D,temp nan,Hypertension 5649," A 35-year-old woman is referred for evaluation of hypertension that developed at 27 weeks’ gestation. She has no significant medical history, and her condition during prior prenatal visits, including BP measurements were normal. Recent onset of hypertension was confirmed by home readings. Her BP during the visit is 149/89 mm Hg. The result of screening urinalysis was negative for hematuria and proteinuria. Urine protein-to-creatinine ratio was 0.15 mg/mg. Laboratory evaluation demonstrated creatinine of 0.6 mg/dl, normal blood counts including platelets, and normal liver function test results, aspartate aminotransferase and alanine aminotransferase. Serum levels of fms-like tyrosine kinase and placental growth factor ratios were within normal for gestational age..",All of the following support gestational hypertension rather than preeclampsia EXCEPT:,"1. Absence of increased proteinuria 2. Onset of hypertension after 20 weeks’ gestation 3. Normal serum creatinine, platelet count and liver function tests. 4. Normal values and ratios of angiogenic markers, fms-like tyrosine kinase and placental growth factor",B,temp nan,Hypertension 5650," A 41-year-old woman is noted to be hypertensive at her first prenatal appointment at 12 weeks. She describes being mildly hypertensive during her prior pregnancy 6 years ago, but she has not been followed up by a physician since then. She states that both her parents have hypertension. She further notes that she has measured her BP at home using her mother’s BP cuff, and it runs 139/92 mm Hg. Her BMI is 33, and her examination results are otherwise unremarkable. Laboratory investigation results, including urinalysis, protein-to-creatinine ratio, and serum creatinine, are normal..",What is the MOST likely diagnosis?,"1. Essential hypertension 2. Gestational hypertension 3. Preeclampsia 4. White-coat hypertension",A,temp nan,Hypertension 5651," A 29-year-old woman presents at 16 weeks’ gestation. She has a history of type 2 DM, hypertension, and CKD, with eGFR 42 ml/min per 1.73 m2. She has a history of prior pregnancy complicated by preeclampsia. She asks you, what is her risk having preeclampsia again?.",Which of the following statements is the MOST accurate?,"1. Her risk of preeclampsia is high, primarily because of her history of preeclampsia and hypertension; type 2 DM and CKD also contribute to risk but to a lesser degree than hypertension and prior preeclampsia 2. CKD confers the highest risk of preeclampsia; all other factors play a lesser role including history of preeclampsia 3. She is not at high risk for preeclampsia because she is < 30 years of age 4. Type 2 DM, CKD, hypertension, and prior history of preeclampsia all contribute equally to risk of preeclampsia",A,temp nan,Hypertension 5652," A 39-year-old woman at 16 weeks’ gestation presents with BP 142/86 mm Hg. She has a history of type 2 DM and hypertension, and has stable protein excretion for the past 2 years of approximately 1 gram per day. She is taking losartan 100 mg daily..","According to the most recent data, what is the BEST approach to treating her hypertension?","1. Continue losartan because her BP is well controlled. 2. Stop the losartan and start labetalol 3. Continue losartan but add another agent to target BP < 140/90 mm Hg 4. Stop the losartan and add a calcium channel blocker",B,temp nan,Hypertension 5653," A 29-year-old woman presents at 13 weeks of her second pregnancy. During her first pregnancy, she was treated with 81 mg/day of aspirin, based on her history of chronic hypertension. Her BP was well controlled on labetalol, but she went into preterm labor and delivered at 35 weeks’ gestation. Now, she presents for counseling about whether she needs aspirin again in this pregnancy. Her BP is 132/84 mm Hg on methyldopa 250 mg orally three times daily. This medication was started in place of losartan, which she took before pregnancy..",Which ONE of the following is the best advice for this woman?,"1. The woman should be prescribed 150 mg/day of aspirin to be taken from 20 weeks’ until 36 weeks’ gestation 2. The woman should understand that aspirin may not be effective for preeclampsia prevention in women with chronic hypertension, but if prescribed, it should be started before 16 weeks’ gestation 3. This woman did not experience preeclampsia in her first pregnancy, and so the risk of preeclampsia in this pregnancy is very low, despite the his-tory of chronic hypertension 4. Aspirin is associated with no increased risk of bleeding in pregnancy",B,temp nan,Hypertension 5654," A 23-year-old man presents for evaluation of edema. He is found to have 6.2 g of protein over 24 hours. Serum albumin is 2.3 g/dl, and serum creatinine is 0.9 mg/dl. The patient undergoes a kidney biopsy that shows normal light microscopy. The IF shows evidence of fine punctate staining for IgG. The EM results are pending..",What is the MOST likely explanation for the finding on the immunofluorescence?,"1. Nonspecific entrapment of IgG in the glomeruli 2. Presence of anti-PLA2R antibody 3. Presence of antinephrin antibody 4. Presence of anti–NELL-1 antibody",C,temp nan,Primary and Secondary Glomerular Diseases 5655," A 43-year-old woman is seen in your clinic for evaluation of proteinuria. Her serum creatinine is 2.5 mg/dl, which has been slowly increasing over the past 10 years. Her serum albumin is 4.2 mg/dl. Her 24-hour urinary protein shows 5.6 g of protein (up from 4.2 g/24 h last year). Her renal US shows two kidneys about 11 cm each in size. Her only other medical his-tory is hypertension, for which she is on losartan 100 mg daily with BP averaging about 120/75 mm Hg. Her body mass index (BMI) is normal, and she has no known family history of kidney disease. A kidney biopsy from 5 years ago was consistent with FSGS on the basis of light microscopy with negative immunofluorescence and EM that showed segmental foot process effacement. She has never received immunosuppressive therapy..",What is the next BEST step in management of this patient?,"1. Add an antihypertensive agent to tighten her BP control 2. Initiate a trial of prednisone therapy 3. Repeat her kidney biopsy 4. Send for genetic testing",D,temp nan,Primary and Secondary Glomerular Diseases 5656," A 33-year-old woman is found to have proteinuria on insurance examination. Additional workup reveals serum creatinine of 0.7 mg/dl and serum albumin of 3.3 g/dl. She has 4.2 g of protein per 24 hours. She undergoes a kidney biopsy that is consistent with membranous nephropathy (MN). PLA2R staining on biopsy is negative, but the biopsy stained positive for exostosin 1/exostosin 2 (EXT1/EXT2)..",Which of the following is MOST correct with regard to EXT1/EXT2 MN?,"1. The patients typically have a poor prognosis and are more likely to progress to ESKD 2. EXT1/EXT2 is seen more commonly in patients with underlying autoimmune diseases 3. EXT1/EXT2 is commonly associated with an underlying malignancy 4. EXT1/EXT2 is more common in pediatric population",B,temp nan,Primary and Secondary Glomerular Diseases 5657," A 56-year-old man is referred for a second opinion with regard to management of FSGS. He was diagnosed a year ago after presenting with creatinine of 1.5 mg/dl, serum albumin of 3.9 g/dl, and proteinuria of 3.9 g/24 h. His biopsy showed FSGS with 15% interstitial fibrosis and tubular atrophy; electron microscopy showed segmental foot process effacement. He was initiated on lisinopril; it was up titrated to 40 mg daily, which he has been tolerating. He is also on amlodipine 5 mg daily and metoprolol 50 mg twice daily. His BP in the office is 115/72 mm Hg, and heart rate is 68 beats per minute. Creatinine is 1.7 mg/dl, albumin is 3.8 mg/dl, and proteinuria is down to 2.3 g/24 h. His BMI is 39 kg/m2, and he has sleep apnea that is treated with CPAP. He has no known family history of FSGS..",Which of the following is the next BEST step?,"1. Initiate an SGLT2 inhibitor 2. Make no changes 3. Refer for genetic testing 4. Add ARB to help lower proteinuria",A,temp nan,Primary and Secondary Glomerular Diseases 5658," A 53-year-old man is referred by his primary care provider for assessment of nephrotic syndrome. He recently experienced weight gain and ankle edema. He was found to have 3+ proteinuria and increased creatinine to 1.8 mg/dl, from a prior baseline of 0.8 mg/dl. His 24-hour urine collection shows 9.8 g proteinuria, and his PLA2R antibody is positive at 93 RU/ml. A kidney biopsy is performed that shows MN, and PLA2R staining is also positive..","When considering a treatment plan, which of the following is MOST correct?","1. Holding immunosuppressive therapy, initiating an ACE-I, and monitoring PLA2R change are the first steps in managing all patients with MN 2. Cyclophosphamide in combination with corticosteroids has been shown to be superior to rituximab on the basis of the RI-CYCLO trial 3. In the MENTOR study, patients treated with rituximab and cyclosporin had similar eGFR at 24-month follow-up 4. In the MENTOR study, those treated with rituximab had higher rates of remission at 24-month follow-up",D,temp nan,Primary and Secondary Glomerular Diseases 5659," A 62-year-old man is seen in your clinic for evaluation of nephrotic syndrome. Workup reveals serum creatinine of 0.8 mg/dl, serum albumin of 3.2 g/dl, and 3.4 g/24 h of proteinuria. BP is 133/82 mm Hg. Additional serologic workup reveals that PLA2R antibody is positive at 35 RU/ml..",Which of the following is the next BEST step in man-aging this patient?,"1. Obtain a kidney biopsy 2. Initiate immunosuppressive therapy with a calcineurin inhibitor 3. Initiate immunosuppressive therapy with rituximab 4. Initiate an ACE-I",D,temp nan,Primary and Secondary Glomerular Diseases 5660," A 34-year-old woman is referred for evaluation of new-onset edema. She has noted increasing lower-extremity edema and weight gain over the last 2 weeks. Her history is significant for morbid obesity with BMI of 37 kg/m2 and diabetes mellitus type 2, with the most recent A1c being 7.2% on metformin therapy. Her creatinine is 0.7 mg/dl, serum albumin is 2.2 g/dl, and 24 hours show 8.8 g/24 h of protein. She undergoes a kidney biopsy that showed normal appearance on light microscopy but diffuse foot process effacement on electron microscopy..","Regarding the management of minimal change disease, which of the following is MOST correct?","1. In patients with contraindication to glucocorticoids, calcineurin inhibitors would be an appropriate first line of therapy 2. Rituximab has been shown to be beneficial in those who fail to respond to steroid or calcineurin inhibitor therapy 3. Calcineurin inhibitor is associated with a lower rate of remission compared with corticosteroids 4. When compared with a tacrolimus-based treatment protocol, treatment with rituximab was associated with a greater cumulative steroid dose",A,temp nan,Primary and Secondary Glomerular Diseases 5661," You are asked to give a second opinion on a 67-year-old man with biopsy-demonstrated IgA nephropathy with hematuria on urinalysis. His creatinine is 0.9 mg/dl. He has microalbuminuria of 600 mg/g creatinine..","BP is 119/80 mm Hg on losartan 100 mg. On the basis of current data, what is the next BEST course of treatment?","1. Add dapagliflozin 10 mg/d 2. No additional therapy is needed; the normal eGFR and proteinuria < 1 g suggest that he will not progress 3. Add chlorthalidone 25 mg daily to target lower BP 4. Add lisinopril 10 mg/d",A,temp nan,Primary and Secondary Glomerular Diseases 5662," You are evaluating a 69-year-old man for hematuria and proteinuria approximately 1.5 g/d. eGFR is stable at 72 ml/min per 1.73 m2. Biopsy demonstrates IgA nephropathy with an MEST score T0. He has a history of diabetes, hypertension, and obesity, with a BMI of 32. He is on lisinopril 40 mg/d and chlorthalidone 25 mg/d, with BP of 124/80 mm Hg. You add dapagliflozin. Six months later, his eGFR is still 70 ml/min per 1.73 m2, but his proteinuria has increased to 2.4 g/d..",Which of the following statements is MOST accurate?,"1. The 2021 Kidney Disease Improving Global Out-comes (KDIGO) guidelines recommend glucocorticoids for all patients with IgA nephropathy who have high-grade proteinuria after a trial of nonimmunosuppressive therapy 2. Rituximab reduces proteinuria in patients with IgA nephropathy and should be used rather than glucocorticoids 3. Glucocorticoids have not been shown to be effective in preventing decline in GFR in patients with IgA nephropathy 4. Glucocorticoids reduce the risk of progression of kidney disease but are associated with an increased risk of adverse events; this risk is markedly attenuated, although still present, at lower doses (0.4 mg/kg per day)",D,temp nan,Primary and Secondary Glomerular Diseases 5663," You are evaluating a 30-year-old woman who was diagnosed with IgA nephropathy by biopsy 1 year ago. She initially presented with 1.2 g proteinuria and with eGFR of 94 mg/min per 1.73 m2. She was normo-tensive on initial presentation. She was treated with lisinopril, but uptitration of the dose has been limited by hypotension. Her insurance did not cover an SGLT2 inhibitor, and she was unable to afford an SGLT2 inhibitor. She now has approximately 900 mg protein with stable creatinine. She asks about the chances of ESKD..",Which ONE of the following is MOST correct regarding the International IgAN Prediction Tool?,"1. The tool is recommended by the 2021 KDIGO guidelines to inform whether or not the patient will respond to glucocorticoids 2. Clinical variables, such as proteinuria, but not histology inform the risk score 3. The tool was updated to more accurately predict risk with variables obtained 1–2 years after biopsy 4. The tool is expensive and requires sophisticated software",C,temp nan,Primary and Secondary Glomerular Diseases 5664," You are evaluating a 19-year-old man with increased creatinine of 1.9 mg/dl, hematuria, and proteinuria 1.2 g/d. He describes a flu-like syndrome about 6 weeks ago. His grandfather died of ESKD, and he has two cousins, both living in Greece, who have undergone kidney transplantation. Tests for hepatitis are negative. ANCA, ANA, RF, and dsDNA are all negative. C3 is mildly low. Biopsy demonstrates a diffuse proliferative pattern. Immunofluorescence reveals both C3 and IgG deposition..",Which ONE of the following statements is MOST correct?,"1. Although exceptions may occur, immunofluorescence microscopy generally shows both IgG and C3 deposits in postinfectious GN but not C3GN 2. Although exceptions may occur, immunofluorescence microscopy generally shows both IgG and C3 deposits in C3GN but not postinfectious GN 3. Light microscopy can distinguish between C3GN and postinfectious GN 4. Genetic screen would be of little utility in distinguishing postinfectious from C3GN",A,temp nan,Primary and Secondary Glomerular Diseases 5665," A 62-year-old man presents with creatinine 1.7 mg/dl, proteinuria 2.5 g, and hematuria. Review of the chart shows multiple urinalyses in the past that were negative for hematuria and proteinuria. Laboratory evaluations, including ANCA, ANA, anti-GBM, hepatitis B virus, and hepatitis C virus (HCV), are negative or normal. C3 is mildly low, and C4 is normal. Biopsy demonstrates a membranoproliferative pattern of histology. Immuno-fluorescence shows 3+ C3 deposition and no Ig. Free kappa and lambda-light chains are elevated, with a kappa-lambda ratio of 12. Family history is negative for ESKD or hematuria..",Which ONE of the following statements is MOST correct?,"1. Among patients with C3GN, monoclonal gammopathy is less common among patients older than 50 2. The absence of IgG on biopsy excludes monoclonal gammopathy–associated C3GN 3. Low C3 strongly suggests heritable complement disorder 4. C3GN in this patient is likely caused by an acquired autoantibody to components of the alter-native complement pathway",D,temp nan,Primary and Secondary Glomerular Diseases 5666," You are evaluating a previously healthy 23-year-old woman who initially presented with hematuria and proteinuria during a brief hospitalization for coronavirus disease 2019 (COVID-19). BP is 135/89 mm Hg, and examination is otherwise unremarkable. Her creatinine is 0.95 mg/d; eGFR 588 ml/min per 1.73 m2. Urine protein measured over 24 hours is 3.6g/24h.SerumC3 is mildly low. Kidney biopsy shows a membranoproliferative pattern with marked C3 deposition and virtually no Ig deposition by immunofluorescence microscopy..",Which of the following variables BEST predicts her risk of developing ESKD?,"1. The presence of electron-dense intramembranous deposits on electron microscopy 2. Low C3 3. A high chronicity score 4. BP",C,temp nan,Primary and Secondary Glomerular Diseases 5667," A 62-year-old woman presents with a 6-week history of systemic illness, with fatigue, weight loss, intermittent fevers, and bloody nasal discharge. She is found to have AKI, with creatinine 2.3 mg/dl, and new-onset hematuria and proteinuria. Additional laboratory tests demonstrate anemia (hemoglobin 10.2 g/dL), thrombocytosis (platelet count 523,109/L), eosinophilia (count 900 cells per µL), and elevated C-reactive protein (81 mg/dl). Chest radiography reveals bilateral cavitation in the lungs..","Regarding the diagnosis of ANCA-associated vasculitis in this patient, which of the following statements is MOST correct?","1. Indirect immunofluorescence is the preferred screening test for detection of circulating ANCA 2. Positive testing for MPO-ANCA excludes a diagnosis of granulomatosis with polyangiitis 3. The demonstration of granulomatous inflammation on tissue biopsy is required to confirm a diagnosis of GPA 4. GN is more common in microscopic polyangiitis than in GPA 5. Serum-soluble CD163 is a sensitive and specific biomarker of GN in patients with both microscopic polyangiitis and GPA",D,temp nan,Primary and Secondary Glomerular Diseases 5668," A 42-year-old man is admitted to the hospital with malaise, nausea, dyspnea, and oligoanuria. His serum creatinine is elevated at 7.1 mg/dl. He is initiated on acute KRT. He is found to be MPO-ANCA positive, and kidney biopsy confirms a diffuse crescentic GN, with a paucity of immune deposits..","Regarding remission induction treatment for this presentation of ANCA-associated GN, which of the following statements is MOST correct?","1. A recent meta-analysis has shown that plasma exchange has no effect on renal outcomes in ANCA-associated GN 2. A reduced dose glucocorticoid regimen is equally effective as a standard dose regimen and associated with fewer infections in patients treated with cyclophosphamide 3. Avacopan is superior to glucocorticoids for achieving sustained disease remission at 6 months 4. A randomized, controlled trial has shown that daily oral cyclophosphamide and rituximab infusions are equally effective for the induction of remission 5. Treatment with mycophenolate mofetil is associated with fewer infections than with cyclophosphamide",B,temp nan,Primary and Secondary Glomerular Diseases 5669," A 37-year-old woman was diagnosed with PR3-ANCA–associated vasculitis 6 months ago and was treated with rituximab and prednisone for induction therapy. She is currently in clinical remission (BVAS=0). Testing for ANCA became negative within 3 months of initiating therapy, and her peripheral B cell count was zero at the most recent check. She has been on tapering dose steroids but remains on prednisone 5 mg daily. She has been reading about the risks of immunosuppression and wants to discuss the bene-fits of azathioprine versus rituximab..","Regarding remission maintenance therapy in AAV, which of the following is MOST correct?","1. In a clinical trial comparing it with maintenance therapy with azathioprine, rituximab resulted in more infections during the maintenance of remission of AAV 2. A rituximab dosing strategy on the basis of repopulation of peripheral B cells and/or changes in ANCA titer led to fewer major relapses and longer relapse-free survival than the regular scheduled 6 monthly dosing of rituximab 3. In a study examining the duration of treatment with oral immunosuppressants to maintain remission, there were no differences in rates of ESKD associated with the use of azathioprine and prednisone for 24 versus 48 months 4. Prolonged maintenance treatment with rituximab for up to 3 years is associated with a lower relapse rate and similar incidence of serious adverse events com-pared with maintenance treatment for 18 months",D,temp nan,Primary and Secondary Glomerular Diseases 5670," A 27-year-old man who is a cigarette smoker presents with pulmonary-renal syndrome. A kidney biopsy shows a diffuse crescentic GN, with linear deposits of IgG and C3 on immunostaining..","Regarding the serologic assessment of anti-GBM disease, which of the following statements is MOST correct?","1. The detection of circulating anti-GBM antibodies is essential for diagnosis 2. The detection of ANCA does not influence treatment or outcome 3. Antibodies to laminin-521 are found more frequently in patients with lung hemorrhage 4. Peroxidasin antibodies are found more frequently in patients with severe kidney disease 5. Anti-GBM antibodies are detected in up to 5% of patients with SLE",C,temp nan,Primary and Secondary Glomerular Diseases 5671," A 62-year-old woman presents with a 1-week history of fatigue and a 3-day history of dark urine and dys-pnea, with reduced urine output within the last 24 hours. On examination, her BP is 182/98 mm Hg, and the examination is notable for JVD, bibasal rales, and 1+ lower limb edema. Chest radiography reveals bilateral ground glass change with apical and peripheral sparing. Her creatinine is 6.8 mg/dl, and she is oliguric. Her urine sediment shows numerous RBCs, including many acanthocytes, and RBC casts. She undergoes renal biopsy, which shows crescentic GN affecting all glomeruli, and linear staining for IgG on immunofluorescence consistent with anti-GBM dis-ease. Her serologic workup reveals dual positivity for ANCA and anti-GBM antibodies..","Regarding the treatment and outcome of anti-GBM disease in this patient, which of the following is MOST correct?","1. The presence of lung hemorrhage is associated with poorer renal outcome 2. Treatment with daily oral cyclophosphamide is preferred to pulse intravenous dosing 3. Following treatment with plasma exchange, cyclo-phosphamide, and glucocorticoids, recovery from dialysis-dependent renal failure can be expected in approximately 30% of cases 4. A randomized, controlled trial has shown that imlifidase is superior to plasma exchange for the rapid removal of anti-GBM antibodies from circulation 5. Maintenance immunosuppression is not required",B,temp nan,Primary and Secondary Glomerular Diseases 5672," You have been asked to give a presentation to your colleagues in primary care on lupus nephritis, with an emphasis on the presentation of lupus nephritis, the interpretation of the serologic and histologic evaluation, and recommendations for follow-up testing..","In the evaluation of lupus nephritis, which of the following is MOST correct?","1. Observational studies suggest that lupus nephritis is being diagnosed at a younger age 2. The time from diagnosis of SLE to the development of lupus nephritis is shorter in Blacks and other minority groups 3. In patients with proliferative lupus nephritis who undergo rebiopsy, histologic findings correlate closely with clinical response to treatment 4. Anti-PLA2R antibodies are frequently identified in patients with class 5 lupus nephritis",B,temp nan,Primary and Secondary Glomerular Diseases 5673," A 28-year-old woman presents to her primary care physician with joint pain, fatigue, and a malar rash. Her laboratory evaluation shows positive ANA and anti-dsDNA antibodies; urinalysis shows 1+ blood and 2+ protein, and serum creatinine is 0.8 mg/dl. Her biopsy reveals class 4 lupus nephritis, and you plan to start her on immunosuppression. You recommend starting mycophenolate mofetil and steroids, but she asks about alternative options and recent developments in treatment..","Which of the following statements is supported by the findings of recent prospective, randomized, con-trolled studies?","1. Rituximab is an effective treatment for lupus nephritis 2. Treatment with obinutuzumab, in addition to standard care, improves renal responses but is associated with a higher risk of infection 3. Belimumab, in addition to standard therapy, has demonstrated efficacy in the treatment of lupus nephritis and extrarenal manifestations of SLE 4. Calcineurin inhibitors, such as voclosporin, are an effective alternative to mycophenolate mofetil for the treatment of proliferative lupus nephritis 5. The addition of anifrolumab to standard therapy results in greater improvements in proteinuria in patients with high type 1 IFN gene signatures",C,temp nan,Primary and Secondary Glomerular Diseases 5674," A 44-year-old morbidly obese Hispanic man who has not received COVID-19 vaccination is admitted to the hospital with respiratory distress 8 days after the initial onset of rhinorrhea and cough. In the emergency room, he is hypoxic and requires high-flow oxygen, and he is started on remdesivir and dexamethasone. His hypoxia worsens over the next 48 hours, and he is transferred to the intensive care unit. His admission creatinine was 1.2 mg/dl (eGFR576 ml/min per 1.73 m2) and has risen to 2.5 mg/dl on hospital day 3. His urinalysis shows 1+ blood and 1+ protein. On hospital day 5, his creatinine has risen to 4.6 mg/dl, and his nephrology team recommends starting continuous KRT..",Which of the following statements is MOST correct?,"1. Remdesivir is the most likely cause of his AKI 2. A kidney biopsy would most likely show thrombotic microangiopathy 3. The mortality of stage 3 AKI in patients admitted to the intensive care unit with COVID-19 exceeds 50% 4. AKI most commonly occurs in the first 5 days of COVID-19 illness",C,temp nan,Primary and Secondary Glomerular Diseases 5675," A 36-year-old Black woman presents to her primary care doctor due to a 15-pound weight gain and lower extremity edema over the last 7 days. She was well until 3 weeks ago, when she developed a sore throat and cough, and she was diagnosed with COVID-19 by RT-PCR test; her COVID-19 symptoms were mild and lasted 8 days, and she felt fully recovered until she noted leg swelling. Her laboratory studies are notable for a creatinine of 2.9 (unknown baseline), and 24-hour urine collection demonstrates 5.5 g of proteinuria and 4.0 g of albuminuria. Urine microscopy shows five to 10 RBCs and zero to five WBCs..",Which of the following statements is MOST correct?,"1. There are numerous glomerular diseases that more commonly occur in patients with COVID-19 com-pared with controls, including COVID-19–associated collapsing glomerulopathy, minimal change disease, MN, and pauci-immune GN 2. COVID-19–associated collapsing glomerulopathy has been described in patients with high-risk APOL1 genotypes 3. The pathogenesis of COVID-19–associated collapsing glomerulopathy involves direct viral infection of podocytes 4. Early antiviral therapy has been shown to improve outcomes in COVID-19–associated col-lapsing glomerulopathy 5. COVID-19–associated collapsing glomerulopathy typically only occurs in patients with severe or critical COVID-19",B,temp nan,Primary and Secondary Glomerular Diseases 5676," A 64-year-old man is referred for evaluation of CKD and new-onset nephrotic-range proteinuria. He is HIV positive and has been taking antiretroviral therapy for >10 years with undetectable viral load. His current antiretroviral regimen is bictegravir, emtricitabine, and tenofovir alafenamide. His other medication problems include hypertension diagnosed at age 45 well controlled on hydrochlorothiazide 25 mg daily. He has zero to five RBCs and zero to five WBCs per high-powered field on urine microscopic examination, 4.6 g of protein on 24-hour collection, and creatinine of 1.2 mg/dl (eGFR575 ml/min per 1.73 m2). Kidney biopsy shows membranous glomerulopathy. Anti-PLA2R antibody titer has been sent and is not yet back..",Which of the following statements is MOST correct?,"1. He does not need to be screened for malignancy 2. His antiviral medications should be changed 3. His biopsy will most likely stain for THSD7 antibody 4. Treatment of HIV alone is unlikely to lead to full or partial remission of kidney disease",D,temp nan,Primary and Secondary Glomerular Diseases 5677," A 70-year-old Black woman is referred for evaluation of CKD and newly detected proteinuria. She was diagnosed with AIDS in 1996 when she presented with a CD4 count of ten; she has been taking antiretroviral therapy for > 25 years with undetectable viral load. Her antiretroviral regimen is efavirenz, emtricitabine, and tenofovir disaproxil fumarate. Her other medication problems include HCV genotype 2 infection that was cured 5 years ago using sofosbuvir and ribavirin and hypertension diagnosed at age 60 that is well controlled with hydrochlorothiazide 25 mg daily and lisinopril 10 mg daily. Her examination shows normal vital signs, BMI of 18, and temporal wasting. She has ten to 20 RBCs and zero to five WBCs per high-powered field on urine microscopic examination. She has a 24-hour urine collection showing 1.1 g of protein and 180 mg of albuminuria. Her serum creatinine is 1.3 mg/dl (eGFR544 ml/min per 1.73 m2), and her serum phosphorus level is 1.9 mg/dl..",Which of the following statements is MOST correct?,"1. Her antiviral medications should be left un-changed because they have been effective in controlling HIV for over 20 years 2. Kidney biopsy is most likely to show immune com-plex GN that may or may not be related to HIV 3. Kidney biopsy will most likely show HIVAN 4. PLA2R levels can rule out MN 5. Cessation of tenofovir disoproxil fumarate will lead to improvement in proteinuria and hypophosphatemia",E,temp nan,Primary and Secondary Glomerular Diseases 5678," A 62-year-old woman receives a diagnosis of HCV infection after routine screening by her primary care doctor. She was referred to you because CKD screening performed as part of her HCV evaluation demonstrated creatinine 1.4 mg/dl, 2+ proteinuria, and 2+ hematuria. When evaluated in your office, she has normal vital signs; her repeat laboratory studies con-firm a creatinine of 1.4, and 24-hour proteinuria collection demonstrates 2.9 g of proteinuria. Her cryocrit level is 2%. She undergoes a kidney biopsy showing membranoproliferative GN with histologic activity but no crescents. She feels well and is asymptomatic. She has not yet been prescribed any therapy..",Which of the following statements is MOST accurate?,"1. A direct-acting antiviral (DAA) regimen, such as sofosbuvir and velpatasvir, and rituximab should be started concurrently because she has an active GN 2. She should be treated with a nonhepatically metabolized DAA regimen because of her kidney disease 3. She should be treated with rituximab first followed by any DAA 4. She should undergo plasmapheresis, rituximab, corticosteroids, and DAA therapy 5. She should be treated with a pangenotypic DAA, and rituximab should not be used, at least initially",E,temp nan,Primary and Secondary Glomerular Diseases 5679," A 14-year-old boy is admitted for severe unilateral flank pain. He was perinatally infected with hepatitis B and is known to be hepatitis B virus positive, but he has had poor compliance with antiviral therapy. On examination, he is uncomfortable, writhing in bed, but he has no costoverterbral angle tenderness. Urinalysis shows 4+ proteinuria and 3+ hematuria; leukocyte esterase is negative, with sediment showing five to ten nondysmorphic RBCs and zero to five WBCs. Serum creatinine is 0.9 mg/dl. A 24-hour urine collection demonstrated 14 g protein. Abdominal imaging shows normal kidneys, no obstruction, and no stone..",What is the next BEST step in his management?,"1. Perform a kidney biopsy to diagnose the cause of nephrotic syndrome 2. Treat with intravenous antibiotics for presumed pyelonephritis 3. Exclude renal vein thrombosis with appropriate imaging 4. Begin antiviral therapy for presumed hepatitis B virus–associated MN",C,temp nan,Primary and Secondary Glomerular Diseases 5680," A 61-year-old man with HIV infection who has stage 5 CKD (attributed to long-standing hypertension and diabetes) is referred for transplant evaluation..",Which of the following statements is MOST COR-RECT regarding transplantation?,"1. He would do better with dialysis than transplantation because the immunosuppressive medications required for transplantation would lead to loss of HIV control 2. There is no need to look for a live donor because he can get an HIV-infected deceased donor very rapidly 3. He should do fine with transplantation because the risks of transplantation are identical to those of HIV-negative patients, including the risk of rejection 4. He will need to avoid calcineurin inhibitors after transplant due to interactions with antiretroviral therapy for HIV 5. Providing that institutional safeguards have been established and the patient’s HIV is controlled with antiretroviral therapy, he is a good candidate for transplantation",E,temp nan,Primary and Secondary Glomerular Diseases 5681," A 45-year-old man with type 2 diabetes mellitus has been referred to you for further evaluation. On review of his medical record, you observe that he was diagnosed with diabetes 5 years earlier and has lost > 12 ml of GFR over the last 2 years. You advise him that a kidney biopsy is required to exclude the diagnosis of an alternative nondiabetic kidney disease to explain his progressive GFR loss. He asks you to explain the risks associated with kidney biopsy. The largest meta-analysis of clinically indicated kidney biopsy complications to date emanated from the Kidney Precision Medicine Program..","On the basis of these data, which ONE of the following is correct?","1. Death does not occur as a result of kidney biopsy 2. Complication rates are equivalent in outpatient and inpatient kidney biopsy procedures 3. The need for intervention to halt excessive bleeding from a clinically indicated kidney biopsy occurs in three of 1000 4. Gross hematuria from a clinically indicated kidney biopsy is extremely rare, occurring in <1%",C,temp nan,Primary and Secondary Glomerular Diseases 5682," A 63-year-old woman with type 2 diabetes mellitus and stage A3G3a (baseline creatinine 1.3 mg/dl) is hospitalized for a urinary tract infection and AKI. Her metformin and valsartan were both stopped; her creatinine peaked at 4.2 mg/dl but was down to 2.8 mg/dl on discharge. You are seeing her 2 weeks later for a posthospital follow-up. Her BP is 128/76 mm Hg, heart rate is 72 bpm, creatinine is 1.7 mg/dl (eGFR is 33 ml/min per 1.73 m2), hemoglobin A1c is 7.6%, and UACR is 480 mg/g. Her discharge medications include sitagliptin, amlodipine, carvedilol, and simvastatin..",What is the BEST medication change for her long-term cardiovascular and kidney health?,"1. Start her on an SGLT2 inhibitor 2. Restart her angiotensin receptor blocker 3. Restart her on metformin 4. Start her on a GLP1 agonist",B,temp nan,Primary and Secondary Glomerular Diseases 5683," You are seeing a 46-year-old man for follow-up after his initial consultation for CKD stage A3G4 in the setting of type 2 diabetes mellitus. He has a history of coronary artery disease and had a stent of his right coronary artery last year. His urine sediment, serologies, and ultrasound were all unremarkable. Today, his vital signs show a BP of 136/84 mm Hg, heart rate of 80 bpm, and BMI of 42. His hemoglobin A1c is 7.6%, creatinine is 3.0 mg/dl (eGFR526 ml/min per 1.73 m2), UACR is 280 mg/g, and potassium is 4.6 mEq/dl. His medications include insulin glargine 120 units nightly, A spart insulin 22 units with meals, empagliflozin 10 once mg daily, losartan 100 mg once daily, amlodipine 10 mg once daily, carvedilol 25 mg once daily, and chlorthalidone 50 mg once daily. You discuss goals for nutrition and exercise..",What is the BEST medication change you can make for his overall medical prognosis?,"1. Increase his empagliflozin to 25 mg daily 2. Change his losartan 100 mg daily to valsartan 320 mg daily 3. Add semaglutide starting at a dose of 0.5 mg subcutaneously once weekly 4. Add finerenone 10 mg daily",C,temp nan,Primary and Secondary Glomerular Diseases 5834," A 72-year-old hemodialysis patient is found to have a plasma phosphate of 9.0 mg/dl on routine monthly lab testing. He takes calcitriol 0.5 mcg daily for treatment of secondary hyperparathyroidism but reports noncompliance with phosphate binders and dietary phosphate restriction. He is educated regarding the need to comply and is referred to meet with a nutritionist at his hemodialysis unit to reduce dietary phosphate intake..",Which of the following is MOST CORRECT regarding intestinal phosphate absorption?,"A. The activity of Pit-1 and Pit-2 is increased by calcitriol. B. Dietary phosphate within phytic acid is readily absorbed owing to the widespread activity of phytases in the intestine. C. Tenapanor inhibits the Na+ /H+ -exchanger isoform 3 (NHE3) in the intestine, thereby reducing paracellular phosphate transport. D. The phosphate transporter NaPillb is expressed across the entire small and large intestine.",C,temp nan,Disorder of Divalent Ions Renal Bone Disease and Nephrolithiasis 5835," A liver transplant recipient with hypophosphatemia (serum phosphate, 1.0 mg/ dl) is started with phosphate supplementation. His medical history, lab data, and medications are reviewed to identify a cause..",Which is the BEST test to investigate the underlying cause?,"A. Measure fractional phosphate excretion B. Measure tubular maximal reabsorption of phosphate/ glomerular filtration rate (TmP/GFR) in a second morning urine after an overnight fast C. Determine serum FGF23 and PTH concentrations D. Determine trough levels of immunosuppressants",B,temp nan,Disorder of Divalent Ions Renal Bone Disease and Nephrolithiasis 5836," A 52-year-old woman with type 2 diabetes, hypertension, and dyslipidemia is following up with her primary care physician (PCP). Routine labs reveal her eGFR is 70 ml/min per 1.73 m2, with albuminuria 40 mg/ g on spot urine sample and plasma phosphate 4.3 mg/ dl (upper limit of normal, 4.5 mg/ dl)..",Which of the following is MOST CORRECT regarding hyperphosphatemia?,"A. In the general population, plasma phosphate levels at the upper end of the normal range have been associated with increased cardiovascular mortality B. Once eGFR levels decrease below 60 ml/min per 1.73 m2, fibroblast growth factor (FGF)-23 levels increase to protect against hyperphosphatemia C. Metabolic or respiratory alkalosis is associated with hyperphosphatemia D. Dietary phosphate in food additives is poorly bioavailable",A,temp nan,Disorder of Divalent Ions Renal Bone Disease and Nephrolithiasis 5837," A 68-year-old woman presents to the emergency department with a 5-month history of gradually progressive fatigue, lower extremity edema, and headaches. She has not been seen by a physician for more than 15 years and is taking no medications. BP on arrival is 180/110 mm Hg, and her creatinine level is 5 mg/dl. Her kidneys are small and echogenic on ultrasound, and an electrocardiogram shows left ventricular hypertrophy. Of note, her serum calcium is 7 mg/ dl, ionized calcium is 0.9 mmol/L, phosphate is 6.5 mg/ dl, and PTH is 465 pg/ml..",Which of the following is NOT a direct action of PTH?,"A. Increased intestinal absorption of calcium B. Increased activity of lα-hydroxylase C. Increased tubular reabsorption of calcium D. Increased bone resorption",A,temp nan,Disorder of Divalent Ions Renal Bone Disease and Nephrolithiasis 5838," A 5-year-old boy was referred to a renal pediatric service because of recurrent urinary tract infections. Labo­ratory findings were notable for eGFR creatinine of 65 ml/min per 1.73 m2, hypomagnesemia (1.1 mg/dl; reference range, 1.6-2.6 mg/ dl), elevated fractional excretion of magnesium (FeMg, 12%; reference range, < 4%), hypercalciuria (12 mg/kg per day; reference range, < 4 mg/kg per day), hypocitraturia (120 mg/ d; reference range, 300-600 mg/ d), and elevated serum PTH (110 pg/ml; reference range, 10-65 pg/ml). On ultrasound, nephrocalcinosis was observed. His parents were cousins, and his 20-year-old brother had recently a diagnosis of advanced CKD..",Which ONE of the following genetic abnormalities is MOST likely causing this clinical condition?,"A. Gitelman syndrome B. Distal renal tubular acidosis C. Bartter syndrome D. A loss-of-function mutation in the CLDN16 gene",D,temp nan,Disorder of Divalent Ions Renal Bone Disease and Nephrolithiasis 5839," A 25-year-old man presented to the emergency unit after a seizure episode. He had two previous episodes, but no investigation was done. Laboratory values were calcium 6.8 mg/ dl, phosphorus 7.0 mg/ dl, and PTH 70 pg/ml (reference range, 12-72 pg/ml). A diagnosis of autosomal dominant hypocalcemia type 1 was performed. Calcium and vitamin D were prescribed..",Which ONE of the following is not a potential complication of this therapy?,"A. Nephrocalcinosis B. Brain calcifications C. Nephrolithiasis D. Hypocalciuria",D,temp nan,Disorder of Divalent Ions Renal Bone Disease and Nephrolithiasis 5840," A 72-year-old woman has CKD stage 5 (not yet on dialysis), hypertension, secondary hyperparathyroidism (PTH, 380 pg/ml; reference range, 10-65 pg/ml), and osteoporosis with a prior radial fracture. Her serum cal­cium is 7.8 mg/ dl, and phosphate is 3.9 mg/ dl. Her PCP contacts you for advice on adding calcimimetic or denosumab given her combination of CKD-MBD and osteoporosis..",Which of the following is correct regarding drug-induced hypocalcemia?,"A. A recent meta-analysis reported that the risk of hypocalcemia is doubled in patients treated with calcimimetics compared with controls B. Denosumab rarely causes hypocalcemia in patients on hemodialysis C. High alkaline phosphatase levels are a risk factor for calcimimetic-induced hypocalcemia D. Denosumab has not been shown to be effective in kidney transplant recipients",C,temp nan,Disorder of Divalent Ions Renal Bone Disease and Nephrolithiasis 5841," A 62-year-old woman with ESKD has been receiving chronic hemodialysis for the last 3 years. Her PTH level has been increasing and has been between 900 and 1000 pg/ ml over the past 4 months. She has previously tried taking cinacalcet but could not tolerate it because of gastrointestinal side effects. You are contemplating prescribing etelcalcetide to this patient..",Which ONE of the following is CORRECT given US data on new users of etelcalcetide?,"A. Patients who had previously used cinacalcet for secondary hyperparathyroidism (SHPT) without success are unlikely to receive a benefit from etelcalcetide, so other approaches to treating SHPT are advisable. B. Recipients of etelcalcetide who received the drug for a year experienced a decrease in PTH level of approximately 40% at 1 year. C. Circulating levels of phosphate tended to begin to decrease with etelcalcetide after approximately 4-6 months of sustained use. D. At any given time point in the first year of etelcalcetide use, approximately 8% of patients will have a corrected serum calcium level of < 7.5 mg/ dl.",B,temp nan,Disorder of Divalent Ions Renal Bone Disease and Nephrolithiasis 5842," A 48-year-old patient with CKD stage 4 secondary to diabetes is seen in clinic to discuss blood results, and he asks about the significance of his elevated PTH. You counsel the patient about the potential impact of developing SHPT, particularly its long-term implications for health..","In studies of patients with CKD and ESKD, which ONE of the following is CORRECT?","A. In observational data, a U-shaped association has been seen between PTH level with mortality, whereas patients with PTH > 1200 pg/ml had an approximately 11 % shorter lifespan than patients with PTH levels at goal. B. Development of SHPT is associated with increased major cardiovascular events and death C. Older age, female sex, and hypertension are pre­dictors of the development of SHPT D. Development of SHPT is not associated with increased risk of CKD progression",B,temp nan,Disorder of Divalent Ions Renal Bone Disease and Nephrolithiasis 5843," You are teaching a group of medical residents about the management of hyperphosphatemia in CKD. After discussing traditional phosphate binders, you move on to discuss newer agents and explain that there are multiple routes of phosphate absorption in the small intestine that can serve as targets for the treatment of hyperphosphatemia..",Which ONE of the following hyperphosphatemia treatments primarily targets intestinal paracellular phosphate absorption?,"A. Tenapanor B. Modified-release nicotinamide C. EOS789 D. Ferric iron oxide adipate",A,temp nan,Disorder of Divalent Ions Renal Bone Disease and Nephrolithiasis 5844," A 58-year-old man with ESKD secondary to type 2 diabetes has been receiving hemodialysis for 4 years. His serum phosphate levels have been > 7 mg/dl for the last several months despite dietary education and increasing his dose of phosphate binders. He has a well-functioning arteriovenous fistula and meets targets for adequate dialysis based on a 4-hour dialysis run. However, he cuts dialysis treatments short several times per month. You are reviewing his treatment regimen in an effort to improve phosphate control..",Which of the following statements regarding phosphate clearance by hemodialysis is CORRECT?,"A. Nearly 80% of phosphate removal by hemodialysis occurs within the first hour of treatment. B. Intracellular phosphate concentrations are reduced by hemodialysis. C. Serum phosphate generally returns to predialysis concentrations within 12 hours of therapy. D. The length of hemodialysis treatment has little impact on the total amount of phosphate removal.",B,temp nan,Disorder of Divalent Ions Renal Bone Disease and Nephrolithiasis 5846," A 59-year-old man with ESKD on hemodialysis due to bilateral nephrectomy (first after motorcycle accident at age 30, second because of renal cell carcinoma at 53 years) presented for transplant evaluation. He has had four episodes of low-trauma fractures (wrist, arm, and leg). He described muscle weakness and difficulty with walking and bone pain, all which developed over the past year. He walks with a cane, for support. His blood tests revealed calcium 8.8 mg/ dl, phosphate 2.2 mg/dl, iPTH 380 pg/ml, 25-hydroxyvitamin D 33.8 ng/ml, alkaline phosphatase 70 U /L (normal for men is 45-115 U/L). Dual-energy x-ray absorptiometry (UXA scan) of femoral neck revealed a T-score of - 3 (osteoporosis diagnosis), and a whole-body bone scan revealed symmetrical uptake of the nucleotide in ribs..",Which one of the following is the MOST CORRECT approach for this patient?,"A. The patient likely has osteoporosis and should have a steroid minimization protocol after the transplant, along with antiresorptive medication. B. The patient likely has osteoporosis and should have a steroid minimization protocol, and no antiresorptive medication will be needed. C. The patient likely has low bone turnover disease along with low bone volume, and, for this reason, the kidney transplant is contraindicated. D. Ideally, this patient should have a bone biopsy in order to identify the optimal therapy.",D,temp nan,Disorder of Divalent Ions Renal Bone Disease and Nephrolithiasis 5847," A 62-year-old man, with membranous nephropathy, underwent kidney transplantation 2 years ago. His kidney graft has good function (creatinine 1.3 mg/dl, CKD-EPI 58 ml/min per 1.73 m2), but he presents with chronic hypercalcemia (calcium 10.8 mg/dl), and high PTH level (iPTH, 520 pg/ml), with normal alka­line phosphatase and phosphate serum levels..",Which ONE of the following is the most correct approach for this patient?,"A. Start an oral calcimimetic, because the patient has hypercalcemic hyperparathyroidism and high bone turnover B. Try to obtain a bone biopsy, because hypercalcemic hyperparathyroidism in kidney-transplant recipients commonly does not represent high bone turn­over but instead normal or low bone turnover C. Schedule a parathyroidectomy, because the patient has hypercalcemic hyperparathyroidism and high bone turnover D. Perform a DXA examination",B,temp nan,Disorder of Divalent Ions Renal Bone Disease and Nephrolithiasis 5848," A 57-year-old woman with ESKD in peritoneal dialysis had a traumatic fracture of her wrist. A DXA exam revealed a T-score of femoral neck of -2. Her rheumatologist wants to start therapy for osteoporosis. PTH is 58 pg/ml, calcium is 8.5 mg/dl, phosphorus is 4.8 mg/ dl, and alkaline phosphatase is 45 U /L (within normal range)..",Which ONE of the following is the MOST CORRECT answer?,"A. Advise denosumab B. Advise a bisphosphonate C. Advise vitamin D supplementation D. Schedule a bone biopsy before starting therapy",D,temp nan,Disorder of Divalent Ions Renal Bone Disease and Nephrolithiasis 5849," A 72-year-old woman with history of hypertension for more than 20 years and diabetes mellitus type 2 diagnosed 12 years ago presented to the nephrology clinic, where she is followed for presumed diabetic nephropathy. Her physical examination reveals BP 144/80 mm Hg, with normal heart rate, and symmet­ric pitting edema in both legs. Her lab results showed a hemoglobin of 11.2 g/ dl, serum creatinine 3.7 mg/ dl and 24 hour urine protein determination of 1.6 g. Her medication consists of lisinopril 20mg day, dapagliflozin 10 mg, atorvastatin 20 mg, and subcutaneous insulin..",Which one of the following is MOST correct?,"A. The nephrologist should be worried about mineral and bone-related parameters, as CKD-MBD is associated with poor outcomes B. The nephrologist should not be worried about mineral and bone-related parameters at this time, as the patient is not in stage 5 C. Cardiovascular morbidity and mortality is similar across the different stages of CKD D. Evaluation of extraosseous calcifications is a better way to assess cardiovascular risk in CKD patients, compared to clinical history and/ or lab results",A,temp nan,Disorder of Divalent Ions Renal Bone Disease and Nephrolithiasis 5850," A 79-year-old woman with ESKD secondary to diabetes and hypertension fractured her hip while ambulating to a wheelchair after dialysis. She is dialyzed three times weekly. Her most recent PTH is 89 mg/ dl (range, 15--65 pg/ml), phosphorus is 4.9 mg/ dl (range 2.5-4.5 mg/ dl), and corrected calcium is 8.8 mg/ dl (range, 8.8-10.7 mg/ dl)..",Which statement below is MOST correct?,"A. In her age group, the risk of fracture is the same for ESKD compared with non-ESKD patients B. The mortality risk for ESKD patients is the same after the fracture compared with mortality risk in non-ESKD patients after fracture C. Factors that increase fracture risk for ESKD patients include age and CKD-MBD and postmenopausal osteoporosis D. Length of hospital stay following the fracture is unlikely to be changed by her ESKD status",C,temp nan,Disorder of Divalent Ions Renal Bone Disease and Nephrolithiasis 5851," A 61-year-old postmenopausal woman has CKD stage 3 related to class IV lupus nephritis treated successfully with Cytoxan and glucocorticoids when she was 30 years old. She has mild hypertension and is taking lisinopril 20 mg daily, atorvastatin, and a multivitamin. PTH and phosphorus have always been in normal range. She has never been screened for osteoporosis because her nephrologist told her the results of common screening methods would be difficult to interpret. Her current PCP asks you for advice regarding screening..",Which of the following statements is MOST accurate?,"A. There is little utility to screening by DXA because she likely has renal osteodystrophy, and DXA would not be sufficiently predictive of fracture risk. B. There is little utility to screening because the treatments for osteoporosis are contraindicated by her reduced eGFR C. DXA should be done, but clinical variables should also be assessed using the fracture risk assessment tool to increase the predictive capability of DXA. D. She should be screened by DXA as per general population screening guidelines.",D,temp nan,Disorder of Divalent Ions Renal Bone Disease and Nephrolithiasis 5852," A 72-year-old man has CKD stage 4 related to renovascular disease. He has coronary artery disease and is s/p ST elevation myocardial infarction, and s/p multiple angioplasties. He has peripheral vascular disease and severe chronic obstructive pulmonary disease. He has been taking glucocorticoids for years for chronic obstructive pulmonary disease. He has had a number of fractures, related to falls, and also has a nontraumatic vertebral fracture. He has been taking calcitriol for many years, but this was stopped when his PTH was noted to be < 100 pg/ml. Follow-up labs after stopping vitamin D receptor activator (VORA) show a low bone-specific alkaline phosphatase and PTH 120 pg/ml. Vitamin D levels are normal. Phosphorus is normal, and he is not taking phosphate binders. Calcium is normal at 9.8 mg/dl..","Of the following, which is the best agent for his osteoporosis?","A. Pamidronate B. Denosumab C. Teriparetide D. Romosozumab",C,temp nan,Disorder of Divalent Ions Renal Bone Disease and Nephrolithiasis 5853," A 59-year-old man has ESKD secondary to diabetes and is receiving dialysis three times weekly. He has hypertension and hyperlipidemia. He is noted to have osteoporosis on a routine screen. PTH is >1200 pg/dl while taking maximum-dose etalcalcetide and calcitriol. Bone­specific alkaline phosphatase is high. Vitamin D is normal, phosphorus is 5--6 mg/ dl, he is taking sevelamer 2400-mg three times a day, and calcium is normal..",Which of the following agents is best for his osteoporosis?,"A. Alendronate B. Denosumab C. Teriparatide D. Parathyroidectomy",D,temp nan,Disorder of Divalent Ions Renal Bone Disease and Nephrolithiasis 5854," An 84-year-old woman with creatinine 1.8 mg/ dl is treated with denosumab following a low trauma vertebral fracture. Laboratory tests prior to treatment demonstrated PTH 286 pg/ml, vitamin D concentration 16 ng/ml, total calcium of 8.3 mg/dl, and phosphorus 4.7 mg/ dl. Laboratory tests were repeated 3 weeks later..",What is the most likely adverse effect of denosumab to be observed on laboratory tests?,"A. Hypercalcemia B. Acute kidney injury C. Hypocalcemia D. Toxic levels of denosumab",C,temp nan,Disorder of Divalent Ions Renal Bone Disease and Nephrolithiasis 5855," A 54-year-old woman with type 2 diabetes mellitus, hypertension, obesity, atrial fibrillation, and end-stage kidney disease treated with peritoneal dialysis has developed an indurated area over right medial thigh. Her medications include glargine insulin, lispro insulin, sevelamer, warfarin, metoprolol, and lisinopril. Sites for insulin injections include abdominal wall and left lateral thigh. On physical examination, the skin over right medial thigh is not ulcerated but has severe tenderness with areas of purple discoloration..",Which of the following medications is a potential risk factor for this presentation?,"A. Glargine insulin B. Lispro insulin C. Sevelamer D. Warfarin E. Metoprolol",D,temp nan,Disorder of Divalent Ions Renal Bone Disease and Nephrolithiasis 5856," A 62-year-old man with primary focal segmental glomerulosclerosis has been hemodialysis dependent for the past 6 years. He was recently hospitalized for abdominal pain, which was determined to be from sigmoid diverticulitis. His symptoms resolved upon medical management of diverticulitis that included antibiotic treatment, and he was discharged home. As you review his hospitalization course during dialysis rounds, you notice abdominal aortic calcification and bilateral iliac artery calcification on CT..",Which of the following therapeutic approaches should be considered in this patient's management?,"A. A void hypercalcemia B. Induce hypophosphatemia C. Supplement vitamin K D. Sodium thiosulfate infusion",A,temp nan,Disorder of Divalent Ions Renal Bone Disease and Nephrolithiasis 5857," A 42-year-old man with persistent hypercalcemia from primary hyperparathyroidism is diagnosed with calciphylaxis involving his left calf. His kidney function and serum phosphorus are normal, and current medications include a multivitamin tablet and tramadol..",Which of the following statements is MOST CORRECT regarding this patient's condition?,"A. Teriparatide should be immediately started B. The 1-year survival rate is likely higher in this patient compared to dialysis-dependent patients with end-stage kidney disease who develop calciphylaxis C. Calcitriol should be immediately started D. Sevelamer should be immediately started",B,temp nan,Disorder of Divalent Ions Renal Bone Disease and Nephrolithiasis 5858," A 61-year-old woman with type 2 diabetes mellitus and end-stage kidney disease treated with hemodialysis has been receiving wound care and sodium thiosulfate infusion treatments for right lower leg wound caused by calciphylaxis. The clinical team is considering starting hyperbaric oxygen therapy to facilitate wound healing..",Which of the following is an absolute contraindication for hyperbaric oxygen therapy?,"A. Untreated pneumothorax B. Claustrophobia C. Sinus infection D. Diabetes mellitus",A,temp nan,Disorder of Divalent Ions Renal Bone Disease and Nephrolithiasis 5859," A 46-year-old woman with end-stage kidney disease underwent surgical debridement of nonhealing leg ulcer located above the left ankle. The patient has severe diabetic peripheral neuropathy and reports that pain intensity at the ulcer site has been mostly none to mild. The pathologist examining the cutaneous tissue obtained during surgical debridement raises a concern for calciphylaxis..",Which of the following features on histology can help distinguish calciphylaxis from its mimics?,"A. Presence of circumferential calcium deposits in hypodermal vessels B. Presence of noncircumferential calcium deposits in hypodermal vessels C. Absence of thrombosis D. Presence of inflammatory infiltrate in cutaneous vessel walls",A,temp nan,Disorder of Divalent Ions Renal Bone Disease and Nephrolithiasis 5860," A 50-year-old man is consulting you because of recent diagnosis of a kidney stone. He was in a good state of heath until 1 week ago when he developed sudden and severe right-flank pain. He had no hematuria and no fever. A computed tomography (CT) scan of the abdomen without contrast revealed mild left hydroureter with an obstructing 4-mm stone at the left ureterovesical junction. Laboratory tests including serum creatinine and urinalysis were normal. In the emergency department, he received analgesics that relieved the pain temporarily, and he signed himself out against medical advice and got an appointment with you 3 days later. He has not passed any stone, and the pain persists when he does not take analgesics. He recalls having passed a kidney stone spontaneously about 10 years ago but has not had symptoms of colic since then until this most recent episode. He eats a high-protein diet and does a lot of exercise in the gym..",What is the best advice at this point?,"A. Repeat blood tests and order a 24-hour urine collection for assessment of stone risk factors B. Advise him to go back to emergency department for urgent urologist consultation C. Start him with expulsive medical therapy (tamsulosin) and get a follow-up for him with a urologist over the next week to evaluate further therapy D. Advise him to drink plenty of water, move around, cut down the high-protein dietary intake, and return in 2 weeks E. Start him on antibiotics to prevent superimposed infection",C,temp nan,Disorder of Divalent Ions Renal Bone Disease and Nephrolithiasis 5861," You asked to see a 35-year-old man with recurrent nephrolithiasis since age 18. He has passed about 15 stones in his lifetime but has not had an episode of colic in the past 3 years. He has a strong family history of kidney stones on both sides of his family. He had removal of one kidney stone with lithotripsy a few years ago, but he is not sure about stone composition. He has taken thiazide diuretics in the past, but he discontinued them because ""they made him urinate too much."" He is currently asymptomatic. Physical examination is unremarkable. Serum creatinine is 1.1 mg/dl, calcium is 8.9 mg/dl, and total CO2 is 26 mmol/L. He has documented 24-hour urine collection of calcium at 360 mg/dl (normal, < 250 mg/dl). A CT scan of the abdomen shows no stones in the kidneys at present..",What is the best course of action for this patient?,"A. Advise a low calcium intake in view of his hypercalciuria B. Perform a complete metabolic assessment, including 24-hour urine for calcium, oxalate, citrate, uric acid, and electrolytes, to establish mechanisms for his hypercalciuria C. Do nothing and follow up in a few months because there is no evidence of new formation of kidney stones over the past 3 years D. Advise against genetic testing as there are no indications for this",B,temp nan,Disorder of Divalent Ions Renal Bone Disease and Nephrolithiasis 5862," A 38-year-old obese woman presents with history of recurrent kidney stones. She is not aware of any other past or present significant medical problems. She recalls that previous clinicians found ""too much uric acid"" in a 24-hour urine collection done last year, but she did not go to follow-up appointments. There is no significant medical family history of kidney stones..",Which ONE of the following is the best advice for this patient at this moment?,"A. Any interventions done to control obesity will be of benefit to prevent new stone formation B. All obesity surgical procedures have been found to be equally associated with tendency to form kidney stones C. Patient should be started on allopurinol D. A 24-hour urine collection for measuring stone risk factors is indicated to identify metabolic abnormalities that may need to be addressed E. Use of thiazide diuretics would be advisable at this point",D,temp nan,Disorder of Divalent Ions Renal Bone Disease and Nephrolithiasis 5863," A 56-year-old man with history of recurrent kidney stones is now referred to you because of the incidental findings of nephrocalcinosis. He was born in Iran and has been living in the United States since age 14 years. Family history is unknown. Physical examination is unremarkable. His serum calcium and serum bicarbonate levels are normal. His main concern is that his serum creatinine was 1.5 mg/ dl in 2019 and 1.7 mg/ dl in 2022. In the past, he has been advised to take potassium citrate, but he has difficulty tolerating it because of ""chronic esophagitis."" A CT scan shows two non-obstructing kidney stones in the right kidney measuring up to 5 mm..",What is the best course of action for this patient at this moment?,"A. 24-hour urine collection for measurement of stone risk factors and blood for genetic analyses B. Refer to a urologist for surgical management of his two kidney stones C. Refer to a nutritionist for detailed evaluation of his habitual dietary intake D. Assure him that nephrocalcinosis, despite its dramatic radiologic appearance, is a benign condition unlikely to progress to renal failure",A,temp nan,Disorder of Divalent Ions Renal Bone Disease and Nephrolithiasis